You are on page 1of 274

Oral Implantology Review, Second Edition

ORAL
IMPLANTOLOGY
REVIEW
SECOND EDITION

LOUIE ALFARAJE, DDS


Founder and Director
California Implant Institute
San Diego, California
One book, one tree: In support of reforestation
worldwide and to address the climate crisis, for
every book sold Quintessence Publishing will
plant a tree (https://onetreeplanted.org/).

Library of Congress Control Number: 2022920494

A CIP record for this book is available from the British Library.
ISBN: 978-1-64724-156-8

97%
© 2023 Quintessence Publishing Co, Inc

Quintessence Publishing Co, Inc


411 N Raddant Road
Batavia, IL 60510
www.quintpub.com

5 4 3 2 1

All rights reserved. This book or any part thereof may not be reproduced, stored in a re-
trieval system, or transmitted in any form or by any means, electronic, mechanical, photo-
copying, or otherwise, without prior written permission of the publisher.

Editors: Kristen Clark and Leah Huffman


Design: Sue Zubek
Production: Angelina Schmelter

Printed in Croatia
CONTENTS

Dedication vii
Preface viii
Acknowledgments ix
Contributors x

1 Medical Evaluation of the Implant Patient 1

2 Oral Evaluation and Treatment Planning 35

3 Head and Neck Anatomy 55

4 Surgical Procedures and Complications 87

5 Pharmacology 129
6 Biomechanics 163

7 Implant Prosthodontics 181

8 Bone Grafting 225

9 Zygomatic and Pterygoid Implants 235

10 Blood Concentrates and Growth Factors 241


Bibliography 253

vi
Dedicated to
Ibn Zuhr (1094–1162)

Biography
Abu-Marwan Abd-al-Malik ibn Zuhr Al Eyadi Al-Ishbily (Avenzoar) was a Muslim Arab physician
and surgeon who was influential in advancing the progress of surgery. His major work, Al-Taysıˉr
fil-Mudaˉwaˉt wal-Tadbıˉr (Book of Simplification Concerning Therapeutics and Diet), reflects Ibn
Zuhr’s reliance on his own clinical observations, skill in differential diagnosis, and interest in
clinicopathologic correlations. Based on his own experience, he staged and classified diseas-
es in a practical way relevant to their management and prognosis. Furthermore, he enriched
surgical and medical knowledge by describing many diseases never described before, includ-
ing pericarditis, mediastinitis, mediastinal tumors, empyema, meningitis, intracranial throm-
bophlebitis, inflammation of the middle ear, pharyngeal and esophageal paralysis, verrucous
malignancy of the colon, fecal fistula, Peyronie’s disease, purpuric skin rash, and scabies.

Contributions
1.  Experimental surgery: Ibn Zuhr introduced animal testing as an experimental method of
testing surgical procedures before applying them to human patients to know if they would
work, performing the first experimental tracheotomy on a goat before performing it on
humans. He was the first surgeon of his time to apply experimental methodology in evalu-
ating new or controversial surgical procedures. Hence, he was given the title “The Father
of Experimental Surgery.”
2.  Clinical anatomical knowledge: Ibn Zuhr emphasized the great importance of a practical
knowledge of anatomy for the surgical trainee. Here is a translation of his words regarding
the management of inflammatory swellings of the neck that are ripe and ready for bursting/
drainage:
And in case you have mastered the science of dissection then drain by the scalpel in the way
that you will not come across a vein, artery or a nerve or anything that its injury will lead to an
extra harm to the patient. But if you were one of the group like me and did not practice dis-
section but knew it only by imitation, keep away from the knife as nothing you know by mere
imagination will be the same in real life; especially in the case of small organs.

 ccording to Ibn Zuhr, only the practitioner who has practiced dissection himself and mas-
A
tered the science is entitled to perform an operative intervention. He therefore advocated
that mastering anatomy is essential training for a surgeon.
3.  Adequate supervised training: Ibn Zuhr insisted on an adequately supervised and structured
training program for the surgeon-to-be before allowing him to operate independently.
4.  Established limits: Ibn Zuhr drew emphatic red lines at which a physician should stop during
his general management of a surgical condition. This was a major step forward in the evo-
lution of general surgery as a specialty of its own. Here is a translation of an example of Ibn
Zuhr’s demarcation:
If the wound caused by a sharp iron has taken into the bones and not extended to the interior,
then the treatment I just mentioned is enough for you, so stick to it. However, if it did penetrate
the bone then in such a case, the surgeon should come and see.

Legacy
Ibn Zuhr was the most well-regarded physician of his era, and his ideas about medicine and
surgery helped to shape our modern concept of standard care. He is an inspiration to those of
us who seek to make the best decisions for our patients and our discipline.

vii
1 MEDICAL EVALUATION OF THE IMPLANT PATIENT

PREFACE
The goal of this study guide is twofold. First, it aims to serve as a comprehensive review of
the topics and disciplines relevant to the field of oral implantology. The successful place-
ment of dental implants with good long-term functional and esthetic results involves much
more than just knowing the basic procedures and protocols involved in implant surgery. Of
course, it is important to know what size and type of implant to choose for various clinical
situations and how to drill safely into bone. But there are many other factors that influence
implant treatment planning, ranging from the patient’s systemic health, habits, and anatomy
to pharmacology, biomechanics, and prosthodontics. Placing dental implants also requires
an understanding of adjunctive methods, such as bone grafting and the use of blood concen-
trates and growth factors. This book synthesizes all the information a clinician must consider
at each stage of oral implant treatment in order to elevate the standard of patient care they
provide and round out the knowledge and skill set they bring to each dental implant case.
Second, this study guide serves as a tool for professional development, providing excel-
lent preparation for any oral implantology certification examination. With the knowledge
presented in this study guide, the dental professional can not only achieve certification but
also feel confident in their ability to provide the highest level of care when treating patients.
The reason for updating this study guide, first published in 2016, is also twofold. First, many
new techniques that were under development at the time the previous edition was published
have now become mainstream in dental implant treatment. As a result, three new chapters
have been added, covering bone grafting, zygomatic and pterygoid implants, and blood con-
centrates and growth factors. Knowledge of these topics is important both to provide the best
and most advanced patient care and to achieve certification in oral implantology.
Second, a greater emphasis has been placed on contextualizing dental implant treatment
within an understanding of the patient’s general health and its influence on outcomes. For
example, a systemic disease like diabetes mellitus has an enormous impact on oral implan-
tology. Clinicians must understand and consider the biologic effects of systemic health when
planning treatment in order to provide patient-centered care that achieves the highest level
of success possible.
As evidenced by the advances that have occurred in the 7 years since the publication of the
first edition of this guide, implant dentistry continues to evolve rapidly. Only by continuously
updating our knowledge base will we be able to keep pace with current trends. This book pro-
vides an overview of the discipline of oral implantology as it is practiced today. It is my hope
that it will not only prepare future implantologists to pass certification examinations but also
improve the patient care provided by all practicing implantologists.

viii
ACKNOWLEDGMENTS
To God, who made everything I have accomplished possible through his guidance
and gracious love.
To my family, thank you for your support and for giving up our personal time.
To my teammates at the California Implant Institute and Novadontics, you have shown
dedication to your jobs for years. You take so much off my plate so that I can have the
time to write, to lecture, and to innovate. Thank you.
Special thanks to Dr James Rutkowski, Dr Mamaly Reshad, and Dr Christopher Church
who unselfishly shared their knowledge and experience to contribute greatly to the
development of this book.
My deepest thanks to Bill Hartman and Leah Huffman and the entire staff at
Quintessence Publishing for their support and patience. They contributed tremendously
to the organization and the design of this book.
To my colleagues and students at the California Implant Institute, you and your patients
are the ultimate reason for this book. Elevating the standard of care for our patients and
giving the best for their well-being has always been the goal of everything I have done
since I started practicing dentistry.

ix
1 MEDICAL EVALUATION OF THE IMPLANT PATIENT

CONTRIBUTORS
Christopher Church, md
Professor
Department of Otolaryngology, Head and Neck Surgery
Loma Linda University School of Medicine
Loma Linda, California

Mamaly Reshad, dds, msc


Clinical Lecturer
Eastman Dental Institute
University College London
London, England
Former Chairman
Section of Fixed Prosthodontics and Operative Dentistry
University of Southern California
Los Angeles, California
Private Practice
Los Angeles, California

James L. Rutkowski, dmd, phd


Adjunct Clinical Instructor
Department of Restorative Dentistry
The State University of New York at Buffalo School of Dental Medicine
Buffalo, New York
Private Practice
Clarion, Pennsylvania

x
MEDICAL 1
EVALUATION OF
THE IMPLANT
PATIENT
An accurate and thorough medical evaluation is a critical
component of implant therapy. This chapter discusses the
many medical factors that must be considered when a
patient presents for treatment, including pathologic condi-
tions, bleeding risk, allergy, and medical contraindications.
Implant therapy is not without risk, and medical emergencies
can occur even when the proper precautions are followed;
it is therefore imperative that all clinicians understand what
to do in such situations, especially for individuals already
compromised by certain medical conditions.

1
1 MEDICAL EVALUATION OF THE IMPLANT PATIENT

1. What key medical considerations must the clinician take into account when formulating a
treatment plan for a dental implant patient?
a. Hemostasis
b. Drug actions and/or interactions
c. Predisposition to infection
d. All of the above

d: All of these could have a profound effect on the healing response and thereby compromise the
treatment result. If there is a hemostasis problem, excessive bleeding may result. Drug actions
may interfere with proper healing, and drug interactions may affect cardiovascular integrity. A
compromised immune system could lead to postoperative infections.

2. The risk of a dental practitioner encountering a medical emergency during placement of a


dental implant is related to:
a. The clinician’s medical training
b. The patient’s medical health
c. Staff training
d. Complexity of the procedure
e. All of the above

b: The patient’s systemic health will dictate how well he or she will be able to sustain the stress
of the procedure and the response to administered medications.

3. W
 hich of the following are essential components of a medical history? (MULTIPLE ANSWERS)
a. Medications
b. Previous hospitalizations, illnesses, and/or surgeries
c. Information regarding prosthetic joint replacements
d. Childhood immunizations
e. All of the above

 , b, c: A complete medical history should include an organ systems review, height, weight, exer-
a
cise tolerance, present illnesses, as well as any medications the patient is taking, any previous
hospitalizations or illnesses, and information regarding prosthetic joint replacements. The medical
history can be done as an interview of the patient or as a printed questionnaire that the clinician
reviews with the patient.

4. According to the ASA (American Society of Anesthesiologists) Physical Status (PS) classifica-
tion, what would the classification be for a patient who can walk up a flight of stairs or the
equivalent of two city blocks but has to stop along the way because of distress or shortness
of breath?
a. ASA I
b. ASA II
c. ASA III
d. ASA IV

c: ASA III is defined as a patient with severe systemic disease. A consultation with this patient’s
physician is recommended prior to initiating dental treatment for this individual. Perioperative
sedation and special monitoring may be necessary in the treatment of ASA III patients.

2
5. What would the ASA classification be for a patient who is able to walk up a flight of stairs or
the equivalent of two city blocks but has to rest at the end of the walk because of distress?
a. ASA I
b. ASA II
c. ASA III
d. ASA IV

b: ASA II is defined as a patient with mild systemic disease.

6. A healthy 38-year-old woman presents for a dental implant. She takes no medications and is
not anxious about the treatment. What is her ASA classification?
a. ASA I
b. ASA II
c. ASA III
d. ASA IV

a: ASA I is defined as a normal healthy patient.

7. What would the ASA classification be for a patient with well-controlled diabetes who is
insulin dependent?
a. ASA I
b. ASA II
c. ASA III
d. ASA IV

c: ASA III

8. What would the ASA classification be for a patient whose diabetes is well controlled with
diet and oral hypoglycemic agents?
a. ASA I
b. ASA II
c. ASA III
d. ASA IV

b: ASA II

9. What percentage of patients, when asked “Are you in good health?”, respond “yes” but are
actually found to be medically compromised on closer examination?
a. 10%
b. 20%
c. 30%
d. 40%

c: Studies reveal that 30% of patients who respond in the affirmative are actually deemed medi-
cally compromised by the treating clinician. (Source: Brady WF, Martinoff JT. Validity of health
history data collected from dental patients and patient perception of health status. J Am Dent
Assoc 1980;101:642–645.)

3
1 MEDICAL EVALUATION OF THE IMPLANT PATIENT

10. W
 hen a patient presents with a burning mouth or tongue, which of the following could be
the possible medical cause?
a. Alcoholism
b. Neoplasm
c. Renal failure
d. Primary or secondary neuropathy

d: Patients with primary or secondary neuropathy often present with the symptom of a burning
mouth or tongue.

11. When a patient presents with gingival overgrowth, which of the following could be a
possible medical cause?
a. Leukemia
b. Gastroesophageal reflux disease (GERD)
c. Immune suppression from HIV
d. Mouth breathing

a: Gingival overgrowth can be a sign of leukemia.

12. When a patient presents with rampant dental caries, which of the following could be a
possible medical cause?
a. Addison’s disease
b. Sjögren’s syndrome
c. Vitamin deficiency
d. Liver cirrhosis

b: Patients with Sjögren’s syndrome often present with a dry mouth that leads to rampant dental
caries. In elderly patients, it often presents as root caries.

13. When a patient presents with ptosis of the chin, which of the following could be a possible
medical cause?
a. Anemia
b. Use of skeletal muscle relaxants
c. Scleroderma
d. Myasthenia gravis

d: Myasthenia gravis is a neuromuscular disease that results in muscle fatigue and weakness.
Patients with myasthenia gravis will have decreased muscle tone that can result in ptosis.

14. W
 hen a patient presents with a radiographic finding of reduced cortical bone density, which
of the following could be a possible medical cause?
a. Primary hyperparathyroidism
b. Scleroderma
c. Osteoarthritis
d. Multiple myeloma

a: Hyperparathyroidism results in the secretion of excess parathyroid hormone, which stimulates
osteoclast catabolic effects on bone, resulting in the loss of calcium and density.

4
15. When a patient presents with a radiographic finding of degenerative damage to the condyle
or temporomandibular joint (TMJ), which of the following could be a possible medical cause?
a. Osteonecrosis
b. Paget disease
c. Hyperparathyroidism
d. Rheumatoid arthritis

 : Rheumatoid arthritis has an unknown etiology; however, genetic, environmental, hormonal, and
d
immunologic factors as well as infection are possibly involved in the process. A genetic suscep-
tibility may provoke an autoimmune reaction that leads to hypertrophy of the synovial lining of
the TMJ and endothelial cell activation that result in an uncontrolled inflammatory response and
destruction of the bone.

16. When a patient presents with a radiographic finding of carotid artery calcification, which of
the following could be a possible medical cause?
a. Cardiac disease
b. Sickle cell anemia
c. Hyperparathyroidism
d. Renal disease

a: Carotid artery calcium deposits have been identified as an independent predictor of coro-
nary heart disease events. Therefore, clinicians should be surveying panoramic radiographs and
computed tomography (CT) scans that are obtained for dental reasons for these calcium deposits
in the coronary artery.

17. When assessing the bleeding risk for a dental implant procedure, the clinician must consider
which of the following?
a. Inherited defects of hemostasis
b. Medications
c. Acquired defects of hemostasis
d. All of the above

d: Each of these factors can interfere with coagulopathy.

18. On review of the medical history, you find that the patient has severe Addison’s disease.
Why is severe adrenal insufficiency significant?
a. T he stress of an extensive dental implant surgical procedure may induce cardiovascular
collapse.
b. Soft tissue healing will be severely compromised.
c. Implants may not integrate.
d. The patient may experience a hypertensive crisis with the administration of more than 72 μg
of epinephrine within a 10-minute time period.

a: A patient with Addison’s disease will not be able to release the extra cortisol needed to deal
with the stress of the surgical procedure. Cortisol is a glucocorticosteroid that is responsible for
glucose metabolism as well as potentiation of catecholamines that assist in maintaining circula-
tory pressure.

5
1 MEDICAL EVALUATION OF THE IMPLANT PATIENT

19. What oral clinical finding may indicate that a patient has adrenal insufficiency?
a. Severe tooth erosion
b. Sloughing of the buccal mucosal tissues
c. Hyperpigmentation of the buccal or labial mucosal tissues
d. Gingival hyperplasia

c: Increased diffuse melanin pigmentation is a documented sign of Addison’s disease.

20. W
 hat is a medical reason for a patient to take long-term systemic glucocorticosteroids?
a. Liver, lung, or heart transplant recipient
b. Lupus erythematosus
c. Inflammatory bowel disease
d. All of the above

 : Long-term glucocorticosteroid therapy is indicated for each of these conditions. Dental clinicians
d
should consider increasing the patient’s normal daily steroid dose when the patient undergoes a
surgical or stressful dental procedure.

21. W
 hich of the following blood tests are generally thought to identify a patient with a possible
bleeding disorder? (MULTIPLE ANSWERS)
a. Complete blood count (CBC) and platelet count
b. Prothrombin time (PT) and partial thromboplastin time (PTT)
c. Lipoprotein panel
d. Bleeding time
e. White blood cell (WBC) count
f. All of the above

 , b, d: Each of these laboratory tests will act as a screening test for possible bleeding disorders.
a
The sum of these tests will measure platelet activity and coagulation factors.

22. C
 lassic hemophilia (type A) is a deficiency of which clotting factor?
a. Factor VII
b. Factor VIIa
c. Factor VIII
d. Factor VIIIa

c: Factor VIII

23. Type B hemophilia is a deficiency of which clotting factor?


a. Factor IX
b. Factor IXb
c. Factor X
d. Factor Xb

a: Factor IX

6
24. The dental implant patient who presents with chronic liver failure should have which of the
following hematology tests performed prior to the surgical procedure?
a. CBC, platelet count, PT
b. CBC, bleeding time, PTT
c. Platelet activation study (PAS), platelet count, WBC
d. PAS, bleeding time, PTT

a: Patients with chronic liver failure are likely to have problems with blood coagulation. The CBC,
platelet count, and PT will evaluate the coagulation factors that can be affected by the liver. The
CBC and platelet count will screen for anemia and thrombocytopenia, while the PT will confirm
a deficiency of vitamin K.

25. “Ageusia” refers to which of the following?


a. Diminished taste
b. Altered or distorted taste
c. Salty taste
d. Absence of taste

d: The tongue loses the ability to taste sweetness, sourness, bitterness, and saltiness. Complete
or true ageusia is rare, and what patients most often have is the partial loss of taste, known as
hypogeusia.

26. “Dysgeusia” refers to which of the following?


a. Diminished taste
b. Altered or distorted taste
c. Salty taste
d. Absence of taste

b: Altered or distorted taste

27. What are the most common reasons for alteration in taste? (MULTIPLE ANSWERS)
a. Autoimmune disease
b. Periodontal disease
c. Infection
d. Poor oral hygiene
e. GERD
f. All of the above

 , c, d: Periodontal disease, infection, and poor oral hygiene are known to alter the sensation
b
of taste.

7
1 MEDICAL EVALUATION OF THE IMPLANT PATIENT

28. “
 Tic douloureux” is also known as which of the following?
a. Idiopathic trigeminal neuralgia
b. Bell’s palsy
c. Facial paralysis
d. Trigeminal dysesthesia

a: Tic douloureux, or idiopathic trigeminal neuralgia, is a condition that creates episodes of
acute-onset, severe facial pain. It is most frequently found in patients of middle to old age. Intraoral
or facial trigger points initiate the pain, which can be excruciating but is usually not long lasting.
The trigeminal nerve’s mandibular branch is most often involved, but the etiology is unknown.

29. Which of the following endogenous pigmentation sources is the most common?
a. Melanin
b. Bilirubin
c. Iron
d. Heavy metals

a: Melanin is a term used to describe natural pigments in the body. It is produced by melanocytes
via the oxidation of tyrosine.

30. W
 hich of the following diseases can cause an abnormal melanin deposit in the oral mucosa?
a. Diabetes mellitus type 1
b. Acute myelogenous leukemia (AML)
c. Addison’s disease
d. Thrombocytopenia purpura

c: Patients with Addison’s disease frequently have bluish-black or dark-brown areas on the buccal
or labial mucosa and possibly on the gingiva.

31. Which of the following laboratory tests measures the intrinsic coagulation pathway?
a. PT
b. PTT
c. International normalized ratio (INR)
d. PAS

b: The PTT is a measure of the efficacy of the intrinsic pathway that mediates fibrin clot formation.
All coagulation factors are measured by this test except factor VII. Normal values are between 25
and 40 seconds. Values that are extended by 5 to 10 seconds represent a mild bleeding disorder;
values beyond 10 seconds may be an indicator of a clinically significant bleeding problem.

32. What is the recommended INR therapeutic range for standard oral anticoagulant therapy?
a. 1.0 to 2.0
b. 1.5 to 2.5
c. 2.0 to 3.0
d. 2.5 to 3.5

c : A value between 2.0 and 3.0 is the recommended therapeutic range for the prevention of deep
vein thrombosis, pulmonary embolism, hypercoagulable states, transient ischemic attack, atrial
fibrillation, dilated cardiomyopathy, rheumatic mitral valve disease, and stroke.

8
33. A patient who presents with a prosthetic heart valve replacement should have an
anticoagulant therapeutic range (INR) of:
a. 1.0 to 2.0
b. 1.5 to 2.5
c. 2.0 to 3.0
d. 2.5 to 3.5

d: 2.5 to 3.5

34. Known risk factors for ischemic heart disease (IHD) include which of the following?
a. Smoking
b. Obesity
c. Diabetes mellitus
d. All of the above

d: Each of these conditions can either decrease the oxygen supply to the heart or increase the
cardiac workload.

35. An implant patient presents with a recent history of myocardial infarction (MI). At what time
point post MI is the typical cardiac patient’s normal reinfarction risk level back to baseline?
a. 3 months
b. 4 months
c. 5 months
d. 6 months

d: The highest risk of reinfarction is between 0 and 3 months post MI. A lower risk is present
from 3 to 6 months post MI, with the risk returning to baseline after 6 months. Naturally, this is
somewhat patient dependent.

36. Stage 1 hypertension is associated with which of the following?


a. Systolic blood pressure 130–139 mm Hg, diastolic blood pressure 90–99 mm Hg
b. Systolic blood pressure 140–159 mm Hg, diastolic blood pressure 90–99 mm Hg
c. Systolic blood pressure 160–179 mm Hg, diastolic blood pressure 90–99 mm Hg
d. Systolic blood pressure 180–189 mm Hg, diastolic blood pressure 90–99 mm Hg

b: Systolic blood pressure 140–159 mm Hg, diastolic blood pressure 90–99 mm Hg

37. Stage 2 hypertension is associated with which of the following?


a. Systolic blood pressure 170–179 mm Hg, diastolic blood pressure ≥ 100 mm Hg
b. Systolic blood pressure 180–189 mm Hg, diastolic blood pressure ≥ 100 mm Hg
c. Systolic blood pressure ≥ 160 mm Hg, diastolic blood pressure ≥ 100 mm Hg
d. Systolic blood pressure ≥ 170 mm Hg, diastolic blood pressure ≥ 110 mm Hg

c: Systolic blood pressure ≥ 160 mm Hg, diastolic blood pressure ≥ 100 mm Hg

9
1 MEDICAL EVALUATION OF THE IMPLANT PATIENT

38. A
 bilateral submandibular, sublingual, and submental space infection is often referred to as
which of the following?
a. Ludwig’s triangle
b. Ludwig’s angina
c. Ludwig’s infection
d. Ludwig’s metastasis

 : Ludwig’s angina is a serious, frequently dentally induced cellulitis that can be life-threatening. If
b
left untreated or treated incorrectly, it can lead to airway obstruction.

39. A
 patient who presents with sudden-onset substernal chest pain that radiates to the left arm,
neck, jaw, and back may be experiencing which of the following?
a. Angina pectoris
b. Costochondritis
c. Herpes zoster
d. Esophageal spasm

a: The pain is due to myocardial ischemia. When it is limited to angina pectoris, there is no necro-
sis of the cardiac muscle. Angina is classified as stable, unstable, or Prinzmetal. Costochondritis,
herpes zoster, and esophageal spasm may present with similar symptoms, but they do not usually
involve the jaw.

40. Y
 our patient informs you that he has chest pain when he exercises, but the pain is always
gone in 15 minutes or less. Which of the following may he be experiencing?
a. Stable angina
b. Unstable angina
c. Prinzmetal angina
d. Kishimoto’s angina

a: Angina that lasts less than 15 minutes and only occurs on exercise or exertion is classified
as stable angina. Unstable angina may occur at any time for any reason and will last longer than
15 minutes, and the pain is more severe. Prinzmetal angina can occur when the patient is at rest,
and there are associated changes in the electrocardiogram; it is most likely due to a coronary
artery spasm.

41. In preparation for the placement of a dental implant at a previously grafted maxillary right
central incisor edentulous site, you administer 2 g of cefazolin intravenously. Within 3 min-
utes, the patient appears to be having difficulty breathing, which is worsening rapidly. This
reaction is an allergic response to the cefazolin antibiotic that was just administered. What
type of allergic reaction is this?
a. A type I immediate-onset reaction induced by an IgE-mediated activation of mast cells
and basophils
b. A type II immediate-onset reaction induced by an IgG-mediated activation of cell destruction
c. A type III immediate-onset reaction induced by an IgG-mediated activation of the immune
complex and complement system
d. A type IV immediate-onset reaction induced by T-cell activation, resulting in massive
histamine release

a: A type I reaction is the only immediate-onset reaction; all others are delayed in onset. Choice d
is also incorrect because T-cell activation leads to antibody production, activation of phagocytes,
and direct cell killing.

10
42. For an allergic reaction to be considered an immediate allergic response, it must occur within
how many minutes?
a. < 15 minutes
b. < 30 minutes
c. < 45 minutes
d. < 60 minutes

d: The World Allergy Organization defines an immunologic immediate drug reaction as one
occurring within 60 minutes of the administered dose. Delayed reactions are defined as those
occurring after 1 hour, although they usually occur at a time period greater than 6 hours and
can occur weeks or months after the start of drug administration. (Source: Lockhart PB [ed]. Oral
Medicine and Medically Complex Patients, ed 6. Hoboken, NJ: Wiley-Blackwell, 2013.)

43. Your patient presents with an infected dental implant and also reports that she currently has
infectious mononucleosis caused by the Epstein-Barr virus. With this current medical history,
which antibiotic should be avoided?
a. Azithromycin
b. Clindamycin
c. Amoxicillin
d. Metronidazole

c: Patients who present with the Epstein-Barr virus and are prescribed amoxicillin frequently
develop a bright red, generalized morbilliform rash. The other antibiotics are not known to do this.

44. Which local anesthetic is known to have the highest incidence of allergic reactions?
a. Articaine
b. Lidocaine
c. Procaine
d. Prilocaine

c: Procaine is an ester local anesthetic, whereas the others listed are amide local anesthetics.
Procaine is well documented as having the greatest incidence of allergic reactions due to its
inclusion of an ester moiety in its chemical structure. The amide local anesthetics have a less than
1% confirmed allergic reaction rate.

45. A 65-year-old man with extensive tooth decay desires to be restored with a fixed implant-
supported prosthesis. He reports that he lost his teeth because he was afraid of the dentist.
The last time he was treated (40 years earlier), he was given a local anesthetic and became
faint, nauseated, and short of breath, his heart raced, and he had to be transported to the hos-
pital for emergency care. The clinician in attendance at that time has since retired, and the pa-
tient records have been destroyed. Which of the following is the most appropriate action to take?
a. Provide intravenous conscious sedation for the patient.
b. Review the medical history for possible drug allergies.
c. Refer the patient for allergy testing and/or desensitization before administering any local
anesthetics.
d. Perform the procedure using articaine as the local anesthetic because it is unlikely that he was
given this agent 40 years ago.

c: The reaction the patient had 40 years ago seems to have been a severe type I allergic reaction.
Therefore, the most appropriate next step is to refer the patient for allergy testing.

11
1 MEDICAL EVALUATION OF THE IMPLANT PATIENT

46. Y
 our implant patient provides you with a history of extensive bleeding following a
previous extraction 18 months earlier. Which of the following would be appropriate
as an initial screening test?
a. Platelet count
b. PT/INR
c. Activated PTT (aPTT)
d. All of the above

d: These three tests will provide the broadest survey of the clotting mechanisms. The platelet
count measures the platelet concentration in whole blood. The aPTT assesses the intrinsic coag-
ulation pathway and the final common pathway; it also monitors heparin therapy. The PT/INR
assesses the extrinsic pathway of clotting. The PT and aPTT are also used to assess hemophilia
A and hemophilia B.

47. If your patient presents with severe hemophilia A or B, why will the management of
postoperative bleeding episodes be difficult?
a. The patient may have developed IgG antibodies against his deficient factor.
b. The deficient factors may not be available to induce coagulation.
c. The administration of postoperative antibiotics would most likely prevent coagulation.
d. The administration of postoperative narcotics would most likely prevent coagulation.

a: About 25% of patients with severe hemophilia A and 3% to 5% of patients with severe hemo-
philia B develop antibodies (primarily IgG) against their deficient factor. Antibodies are less likely
in those with mild or moderate hemophilia A or B.

48. W
 hich of the following medications is contraindicated in patients with hemophilia A?
a. Metronidazole
b. Ibuprofen
c. Oxycodone
d. Dexamethasone

b: Ibuprofen is a nonsteroidal anti-inflammatory drug (NSAID). All NSAIDs may interfere with
clotting mechanisms, so postoperative pain should be managed with either acetaminophen or
a narcotic.

49. Y
 our patient provides you with a history of immediate severe postoperative bleeding follow-
ing previous tooth extractions. This is most likely due to what bleeding disorder?
a. Thrombocytopenia or platelet dysfunction
b. Hemophilia A
c. Hemophilia B
d. Von Willebrand disease

a: Coagulation disorders usually induce delayed bleeding several hours or days after surgery,
whereas patients presenting with thrombocytopenia or platelet dysfunction experience immediate
bleeding after vascular injury.

12
50. What is the incidence of bacteremia following a tooth extraction?
a. 20%
b. 40%
c. 80%
d. 90%

d: 90%

51. Infective endocarditis is an infection of the:


a. Pericardium outer lining and mural endocardium
b. Pericardium inner lining and septal wall
c. Endocardium and heart valves
d. Blood vessels supplying the myocardium

c: Infective endocarditis is an infection/inflammation of the endocardium (inner surface of the


heart) and may include one or more heart valves, the mural endocardium, or a septal defect.

52. Prosthetic heart valves may be made of which of the following?


a. Synthetic (carbon alloys) or biologic (porcine in origin)
b. Synthetic (titanium alloy) or biologic (bovine in origin)
c. Recombinant porcine in origin
d. Recombinant bovine in origin

a: Synthetic (carbon alloys) or biologic (porcine in origin)

53. Which of the following cardiac conditions DOES NOT require antibiotic prophylaxis prior to
surgical implant treatment?
a. History of infective endocarditis
b. History of heart transplant with a new valvular lesion
c. Presence of a prosthetic synthetic heart valve
d. History of rheumatic fever

d: According to the American Heart Association (AHA) 2007 recommendations, a history of infec-
tive endocarditis, a history of heart transplant with a new valvular lesion, and the presence of a
prosthetic synthetic heart valve all require antibiotic prophylaxis prior to dental treatment. Given
these histories of cardiac issues, these patients are at a higher risk of infective endocarditis with a
potentially unfavorable outcome. A history of rheumatic fever does not warrant antibiotic prophy-
laxis prior to dental treatment. (Source: American Heart Association. Infective Endocarditis. http://
www.heart.org/HEARTORG/Conditions/CongenitalHeartDefects/TheImpactofCongenitalHeart-
Defects/Infective-Endocarditis_UCM_307108_Article.jsp#.Vxjs7atX--I. Accessed 21 April 2016.)

13
1 MEDICAL EVALUATION OF THE IMPLANT PATIENT

54. P
 eer-reviewed literature supports making no adjustments to the warfarin therapy dosage
prior to a tooth extraction or other invasive dental procedure provided the INR is equal to or
below which of the following values?
a. 2.5
b. 3.0
c. 3.5
d. 4.0

c: The risk for stroke or thrombosis by discontinuing warfarin therapy when the INR is equal to
or below 3.5 outweighs any risk of prolonged bleeding from a tooth extraction or other invasive
dental procedure. (Sources: Lockhart PB [ed]. Oral Medicine and Medically Complex Patients,
ed 6. Hoboken, NJ: Wiley-Blackwell, 2013. / Aframian DJ, Lalla RV, Peterson DE. Management of
dental patients taking common hemostasis-altering medications. Oral Surg Oral Med Oral Pathol
Oral Radiol Endod 2007;103[suppl 45]:1–11.)

55. T
 he dental implant patient presents with a medical history of having an implantable cardio-
verter defibrillator (pacemaker) placed 18 months previously. Due to the history of an active
pacemaker, which of the following does the AHA recommend regarding dental procedures?
a. Prescribe preoperative prophylaxis with appropriate antibiotics.
b. Avoid the use of ultrasonic scalers and electrosurgical units.
c. Avoid the use of an electronic automatic blood pressure cuff.
d. Avoid the use of electrocardiography during dental surgery.

 : The AHA does not recommend antibiotic prophylaxis for patients with nonvalvular cardiovascular
b
devices prior to dental procedures because of a lack of evidence supporting an increased risk of
device infection. However, the use of ultrasonic scalers, electric pulp testers, and electrosurgical
units should be avoided because of possible interference with the pacemaker’s function.

56. A
 patient presents with a history of having a coronary artery bypass graft (CABG) placed
without concurrent myocardial infarction. In consultation with the physician, the patient
should be cleared for dental treatment after how long?
a. 4 weeks
b. 6 weeks
c. 12 weeks
d. 6 months

b: Although it is conventionally thought that the same 6-month delay following CABG with MI
should be observed before resuming dental treatment, there is not sufficient research to support
the need to wait longer than 6 weeks.

57. F
 or patients presenting with a history of angina, it is reasonable to perform elective dental
treatment provided:
a. The angina is variant or Prinzmetal in type.
b. The angina is stable and well controlled by one or two nitroglycerin tablets, with at least
7 days between episodes.
c. The patient has only mild substernal pain with no radiation to the left arm.
d. The patient’s resting heart rate is below 80 bpm, and the resting systolic blood pressure is
below 90 mm Hg.

b: The angina is stable and well controlled by one or two nitroglycerin tablets, with at least
7 days between episodes.

14
58. The concerns about the need to prevent bacterial seeding of a prosthetic joint from
staphylococcal bacteria originating from the oral cavity are:
a. Supported by the literature; therefore, prophylactic antibiotics prior to dental treatment
should be administered in the first 6 months following placement of the prosthetic joint.
b. Supported by the literature; therefore, prophylactic antibiotics prior to dental treatment
should be administered in the first 2 years following placement of the prosthetic joint.
c. Supported by the literature; therefore, prophylactic antibiotics prior to dental treatment
should be administered for the lifetime of the prosthesis following placement of the
prosthetic joint.
d. Not supported by the literature; therefore, prophylactic antibiotics prior to dental treatment
are not necessary.

d: The risk of a dental procedure inducing prosthetic joint infections is extremely low. Prosthetic
joint infections are almost always caused by Staphylococcus originating from sources other than
the oral cavity.

59. Hyperadrenocorticism is also known as:


a. Addison’s disease
b. Beckert’s disease
c. Cushing’s disease
d. Kennedy’s disease

c: Hyperadrenocorticism, or Cushing’s disease, is a combination of signs and symptoms caused


by hyperactivity of the pituitary or adrenal glands that induces hypersecretion of cortisol from the
adrenal gland or from long-term exogenous glucocorticosteroid therapy.

60. Hyperadrenocorticism symptoms include which of the following? (MULTIPLE ANSWERS)


a. Bleeding diathesis
b. Weight loss
c. Overactive bladder
d. Muscle weakness
e. Diabetes
f. All of the above

 , d, e: Bleeding diathesis occurs because microcirculation is affected and cannot respond appro-
a
priately in aiding the clotting process. Cortisol induces gluconeogenesis and the breakdown of
glycogen, thereby increasing blood glucose levels and the incidence of diabetes. The etiology
of muscle weakness is poorly understood but is associated with the disease.

61. What is the most likely cause of a chronic red lesion found on the denture-bearing palatal
mucosa of a patient who wears a maxillary removable dental prosthesis?
a. Staphylococcus aureus
b. Candidiasis (chronic atrophic type)
c. Erythroplakia (also known as speckled leukoplakia)
d. Extravasated blood

b: The patient may be wearing the denture continuously, except for brief periods of time to clean
it. Candida may be present in the tissues as a result of a Candida-positive denture base. Treat-
ment consists of antifungal agent application to both the mucosa and the base of the prosthesis.

15
1 MEDICAL EVALUATION OF THE IMPLANT PATIENT

62. A
 ngular cheilitis is found at the commissures of the lip and appears as fissures and scales
within the folds of the tissue. What is the cause of this condition?
a. Nutritional vitamin deficiency
b. Immunosuppressive drugs
c. Candida albicans or Staphylococcus aureus
d. All of the above

d: All of these are known to cause angular cheilitis. Loss of vertical dimension and continued
wetness of the area are additional known causes.

63. W
 hich of the following statements is true regarding hypothyroidism and dental treatment?
a. P atients with exceedingly low levels of thyroxine are not candidates for usual dental
treatments because they may be experiencing heart failure.
b. The administration of epinephrine may precipitate a hypertensive crisis in patients with
hypothyroidism.
c. Patients with hypothyroidism may experience tachycardia with the administration of
epinephrine.
d. Patients with hypothyroidism may experience a “thyroid storm” if administered epinephrine.

a: Patients with exceedingly low levels of thyroxine may be suffering heart failure and therefore
should not be considered for treatment without a consultation with their physician. Choices b, c,
and d apply to hyperthyroidism.

64. W
 hich of the following medications may progress a hyperthyroid state to a thyroid crisis?
a. Azithromycin
b. Bupivacaine
c. Epinephrine
d. Oxycodone

c: Being a sympathomimetic drug, epinephrine may precipitate the progression from hyperthy-
roidism to a thyroid crisis. The other medications would not.

65. A
 thyroid crisis consists of which of the following?
a. Severe tachycardia
b. Mental disorientation
c. Presence of a precipitating factor such as an ongoing surgical procedure,
diabetic ketoacidosis, or administration of a sympathomimetic agent
d. All of the above

 : A thyroid crisis consists of four items: hyperthermia, severe tachycardia, mental disorientation,
d
and the presence of a precipitating factor.

16
66. What is the proper management of a patient undergoing a suspected thyroid crisis?
a. P
 lace the patient in the Trendelenburg position and administer oxygen; the crisis should
pass within 10 minutes.
b. Activate emergency medical services (EMS) and administer acetaminophen.
c. Massage the carotid artery.
d. Administer a benzodiazepine.

b: When a patient demonstrates the four signs of a thyroid crisis, EMS should be immediately
activated. Acetaminophen administration will help decrease the body temperature. The other
choices listed will be of no benefit.

67. Which of the following is an EARLY sign of local anesthesia overdose?


a. Lethargy
b. Decreased systolic blood pressure
c. Decreased muscle tone
d. Muscle twitching

d: The earliest signs of local anesthesia toxicity are excitatory in nature. These include talkative-
ness, slurred speech, tinnitus, tongue numbness, disorientation, muscle twitching, or tonic-clonic
seizures.

68. What is the most common cause of local anesthetic overdose?


a. The continual administration of local anesthetic injections to patients who are complaining of
pain during the dental procedure
b. The administration of local anesthesia to patients who are taking systemic sodium channel
blockers such as gabapentin
c. The administration of local anesthetics at 4% concentrations as opposed to 2% concentrations
d. Injecting local anesthetic too rapidly

a: The administration of multiple local anesthetic carpules to a patient who complains of being
numb but continues to feel pain is the primary reason local anesthetic overdoses occur. Clinicians
must be aware of the concentration of the local anesthesia in the dental anesthesia carpule, the
total amount being administered, the weight of the patient, and the importance of aspiration
prior to injecting for the prevention of an intravascular injection.

69. What are the best ways to prevent local anesthetic overdose? (MULTIPLE ANSWERS)
a. Only perform one injection of anesthetic prior to the procedure.
b. Be aware of the anesthetic preparation concentration and the volume of each carpule.
c. Aspirate on injection and incremental injections.
d. Know the toxicity of the local anesthetic in milligrams and the number of carpules that rep-
resents for the patient’s particular weight.
e. All of the above

b, c, d: All of these precautions will help prevent a local anesthetic overdose.

17
1 MEDICAL EVALUATION OF THE IMPLANT PATIENT

70. Which of the following is a LATE sign of local anesthesia toxicity?


a. Bradycardia and hypotension
b. Tachycardia and hypertension
c. Perfuse sweating
d. Seizures

a: Bradycardia and hypotension are late signs of local anesthesia toxicity because cardiovascular
system depression is a late-stage event in the sequence of toxicity.

71. W
 hen a patient is suffering from local anesthetia toxicity, which of the following body
systems are affected?
a. Respiratory and central nervous systems
b. Renal and cardiovascular systems
c. Central nervous and cardiovascular systems
d. Respiratory and renal systems

c: The body systems affected by local anesthesia toxicity are the central nervous system and the
cardiovascular system. The initial responses are excitatory, followed by depression for both systems.

72. W
 hat is a clinically significant reason for administering a vasoconstrictor with a local
anesthetic injection? (MULTIPLE ANSWERS)
a. T o maintain the local anesthesia at the site and block the vasodilation that occurs when local
anesthesia is administered
b. To maintain the local anesthesia at the site and decrease the blood level of the local anesthesia
c. To prevent the progressive diffusion of the local anesthesia to other areas
d. To hasten the onset of the local anesthesia

 , b, c: Epinephrine is a vasoconstrictor. The purpose of adding the epinephrine to the local


a
anesthesia carpule is to induce localized vasoconstriction by activating α-receptors on the blood
vessels at the site of administration. By creating an environment where there is localized vaso-
constriction, the local anesthetic will remain at the site, where it is less likely to be picked up by
the blood supply and carried to the central nervous system or heart. Epinephrine therefore has
an effect on the depth, duration, and overall systemic uptake of the local anesthetic.

73. If you are performing an inferior alveolar nerve block and the needle breaks at the hub,
which of the following should you do?
a. Have the patient sit up; use a hemostat to remove the broken needle piece.
b. Ask the patient to keep their mouth open; insert a bite block to prevent closure of the mouth,
and then use a hemostat to remove the broken needle piece.
c. Ask the patient to tilt their head toward the side with the broken needle; close the jaws,
retract the cheek, and use a college pliers to remove the broken needle piece.
d. Ask the patient to tilt their head upwards and towards the clinician; then use a college pliers
to remove the broken needle piece.

 : The patient’s mouth should be held open with a bite block or other device, and then a hemostat
b
or college pliers should be used to remove the broken piece. If the broken needle is not visible,
then the patient should be immediately referred to a maxillofacial surgeon.

18
74. Addison’s disease is also known as:
a. Primary adrenal insufficiency
b. Secondary adrenal insufficiency
c. Tertiary adrenal insufficiency
d. Hashimoto’s adrenal insufficiency

a: Primary adrenal insufficiency can be caused by conditions that lead to the destruction of the
adrenal cortex. Autoimmune idiopathic adrenal insufficiency is the most common subtype.

75. What is a primary function of cortisol in the body?


a. It increases the level of glucose in the plasma by controlling gluconeogenesis.
b. It increases the level of glucose in the plasma by stimulating the release of glucose from
the liver.
c. It is anti-inflammatory by inhibiting prostaglandin production, lysosome release,
and leukocyte function.
d. All of the above

d: It is essential for maintaining homeostasis and is released daily in the body through a circadian
rhythm. Stress or pathology can increase the release of cortisol.

76. Which of the following are prudent preparation steps for effectively managing medical
emergencies in the dental office? (MULTIPLE ANSWERS)
a. A ll staff members have successfully completed a basic life support course and participate in
regular practice emergency drills.
b. All staff members have been trained in advanced cardiac life support.
c. A list of emergency phone numbers is posted in a conspicuous, easily accessible location.
d. There is an appropriately stocked, routinely maintained office emergency drug kit. All office
employees know its location and can retrieve it quickly.
e. One of the staff members is a licensed paramedic.
f. All of the above

 , c, d: If the dentist is the only one who knows basic life support and the location of the emer-
a
gency phone numbers and kit, patients may not receive the care they require in the event of a
medical emergency.

77. Within how many minutes of ventricular fibrillation (VF) should automated external
defibrillator (AED) shock delivery take place?
a. 2 minutes
b. 3 minutes
c. 4 minutes
d. 5 minutes

b: The time from VF to shock delivery should be less than 3 minutes.

19
1 MEDICAL EVALUATION OF THE IMPLANT PATIENT

78. O
 nce VF has occurred and EMS has been activated, the best thing to do until the AED is
available is:
a. Apply a nasal cannula and administer oxygen at a rate of 4 liters per minute.
b. Determine the patient’s level of consciousness.
c. Perform CPR.
d. Administer 30% nitrous oxide via a nasal hood.

c: The dental team should perform CPR once the patient has entered into VF. When VF has
occurred, the myocardium quickly becomes depleted of oxygen. Chest compressions will deliver
oxygen to the heart, thereby increasing the likelihood that a shock from an AED will convert an
existing VF to a sinus rhythm.

79. O
 nce the dental team has (1) determined that the patient is unresponsive and not breathing
(or having difficulty breathing) and (2) activated EMS, which of the following activities should
be performed?
a. Perform 30 chest compressions at a rate of at least 100 per minute.
b. Administer two quick breaths, then begin chest compressions.
c. Place an oral airway, give two quick breaths, and then begin chest compressions.
d. Perform 15 chest compressions at a rate of 60 per minute.

a: The adult sternum should be depressed a minimum of 2 inches, and the rescuer’s arms should
be kept straight during each compression. The rescuer should always allow for complete chest
recoil after each chest compression, but interruptions should be minimized.

80. F
 or an unconscious, nonbreathing patient with no pulse, what should happen after the first
set of chest compressions?
a. T he rescuer gives two long, deep breaths to the victim, which will allow for the patency of the
airway to be checked.
b. The victim’s airway is opened, and the rescuer gives two quick breaths to the victim. The
breaths are given in approximately 1 second.
c. A non-rebreathing face mask is placed on the patient, and oxygen is administered at a rate of
15 liters per minute.
d. A nasal cannula is placed on the patient, and oxygen is administered at a rate of 6 liters per
minute.

b: Excessive ventilation must be avoided. The compression-to-ventilation ratio should be


30:2 for a single rescuer of adults, children, and infant victims.

81. C
 ushing’s syndrome is most often caused by which of the following?
a. Congenital adrenal hyperplasia
b. Hypertrophy of the hypothalamus
c. Prolonged administration of exogenous glucocorticosteroids
d. Hypertrophy of the pituitary gland

c: Cushing’s syndrome, not Cushing’s disease, is a syndrome that presents itself with signs and
symptoms that mimic Cushing’s disease. The prolonged administration of glucocorticosteroids
can induce Cushing’s syndrome.

20
82. Patients taking systemic glucocorticoids for a chronic disease are at risk of acquiring
glucocorticoid-induced osteoporosis. What patient group has the highest risk?
a. Children and postmenopausal women
b. Men over 70 years and postmenopausal women
c. Premenopausal women who are on a daily dose of prednisone 5 mg
d. Patients with chronic hypocalcemia

a: This type of osteoporosis involves the most metabolic site of the bone—the trabeculae.The
steroids disrupt the regulation of calcium metabolism at multiple levels, resulting in a net decrease
in calcium absorption. The parathyroid gland responds to the hypocalcemia by secreting more
parathyroid hormone, which catabolizes the bone to release calcium for the resolution of the
serum hypocalcemia.

83. What are the side effects of long-term glucocorticoid administration? (MULTIPLE ANSWERS)
a. Hyperglycemia and diabetes mellitus
b. Weight loss
c. Osteoporosis
d. Arthritis
e. Peptic ulcers
f. All of the above

 , c, e: Glucocorticosteroids increase gluconeogenesis and glucose secretion by the liver, thereby


a
inducing hyperglycemia and the risk of diabetes mellitus. They also interfere with calcium absorp-
tion, inhibit osteoblasts, and stimulate osteoclast formation, which results in bone resorption. It has
been hypothesized that glucocorticoids cause peptic ulcers by increasing gastric acid secretion and
inhibiting the synthesis of mucopolysaccharides that protect the gastric mucosa from acid.

84. There is an uncommon autoimmune disease that begins as a small flaccid bulla with the
history of being a single lesion for a prolonged time. Over time, several lesions can develop.
The history often includes skin lesions or blisters, and 80% to 90% of patients with this dis-
ease have oral lesions. These bullae can affect all parts of the mouth and will often rupture,
leaving behind oral ulcers; however, skin lesions will remain intact for prolonged periods of
time on the skin. Blister formation induced by gentle rubbing of the affected mucosal site
(Nikolsky sign) is one of the classic signs of the disease. The bullae are intraepithelial and
therefore often filled with a clear liquid. Autoantibodies against antigens are found within
the desmosomes of the epithelium. What is the name of this disease?
a. Mucous membrane pemphigoid
b. Pemphigus
c. Oral erythema multiforme
d. Primary herpetic gingivostomatitis

b: Unlike pemphigus lesions, mucous membrane pemphigoid lesions are usually limited to the
oral cavity. These lesions are below the epithelium, so the bullae are usually blood filled. The
pemphigoid lesions appear as red, nonulcerated gingival lesions. Patients with mucous membrane
pemphigoid may present with ocular lesions as well.
Oral erythema multiforme is also an autoimmune condition and may occur at any age. The
inciting agent may be a drug (such as penicillin or sulfonamides) or an immediately preceding
herpetic infection. Signs of oral erythema multiforme include blood-crusted lips, bullseye skin
lesions, and nonspecific sloughing of the mucosal tissues. Stevens-Johnson syndrome is an extreme
form of erythema multiforme.
Primary herpetic gingivostomatitis is a transmissible infection of the herpes simplex virus. Vesicles
containing a clear or yellowish liquid form intraorally and extraorally. The lesions rupture within
hours and form shallow ulcers that are painful. The gingiva may be red, enlarged, and painful. The
patient may present with regional lymphadenitis, fever, and malaise. The disease is self-limiting
and lasts for 7 to 10 days.

21
1 MEDICAL EVALUATION OF THE IMPLANT PATIENT

85. A
 patient presents with a shallow painful ulcer in the mouth. He reports that the ulcer was
initially a vesicle that ruptured and discharged a clear liquid. The patient has had the ulcer
as well as regional lymphadenitis, fever, and malaise for about a week. This disease is most
likely which of the following?
a. Mucous membrane pemphigoid
b. Pemphigus
c. Oral erythema multiforme
d. Primary herpetic gingivostomatitis

d: Primary herpetic gingivostomatitis (see answer to question #84)

86. A
 patient presents with atrophic, dry, pale mucosal tissues. There is a history of acute or
chronic reduced salivary flow. The tongue is devoid of papillae and is atrophic, fissured,
and inflamed. There are multiple root caries lesions despite the patient reporting many
years of having had “no cavities.” The patient is most likely suffering from which of the
following conditions?
a. Geographic tongue
b. Benign migratory glossitis
c. Xerostomia
d. Erythema migrans

c: Xerostomia

87. P
 ilocarpine (Salagen, Pfizer) is a cholinergic agonist (parasympathomimetic) that stimulates
the flow of saliva and is used in patients suffering from xerostomia. Because it is a cholinergic
agonist, which of the following side effects can it cause?
a. Excessive tearing
b. Flushing
c. Diarrhea
d. All of the above

d: As a cholinergic agonist, pilocarpine stimulates acetylcholine receptors, thereby inducing all
of these side effects.

88. L
 ichen planus has been hypothesized to be a chronic mucocutaneous autoimmune disorder
that may have a genetic origin. Which of the following are known initiators of the disease?
a. Food, dental products, emotional stress, and hypersensitivity to drugs
b. Age, diet, acute topical exposure to irritants, and emotional stress
c. Bicarbonate, acid reflux, emotional stress, and clenching
d. Bruxing, acrylic resin, anticholinergic drugs, and fungal infections

a: Lichen planus can present in various forms. It may appear as Wickham striae (white lines), atro-
phic (erythematous form), or ulcerative (ulceration appearance). The buccal mucosa, gingiva, and
tongue are the most common locations for lesions. A biopsy is indicated for refractory lesions to
rule out malignancy.

22
89. Treatment for lichen planus may include the use of topical steroids. How long can topical
steroids be used before risking secondary fungal infection?
a. 10 days
b. 14 days
c. 16 days
d. 21 days

 : The topical use of glucocorticosteroids for longer than 2 weeks has been implicated in mucosal
b
atrophy and secondary candidiasis. The addition of an antifungal agent may be necessary. Once
the lichen planus is controlled, the topical steroid should be tapered to alternate-day therapy.

90. Immunosuppressive therapy is key to organ transplant success in the 3 months following sur-
gery. Transplant patients often take cyclosporine, prednisone, and other immunosuppressive
agents during this critical time period. Dental treatment considerations include which of the
following?
a. Infection (only emergency dental care is provided)
b. Possible drug interactions between drugs used in the treatment of a dental emergency and
the immunosuppressive therapy the patient is receiving
c. Possible fungal infections due to appliance wear or antibiotics prescribed for the emergency
dental treatment
d. All of the above

d: Immunosuppressive therapy will make patients more susceptible to infections. These immuno-
suppressive agents can interact with medications that the dental emergency may require (eryth-
romycin, ketoconazole, carbamazepine, and phenytoin). Dental appliance care and oral hygiene
must be maintained to prevent possible oral complications.

91. Patients with iron deficiency anemia may experience which of the following oral conditions?
(MULTIPLE ANSWERS)
a. Paresthesias
b. Infection and prolonged bleeding
c. Loss of papillae on the dorsum of the tongue
d. Increased incidence of oral and pharyngeal carcinoma (Plummer-Vinson syndrome)
e. Gingivostomatitis
f. All of the above

 , b, c, d: Iron deficiency anemia (a microcytic anemia) can be induced by excessive blood loss,
a
poor iron intake, poor iron absorption, or an increased demand for iron.

92. A patient who presents with oral paresthesias (such as burning, tingling, and numbness),
delayed healing, infection, a bald red tongue, angular cheilitis, and petechial hemorrhages
may have which of the following conditions?
a. Sickle cell anemia
b. Glucose-6-phosphate dehydrogenase (G6PD) deficiency
c. Pernicious anemia
d. Iron deficiency anemia

c: G6PD traditionally does not present with oral symptoms. Sickle cell anemia and iron deficiency
anemia present with symptoms discussed in other questions.

23
1 MEDICAL EVALUATION OF THE IMPLANT PATIENT

93. A
 patient who presents with loss of the bone’s trabecular pattern on a panoramic radio-
graph, delayed eruption of teeth, growth abnormalities, hypoplasia of teeth, pallor or jaun-
dice of oral mucosa, bone pain, and osteoporosis may have which condition?
a. Acute renal failure
b. Sickle cell anemia
c. Hashimoto’s disease
d. Hemophilia

b: Patients presenting with acute renal failure may demonstrate gingival bleeding and lichenoid
eruptions. Hashimoto’s disease can present with aggressive periodontal disease, extensive dental
caries, and premature loss of primary teeth followed by the early eruption of permanent teeth.
Hemophilia would present as excessive bleeding following dental surgery or treatment.

94. A
 patient who presents with excessive bleeding after scaling and surgical procedures,
petechiae, ecchymosis, and hematomas may have which of the following conditions?
a. Vascular wall alterations such as scurvy (vitamin C deficiency) or infection
b. Thrombocytopenia induced by chemicals, radiation, or leukemia
c. Hodgkin disease
d. Non-Hodgkin lymphoma

a: No laboratory screening tests are reliable in detecting patients with this disorder. The clinician
has to rely on the medical history, physician consultation, and clinical findings to identify these
patients. Vascular wall alterations can be the result of autoimmune diseases, infections, structural
malformation of vessels, hereditary disorders involving connective tissue, scurvy, steroid use,
small-vessel vasculitis, or paraprotein deposits.
Patients with thrombocytopenia may present with all of the same signs and symptoms, but they
tend to experience spontaneous bleeding as well. Patients who present with Hodgkin disease
or Non-Hodgkin lymphoma may have secondary petechiae or ecchymosis if thrombocytopenia
is present due to tumor invasion of the bone marrow. They will also present with extranodal oral
tumors, xerostomia secondary to radiation treatment, burning mouth or tongue symptoms, and
cervical lymphadenopathy.

95. W
 arfarin (Coumadin, Bristol-Myers Squibb) is an anticoagulant whose mechanism of action
involves which of the following pathways?
a. Irreversibly inhibits platelet cyclooxygenase
b. Indirectly inhibits thrombin
c. Inhibits the fibrinogen receptor glycoprotein IIb/IIIa
d. Inhibits the biosynthesis of vitamin K–dependent coagulation proteins (factors VII, IX, and X
and prothrombin)

d: Warfarin’s mechanism of action is to prevent the reduction of vitamin K. Reduced vitamin K is
necessary for the formation of coagulation protein factors VII, IX, and X and prothrombin.
Aspirin’s mechanism of action is based on irreversibly inhibiting platelet cyclooxygenase, prevent-
ing the synthesis of thromboxane A2 and interfering with platelet secretion and aggregation.
NSAIDs’ mechanism of action is based on the reversible inhibition of cyclooxygenase. Heparin
and low–molecular weight heparin act by indirectly inhibiting thrombin. Direct thrombin inhibi-
tors are lepirudin, desirudin, argatroban, and bivalirudin. Ticlopidine and clopidogrel inhibit the
fibrinogen receptor glycoprotein IIb/IIIa.

24
96. Factor V Leiden thrombophilia:
a. Leads to severe postsurgical bleeding.
b. Increases the patient’s chance of developing abnormal blood clots in veins (deep vein thrombosis).
c. Causes irreversible renal damage.
d. Results in interference with cytochrome P450 enzymes that are responsible for thrombin formation.

b: Most people who have factor V Leiden never develop signs or symptoms. An early indicator
of this disorder is the development of a blood clot (thrombosis). Clots can be of no consequence
and disappear on their own or may be life threatening.

97. A patient who presents with erosion of the lingual surfaces of the maxillary teeth may be
suspected of which of the following eating disorders?
a. Anorexia nervosa
b. Bulimia nervosa
c. Binge eating
d. All of the above

b: These patients may also experience tooth sensitivity to thermal changes. Usually there are no
oral changes with anorexia nervosa or binge eating.

98. Patients who are methamphetamine abusers often appear with “meth mouth.” Signs of
“meth mouth” include which of the following?
a. Multiple tooth fractures and periodontal disease
b. Thermal burning of lips and oral mucosa
c. Xerostomia, rampant caries, and enamel erosion
d. All of the above

c: Signs of “meth mouth” include xerostomia, rampant caries, enamel erosion, and periodontal
disease. These patients may also experience a chronic bad taste, bruxism, and jaw clenching
that results in muscle trismus. Multiple tooth fractures, on the other hand, would be a sign of ice
chewing, physical abuse, or falls (in older adults). Thermal burning of the lips and oral mucosa
may be a somatoform or factitious disorder.

99. Which of the following statements are true about stress-reduction protocols for patients who
demonstrate anxiety regarding dental treatment? (MULTIPLE ANSWERS)
a. Do not discuss the patient’s fears—the less they know, the better.
b. Use longer appointments—get as much treatment done as you can in a single setting and get
it over with.
c. Short morning appointments are often best.
d. Openly discuss the patient’s fears.

c , d: For patients who demonstrate anxiety regarding dental treatment, it is best to openly discuss
their fears; use short appointments in the early morning; consider preoperative and intraoperative
sedation, substantial local anesthesia, and prevention of postoperative pain; and communicate
with the patient on the eve of the treatment.

25
1 MEDICAL EVALUATION OF THE IMPLANT PATIENT

100. A
 heart murmur can best be described as which of the following?
a. A turbulence of blood flow that produces a vibratory sound during the beating of the heart
b. Defective closing of the heart valves
c. Inflammation of the pericardium
d. Slow electrical conduction at the atrioventricular node resulting in “overlapping” heartbeats

 : This turbulence can be produced from normal physiologic factors or pathologic abnormalities of
a
the vessels, heart valves, or both. A heart murmur may be an indication of underlying heart disease.

101. W
 hen a clinician uses a patient medical questionnaire, there is an obligation to do which of
the following? (MULTIPLE ANSWERS)
a. Review the information and discuss or clarify the patient’s responses to determine the impor-
tance of the findings.
b. Have another dental team member review the responses with the patient.
c. Determine if any modifications are necessary in dental treatment before beginning treatment.
d. The clinician only needs to sign the questionnaire in the presence of the patient.

a, c: The clinician must discuss the questionnaire with the patient in order to determine any
necessary modifications to treatment.

102. T
 he clinician should obtain a medical consultation if the dental patient presents with which of
the following? (MULTIPLE ANSWERS)
a. A poorly controlled or undiagnosed medical condition
b. Uncertain health status
c. Well-controlled American Society of Anesthesiologists (ASA) II classification
d. A clear medical record but a history of heavy smoking

a, b: The clinician should seek a medical consultation if the patient presents with a poorly controlled
or undiagnosed medical condition or has an uncertain health status.

103. W
 hich bacteria are the most common cause of community-acquired native valve
endocarditis (NVE)?
a. Helicobacter pylori
b. Lactobacillus acidophilus
c. Viridans streptococci (a-hemolytic streptococci)
d. Bifidobacteria biavatii

c: Viridans streptococci are the most common cause of community-acquired NVE.

104. The interval between the presumed initiating cause of the bacteremia and the commencement
of symptoms of infective endocarditis (IE) is estimated to be less than 2 weeks in what
percentage of cases?
a. < 2%
b. 5% to 10%
c. 50% to 65%
d. > 80%

d: > 80%

26
105. Which of the following statements is true regarding IE? (MULTIPLE ANSWERS)
a. The infective dose required to cause IE in humans has been defined.
b. The magnitude of bacteremia resulting from a dental procedure is relatively high and exceeds
the bacteremia that results from normal daily activities.
c. IE caused by oral bacteria probably results from frequent exposure to low inocula of bacteria
in the bloodstream brought on by routine daily activities.
d. Emphasis on maintaining good oral hygiene and reducing dental and oral disease is key to
decreasing the frequency of bacteremia produced by normal daily activities.

c, d: Both are true statements regarding IE.

106. TRUE OR FALSE: The incidence of bacteremia after toothbrushing is related to poor oral
hygiene and gingival bleeding.

True

107. Antibiotic prophylaxis with dental procedures is recommended for patients with cardiac
conditions associated with the highest risk of adverse outcomes from endocarditis. These
conditions include which of the following? (MULTIPLE ANSWERS)
a. Prosthetic cardiac valve
b. Previous incidence of IE
c. Cardiac valvulopathy in heart transplant recipients
d. Congenital heart disease (CHD) conditions involving: (1) unrepaired cyanotic CHD, including
in patients with palliative shunts and conduits; (2) CHD completely repaired with a prosthetic
device/materials by surgery or catheter intervention less than 6 months ago; and (3) repaired
CHD with residual defects at or adjacent to the site of a prosthetic patch or prosthetic device
that inhibits endothelialization

a, b, c, d: All are cardiac conditions for which antibiotic prophylaxis is recommended with dental
procedures.

108. The preferred antibiotic for prevention of IE prior to a dental implant procedure in a patient
not allergic to penicillin is:
a. Amoxicillin 2 g orally 30 to 60 minutes prior to the procedure
b. Clindamycin 600 mg orally or by IV 30 to 60 minutes prior to the procedure
c. Azithromycin 500 mg orally 30 to 60 minutes prior to the procedure
d. Ampicillin 3 g orally 30 to 60 minutes prior to the procedure

a: Amoxicillin 2 g orally 30 to 60 minutes prior to the procedure

109. Clindamycin is no longer recommended as a prophylactic antibiotic because even a single


dose can cause which of the following?
a. Serious cardiovascular events, especially Torsades de Pointes with ventricular tachycardia (VT)
b. Fungal IE
c. Increased risk of operative and postoperative bleeding
d. Clostridioides difficile infections that can lead to death

d: Clostridioides difficile infections that can lead to death

27
1 MEDICAL EVALUATION OF THE IMPLANT PATIENT

110. S
 tage 1 hypertension is defined by the ACC (American College of Cardiology) and the AHA
(American Heart Association) as blood pressure (BP):
a. < 120/< 80
b. 120–129/< 80
c. 130–139/80–89
d. ≥ 140/≥ 90

c: 130–139/80–89 (Source: Flack JM, Adekola B. Blood pressure and the new ACC/AHA hyper-
tension guidelines. Trends Cardiovasc Med 2020;30:160–164.)

111. The peak ventricular contraction represents which of the following?


a. Average of the systolic and diastolic blood pressure
b. Mean arterial pressure
c. Systolic blood pressure
d. Diastolic blood pressure

d: Diastolic blood pressure

112. W
 hich represents the total resting resistance in the arterial system after passage of the
pulsating force produced by contraction of the left ventricle?
a. Average of the systolic and diastolic blood pressure
b. Mean arterial pressure
c. Systolic blood pressure
d. Diastolic blood pressure

d: Diastolic blood pressure

113. In a routine clinical setting with a patient sitting alone in a quiet place, which is the most
accurate and preferred method of determining BP?
a. Automated BP readings.
b. Auscultatory method with a manual aneroid sphygmomanometer (with a dial) and a stethoscope.
c. There is no difference in accuracy between automated and manual methods for determining BP.
d. BP readings should always be taken by the same clinician for accuracy and consistency.

a: Automated BP readings

114. Which of the following is an early symptom of hypertension?


a. Rupture and hemorrhage of retinal arterioles
b. Proteinuria
c. Elevated blood pressure readings
d. Dementia

c: Elevated blood pressure readings

28
115. The difference between diastolic and systolic pressure is called:
a. BP difference (BPD)
b. Pulse pressure (PP)
c. Mean arterial pressure (MAP)
d. Heart rest pressure (HRP)

b: PP

116. Cardiac output is calculated as:


a. Heart rate times stroke volume
b. Systemic vascular resistance minus diastolic BP
c. Systolic BP minus diastolic BP
d. Diastolic BP plus one-third of the difference between systolic BP and diastolic BP

a: Heart rate times stroke volume

117. A patient will undergo bilateral maxillary sinus augmentation with a lateral window approach.
The patient reports taking long-acting propranolol 40 mg (Inderal LA, ANI Pharmaceuticals)
for hypertension. Which of the following medications should the clinician use with caution
during the procedure? (MULTIPLE ANSWERS)
a. Lidocaine
b. Septocaine
c. Epinephrine
d. Levonordefrin

c, d: Epinephrine and levonordefrin. Propranolol is a nonspecific beta blocker. There is a dose-


related drug interaction between sympathomimetic amines (epinephrine and levonordefrin—
vasoconstrictors) and nonspecific beta blockers. Nonspecific beta blockers block b-1 and b-2
receptors, leaving only a-1 receptors for the sympathomimetic amines to act upon. If a-1 receptors
are not balanced with a compensatory b-2–induced vasodilation, the patient will experience an
unopposed vasoconstriction from a-1–induced vasoconstriction. Thus, the patient could experience
an increase in both systolic and diastolic blood pressures with a concomitant reflex bradycardia.
Because this drug interaction is dose-related, if the patient is on a lower dose of the nonspecific
beta blocker or if the clinician uses less vasoconstrictor (epinephrine), then the resultant increase
in BP is less than if higher doses are used. The dose of the nonspecific beta blocker should not
be changed, but the clinician does have the option of using lower doses of the epinephrine
with caution. An example is to use local anesthetics with epinephrine 1:200,000 rather than epineph-
rine 1:100,000 or 1:50,000. Baseline blood pressure should be taken, and if acceptable, then
0.5 to 1 carpule of local anesthetic with epinephrine (1:200,000) should be administered and the
BP rechecked in 3 to 5 minutes. If the BP is back to baseline (or close to baseline), then more
local anesthesia with a low dose of epinephrine (1:200,000) is administered. This process should
be followed each time local anesthesia with epinephrine is administered.

29
1 MEDICAL EVALUATION OF THE IMPLANT PATIENT

118. A
 72-year-old male patient presents for treatment involving removal of the maxillary left
lateral incisor and immediate placement of a dental implant. The patient normally has a
baseline BP of 156/94 (stage 1 hypertension). He takes lisinopril 10 mg (an angiotensin-
converting enzyme [ACE] inhibitor) daily and hydrochlorothiazide 50 mg (a diuretic) daily.
The patient’s physician said the patient’s hypertension is stable for treatment. The BP at the
start of this treatment appointment is 172/96 with no symptoms. Which of the following
would be appropriate treatment protocols at this appointment? (MULTIPLE ANSWERS)
a. Rescheduling the patient for a future appointment
b. Leaving the BP cuff on the patient during the appointment and periodically checking the BP
during the appointment
c. Using less epinephrine during the appointment
d. Raising the dental chair slowly at the end of the appointment

b, c, d: Rescheduling is always an option, but because the patient is asymptomatic (no headache,
altered behavior, anger, tinnitus, or dizziness) and his current BP is within 20% of his normal base-
line BP, there is no need to cancel the appointment.
Option b, monitoring the BP periodically during the appointment, is correct because during
treatment there is always the possibility for a sudden, acute elevation in BP that could lead to
serious outcomes, including a stroke or myocardial infarction (MI). There may be the release of
endogenous catecholamines during treatment in response to stress and anxiety or from use of
exogenous catecholamines (vasoconstrictors combined with local anesthesia).
Option c, using less epinephrine, is correct because epinephrine activates b-1 receptors, leading
to vasoconstriction and a resultant increase in BP. The increase in BP may be of short duration due
to the COMT (catechol-O-methyltransferase) metabolism of the epinephrine, but even a transient
increase in BP may be enough to induce a stroke or MI.
Option d, raising the dental chair slowly at the end of treatment, is correct because the patient is
taking a thiazide diuretic (hydrochlorothiazide) that can cause orthostatic hypotension, resulting in
vertigo if the patient is righted too quickly

119. A
 CC and AHA guidelines provide recommendations for noncardiac surgery, including surgical
and nonsurgical dental treatments. Which of the following evaluations of risk provide guid-
ance to the dental clinician performing dental implant treatments? (MULTIPLE ANSWERS)
a. Evaluation of the risk imposed by the patient’s cardiovascular disease
b. Evaluation of the risk imposed by the dental implant procedure to be performed
c. Evaluation of the risk imposed by the number of clinical personnel necessary to perform the
procedure
d. Evaluation of the risk imposed by the functional reserve or capacity of the patient

a, b, d: Are all risk evaluations taken from the ACC/AHA guidelines. The number of clinical
personnel necessary to perform a procedure does not affect patient risk. (Source: Little JW, Miller
CS, Rhodus NL. Little and Falace’s Dental Management of the Medically Compromised Patient,
ed 9. St Louis: Elsevier, 2018:46.)

120. F
 or an asymptomatic patient, at what BP reading should the implant dentist defer elective
treatment and make a referral to a physician?
a. ≥ 120/80 and < 140/90
b. ≥ 140/90 and < 160/100
c. ≥ 160/100 and < 180/110
d. ≥ 180/110

d: There is very little risk of an adverse outcome if the BP reading is ≤ 180/110.

30
121. A 68-year-old male patient presents with an infection and exudate as a result of peri-
implantitis surrounding the implant at the site of the mandibular left second premolar.
There is slight localized edema and pain on percussion and chewing but none otherwise.
He reports that he had an MI 8 months earlier. He has not had an episode of angina since
the MI occurred. His medications include daily (1) nitroglycerin extended release 2.5 mg
capsules, (2) metoprolol 100 mg, (3) aspirin 325 mg, (4) clopidogrel (Plavix, Bristol-Myers
Squibb) 75 mg, and (5) simvastatin 20 mg. He has had two coronary artery stents placed.
Which of the following statement(s) is true for this patient?
a. Orthostatic hypotension is not an issue for this patient.
b. There is a significant drug interaction between the epinephrine used in the dental carpule and
the metoprolol.
c. The aspirin and clopidogrel should be discontinued prior to any dental surgical procedure.
d. There may be a need for antibiotics due to the localized infection, but there is no need to
premedicate with antibiotics.

d: An antibiotic may be in order due to the infection, but premedication is not necessary for the
cardiac stents.
Option a is false because the patient is taking nitroglycerin, which may cause orthostatic hypo-
tension. Option b is false because metoprolol is a selective b-1 blocker only. Metoprolol does not
block b-2 receptors. Therefore, the epinephrine would provide vasoconstriction due to activation
of a-1 receptors and the normal compensatory vasodilation due to the activation of b-2 receptors.
Thus, there is no drug interaction at normal doses. Option c is false because the risk of a cardio-
vascular event from clot formation exceeds the risk of a major bleeding event by continuing the
aspirin and clopidogrel.

122. The premature occurrence of an abnormally shaped QRS complex (ventricular contraction)
followed by a pause is known as a premature ventricular complex (PVC). PVCs are common
cardiac arrhythmias that are often present in older male patients seeking dental implant
therapy. A patient with no known structural heart disease and no history of MI, valvular
heart disease, or heart failure is 90 minutes into the surgical removal of 10 mandibular
teeth, an accompanying alveolectomy, and placement of 6 mandibular implants for an
eventual implant-supported fixed prosthesis. The patient is being treated with IV conscious
sedation with an electrocardiogram (ECG) recording in place. Which of the following PVC
patterns is of concern and represents prognostic significance for a possible pending cardio-
vascular event? (MULTIPLES ANSWERS)
a. A single PVC
b. A run of three or more PVCs in a row with a rate of 140 or more (VT)
c. Two repetitions of sustained VT for 3 minutes or more
d. A single PVC that repeats every 3 to 5 minutes

b, c: Both are PVC patterns of concern.

123. TRUE OR FALSE: Patients with pacemakers or implantable cardioverter-defibrillators (ICDs)


are at risk for IE. Therefore, if they are not allergic to beta-lactam antibiotics, they should
always be premedicated with amoxicillin 2 grams 30 to 60 minutes prior to all dental
implant therapies.

False: Patients with ICDs or pacemakers are not at risk for IE. Therefore, there is no need to
premedicate with antibiotics.

31
1 MEDICAL EVALUATION OF THE IMPLANT PATIENT

124. A
 75-year-old female patient with a history of atrial fibrillation is scheduled to have two
periodontally involved teeth removed (the mandibular right first and second molars). She
has been taking warfarin (Coumadin, Bristol-Myers Squibb) 2 mg daily for the past 6 months.
Her international normalized ratio (INR) is 3.0. How should the dental clinician manage the
warfarin for the pending surgical procedure?
a. Inform the physician of the procedure and ask if the warfarin should be discontinued.
b. Ask the physician if the patient should stop the warfarin 3 days prior to surgery and then start
again 2 days after surgery.
c. Ask the physician if the patient should stop the warfarin 3 days prior to surgery and then start
again 1 day after surgery.
d. Inform the physician that surgery will be performed without stopping the warfarin.

d: The dental clinician should inform the physician that surgery will be performed without stop-
ping the warfarin.

125. U
 pon examination of a panoramic radiograph taken for dental implant evaluation, the
clinician notes carotid artery calcifications. Possible causes of carotid artery calcifications
include which of the following? (MULTIPLE ANSWERS)
a. Carotid arteritis
b. Stroke or transient ischemic attack–related disease
c. Hyperlipidemia
d. Hypertension

a, b, c, d: All are possible causes of carotid artery calcifications.

126. H
 ypoparathyroidism would appear radiographically as which of the following?
a. Radiolucency (less bone density)
b. Diffuse sclerotic lesions
c. Radiopacity (greater bone density)
d. Increased spacing of bony trabeculae

c: There is less stimulation of the osteoclasts in hypoparathyroidism. This results in less bone
breakdown, detectable as radiopacity.

127. A 45-year-old male patient presents with an infection that began more than 10 days ago.
At the hospital emergency room 5 days ago, he was given a prescription for amoxicillin 500 mg,
with the instruction to take two pills the first time and then one every 8 hours until the pills are
gone. The infection seems to have only gotten worse as he now has noticeable facial swelling.
Examination reveals radiographic evidence of severely infected dental implants at the sites of the
mandibular right first and second molars. What is the most logical treatment for this patient?
a. Immediately remove both implants.
b. Double the dose of amoxicillin to 1,000 mg every 8 hours and remove the implants within 24 hours.
c. Change the antibiotic to metronidazole 250 mg, with the same instruction to take two pills the
first time and then one every 8 hours, and remove both implants immediately.
d. Keep the patient on amoxicillin and add metronidazole 250 mg, with the instruction to take two
pills the first time and then one every 8 hours for 10 days. Re-examine the patient 48 hours after
starting the metronidazole and remove both implants once the signs of infection have subsided.

d: Although amoxicillin is a broad-spectrum antibiotic and has bactericidal properties against


many gram-positive and gram-negative aerobic and anaerobic microorganisms, it does not resist
destruction by b-lactamase–producing bacteria; therefore, it may not always be effective. Often
it is necessary to improve the obligate anaerobic bacteria coverage, and metronidazole is an
excellent option for this.

32
128. The implant patient should be made aware that they may experience a disulfiram-like reac-
tion of nausea, vomiting, headache, and flushing if alcohol is consumed while taking certain
antibiotics. Admittedly, reports of the interaction in the literature are controversial. Patients
should be cautioned not to consume alcohol when prescribed which antibiotic?
a. Azithromycin
b. Ciprofloxacin
c. Minocycline
d. Metronidazole

d: Metronidazole (Source: Mergenhagen KA, Wattengel BA, Skelly MK, Clark CM, Russo TA.
Fact versus fiction: A review of the evidence behind alcohol and antibiotic interactions. 
Antimicrob Agents Chemother 2020;64:e02167-19.) 

129. A 57-year-old female implant patient with a history of smoking presents with angioedema
that began 3 weeks ago and has gotten progressively worse. The dental examination reveals
no problems with the implants placed 10 years earlier. She does report having recently start-
ed drug therapy for hypertension. Which of the following antihypertensive drugs would be
suspect as causing angioedema?
a. Lisinopril (ACE inhibitor)
b. Losartan (angiotensin receptor blocker)
c. Metoprolol (selective b-1 blocker)
d. Propranolol (nonselective b-blocker)

a: ACE inhibitors inhibit the breakdown of bradykinin and substance P by ACE (kinase II), which
leads to vasodilation and plasma extravasation.

130. A patient with long-standing implants presents for a routine examination, and it is evident
that her gingival tissues are enlarged. They do not bleed or look inflamed but rather have
an enlarged fibrous appearance. She reports that her physician changed her blood pressure
medicine about 6 months ago. Which of the following antihypertensive agents is known to
cause gingival overgrowth?
a. Metoprolol (selective b-1 blocker)
b. Propranolol (nonselective b-1 blocker)
c. Losartan (Cozaar, Merck; an angiotensin receptor blocker)
d. Amlodipine (Norvasc, Pfizer)

d: Amlodipine

33
ORAL 2
EVALUATION
AND TREATMENT
PLANNING
Evaluation of the implant patient goes beyond an under-
standing of any medical issues. Clinical examination should
also include evaluation for any functional, occlusal, biome-
chanical, or esthetic risk factors. This chapter includes many
case descriptions and discusses the evaluation of implant
sites as well as the indications for immediate loading and
placement of implants into extraction sites.

35
2 ORAL EVALUATION AND TREATMENT PLANNING

1. Bone modeling is characterized by which of the following?


a. Change in the shape or size of the bone
b. Change in the density or turnover of the bone

 : The bone modeling process involves independent sites of formation and resorption and results
a
in change in the shape or size of the bone.

2. Bone remodeling is characterized by which of the following?


a. Change in the shape or size of the bone
b. Change in the density or turnover of the bone

b: Bone remodeling is a process of resorption and formation at the same site that replaces
previously existing bone. In other words, it is the process of replacement of old bone by new bone.

3. Macroscopic bone types include which of the following?


a. Trabecular
b. Cortical
c. Woven
d. Lamellar
e. Composite
f. Bundle
g. a and b
h. a, b, c, and d
i. All of the above

g: Two macroscopic bone types exist: trabecular (or cancellous or spongy) bone and cortical
(or compact) bone.

4. TRUE OR FALSE: The bone density on a computed tomography (CT) scan is determined by
the number of pixels in the image.

True: Each axial image on the CT scan has 260,000 pixels, and each pixel has a CT number
(Hounsfield unit) related to the density of the tissues within the pixel. The higher the CT number,
the denser the tissue.

5. TRUE OR FALSE: D1 bone usually has a density measurement of 1,250 Hounsfield units (HU)
or more.

 rue: D1 bone usually measures greater than 1,250 HU, D2 between 850 and 1,250 HU,
T
D3 between 350 and 850 HU, and D4 between 150 and 350 HU.

36
6. At what HU value is dental implant placement probably not a good idea?
a. ≤ 100 HU
b. ≤ 150 HU
c. ≤ 200 HU

b: A value of 150 HU or less presents a very low bone density that’s generally not suitable for
implant placement. If the operator decides to go ahead with implant placement, then the implant
must be given an extended period of time to heal/osseointegrate (7 to 9 months), an osteotome
placement protocol must be followed, and the patient must be warned about the possibility of
unsuccessful osseointegration despite these measures.

7. The density of available bone is a determining factor in which of the following?


a. Treatment planning
b. Implant design selection
c. Surgical technique
d. Healing period length
e. Timing for loading
f. All of the above except e
g. All of the above

g: All of the above

8. In the formula for stress (S = F/A), which of the following is NOT a force factor applied onto
the implant prosthesis?
a. Parafunctions (bruxism, clenching, and tongue thrusting)
b. Location of the implant in the mouth
c. Masticatory dynamics
d. Direction of the load
e. Nature of the opposing arch
f. Implant-to-bone contact ratio

f: The implant-to-bone contact ratio is an area factor in the formula S = F/A.

9. In the formula for stress (S = F/A), which of the following are area factors applied onto the
implant prosthesis?
a. Parafunctions (bruxism, clenching, and tongue thrusting)
b. Location of the implant in the mouth
c. Nature of the opposing arch
d. Implant-to-bone contact ratio
e. Surface area of the implant (length, width, and surface texture)
f. Bone density
g. b, d, and e
h. e and f only
i. d, e, and f
j. All of the above

i: Of these factors, the implant-to-bone contact area, the surface area of the implant, and the
bone density are considered to be area factors in the formula S = F/A.

37
2 ORAL EVALUATION AND TREATMENT PLANNING

10. S
 igns of bruxism include which of the following?
a. Tooth mobility
b. Cervical abfraction of teeth
c. Wear facets
d. Occlusal dimples
e. Exostosis
f. Temperature sensitivity
g. Stress lines in enamel
h. a, b, c, d, and e
i. All of the above except a
j. All of the above

h: Temperature sensitivity and stress lines in enamel are usually signs of clenching and not bruxism.

11. Bruxism and clenching cause which of the following?


a. Increase in the magnitude of force applied to the teeth
b. Increase in the duration of force
c. Change in the direction of force
d. Change in the type of force from shear to compressive
e. a, b, and c
f. All of the above

 : Bruxism and clenching change the type of force from compressive to shear. Bone is 70% weaker
e
under shear forces compared with compressive loads.

12. Management of parafunctions includes which of the following?


a. Educating the patient
b. Occlusal equilibration
c. Nightguard therapy
d. All of the above

d: All of the above

13. T
 RUE OR FALSE: It is important that nightguards fabricated for bruxers allow only anterior
tooth contact during centric occlusion and excursions.

True: The amount of muscle force in general is reduced when bilateral posterior regions are out
of occlusion, therefore decreasing the forces on the implants.

14. T
 RUE OR FALSE: If the implant exhibits any mobility within a few months of placement,
it should be removed.

True: Once lost, osseointegration cannot be reconstituted. Connective tissue can become orga-
nized to a certain degree, but it is not a proper anchoring tissue because of its inadequate
mechanical and biologic capacities.

38
15. Absolute contraindications to implant treatment include which of the following?
a. Acute, terminal illness (evolving cancer)
b. Tumoricidal radiation to the implant site
c. Impossibility of prosthodontic restoration
d. Collagen and bone diseases (osteomalacia, osteogenesis imperfecta, Paget disease)
e. Immunosuppressive disorders (AIDS)
f. Hyperactive involuntary muscle movements (Parkinson disease, Huntington chorea)
g. Unrealistic expectations
h. Lack of operator experience
i. a, b, c, d, e, and f
j. All of the above

j: All of the above

16. Relative contraindications to implant treatment include which of the following?


a. Blood dyscrasia (anemia, sickle cell anemia, polycythemia vera and purpura, granulocytopenia)
b. Pulmonary problems (asthma, bronchitis, emphysema)
c. Anticoagulant therapy
d. Psychiatric or psychologic disorders
e. Mental retardation
f. Chemotherapy
g. Immunosuppressive therapy
h. Hemophilia
i. Unrealistic expectations
j. All of the above except i
k. All of the above

j: Unrealistic expectations are an absolute contraindication for implant treatment.

17. The best way to communicate the end results of the proposed treatment to the patient is by:
a. Showing him or her the end results that were achieved with previous patients.
b. Showing him or her the end results using sophisticated software.
c. Ordering a diagnostic wax-up.

c: Ordering a diagnostic wax-up

18. Benefits of the diagnostic wax-up model to the patient include which of the following?
a. Patients can visualize a model of their current pathologic situation and compare it to the ideal
restored situation.
b. The retention of natural teeth and their role in the overall treatment plan can be visualized.
c. The case presentation is simplified.
d. The patient’s understanding of the proposed treatment can be enhanced.
e. All of the above

e: All of the above

39
2 ORAL EVALUATION AND TREATMENT PLANNING

19. Benefits of the diagnostic wax-up model to the implant surgeon include which of the following?
a. The final gingival contours and emergence profile can be determined, which allows for the
soft tissue to be replaced.
b. The required amount of bone to be replaced (in case of bone loss) can be determined.
c. The occlusal relationships can be re-established, simplifying occlusal modifications.
d. The required number of implants can be determined.
e. The implant angulations can be determined.
f. A transitional prosthesis can be fabricated.
g. The case presentation is simplified.
h. Medical legalities and proper and complete pretreatment planning are accomplished.
i. All of the above

i: All of the above

20. Benefits
 of the diagnostic wax-up model to the laboratory technician include which of
the following?
a. Communication between the dentist and the laboratory is clarified.
b. The fabrication of provisional restorations is simplified.
c. The shapes and contours of the definitive prosthesis can be determined.
d. All of the above

d: All of the above

21. M
 outh opening should be checked before any intraoral examination is performed when
treatment planning implant therapy. If the goal is to place implants in the posterior regions
of the mouth, what should be the normal opening from the maxillary incisal edge to the
mandibular incisal edge?
a. 30 mm
b. 35 mm
c. 40 mm
d. 45 mm

c: 40 mm

22. D
 espite adequate mouth opening for the placement of implants in the posterior regions, the
interarch distance at maximal opening can still be compromised by an overerupted opposing
tooth, which could interfere with implant instruments. What are the potential solutions to
this problem? (MULTIPLE ANSWERS)
a. Orthodontic intrusion of the overerupted tooth
b. Extraction of the overerupted tooth
c. Root canal therapy followed by placement of a fixed prosthesis on the overerupted tooth
d. None of the above. The clinician in this case must change the treatment plan because it is
not possible to place implants under the overerupted tooth.

a, b, c: All are valid solutions to this clinical problem.

40
23. Which of the following can be considered gingival risk factors when treatment planning
implant therapy in the esthetic region? (MULTIPLE ANSWERS)
a. Low smile line in the maxilla or high smile line in the mandible
b. Long and fine papillae
c. Thin biotype
d. Scalloped papillae of adjacent teeth

 , c, d: A high smile line in the maxilla or low smile line in the mandible presents an additional
b
gingival risk factor.

24. TRUE OR FALSE: The papillary morphology of the adjacent natural teeth is important to
consider. With long and fine papillae, perfect esthetic results are hard to obtain. With thick
and short papillae, natural regeneration is facilitated.

True: Complete regeneration of long and fine papillae around an implant is usually difficult to
achieve.

25. Which of the following can be considered a dental risk factor when treatment planning
implant therapy in the esthetic region?
a. Form of natural teeth
b. Position of the interdental point of contact
c. Shape of the interdental contact
d. All of the above

d: All of the above

26. TRUE OR FALSE: The literature supports the idea that if the position of the interdental point
of contact is less than 6 mm from the bone margin, regeneration of the papilla will take place
in practically all cases. The chances of regeneration decrease as the distance increases.

False: Papillae will almost always fill the gap between teeth if the distance between the crestal
bone and the interproximal contact is less than 5 mm. If this distance is more than 5 mm, the
chances are high that a black triangle will be formed.

27. Which of the following can be considered a bone risk factor when treatment planning
implant therapy in the esthetic region? (MULTIPLE ANSWERS)
a. Vestibular concavity
b. Presence of an adjacent implant
c. Horizontal bone resorption
d. Absence of bony papillae

 , b, d: A pronounced vestibular (buccal) concavity requires bone regeneration or grafting before


a
implant placement; otherwise, the implant would have to be placed following the bone crest but
with an unfavorable orientation of the prosthetic axis. It is difficult to achieve papillary regeneration
between two implants because of the absence of a bony papilla (septum). Vertical (and not horizon-
tal) bone resorption resulting from trauma or periodontal disease leads to deep implant placement
unless bone grafting is performed. This situation presents an esthetic risk factor (ie, it is hard for the
papilla to regenerate) and makes proper hygiene difficult to maintain. Loss of proximal bony peaks

41
2 ORAL EVALUATION AND TREATMENT PLANNING

(bony papillae) will also present a difficult situation for papillary regeneration. The radiograph below
shows the absence of bony septae proximally around a maxillary left central incisor that was lost to trauma.

28. W
 hich of the following can be considered a patient risk factor when treatment planning
implant therapy in the esthetic region?
a. Esthetic requirements
b. Hygiene level
c. Provisionalization
d. All of the above

d: All of the above

29. All of the following are considered functional risk factors EXCEPT:
a. Bruxism
b. Clenching
c. Tongue thrusting
d. Tongue size
e. Masticatory dynamics
f. Position of the planned implant in the mouth
g. The nature of the opposing arch
h. Distribution of forces
i. Implant angulation

h: Distribution of occlusal forces is a biomechanical risk factor and not a functional risk factor.

30. All of the following are considered occlusal risk factors EXCEPT:
a. Unbalanced occlusion
b. Presence of wear facets
c. Posterior bite collapse
d. History of cracks and fractures of natural teeth
e. Direction of load
f. Wide occlusal table
g. Lateral occlusal contact on the implant-supported prosthesis
h. Cantilever

h: Cantilever is a biomechanical risk factor and not an occlusal risk factor.

42
31. How many risk factors should be present before considering them a contraindication for
implant treatment?
a. Two or more
b. Three or more
c. Four or more
d. It depends on the type of risk factors

d: Whereas the presence of several risk factors often represents a contraindication to implant
treatment, the presence of one risk factor is not necessarily a contraindication to implant treat-
ment. In certain cases, however, one risk factor is adequate to present a contraindication (eg,
unrealistic esthetic expectations). The higher the esthetic requirements of the patient, the more
cooperative the patient should be and the more important it is that they be aware of the difficul-
ties, limitations, and duration of treatment. Patients with unrealistic esthetic demands present a
contraindication for implant therapy.

32. Contraindications for immediate implant placement after extraction include which of
the following?
a. The implant cannot be inserted with primary stability
b. The tooth is substantially larger than the diameter of the implant
c. The implant cannot be inserted in an appropriate position from the prosthetic point of view
d. Acute infection
e. Chronic infection
f. All of the above except e
g. All of the above

f: All of the above but chronic infection present contraindications for immediate implant place-
ment. It is important to note, however, that chronic infection that cannot be cleared completely
during the extraction appointment is a contraindication.

33. Advantages of immediate implant placement include all of the following EXCEPT:
a. Maintenance of alveolar bone
b. Maintenance of interdental papillae
c. Osteogenic potential is greatest immediately after extraction
d. Less time between implant placement and restoration
e. High patient acceptance

b: Immediate implant placement is crucial for maintaining the alveolar bony architecture and
the soft tissue to the crestal bone level; maintenance of the soft tissue above the crestal bone
level and on the bony papillae requires a provisional immediate crown (using the immediate
loading technique).

34. Disadvantages of immediate implant placement include all of the following EXCEPT:
a. Extension of the healing period by 4 to 5 months because this placement does not provide
optimal initial bone-to-implant contact
b. Difficulty in establishing soft tissue coverage
c. Risk of bacterial invasion of osteotomy site
d. Risk of soft tissue invasion of osteotomy site

a: The healing period does not have to be extended if initial stability and other osseointegration
conditions are met during implant placement.

43
2 ORAL EVALUATION AND TREATMENT PLANNING

35. W
 hen performing immediate implant placement in a fresh extraction site, the gap between
the implant platform and the walls of the socket must be filled when it is:
a. 0.5 mm or more
b. 1.0 mm or more
c. 1.5 mm or more
d. 2.0 mm or more

a: Any gap 0.5 mm or larger should be filled with bone grafting material.

36. W
 hen performing immediate implant placement in a fresh extraction site, the implant
platform should be placed:
a. Flush with the buccal bone level
b. 0.5 mm under the bone level
c. 1.0 mm under the bone level
d. 1.5 mm under the bone level

b: Although the implant’s depth is determined to a great degree by the thickness and type of
the soft tissue above it, in most scenarios proper depth leads to the implant platform being
flush with the buccal bone level. In immediate placement, however, it is important to position
the implant platform 0.5 mm under the buccal bone level to account for slight bone loss after
implant placement.

37. Which of the following medical conditions allows for elective dental implant surgery?
a. Psychotic disorder
b. Diagnosed osteoporosis
c. Severe angina pectoris
d. Pregnancy

b: Diagnosed osteoporosis allows for elective implant placement. The other three conditions
listed in the question are considered contraindications for dental implant therapy.

38. A
 65-year-old man presents for an implant consultation for replacement of his missing
mandibular left premolars and molars with a fixed partial denture. The patient has been
missing his left molars for 25 years, and his left premolars have been missing for 2 years.
Periapical and panoramic radiographs demonstrate a minimal anterior loop to the inferior
alveolar nerve relative to the mental foramen and 8 mm of vertical bone height superior
to the inferior alveolar nerve in the first and second molar region. Which of the following
clinical procedures would be helpful in determining the available bone for placement of
a root form implant?
a. Mounted diagnostic casts
b. Surgical template
c. Sharp caliper measurement
d. Manual palpation
e. Any of the above
f. None of the above

f: When the periapical or panoramic radiograph shows only 8 mm of available bone, then the
procedure presents a high risk. The operator must obtain a CT scan and reflect a full-thickness
flap to expose the mental foramen and verify the exact bony height.

44
39. A 16-year-old, healthy-appearing boy presents with a traumatic avulsion of the right maxillary
central incisor. Radiographs reveal that the root is fractured and nonrestorable and therefore
must be extracted. His medical history is unremarkable except for episodes of epistaxis since
childhood and occasional excessive bruising. Other family members have similar histories.
Preoperative laboratory results reveal a normal platelet count with prolonged bleeding time.
This patient most likely has which of the following disorders?
a. Classic hemophilia (AHF variety)
b. Idiopathic thrombocytopenia purpura
c. Hereditary hemorrhagic telangiectasia
d. Vascular hemophilia (von Willebrand disease)

d: Von Willebrand disease is the most common bleeding disorder, affecting up to 1% of the US
population, and occurs equally in men and women. It is a genetic disorder caused by missing or
defective von Willebrand factor, a clotting protein that helps form a platelet plug during the clotting
process. Symptoms of von Willebrand disease include frequent nosebleeds, easy bruising, and
excessive bleeding during and after invasive procedures, including tooth extractions and surgery.

40. A 60-year-old woman presents with a maxillary denture and complete mandibular natural
dentition. She desires fixed, esthetic teeth in the maxilla that look like they are emerging
from her gums. Removal of the maxillary prosthesis reveals the incisive papilla on the crestal
aspect of the incisive bone. What does this clinical observation absolutely indicate?
a. The incisive bone is deficient and will require osseous grafting to achieve the desired results.
b. The patient will never have her esthetic demands satisfied and should consider a long-term
removable prosthesis.
c. The incisive bone is irreparably damaged due to loss of teeth, so the patient should consider
an overdenture.
d. The incisive papilla, being non-osseous, should be avoided, precluding the use of dental
implants.

a: When the incisive papilla is located on the crest of the incisive bone, the patient has lost
6 to 10 mm of the alveolar bony width in this area. The incisive foramen may be located near
the residual ridge as a result of resorption of the anterior maxillary bone. When replacing lost
maxillary central incisors with dental implants, there is a risk of entering the incisive canal and
thereby compromising the osseointegration of the implants. Implant placement will not be possible
without a bone grafting procedure.

41. A totally edentulous patient desires an implant-retained prosthesis. You and the patient
agree to the placement of a mandibular overdenture supported by two root form implants.
For optimal results, you should place the implants into which of the following areas?
a. Immediately to the right and left of the midline
b. Anterior to the right and left mental foramina
c. Midway between the midline and the right and left mental foramina
d. Bilaterally in the second premolar area

c: Midway between the midline and the right and left mental foramina

45
2 ORAL EVALUATION AND TREATMENT PLANNING

42. A
 patient who underwent extraction of her mandibular posterior teeth some time ago
pre-sents for consultation. After assessing the patient’s needs, you propose a fixed
mandibular prosthesis that is supported by splinted implant abutments in the posterior
area and splinted natural tooth abutments in the anterior region. To avoid damage to either
group of abutments through flexure, which of the following would be MOST effective?
a. Ceka attachment
b. Telescopic copings
c. Mortising attachments
d. Extracoronal T attachments

b: A few conditions must be met when connecting natural teeth to implants. These conditions
include healthy natural roots, precise and passive fit of the implant prosthesis, and the use of
telescopic abutments to prevent the possibility of natural tooth decay.

43. A
 partially edentulous patient who desires an implant-supported prosthesis presents for
consultation. The patient reports that he has episodes of kidney stones. A clinical examination
reveals that the patient’s teeth exhibit class 3 mobility. A radiographic examination reveals that
both the maxilla and the mandible have undergone a generalized demineralization and there is
loss of the lamina dura. Which of the following conditions does the patient most likely have?
a. Addison’s disease
b. Hyperparathyroidism
c. Fibrous dysplasia
d. Paget disease

b: Hyperparathyroidism is a disorder in which too much parathyroid hormone is released into
the bloodstream. In primary hyperparathyroidism, one or more enlarged, overactive parathy-
roid glands secrete too much parathyroid hormone. Secondary hyperparathyroidism, another
disease that initially causes low levels of calcium in the body, gives rise to increased parathyroid
hormone levels. This excess parathyroid hormone triggers the release of too much calcium into
the bloodstream. The bones may in turn lose calcium, and the levels of calcium may increase in
the urine, causing kidney stones.

44. A
 patient who recently underwent extraction of his mandibular teeth and desires a replace-
ment for them presents for consultation. After assessing the patient’s needs, you recommend
the placement of root form implants in the mandibular anterior region. During surgery, you
inadvertently pass the pilot drill through the inferior border of the patient’s mandible while
preparing an implant receptor site. Which of the following steps should you take?
a. Discontinue with the placement of the implant.
b. Continue with the placement of the implant but add collagen.
c. Continue with the placement of the implant as planned.
d. Place the implant in an alternate site.

c: When drilling in the anterior mandibular region, we do not have to worry about damaging the
inferior alveolar nerve; however, a careful evaluation of the patient’s CT scan before surgery must
be performed to rule out the existence of the mandibular incisive canal (see chapter 3). If this
canal does not exist, then the operator can continue with the osteotomy but must stay short of
the mandibular inferior border with any subsequent drills.

46
45. A 79-year-old man who had a full subperiosteal implant placed in his mandibular arch 9 years
ago presents for a maintenance evaluation. For the past several months, you have noted
the presence of localized pergingival granulomas. Although you have removed them with a
curette each time, the granulomas have persisted in recurring. Because of this patient’s con-
dition, which of the following actions should you take at this time?
a. Remove the implant and construct a full mandibular denture for the patient.
b. Prescribe a broad-spectrum antibiotic and chlorhexidine rinses.
c. Evaluate the defect and either place a graft or remove the segment.
d. Continue to remove the granulomas with a curette.

c: When a problem presents, something must be done about it as soon as possible. Choices b
and d are not a solution to the problem described. Choice a is incorrect because there is no need
to remove the implant if the problem can be treated without its removal.

46. A patient presented for treatment following extraction of his mandibular right first premolar.
After initial consultation, you propose a root form implant to replace the missing premolar,
and the patient agrees to the treatment plan. Immediately after inserting the implant, you
observe that two threads are exposed. What should you do?
a. Close the soft tissue and place a graft of demineralized freeze-dried bone and resorbable
hydroxyapatite plus a barrier membrane at stage-two surgery.
b. Close the soft tissue and place a connective tissue graft at stage-two surgery.
c. Perform an immediate graft using nonresorbable hydroxyapatite.
d. Perform an immediate graft using resorbable hydroxyapatite and a barrier membrane.

d: Perform an immediate graft using resorbable hydroxyapatite and a barrier membrane

47. A patient elected to undergo single-stage surgery for the placement of a plate form implant.
After inserting this implant to its full depth with finger pressure at the time of surgery, you
observe that it exhibits slight mobility. To stabilize the implant, which of the following steps
should you take?
a. Splint the implant to a natural tooth with a provisional fixed partial denture.
b. Remove the implant and try again with two-stage surgery.
c. Remove the implant and place a bi-blade implant.
d. Remove the implant, bend its body, and reinsert it.

d: Remove the implant, bend its body, and reinsert. Choices a, b, and c will not resolve the problem.

48. A patient has an acute carious abscess in the mandibular right first molar and periodontal
disease extending into the furcation area. The alveolar bone is 13 mm in height superior to
the mandibular canal and 8.5 mm mesiodistally. The remainder of the patient’s dentition is
intact. After assessing each of the patient’s needs, you recommend extraction of the tooth
and placement of at least one root form implant to support a freestanding crown. After
extracting the tooth and performing curettage of the extraction site, which of the following
steps should you take?
a. Augment the extraction site and place one root form implant.
b. Augment the extraction site and place two root form implants.
c. Use guided tissue regeneration with augmentation of the extraction site.
d. Treat the area after the extraction socket has healed.

47
2 ORAL EVALUATION AND TREATMENT PLANNING

d: In most fresh molar extraction sites, it is a challenge to place an implant immediately after
extraction. It is recommended in most cases to graft the socket, wait for 4 to 5 months, and then
place the implant. Exceptions include primary molar extraction sites and the mesial roots of
mandibular second molars upon extraction.

49. A
 patient with a partially edentulous mandible who desires a prosthetic replacement
pre-sents for consultation. After assessing the patient’s needs, you propose the placement
of an implant-supported fixed partial denture. During the first try-in appointment, you cut
the patient’s framework into three pieces and obtain a solder registration. During the
next visit, you are able to obtain an adequate fit, but the soldered frame no longer fits
the master cast. What should you do to obtain a sufficient fit at this time?
a. Obtain a pickup impression of the soldered framework and fabricate a new master cast.
b. Adjust the ill-fitting analogs on the master cast until a passive fit can be achieved.
c. Obtain a sectional cutout of the ill-fitting analogs and then place new analogs.
d. Take a new impression and make a new frame.

a: Obtaining a pickup impression of the soldered framework and fabricating a new master cast
would be the easiest way to solve this problem.

50. A
 patient with a totally edentulous mandible who desires a prosthetic replacement presents
for consultation. In clinically examining the patient’s arch, you observe that there is a minimal
zone of attached gingival tissue on the edentulous crest. After assessing each of the patient’s
needs, you propose the placement of an eposteal (subperiosteal) implant to support an
overdenture. Postoperatively, the patient complains of numbness in the floor of his mouth
and tongue biting. Which of the following nerves did you injure during the surgery?
a. Mylohyoid
b. Hypoglossal
c. Inferior alveolar
d. Lingual

d: The symptoms described here would result from injury to the lingual nerve.

51. A
 patient has elected to undergo surgery for the placement of an endosseous implant in
her maxillary posterior region. After administering a posterior superior alveolar nerve block,
you observe the immediate formation of a hematoma. Which of the following did you most
likely traumatize?
a. Pterygoid venous plexus
b. Lateral pterygoid muscle
c. Sphenopalatine artery
d. Zygomatic artery

a: If the needle is overinserted during posterior superior alveolar nerve block, it may penetrate
the pterygoid plexus of the vein and the maxillary artery in the infratemporal fossa, thus causing
hematoma. This results in extraoral swelling a few minutes after the injection. The hematoma
will cause tissue tenderness and discoloration that will last until the blood is broken down by
the body. There is also a possibility of infection spreading to the cavernous venous sinus if the
needle is contaminated. A hematoma can also develop during other blocks, such as the infraor-
bital and inferior alveolar nerve blocks. To avoid injection into blood vessels, aspiration should
be attempted in all injections.

48
52. In which of the following regions can you place transosteal implants?
a. Symphysis
b. Anterior maxilla
c. Mandibular body
d. Tuberosities

a: Transosteal implants are used only in the mandible and for very atrophic situations. They are
inserted through an extraoral incision below the chin with a series of projections that penetrate
the mandible from its inferior border and are connected by a bone plate that rests on the inferior
border of the mandible. Several of the projections completely traverse the mandible to enter the
oral cavity and anchor the mandibular denture.

53. A patient who underwent surgery for the placement of six root form implants presents for
stage-two permucosal extension surgery. During the procedure, you plan to place the stan-
dard abutments and a healing cap, adjust the patient’s full denture, and return it to her for
use. In modifying the patient’s denture to fit on the abutments, the GREATEST consideration
should be given to which of the following?
a. It must place no pressure on the abutments.
b. It can place a limited pressure on the abutments.
c. It will acquire retention.
d. The soft liner will adapt well to the soft tissue configuration.

b: Choice a would be the correct answer if the denture were to be adjusted immediately after
surgery. However, once the implants have osseointegrated, a limited pressure on the abutments
is acceptable.

54. A patient who has undergone extraction of her mandibular right first and second molars and
has a sharp undercut on her lingual cortical plate presents for implant treatment. The under-
cut begins approximately 7 mm inferior to the crest of her mandibular alveolar ridge. There
is approximately 7 mm of bone superior to the inferior alveolar nerve canal. For this patient,
which of the following treatment options is most appropriate?
a. Place two 6- or 7-mm-long root form implants in the positions of the first and second molars.
b. Place a subperiosteal (eposteal) implant in the positions of the first and second molars.
c. Place a ramus blade (plate) implant in the positions of the second and third molars.
d. Reposition the patient’s inferior alveolar nerve.

a: Place two 6- or 7-mm-long root form implants in the positions of the first and second molars;
however, you must consider splinting and any stress factors, including the crown-to-implant ratio.

49
2 ORAL EVALUATION AND TREATMENT PLANNING

55. A
 58-year-old man who had four root form implants placed to support a maxillary anterior
fixed partial denture returns for a maintenance evaluation. He reports that since the time
of his dental implant treatment, he has begun taking disulfiram (Antabuse, Wyeth-Ayerst)
250 mg on a daily basis. He also reports that he smokes 1½ packs of cigarettes each day
and that he is in a supportive psychotherapeutic program. In clinically examining the
patient’s soft tissues, you observe that he has nicotine stomatitis and that his oral mucous
membranes and gingival tissues do not look normal. Later, the patient admits to experi-
encing facial flushes, throbbing headaches, nausea, vomiting, thirst, and hyperventilation.
Usually these symptoms appear after he ingests salads and certain foods with sauces or after
he takes cough medicine. Sometimes they appear after he uses aftershave. Which of the
following conditions is the patient’s psychotherapeutic program most likely designed to treat?
a. Alcoholism
b. Anorexia
c. Bulimia
d. Narcotic addiction

a: Alcoholism

56. A
 65-year-old woman who underwent extraction of her maxillary anterior teeth some time
ago and who desires an implant-supported prosthesis presents for consultation. You observe
that the ridge lacks sufficient thickness for the placement of root form implants. Its height,
however, is adequate. To prepare the bone for the placement of root form implants, you
propose augmentation procedures. Which of the following is the most effective grafting
material for this situation?
a. Freeze-dried demineralized bone
b. Irradiated cortical bone chips
c. Resorbable hydroxyapatite
d. Xenograft material
e. Combination of a and d
f. Combination of a and c

 : In most bone grafting procedures, it is a good practice to combine an allograft with a xenograft
e
because osteoinductive and osteoconductive properties will be needed.

57. Which of the followings is NOT correct about preliminary evaluation of the implant site?
a. Presurgical assessment of the implant site by suitable imaging procedures is a compulsory
procedure.
b. One must use CT scans before implant placement when conventional radiographs do not
provide satisfactory information about the bone volumes in the implant site.
c. Intraoral and panoramic radiographs are essential screening tools that can be used for detec-
tion of pathologic conditions and preliminary evaluation of the implant sites.
d. Intraoral and panoramic radiographs usually do not provide adequate information for success-
ful implant placement, especially for the mandibular posterior region.
e. Radiologic evaluation of the implant site is an optional procedure, and we do not have to
provide it for every patient.

e: Placing dental implants without radiologic evaluation, particularly 3D CT scans, is considered
to be below the standard of care.

50
58. Radiologic evaluation of the implant site using intraoral and ordinary panoramic radiographs
is useful for:
a. Detection of pathologic processes and identification of gross bone changes.
b. Rough estimation of the superio-inferior and mesiodistal dimensions of the available bone.
c. Relative evaluation of gross osteoporotic changes; however, these findings may not be
reliable, and the suspected conditions should be confirmed by other imaging methods.
d. Rough estimation of the buccolingual shape of the alveolar bone and detection of mild lingual
concavities.
e. Precise estimation of the buccolingual dimensions of the bone.
f. a and b
g. a, b, and c
h. a, b, c, and d
i. All of the above

g: It must be noted that panoramic radiography can distort and increase image size by 1.5 times
the original, so this type of radiography cannot be relied on for measuring purposes. It also must
be emphasized that 3D imaging is required for planning implant placement.

59. Detection of which of the following conditions on the radiograph absolutely contraindicates
placement of an implant until complete treatment and resolution of the condition or disease
has been achieved?
a. A radiologic shadow suspected to be induced by a neoplastic lesion
b. Shadows compatible with infection of the jawbone, including osteomyelitis
c. Bone rarefaction (loss of bone density) due to a systemic disease such as primary
hyperparathyroidism
d. Condensing osteitis (sclerotic bone associated with infection or inflammation) located in the
area of the implant site
e. Idiopathic sclerotic bone
f. a or b
g. a, b, or c
h. a, b, c, or d
i. Any of the above

 : Idiopathic osteosclerosis is a nonpathologic lesion that will not interfere with implant placement.
h
It may be found around the roots of a tooth during routine radiography and is usually painless.
It appears as a radiopacity peripheral to a root surface, usually a premolar or molar, and there is
no sign of inflammation of the tooth.

60. W
 hich of the following criteria plays an important role in selection of the width of the implant?
a. Presence or absence of a bone defect or concavity in the buccal or lingual surfaces
of the bone
b. Buccolingual dimensions of the alveolar bone
c. Mesiodistal dimensions of the bone in the implant site
d. Size of the planned implant prosthesis
e. Density of the bone in the implant site
f. All of the above except e
g. All of the above

g: All of the above

51
2 ORAL EVALUATION AND TREATMENT PLANNING

61. W
 hich of the following imaging techniques is most reliable for detection of abnormal bone
densities in the implant site?
a. Intraoral radiography
b. Panoramic radiography
c. Conventional CT
d. Conventional complex motion tomography
e. Conventional tomography with linear or circular motion

c: Conventional CT scans provide 3D views of any lesion.

62. A
 patient is scheduled for placement of an implant in his mandibular first molar area.
Cross-sectional images of the implant site show a steep concavity in the lingual surface of
the bone. This condition dictates which of the following options during implant selection to
avoid perforation of the lingual concavity and possible damage to the lingual artery?
a. Use of a shorter implant
b. Selection of a different path of insertion to avoid the lingual concavity
c. Use of a tapered implant
d. Use of a narrower implant
e. a and b
f. a, b, and c
g. All of the above can be correct

g: Perforation of the lingual concavity and damage to the lingual artery can cause rapid and
life-threatening bleeding, leading to a blocked airway and death.

63. W
 hich of the following criteria is most important to consider for determining an accurate
path of insertion of the implants?
a. Width and height of the available alveolar bone at the implant site
b. Location of the neurovascular bundles or maxillary sinuses in the alveolar bone, considering
the length and width of the selected implants
c. Shape of the alveolar bone and presence or absence of concavities in the lingual or buccal
surfaces of the bone
d. Mesiodistal dimension of the available bone and alignment of the adjacent teeth
e. All of the above

e: An experienced clinician will evaluate all of these factors both radiographically and clinically. A
fundamental aspect of implant dentistry is avoiding vital anatomical structures. Proper planning
and execution are essential.

64. A
 man presents with only maxillary anterior teeth and is planned to receive several
implants in his mandible and posterior maxilla. Which of the following imaging methods is
most reasonable to use prior to treatment?
a. Selective periapical radiographs of the remaining teeth and a panoramic radiograph for rou-
tine dental and jawbone evaluation as well as conventional motion complex tomography for
providing cross-sectional images
b. Selective periapical radiographs of the remaining teeth and a panoramic radiograph for rou-
tine dental and jawbone evaluation as well as conventional CT for cross-sectional images
c. A full-mouth series
d. Selective periapical radiographs of the remaining teeth and a panoramic radiograph

52
b: Each type of radiography in choice b will provide additional information to help with treatment
planning. When natural teeth are present, traditional periapical radiographs provide important
information about the prognosis of those teeth. Traditional techniques should not be overlooked,
particularly as they can be diagnostic with minimal dose to the patient.

65. The term immediate implant placement refers to which of the following?
a. Placing a dental implant on the same day you meet your patient for the first time
b. The extraction of a tooth and placement of a dental implant in the extraction socket on the
same day
c. Shortening the residual root, extracting the tooth, and placing an implant 4 to 6 weeks later
d. Implant placement after healing of the extraction site
e. None of the above

b: Immediate implant placement refers to surgical implant insertion on the same day as the
extraction. Immediate implant placement requires very specific criteria and planning to achieve
a successful outcome. Not every tooth to be extracted will qualify for this procedure.

66. TRUE OR FALSE: The surgical protocol used with immediate implant placement requires that
the implant engage bone apical to the location of the original tooth apex.

 alse: While engaging bone apical to the original root apex helps to ensure primary stability, which
F
is a requirement for osseointegration, it is not necessary. In narrow sockets, it is often possible to
achieve good initial stability without drilling apical to the extraction socket.

67. TRUE OR FALSE: Immediate implant placement into the socket of a single-rooted tooth
places the implant where the root of the tooth was located.

True: However, the implant may need to be placed beyond the bony housing of the root in order
to engage bone for primary stability. In the anterior region, if a screw-retained restoration is
required, the head of the implant will need to be slightly more palatal than the root.

68. TRUE OR FALSE: Immediate implant placement decreases total treatment time.

True: Immediate implant placement can be a great practice builder, but the prudent clinician will
need to be highly skilled and very selective about which situations are suitable for this procedure.

69. TRUE OR FALSE: Implants that are immediately placed have a significantly lower success rate
than those placed with the delayed placement technique.

 alse: Immediately placed implants have a very high success rate; if proper diagnostic procedures
F
and treatment protocols are followed, success can be achieved with fewer surgical procedures
and much earlier than with traditional techniques.

53
2 ORAL EVALUATION AND TREATMENT PLANNING

70. T
 o predictably achieve a soft tissue papilla between two implants, what is the ideal space to
maintain between the adjacent implants?
a. 2 mm
b. 3 mm
c. 5 mm
d. 6 mm

b: Many studies have shown that if the implants are any closer than 3 mm, interproximal
bone and papilla loss will result. (Sources: Tarnow DP, Cho SC, Wallace SS. The effect of inter-
implant distance on the height of inter-implant bone crest. J Periodontol 2000;71:546–549. /
Kourkouta S, Dedi KD, Paquette DW, Mol A. Interproximal tissue dimensions in relation to adja-
cent implants in the anterior maxilla: Clinical observations and patient aesthetic evaluation. Clin
Oral Implants Res 2009;20:1375–1385. / Teughels W, Merheb J, Quirynen M. Critical horizontal
dimensions of interproximal and buccal bone around implants for optimal aesthetic outcomes:
A systematic review. Clin Oral Implants Res 2009;20[suppl 4]:134–145.)

71. The concept of osseointegration was first described by:


a. Ingemar Stenmark
b. Harald Löe
c. Jan Lindhe
d. Per-Ingvar Brånemark

d: Per-Ingvar Brånemark

72. P
 rolonged (greater than 2 weeks) steroid medication (prednisone 10 mg/day) in an implant
patient may cause:
a. An increase in blood glucose
b. A decrease in blood glucose
c. An increase in inflammation
d. An enhanced healing potential
e. None of the above

a: Cushing’s syndrome, characterized by high blood pressure, abdominal obesity with thin arms
and legs, reddish stretch marks, a round red face, a fat lump between the shoulders, weak muscles,
weak bones, acne, and fragile skin that heals poorly, is caused by excessive cortisol-like medication
such as prednisone that either produces or results in the production of excessive cortisol by the
adrenal glands. Blood glucose levels may also increase.

54
HEAD AND NECK 3
ANATOMY

Detailed knowledge of anatomy is vital for the execution


of implant-related surgical procedures in the oral cavity, an
area that presents special topographic and anatomical diffi-
culties. The aim of this chapter is to present an adequate
amount of anatomical material for review. Reviewing the
questions in this chapter should provide the reader with suffi-
cient information to prepare for the American Board of Oral
Implantology certification examination. For more detailed
information regarding anatomy, refer to Clinical Anatomy
for Oral Implantology, Second Edition (Quintessence, 2021).

55
3 HEAD AND NECK ANATOMY

1. Which bone of the skull contains the optic foramen, foramen ovale, and foramen spinosum?
a. Frontal
b. Temporal
c. Sphenoid
d. Ethmoid

c: Foramina of the sphenoid include the optic foramen, foramen ovale, foramen rotundum,
foramen spinosum, and superior orbital fissure.

2. The internal jugular vein:


a. Is located in the carotid sheath.
b. Receives venous drainage from the brain.
c. Is a tributary of the brachiocephalic vein.
d. Receives blood from the facial vein.
e. All of the above

e: All of the above

3. A
 patient is affected by left-side Bell’s palsy. Which of the following problems does this
patient likely have?
a. He is unable to open his left eye.
b. He is unable to raise his eyebrow on the left side.
c. He is unable to protrude his tongue to the right.
d. He is unable to feel pressure on his left cheek.

 : Cranial nerve (CN) VII, the facial nerve, exits the stylomastoid foramen and is responsible for the
b
muscles of facial expression, including the occipitofrontalis muscle, which elevates the eyebrows.

4. Which nerve would most likely be affected by a mandibular fracture?


a. Lingual nerve
b. Hypoglossal nerve
c. Inferior alveolar nerve
d. Chorda tympani nerve

c: The inferior alveolar nerve courses within the mandibular canal and would therefore be injured
by a mandibular fracture.

5. W
 hich muscle of mastication inserts into the condylar process of the mandible and its
articular disc and the capsule of the temporomandibular joint (TMJ)?
a. Medial pterygoid muscle
b. Lateral pterygoid muscle
c. Temporalis muscle
d. Masseter muscle
e. Buccinator muscle

b: Lateral pterygoid muscle

56
6. Which muscle of mastication inserts into the pterygoid tuberosity on the medial surface of
the angle of the mandible?
a. Medial pterygoid muscle
b. Lateral pterygoid muscle
c. Temporalis muscle
d. Masseter muscle
e. Buccinator muscle

a: Medial pterygoid muscle

Lateral
pterygoid,
superior part

Articular disc
Lateral
pterygoid,
inferior part

Medial
pterygoid

Lateral plate,
pterygoid process

7. Which muscle of mastication inserts into the apex and medial surface of the coronoid process
of the mandible?
a. Medial pterygoid muscle
b. Lateral pterygoid muscle
c. Temporalis muscle
d. Masseter muscle
e. Buccinator muscle

c: Temporalis muscle

Temporalis

Zygomatic arch

Joint capsule

Lateral ligament
Lateral pterygoid

Coronoid process Masseter

57
3 HEAD AND NECK ANATOMY

8. Which muscle of mastication inserts into the angle of the mouth and orbicularis oris?
a. Medial pterygoid muscle
b. Lateral pterygoid muscle
c. Temporalis muscle
d. Masseter muscle
e. Buccinator muscle

e: Buccinator muscle

Occipitofrontalis,
Galea frontal belly
aponeurotica
Orbicularis oculi
Superior
auricularis Procerus
Temporoparietal Levator labii superioris
alaeque nasi muscle
Anterior Nasalis
auricularis Dilator naris,
Occipitofrontalis, anterior muscle
occipital belly Compressor narium
minor muscle
Posterior auricularis
Articular capsule Alar nasalis muscle
Parotid gland Levator nasalis muscle
Accessory parotid gland Orbicularis muscle
Zygomaticus major and minor
Parotid duct
Masseter Depressor labii inferioris
Mentalis
Risorius
Buccinator
Platysma Depressor anguli oris

9. Which muscle of mastication inserts into the masseteric tuberosity at the angle of the mandible?
a. Medial pterygoid muscle
b. Lateral pterygoid muscle
c. Temporalis muscle
d. Masseter muscle
e. Buccinator muscle

d: Masseter muscle

Temporalis

Lateral
pterygoid

Medial pterygoid

Masseter

58
10. Which bone forms the inferior and posterior parts of the nasal septum?
a. Vomer
b. Ethmoid
c. Palatine
d. Temporal
e. Sphenoid

a: Vomer

Crista galli

Cribriform
plate

Nasal bone

Perpendicular plate,
ethmoid bone

Septal
cartilage

Vomer

Incisive canal

11. Which bone forms the superior and posterior parts of the nasal septum?
a. Volmer
b. Ethmoid
c. Palatine
d. Temporal
e. Sphenoid

b: The ethmoid bone, through its perpendicular plate, makes up the superior and posterior parts
of the nasal septum (see illustration in question #10).

59
3 HEAD AND NECK ANATOMY

12. W
 hen performing an emergency airway, where is the most superior site to perform this
procedure so that the airway is secured below the vocal folds?
a. Between the hyoid bone and the thyroid cartilage
b. Between the thyroid cartilage and the cricoid cartilage
c. Between the thyroid cartilage and the arytenoid cartilage

b: The quickest access to secure an airway is by incising the cricothyroid membrane. This proce-
dure is called an emergency tracheotomy (or cricothyrotomy).

Thyroid
cartilage
Cricoid
cartilage
Thyroid
gland

13. Which is the only laryngeal cartilage to completely encircle the airway?
a. Thyroid cartilage
b. Cricoid cartilage
c. Epiglottic cartilage
d. Arytenoid cartilage
e. Corniculate cartilage

b: Cricoid cartilage

14. T
 he vagus nerve (CN X):
a. Leaves the skull through the carotid foramen.
b. Innervates the carotid sinus.
c. Is transmitted through the superior orbital fissure.
d. Innervates all of the muscles of the soft palate.
e. None of the above

e: The vagus nerve leaves the skull through the jugular foramen with the glossopharyngeal and
spinal accessory nerves.

60
15. The soft palate receives innervation from the:
a. Oculomotor nerve (CN III)
b. Trochlear nerve (CN IV)
c. Abducent nerve (CN VI)
d. Vagus nerve (CN X) and the mandibular division of the trigeminal nerve

d: The vagus nerve innervates all of the muscles of the soft palate except the tensor veli palatini,
which is supplied by the mandibular division of the trigeminal nerve.

16. To inject anesthetic into the pterygopalatine fossa through a lateral approach, the needle
passes through the mandibular notch of the mandible, traverses the infratemporal fossa,
and enters the pterygomaxillary fissure. Which of the following structures would be at risk
during this procedure?
a. Maxillary artery
b. Mandibular division of the trigeminal nerve (CN III)
c. Pterygoid venous plexus
d. Otic ganglion
e. All of the above

 : The infratemporal fossa contains the maxillary artery and many of its branches, the mandibular
e
nerve, the pterygoid plexus, and the otic ganglion, as well as the medial and lateral pterygoid muscles.

17. Which of the following drains into the inferior meatus of the nasal cavity?
a. Nasolacrimal duct
b. Frontal sinus
c. Ethmoidal sinus
d. Sphenoidal sinus
e. Maxillary sinus

a: The nasolacrimal duct drains tears from the medial corner of each eye into the inferior meatus.

18. Which of the following drains into the middle meatus of the nasal cavity?
a. Nasolacrimal duct
b. Frontal, ethmoidal, and maxillary sinuses
c. Ethmoidal and sphenoidal sinuses
d. Sphenoidal sinus only
e. Maxillary sinus only

b: The frontal sinus, the maxillary sinus, and part of the ethmoidal sinus drain into the middle meatus.

61
3 HEAD AND NECK ANATOMY

19. Intubation during any surgery requiring general anesthesia is necessary because, due to
muscle relaxants and other drugs administered during the procedure, the patient is unable
to fully relax the vocal folds. Paralysis of which muscle(s) prevents the relaxation of the
vocal folds?
a. Thyroarytenoid muscles
b. Posterior cricoarytenoid muscles
c. Transverse arytenoid muscles
d. Lateral arytenoid muscle

b: The only muscles that relax the vocal folds are the posterior cricoarytenoid muscles. The other
three muscles listed here close the vocal folds. Note that there are also other effects of anesthesia
that necessitate intubation.

20. A
 ll of the following bones form a part of the lateral nasal wall EXCEPT the:
a. Ethmoid
b. Maxilla
c. Palatine
d. Inferior concha
e. Vomer

e: Vomer

21. All of the following bones form a part of the medial nasal wall EXCEPT the:
a. Ethmoid
b. Maxilla
c. Palatine
d. Inferior concha
e. Vomer

d: The medial wall of the nasal cavity is formed by both bony elements and cartilage. Posteriorly,
the perpendicular plate of the ethmoid bone forms the superior septum, and the vomer forms the
inferior septum. Further posteriorly, the crest of both the maxilla and the palatine bone complete
the posterior septum. The anterior septum is formed entirely of the quadrangular cartilage, which
divides the cavity in the midline. The nasal septum can be deviated in some individuals and is a
sign of nasal trauma or abnormal growth.

22. The action of the medial pterygoid muscle is to:


a. Elevate the mandible.
b. Tense the soft palate.
c. Elevate the soft palate.
d. Elevate the hyoid bone.
e. Retract the mandible.

a: The medial pterygoid muscle forms a sling with the masseter muscle to elevate the mandible.

62
23. The action of the superior part of the temporalis muscle is to:
a. Elevate the mandible.
b. Tense the soft palate.
c. Elevate the soft palate.
d. Elevate the hyoid bone.
e. Retract the mandible.

e: R
 etract the mandible

24. The mandibular division of the trigeminal nerve (V3) passes through the foramen ovale into the:
a. Infratemporal fossa
b. Pterygopalatine fossa
c. Orbit
d. Nasal cavity
e. Oral cavity

a: The mandibular division of the trigeminal nerve enters the infratemporal fossa, where it sends
motor branches to the muscles of mastication and sensory branches to the tongue, mandibular
teeth, oral cavity, and skin of the lower and lateral face.

25. T
 he maxillary division of the trigeminal nerve (V2) passes through the foramen rotundum into the:
a. Infratemporal fossa
b. Pterygopalatine fossa
c. Orbit
d. Nasal cavity
e. Oral cavity

b: Pterygopalatine fossa

26. Which of the following arteries is a branch of the internal carotid artery?
a. Posterior cerebral artery
b. Occipital artery
c. Labyrinthine artery
d. Ophthalmic artery
e. Middle meningeal artery

d: The ophthalmic artery is the first major branch of the internal carotid artery in the anterior
cranial fossa.

27. Which of the following arteries is a direct branch of the external carotid artery?
a. Posterior cerebral artery
b. Occipital artery
c. Labyrinthine artery
d. Ophthalmic artery
e. Middle meningeal artery

b: Occipital artery

63
3 HEAD AND NECK ANATOMY

28. The paranasal sinus most developed at birth is the:


a. Maxillary sinus
b. Ethmoidal sinus
c. Frontal sinus
d. Sphenoidal sinus

a: Averaging 8  4  4 mm, the maxillary sinus is the most developed sinus at birth. The sinus
cavity expands rapidly throughout the first year of life, extending laterally to the infraorbital nerve
(midpupillary line) by 12 months of age. The other sinuses are less developed at birth, with the
frontal sinuses often not developing until adolescence.

29. P
 eriods of expansion of the maxillary sinus cavity correspond to each of the following
events EXCEPT:
a. Eruption of the third molars
b. Eruption of permanent teeth
c. Extraction or loss of maxillary molars
d. Episodes of acute sinusitis

d: The maxillary sinus cavity expands in size throughout childhood, with periods of more rapid
expansion corresponding to dental events including eruption of the primary and permanent teeth,
eruption of the third molars, and extraction or loss of maxillary molars.

30. T
 he roots of which maxillary teeth are closest in proximity to the maxillary sinus cavity?
a. Incisors
b. Canines
c. Premolars
d. Second molars

d: The roots of the maxillary incisors and canines are not typically in close proximity to the sinus
cavity. The roots of the maxillary premolars and molars, however, are consistently just inferior to
the maxillary sinus floor, with the roots of the second molars nearest to the sinus cavity, followed
by those of the first and third molars.

31. The maxillary sinus drains into which of the following intranasal structures?
a. The inferior meatus
b. The middle meatus
c. The superior meatus
d. The sphenoethmoidal recess

 : The maxillary sinus drains into the middle meatus, as do the frontal sinus and the anterior ethmoidal
b
sinuses. The nasolacrimal duct drains into the inferior meatus. The posterior ethmoidal sinuses drain
into the superior meatus. The sphenoidal sinus drains into the sphenoethmoidal recess.

64
32. Creating a lateral window too high on the maxillary sinus wall puts which of the following at risk?
a. The infraorbital nerve
b. The roots of the maxillary molars
c. The posterior superior alveolar artery
d. The natural ostium of the maxillary sinus

a: The infraorbital nerve (second division of the trigeminal nerve [CN V]) exits through the infra-
orbital foramen high on the anterior face of the maxilla. This is located at approximately the
midpupillary line. Care must be taken when elevating the soft tissues superiorly during a lateral
window approach to avoid injury to this nerve, which gives sensation to the upper lip, cheek,
maxillary sinus, and teeth.

33. The sinus membrane is composed of which type of lining?


a. Stratified squamous epithelium
b. Simple columnar epithelium
c. Pseudostratified ciliated columnar epithelium
d. Stratified columnar epithelium

c : The lining of much of the nose and paranasal sinuses (including the maxillary sinus) is composed
of respiratory epithelium, or pseudostratified ciliated columnar epithelium. Healthy sinus lining
is 0.2 to 0.8 mm thick, includes goblet cells, and features a thick lamina propria and a relatively
thin basement membrane.

34. Which of the following is NOT true of the posterior superior alveolar artery?
a. On average, it is found about 11.25 mm above the maxillary crest.
b. It forms an anastomotic loop with the infraorbital artery.
c. It originates from the internal maxillary artery.
d. It is more extraosseous posteriorly and more intraosseous anteriorly.

d: The posterior superior alveolar artery originates from the internal maxillary artery, a branch
of the external carotid artery. It forms an anastomotic loop with the infraorbital artery. It tends
to become more extraosseous as it extends anteriorly, often being found within the elevated
membrane during a lateral window approach.

35. Which of the following is true of bony septa arising from the floor of the maxillary sinus?
a. They are rare, occurring in less than 10% of patients.
b. They are only found in the middle third of the sinus floor.
c. Most septa are tallest at their medial and lateral extents.
d. Most septa run from anterior to posterior.

c: As described by Underwood in 1910, bony septa arising from the floor of the maxillary sinus
are relatively common, occurring in about one-third of patients. The most common orientation is
from medial to lateral, with the tallest portions at the medial and lateral extents, like an inverted
flying buttress. While they are most commonly located in the middle third of the sinus, they can be
found in the anterior and posterior portions as well and may assume a multitude of orientations.

65
3 HEAD AND NECK ANATOMY

36. As viewed from inside the sinus, the natural ostium of the maxillary sinus is located:
a. Low on the medial wall.
b. High on the medial wall.
c. Low on the anterior wall.
d. Low on the posterior wall.

b: High on the medial wall

37. W
 hich of the following is NOT true of accessory maxillary ostia?
a. They may be located posterior or anterior to the natural ostium.
b. They are found in up to 25% of sinuses.
c. They ensure that the sinus will not be diseased.
d. They are a result of incomplete bony formation of the medial maxillary wall.

c: Accessory sinus ostia occur in up to 25% of sinuses. These are formed through incomplete
bony development of the medial maxillary wall and, while typically located posterior to the natural
ostium, may also be found anterior to it. Because the cilia are programmed to sweep mucus to
the natural ostium, the presence of an accessory ostium is largely ignored and does not prevent
development of sinus disease related to natural ostium obstruction.

38. T
 he bilayered secretory blanket lining the sinus membrane consists of which two layers?
a. A superficial “gel” layer and an inner “sol” layer
b. A superficial “sol” layer and an inner “gel” layer
c. A superficial “gel” layer and an inner “mucinous” layer
d. A superficial “sol” layer and an inner “mucinous” layer

a: The secretory blanket consists of an inner “sol” layer, which is serous and rich in proteins,
immunoglobulins, and complement. Floating on this is the more superficial “gel” layer, which
is a viscous mucus that is swept across the respiratory mucosal surface by the action of the cilia.

39. The lateral wall of the adult maxillary sinus is formed by the:
a. Orbital floor
b. Ethmoid bone
c. Zygoma
d. Alveolar portion of the maxilla

c: The zygoma forms the lateral wall of the maxillary sinus. The alveolar portion of the maxilla
forms the floor of the sinus, while the orbital floor doubles as the roof of the sinus.

40. H
 ypoplasia of the maxillary sinus occurs in about what percentage of patients?
a. 6%
b. 19%
c. 25%
d. 43%

a: Hypoplastic maxillary sinuses are relatively rare and are thought to be related to failure of
pneumatization due to deficient bone absorption or obstruction of the sinus. Hypoplasia is most
commonly seen in patients with cystic fibrosis, childhood facial trauma, or maxillary teeth that
fail to erupt.

66
41. Which of the following is NOT true of Haller cells?
a. They are also known as infraorbital ethmoid cells.
b. They pneumatize along the orbit from the anterior ethmoidal sinuses.
c. They may narrow the maxillary sinus ostium and contribute to obstruction.
d. They should always be surgically removed when noted.

d: Ethmoid cells that extend along the orbit into the drainage pathway of the maxillary sinus
are called Haller cells, or infraorbital ethmoid cells. While these may contribute to obstruction
or narrowing of the maxillary sinus drainage pathway, they generally do not cause symptoms.

42. The volume of an average adult maxillary sinus is about:


a. 5–10 mL
b. 10–15 mL
c. 15–20 mL
d. 20–25 mL

c : The adult maxillary sinus cavity is usually the largest of the paranasal sinuses by volume, typically
about 15 to 20 mL. It is roughly a laterally directed pyramid in shape, with average dimensions
of 3.75 cm high by 2.5 cm deep by 3 cm wide.

43. The cilia of the sinus membrane sweep the mucus toward the natural ostium of the maxillary
sinus in a consistent pattern of drainage originating where?
a. The posterior wall of the maxillary sinus
b. The lateral wall of the maxillary sinus
c. The floor of the maxillary sinus
d. The medial wall of the maxillary sinus

c: Mucus is swept across the surface of the sinus membrane at a rate of 3 to 25 mm/minute,
originating from the floor of the sinus and extending in a radial pattern toward the natural ostium
high on the medial maxillary wall.

44. All of the following are true of the uncinate process EXCEPT that:
a. It is crescent-shaped.
b. It forms the lateral boundary of the maxillary sinus drainage pathway.
c. It comes off the lateral wall of the nose at a 30-degree angle.
d. It acts as a bony baffle, protecting the natural ostium of the maxillary sinus.

b: The uncinate process is an extension of the ethmoid bone that projects posteriorly from the
lateral wall of the nose at about a 30-degree angle. It acts as a baffle, protecting the natural
ostium of the maxillary sinus. It also forms the medial boundary of the ethmoid infundibulum, a
bony groove into which the natural ostium of the maxillary sinus drains.

67
3 HEAD AND NECK ANATOMY

45. T
 he sensory innervation of the maxillary sinus, maxillary teeth, lips, and cheek is from
what nerve?
a. First division of the trigeminal nerve (V1)
b. Second division of the trigeminal nerve (V2)
c. Third division of the trigeminal nerve (V3)
d. Greater petrosal nerve

b: The first division of the fifth cranial nerve (V1, or the ophthalmic nerve) gives sensation to the
scalp, forehead, periorbital structures, frontal sinus, and dorsal nose. The second division (V2, or
the maxillary nerve) gives sensation to the cheek, lateral nose, interior nose and sinuses, palate,
and maxillary teeth. The third division (V3, or the mandibular nerve) gives sensation to the lower
lip, mandibular teeth and gums, chin, and jaw. The greater petrosal nerve provides parasympa-
thetic innervation to the sinus mucosa.

46. S
 inus membrane elevation has the following effect on the function of the cilia found on the
floor of the maxillary sinus:
a. No effect
b. Transient dysfunction
c. Mild permanent dysfunction
d. Severe permanent dysfunction

b: Timmenga et al performed endoscopically guided biopsies of the floor of the maxillary
sinus before, 3 months after, and 9 months after sinus elevation and found only transient ciliary
dysfunction. (Source: Timmenga NM, Raghoebar GM, Liems RS, van Weissenbruch R, Manson
WL, Vissink A. Effects of maxillary sinus floor elevation on maxillary sinus physiology. Eur J Oral
Sci 2003;111:189–197.)

47. W
 hat effect does the angle between the buccal alveolus (lateral maxillary wall) and the
palatal alveolus (medial maxillary wall), as viewed from the anterior perspective, have on
membrane perforation rates in a lateral window approach?
a. The narrower the angle, the higher the perforation rate.
b. The wider the angle, the higher the perforation rate.
c. The narrower the angle, the lower the perforation rate.
d. There is no correlation between this angle and the perforation rate.

a: Cho et al found that when this angle was greater than 60 degrees, there were no perforations.
With an angle of 30 to 60 degrees, they found a 29% perforation rate. When the angle was less
than 30 degrees, they found a 63% perforation rate. (Source: Cho SC, Wallace SS, Froum SJ,
Tarnow DP. Influence of anatomy on Schneiderian membrane perforations during sinus elevation
surgery: Three-dimensional analysis. Pract Proced Aesthet Dent 2001;13:160–163.)

68
48. Which of the following is contained within the space indicated by the white arrow?

a. V1
b. Posterior superior alveolar artery
c. V3
d. Infraorbital artery

d: The infraorbital canal contains the infraorbital nerve (part of V2, the trigeminal nerve) and the
infraorbital artery. The infraorbital artery is a branch of the maxillary artery. It runs through the
infraorbital canal and emerges onto the face through the infraorbital foramen, where branches
anastomose with the angular artery and posterior superior alveolar artery, among others.

49. The structure indicated by the white arrow occurs in about what percentage of patients?

a. 12%
b. 33%
c. 62%
d. 97%

b: As described by Underwood in 1910, bony septa arising from the floor of the maxillary sinus
are relatively common, occurring in about one-third of patients.

69
3 HEAD AND NECK ANATOMY

50. D
 uring the evaluation phase for implants, a patient’s cone beam computed tomography
(CBCT) scan reveals an abnormality of the right maxillary sinus. The patient denies any
history of sinus symptoms. Which of the following is the best course of action?

a. Refer immediately to an ear, nose, and throat specialist (ENT); this appears to be a malignancy.
b. Treat with amoxicillin; this is acute sinusitis.
c. Tell the patient that he or she is not a candidate for implants due to incompatible anatomy.
d. Refer to ENT for clearance, then proceed with implant surgery.

 : Maxillary sinus hypoplasia occurs in a small percentage of patients and is frequently


d
asymptomatic. Referral to ENT for nasal endoscopy may detect chronic mucus drainage from the
sinus, which may be correctable. There is no contraindication to implant surgery.

51. The nasal septum receives its blood supply from which of the following vessels?
a. Anterior ethmoidal artery
b. Sphenopalatine artery
c. Greater palatine artery
d. All of the above

d: The nasal septum has tremendous vascular supply, with contributions from the sphenopala-
tine, greater palatine, and superior labial arteries (from the external carotid circulation) and the
anterior and posterior ethmoidal arteries (from the internal carotid circulation). Together these
form a highly vascular area on the anterior nasal septal mucosa called Kiesselbach’s plexus, the
site of origin of about 95% of nosebleeds.

52. C
 ompared with the mucosa of the nasal cavity, the mucosa of the maxillary sinus:
a. Is thinner.
b. Contains more goblet cells.
c. Has areas of squamous mucosa.
d. All of the above

a: The sinus membrane of the maxillary sinus is thinner than the lining of the nasal cavity (usually
so thin that it cannot be seen on a CT scan). The nasal cavity mucosa contains more goblet cells
and thus produces more mucus than the sinus lining. The anterior nasal mucosa transitions to
squamous mucosa and serves a protective function.

70
53. T
 RUE OR FALSE: The pterygoid venous plexus is of special importance to dentists because if the
needle is overinserted during posterior superior alveolar block, it may penetrate the pterygoid
plexus of the vein and the maxillary artery in the infratemporal fossa, thus causing hematoma.

True: The pterygoid venous plexus is situated on the medial side of the mandibular ramus within
the pterygoid muscles. It is linked to the facial vein via the deep facial vein, the retromandibular
vein via the maxillary vein, and the cavernous sinus via the sphenoidal emissary vein. The pterygoid
plexus drains into the jugular veins.
This plexus is of special importance to dentists because if the needle is overinserted during
posterior superior alveolar block, it may penetrate the pterygoid plexus of the vein and the maxil-
lary artery in the infratemporal fossa, thus causing hematoma. This results in extraoral swelling
a few minutes after the injection. The hematoma will cause tissue tenderness and discoloration,
which will last until the blood is broken down by the body, and possible spread of infection to
the cavernous venous sinus if the needle is contaminated. A hematoma can also result during
other blocks, such as the infraorbital and the inferior alveolar blocks. To avoid injection into blood
vessels, aspiration should always be attempted in all injections.

Cavernous Superficial
Deep temporal veins
sinus temporal vein
Supraorbital
vein
Supratrochlear
vein
Superior
Superior and
ophthalmic vein inferior
petrosal
Angular vein sinuses

Sigmoid
sinus

Deep facial vein


Pterygoid plexus
Maxillary vein

Retromandibular vein

Internal jugular vein

Facial Common facial vein


vein External
palatine vein Lingual vein

54. After bone grafting surgery in the anterior maxilla, the patient complained about continued numb-
ness (paresthesia) in the upper lip. What nerve might have been damaged during the procedure?
a. Inferior alveolar nerve
b. Zygomatic nerve
c. Anterior superior alveolar nerve
d. Infraorbital nerve

 : The infraorbital nerve enters the orbit through the inferior orbital fissure (after branching off into
d
the posterior superior alveolar nerves to the molars and the medial superior alveolar nerves); it
traverses the infraorbital groove and canal in the floor of the orbit (where it branches off into the
anterior superior alveolar nerve) and appears upon the face at the infraorbital foramen. Here it is
referred to as the infraorbital nerve, a terminal branch. At its termination, the nerve lies beneath
the quadratus labii superioris and divides into a leash of branches that spread out on the side of
the nose, the lower eyelid (inferior palpebral nerve), and the upper lip (the superior labial nerve),
joining with filaments of the facial nerve.

71
3 HEAD AND NECK ANATOMY

Nasopalatine nerve
Nasopalatine block
Infraorbital nerve
Infraorbital block
Incisive
Middle superior
foramen
alveolar nerve
MSA block
Greater palatine nerve
GP block

Greater
palatine Posterior superior
foramen alveolar nerve
PSA block

Buccal nerve
Buccal block

55. The greater palatine foramen is usually located:


a. Distal to the third molar area.
b. Between the second and third molars.
c. Between the first and second molars.

 : The greater palatine foramen is located between the second and third molars. Bilateral symmetry
b
usually exists between the sides of the skull. Many studies have reported mean distances from the
greater palatine foramen to the midsagittal suture or to the posterior palatal border, but these
numbers might still be inaccurate for a particular patient because of variation in the location of
the greater palatine foramen. Examining a CT scan of the patient will render a definitive location
of the foramen before surgery for the purpose of anesthesia or flap reflection in this region.

56. T
 RUE OR FALSE: The greater palatine neurovascular bundle is localized at the junction of the
vertical and horizontal palatal walls of the vault.

True

57. T
 he buccal fat pad is confined in the masticatory space between:
a. The masseter muscle and the medial pterygoid muscle.
b. The masseter muscle and the lateral pterygoid muscle.
c. The masseter muscle and the temporalis muscle.
d. The masseter muscle and the buccinator muscle.

d: The buccal fat pad is confined in the masticatory space between the masseter muscle laterally
and the buccinator muscle (and sometimes the periosteum of the posterior wall of the maxilla)
medially, with its main body resting on the buccopharyngeal fascia, which covers the external
surface of the buccinator. Its anterior border goes beyond the anterior wall of the masseter and
posterior to the site where the parotid duct pierces the buccinator; thus, it is covered externally
in this area by the parotid duct, the zygomaticus major, the zygomaticus minor, and the superficial
fascia of the face. The posterior limit is the retromolar area of the mandible. The buccal fat pad
is wrapped within a thin fascial envelope.

72
Skin and
subcutaneous fat

Parotid duct
Superficial
muscular
aponeurotic
system
Masseter

Branches of
facial nerve

Buccal fat pad

Buccinator
Mucosa

58. Functions of the buccal fat pad include:


a. Enhancing the sucking capabilities of the buccinator muscle.
b. Filling the deep tissue spaces to act as gliding pads when masticatory and mimetic
muscles contract.
c. Cushioning important structures from the extrusion of muscle contraction or outer
force impulsion.
d. All of the above

d: At birth, the sucking muscles of the lips, orbicularis oris, and cheeks, principally the buccinator,
are relatively better developed than the muscles of mastication. In well-nourished babies, the fat
pad pushes the buccinator inward and forms a prominent elevation on the external surface of the
face. It is believed by some that the sucking capabilities of the buccinator muscle are enhanced by
the fat pad, which may prevent collapse of the cheeks during suction by counteracting negative
pressure. The buccal fat pad also functions to fill the deep tissue spaces, acting as gliding pads
when masticatory and mimetic muscles contract; as such, it represents a specialized type of fat also
known as syssarcosis, or fat that enhances intermuscular motion. In addition, the buccal fat pad
cushions important structures from the extrusion of muscle contraction or outer force impulsion
(external trauma that may injure the facial neurovascular bundle).

59. TRUE OR FALSE: When performing sinus bone augmentation using the crestal approach with
simultaneous implant placement, it is recommended that 5 mm of bony height exist under
the sinus floor.

True: The judgment of the clinician on the treatment planning of the posterior maxilla (and other
areas of the mouth) is paramount, because each clinical case is unique; however, guidelines are
always helpful in keeping practitioners on a path that will most likely be successful. Considering
the poor quality of the bone in the posterior maxilla, the simultaneous crestal bone augmentation
approach with implant placement should not be attempted with less than 5 mm of bony height
so as to minimize the possibility of implant migration into the sinus cavity during the procedure
or postoperatively.

73
3 HEAD AND NECK ANATOMY

60. The incisive/nasopalatine canal contains the:


a. Nasopalatine nerve
b. Anterior branch of the greater palatine artery
c. Sphenopalatine artery
d. Palatine artery
e. a and b
f. a, b, and d
g. All of the above

e: The incisive canal contains the nasopalatine nerve and the anterior branch of the greater pala-
tine artery. There are at least two bundles in the canal because the nerves and the arteries in the
canal originate from a bilateral source.
The nasopalatine nerve is a branch of the posterior superior nasal nerves that arise from the
pterygopalatine ganglionic branches of the maxillary nerve. It runs downward and forward to the
incisive foramen, through which it passes to supply the anterior part of the palate and commu-
nicate in this area with the greater palatine nerve. Therefore, anesthesia for surgeries involving
the anterior maxilla, maxillary central incisors, nasal septum, or the nasal floor can be achieved
by injection into the incisive foramen.
The greater palatine artery emerges from its canal through the greater palatine foramen and
runs across the hard palate to the incisive foramen, through which it enters the nasal cavity and
anastomoses with the sphenopalatine artery on the nasal septum and sometimes in the canal itself.

Frontal sinus

Ethmoid cell

Median lamina,
ethmoid bone
Nasal septum

Nasolacrimal duct

Nasal
Maxillary sinus septum
Nasal
Piriform aperture floor Stensen
foramen

Nasal cavity
1
⁄5
Anterior nasal spine 4
⁄5

Hard palate
Oral roof
Incisive/nasopalatine
canal

Main trunks of nasopalatine nerve and


greater palatine artery traverse posteriorly
while minor branches supply gingiva/mucosa
of anterior palate

74
61. TRUE OR FALSE: When indicated for ideal prosthetic planning, excising the
incisive/nasopalatine nerve and the blood vessels of the incisive canal and subsequently
placing bone graft material for immediate or delayed implant placement is a viable
technique that can be performed without detriment to the patient.

True: In some cases, the location of the incisive canal may prevent the placement of dental
implants to replace missing maxillary central incisors. The nerve and the artery of the incisive canal
anastomose with the greater palatine artery and nerve, permitting immediate revascularization
and a gradual reinnervation of the region within 3 to 6 months. Nonetheless, loss of sensation in
the anterior palate is a possibility, and patients should be warned about it, although this is rarely
a cause of patient complaint.

62. Although the buccolingual position of the inferior alveolar canal in the alveolar bone is
variable, the distance from the canal to the medial aspect of the buccal cortical plate
(medullary bone thickness) was found to be greatest at the:
a. Mesial half of the first molar.
b. Distal half of the first molar.
c. Mesial half of the second molar.
d. Distal half of the second molar.

b: Distal half of the first molar (mean: 4 mm). When larger ramus block grafts are planned, the
anterior vertical cut should be made in this area (distal half of the first molar). The mean vertical
distance between the superior edge of the canal and the cortical surface along the external
oblique ridge is 7 mm in the second molar region, 11 mm in the third molar region, and 14 mm
at the base of the coronoid process. Therefore, when a smaller graft size is required, the harvest
may be made higher on the ramus, which is usually further from the canal.

Temporalis

External Superior External


oblique ridge ramus cut oblique cut
Buccal cortex
3.5 mm

Masseter

4 mm

Buccal notch
Inferior Anterior body cut
ramus cut

75
3 HEAD AND NECK ANATOMY

63. TRUE OR FALSE: To avoid cutting the lingual nerve, the distal releasing incision in the retro-
molar (triangle) pad area should be 10 degrees toward the buccal (not straight), because the
nerve might be laying on the retromolar pad.

False: To avoid cutting the lingual nerve, the distal releasing incision in the retromolar (triangle)
pad area should be 30 degrees toward the buccal (not straight), because the nerve might be
laying on the retromolar pad. In addition, the lingual side of the flap in the posterior mandibular
region should be reflected carefully and gently, and lingual releasing incisions should be avoided.

Condyle

Angle of
flare
Lingula

Coronoid
process
Lingual nerve
Retromolar pad
External
oblique Angle of
ridge mandible Superior view of the mandible, show-
ing the angle of flare that starts imme-
diately posterior to the third molar, the
two different arches formed by the
teeth and the bony structure of the
mandible, and the possible location of
Curvature of the arch
the lingual nerve on the crestal ridge
Curvature of the arch
(bone level) (tooth level) distal to the third molar toward the
lingual plate.

64. Cutting the lingual nerve will:


a. Anesthetize the tongue.
b. Decrease salivary flow from the submandibular gland.
c. Affect the patient’s sense of taste.
d. All of the above
e. None of the above

d: All of the above

65. TRUE OR FALSE: Unlike the maxillary and the ophthalmic nerves (both entirely sensory),
the mandibular nerve has both sensory and motor divisions.

True: The inferior alveolar nerve carries motor fibers for the mylohyoid muscle and the anterior belly
of the digastric muscle and sensory fibers that enter the canal through the mandibular foramen,
giving inferior dental branches to the mandibular teeth and exiting through the mental foramen
under the name mental nerve. Damaging the inferior alveolar nerve will alter the sensation to
areas supplied by it and by the mental nerve.

76
66. Inferior alveolar nerve injury can occur:
a. During local anesthesia (needle penetration).
b. During incision (by scalpel).
c. During flap reflection (by stretching its mental branch).
d. During osteotomy preparation (by drills).
e. During implant insertion (by compression).
f. All of the above

f: Dental specialists and general practitioners who place implants must discuss the possibility of
nerve injury with the patient and include this possibility in the informed consent form. Detailed
knowledge of the related anatomy, careful planning using CT scan images and diagnostic wax-ups,
the use of all available tools to perform the surgery accurately (ie, drill stoppers, computer-
generated surgical guides), and careful manipulation of soft tissue can help in minimizing to a
great extent the incidence of nerve injury.

67. Why is it important not to perforate the lingual plate of the posterior mandible during
implant surgeries?
a. Possible nerve trauma
b. Possible muscle trauma
c. Possible arterial trauma

c : It is important not to perforate the lingual plate of the posterior mandible during implant surger-
ies because the floor of the mouth is a highly vascularized region. Perforation of the lingual cortical
plate of the posterior mandible in the region of the submandibular fossa by instrumentation or
a drill may cause an arterial trauma resulting in a hemorrhage that may commence immediately
or with some delay after the vascular insult. The progressively expanding lingual, sublingual,
submandibular, and submental hematomas have the tendency of displacing the tongue and the
floor of the mouth to obstruct the airway. This possibility is a rare but potentially fatal complication
of implant surgery. To avoid this possibility, detailed knowledge of the regional arterial anatomy
is imperative for the implant surgeon.

68. Which blood vessel will be injured if the submandibular fossa is perforated during implant
osteotomy preparation with a drill?
a. Facial artery or one of its branches
b. Lingual artery or one of its branches
c. Maxillary artery or one of its branches
d. a and b
e. b and c
f. All of the above

d: The anatomy of the lingual aspect of the mandible cannot be observed on the panoramic
radiograph, nor can it be visualized because the mylohyoid muscle is located above it. Palpation
can help during evaluation of the lingual aspect anatomy; however, a CT scan is best for revealing
the shape of the mandible on its lingual aspect.

77
3 HEAD AND NECK ANATOMY

69. If the submandibular fossa is perforated with subsequent bleeding, what should be the first
action taken by the operator?
a. Stop the bleeding.
b. Keep the airway open.
c. a and b
d. None of the above, as the bleeding is not life threatening and the airway will not be affected
by this complication.

 : Securing and maintaining an adequate airway should be given the highest priority. The implant
b
surgeon should be prepared to deal with the possibility of airway obstruction, which can occur
very rapidly. The clinical signs of airway obstruction (tachypnea, dyspnea, hoarseness, cyanosis,
drooling) can be hidden until there is significant airway occlusion. Persistent intraoral bleeding can
cause mechanical pressure to the pharyngeal lumen and consequent airway obstruction, which
poses a serious threat to the patient’s life. The airway can be secured by nasotracheal or orotra-
cheal intubation or emergency tracheotomy or cricothyrotomy (when endotracheal intubation is
impossible due to extensive hematoma). Manual tongue decompression and tactile intubation
have been successful in one report during hemorrhagic swelling of the tongue. (Source: Piper SN,
Maleck WH, Kumle B, Deschner E, Boldt J. Massive postoperative swelling of the tongue: Manual
decompression and tactile intubation as a life-saving measure. Resuscitation 2000;43:217–220.)
As for bleeding management, many bleeding control measures have been used and described
in the literature, including the use of hemostatic agents, digital compression, and cauterization.
Also, bleeding may eventually stop when the pressure of the extravasated blood exceeds the
vascular pressure of the feeding hemorrhage; thus, hematoma drainage might have a reverse
effect by lowering the pressure of the adjacent soft tissues and hence promoting further drain-
age. Monitoring in anticipation of hemorrhage self-resolution was successfully reported in a few
cases. (Source: Kalpidis CD, Setayesh RM. Hemorrhaging associated with endosseous implant
placement in the anterior mandible: A review of the literature. J Periodontol 2004;75:631–645.)
When conservative measures are ineffective, intraoral or extraoral surgical evacuation and ligation
of the bleeding artery is necessary.

70. P
 revention measures for arterial injury to the floor of the mouth should include but are not
limited to:
a. Good knowledge of the fine regional arterial anatomy.
b. Obtaining a good-quality panoramic radiograph.
c. Appropriate clinical procedures, which must be carried out with the utmost attention,
because the simplest implant procedure can trigger extremely serious complications.
d. Proper selection of the diameter and length of the implant per the available alveolar ridge
as well as the appropriate angulation of the implant.
e. Adequate training in implant surgery and in medical emergencies.
f. Thorough evaluation of the lingual side of the mandible, including the submandibular
fossa and the accessory lingual foramina.
g. Digital palpation of the lingual mandibular surface, which can help to detect any
pronounced concavity in the anterior or posterior areas of the mandible. CT scanning is
highly recommended.
h. All of the above except b and f
i. All of the above

i: All of the above

78
71. The mental nerve provides sensory supply to the:
a. Chin
b. Lower lip
c. Labial mucosa near the mandibular anterior teeth
d. Lingual mucosa near the mandibular anterior teeth
e. Skin over the body of the mandible
f. Skin over the body of the maxilla
g. a, b, c, and e
h. All of the above except f
i. All of the above

g: The mental nerve exits the body of the mandible through the mental foramen, which is usually
located between the apices of the first and second mandibular premolars. As such, it provides
sensory supply to the chin, lower lip, labial mucosa near the mandibular anterior teeth, and the
skin over the body of the mandible.

72. The inferior alveolar nerve usually courses anterior to the mental foramen, turning posteriorly
and superiorly to exit the mental foramen. Because of this usual path (anterior loop), the
nerve may be as much as:
a. 3 mm anterior to the mental foramen.
b. 5 mm anterior to the mental foramen.
c. 7 mm anterior to the mental foramen.

a: Because the nerve may be as much as 3 mm anterior to the mental foramen, if an implant is to
be placed mesial to and below the level of the foramen, the most posterior extent of the implant
should be at least 5 mm anterior to the mesial aspect of the foramen to avoid drill penetration
though the anterior loop. The pilot drill should penetrate the crestal bone 7 to 8 mm anterior to
the most mesial aspect of the mental foramen (3 mm for the loop + 2 mm of safety zone + the
radius of the implant [R]).

Pilot drill

5 mm + R

79
3 HEAD AND NECK ANATOMY

73. In a mandible with extensive resorption, the position of the mental foramen might be on the
crest of the ridge. In such a case, care should be taken not to harm the mental nerve, which
can be accomplished by placing the crestal incision:
a. Slightly toward the lingual.
b. Slightly toward the buccal.
c. In the middle of the ridge. The crestal incision must always be midcrestal.

a: When the mental foramen lies on the crest of the ridge, the crestal incision should be placed
slightly toward the lingual and a full-thickness flap gently reflected until the foramen is identified.
In some scenarios, it is recommended not to open a flap at all but rather to follow a flapless inser-
tion protocol to avoid damaging the mental nerve or any of its branches by the flap reflection.

Mental foramen

74. T
 RUE OR FALSE: The height of the mental foramen can be used as available bone height
without surgical risk because the inferior alveolar nerve always rises as it approaches the
mental foramen (compared with its height in the molar region).

True: Even if the implant is placed all the way to the superior border of the mental foramen
(as seen on a panoramic radiograph; A), in reality it would be located lingual to it (B).

B A

80
75. TRUE OR FALSE: Flap-releasing incisions mesial to the mental nerve should terminate
5 mm inferior to the mucogingival junction to avoid possible injury to major branches of
the mental nerve.

False: Flap-releasing incisions mesial to the mental nerve should terminate just superior to the
mucogingival junction to avoid possible injury to major branches of the mental nerve.

76. TRUE OR FALSE: The existence of the mandibular incisive canal in the anterior mandible can
be problematic; as an extension of the inferior mandibular nerve, it should be considered to
contain the same neurovascular elements, and thus osteotomies should not be made through
this canal. Damage to the mandibular incisive nerve will result in paresthesia of the corner of
the lip.

True: At the vicinity of the molars, the inferior alveolar nerve typically splits into the mental nerve
(to supply the skin of the mental foraminal region, the lower lip, the mucous membrane, and the
gingiva) and the incisive nerves (to supply the mandibular anterior teeth). However, in some cases,
the incisive nerve might present as a true canal with large lumen (0.48 to 2.90 mm) that extends
anteriorly and inferiorly from the mental foramen, located 8 to 10 mm from the inferior border of
the mandible. This canal can be problematic because, as an extension of the inferior mandibular
nerve, it should be considered to contain the same neurovascular elements; thus, osteotomies
should not be made through this canal.
The incisive canal cannot be detected clearly on conventional radiography, so CT scans are
recommended for proper assessment. It is important to note that in the resorbed mandible, the
position of the incisive canal (if it is present) is expected to be closer to the alveolar crest.
In conclusion, the incisive canal should always be taken into consideration when planning for
implant placement in the intraforaminal area.

81
3 HEAD AND NECK ANATOMY

77. A
 fter undergoing implant surgery in the anterior mandible, the patient developed hematoma
in the sublingual region. This is usually the result of:
a. Damage to the submental artery.
b. Damage to the lingual artery.
c. Damage to the sublingual artery.
d. Damage to the mylohyoid artery.

c: The inferior alveolar arteries supply the symphysis of the mandible by the incisive arteries;
however, an additional blood supply to the intercanine area is derived from the alveolar branches
of the sublingual artery through the accessory lingual mandibular foramina. This complementary
blood supply is especially important in edentulous mandibles, because arteriosclerotic changes of
the inferior alveolar artery after tooth loss make the blood circulation in the mandible increasingly
dependent on the external blood supply provided by the periosteum and the accessory lingual
canals. This fact should be taken into consideration when performing extensive reflection of lingual
mucoperiosteal flaps or when drilling close to the midline of the mandible. Disruption in the blood
supply to the anterior mandible can cause hematoma in the sublingual region; several of the arter-
ies associated with accessory lingual mandibular foramina are of sufficient size to be implicated
in severe hemorrhaging episodes during implant placement in the mandibular anterior region.

78. W
 hen harvesting a bone block from the chin of a dentate patient, what is the minimum
recommended clearance between the superior cut and the apices of the mandibular
anterior teeth?
a. 3 mm
b. 4 mm
c. 5 mm
d. 6 mm

c : To avoid damage to the anterior teeth and prevent disruption to their innervation, the minimum
recommended clearance between the superior cut and the apices of the anterior mandibular
teeth is 5 mm.

82
5-mm minimum
between the
superior border of
the block and the
apices of the roots

3-mm minimum from


the inferior border of
5-mm minimum the mandible to the
from the mesial inferior border of the
border of the chin block
mental foramen

79. TRUE OR FALSE: Chin block graft harvest should not be attempted in D1 bone.

True: D1 bone will be impossible to harvest without complications (eg, possible through-and-
through window formation upon harvesting or even mandibular fracture).

80. TRUE OR FALSE: The pulmonary veins in the chest carry oxygenated blood from the lungs to
the left side of the heart.
a. True. Although arteries usually carry oxygenated blood, the pulmonary veins are an exception.
b. False. Veins carry only deoxygenated blood.

a: The majority of veins in the body carry deoxygenated (blue) blood and return blood to the
heart from the various organs and tissues of the body. However, there are two exceptions: (1) The
pulmonary veins in the chest carry oxygenated blood from the lungs to the left side of the heart.
(2) The portal vein carries nutrient-rich blood from the intestines to the liver.

81. The primary vein(s) for venipuncture in the antecubital fossa are the:
a. Cephalic vein
b. Basilic vein
c. Median vein
d. a and b only
e. All of the above

e: The cephalic vein, basilic vein, and median vein (which divides into the median basilic and
median cephalic) are all primary veins in the antecubital fossa for venipuncture. The median basilic
vein is the largest and most common for phlebotomy.

83
3 HEAD AND NECK ANATOMY

82. T
 he primary vein(s) for venipuncture in the forearm are the:
a. Cephalic vein
b. Basilic vein
c. Median vein
d. a and b only
e. All of the above

d: a and b only

Brachial a.

Median nerve Cephalic v.

Basilic v.

Median Median
basilic v. cephalic v.
Median
cubital v.

Median Median
antebrachial v. antebrachical
cephalic v.
Antebrachial
basilic v. Median
antebrachial
basilic v.
Ulnar a.
Radial a.

83. T
 RUE OR FALSE: Veins do not pulsate.

True

84. T
 RUE OR FALSE: Just like arteries, veins do not collapse.

False: Unlike arteries, veins collapse when they are empty. When they are lower than the heart,
they become full of blood and stand out beneath the skin. When the arm is raised, the veins
normally collapse at a level approximately 9 cm above the level of the heart. Raising an injured
part of the body above the level of the heart will stop bleeding from veins but not from arteries.

Veins collapsed

Veins collapsing

Veins congested

84
85. TRUE OR FALSE: All veins have valves.

False: Most but not all veins (including in the venae cavae and head) have valves inside them
to prevent the backflow of blood. Failure of these venous valves permits backflow between
movements. As a sequence, the venous pressure increases and the veins grow large, a condition
known as varicose veins.

Vein

Orifice
of small
tributary Venous
branch

Cusp

86. TRUE OR FALSE: Blood supply to the nasal cavity is received from both the internal carotid
and the external carotid arteries.

True: The nasal cavity receives its blood supply through the internal carotid artery via the ante-
rior and posterior ethmoid arteries (branches of the ophthalmic artery) and through the external
carotid artery via the sphenopalatine and greater palatine arteries, all of which are branches of
the maxillary artery.

85
SURGICAL 4
PROCEDURES AND
COMPLICATIONS

Because implants are now a widely accepted therapy,


complications with this treatment modality will be
encountered with increasing frequency. Consequently,
comprehensive knowledge of the etiology of implant-
related complications is required to implement proper
prevention and management protocols.

87
4 SURGICAL PROCEDURES AND COMPLICATIONS

1. TRUE OR FALSE: Vertical restorative space refers to the distance from the crest of the
residual alveolar ridge to the occlusal plane of the restoration or the opposing dentition.
This space influences the type of prosthesis and the restorative material but does not
influence the surgical technique.

False: The vertical restorative space is the distance from the crest of the residual alveolar ridge to
the occlusal plane of the restoration or the opposing dentition. This space will influence the type
of prosthesis, the restorative material, and the surgical technique. Therefore, it should be carefully
measured clinically (intraorally) or on the properly articulated diagnostic wax-up or master casts
before surgery. Soft tissue thickness should also be identified prior to surgery because thick soft
tissue might lead to more vertical space than anticipated.

2. The minimum vertical restorative space required for the fabrication of a single posterior
cement-retained porcelain implant prosthesis is:
a. 7 mm
b. 9 mm
c. 11 mm
d. 13 mm

b: 9 mm (or 6 mm from the soft tissue level to the occlusal plane). The cement, ceramic core, and
porcelain are together 3 mm in height, the abutment height is 5 mm (1 mm of which is usually
under the soft tissue), and the peri-implant soft tissue is 2 mm in height. (For an anterior crown,
this distance should be 1 to 2 mm longer because of the need for a longer abutment for proper
retention.)
If a metal occlusal material is used, the vertical restorative space can be as low as 8 mm for a
cement-retained prosthesis and 5 to 6 mm for a screw-retained prosthesis.

3. T
 he minimum vertical restorative space required for the fabrication of a bar overdenture
implant prosthesis is:
a. 14 mm
b. 15 mm
c. 17 mm
d. 20 mm

c : Depending on the type of attachment, a bar overdenture requires a minimum of 17 mm of verti-


cal restorative space. The space is divided as follows: 3 mm for bone and soft tissue, 1 mm from
the soft tissue to the bar (necessary for proper hygiene), and 5 mm of bar height. The remaining
8 mm or more is the minimum distance required between the superior edge of the bar and the
incisal edge for the acrylic material.

4. The minimum vertical restorative space required for the fabrication of a ball/Locator
overdenture implant prosthesis is:
a. 14 mm
b. 15 mm
c. 17 mm
d. 20 mm

a: 14 mm (usually 3 to 4 mm less than the space needed for a bar overdenture). The space is
divided as follows: 3 mm for bone and soft tissue, 2.5 to 3.5 mm for the abutment, and 8 mm or
more between the top of the ball/Locator abutment cap and the incisal edge.

88
5. Excessive vertical restorative space leads to excessive vertical cantilever. Solutions to this
problem include all of the following EXCEPT:
a. Block grafting
b. Guided bone regeneration
c. Distraction osteogenesis
d. Alveoplasty
e. Vertical interpositional osteotomy

d: Alveoplasty leads to removal of hard tissue and is a solution for vertical restorative space
deficiency.

6. What is the recommended distance edge to edge between two implants when the planned
restoration is a fixed prosthesis?
a. 2 mm
b. 3 mm
c. 4 mm
d. 5 mm

b: The distance between implants (edge to edge) should not be less than 3 mm. When this
distance is maintained, any vertical bone loss resulting from crestal bone remodeling (due to the
establishment of the biologic width at the implant-abutment interface) will not influence the bone
height between the adjacent implants or subsequently the level of the soft tissue (the papilla).

7. What is the recommended distance edge to edge between an implant and an adjacent
natural tooth when the planned restoration is a fixed prosthesis?
a. 2 mm
b. 3 mm
c. 4 mm
d. 5 mm

a: Implants should be placed 1.5 to 2 mm edge to edge from the adjacent root surface. This
distance allows the bone on the adjacent tooth to stay at its current level. Also, if bone loss around
the implant occurs, it will not affect the adjacent tooth and vice versa.

8. What is the minimum recommended distance that must be kept between an implant and its
adjacent root?
a. 0.5 mm
b. 1.0 mm
c. 1.5 mm
d. 2.0 mm

b: 1.0 mm

89
4 SURGICAL PROCEDURES AND COMPLICATIONS

9. Management of deficient horizontal restorative space includes:


a. Orthodontic treatment
b. Enameloplasty
c. Selecting small-diameter implants
d. Using mini-implants
e. b, c, and d
f. All of the above

f: All of the above

10. F
 or the last few years, applications for narrow-platform (or mini-) implants have included:
a. Stabilization of full-arch dentures
b. Single tooth replacement in the compromised intertooth space
c. Single tooth replacement in the compromised interroot space
d. Orthodontic anchorage
e. Tooth replacement for patients requiring minimally invasive surgery due to health concerns
f. All of the above except d
g. All of the above

g: All of the above. However, care should be taken to carefully analyze the biomechanical risk
factors before using narrow-diameter implants.

11. W
 hat is the optimal mouth opening—the distance between the maxillary and mandibular
incisal edges—for the placement of implants in the posterior region?
a. 35 mm or more
b. 40 mm or more
c. 45 mm or more

 : The mouth opening should be checked before the intraoral examination begins. Normal opening
b
should be 40 mm from the maxillary incisal edge to the mandibular incisal edge.
If the mouth opening is less than 40 mm, the implant surgeon might encounter difficulties when
placing implants in the posterior region of the mouth. Placement of implants in the posterior region
with limited mouth opening will lead to implants placed with excessive angulation.

12. T
 RUE OR FALSE: When evaluating the patient for implant therapy, attention must be paid to
hypererupted teeth as they could interfere with implant placement.

True: Hypererupted opposing teeth could interfere with implant placement even if the mouth
opening is adequate. In these scenarios, the occlusal plane of extruded teeth should be corrected
by enameloplasty, orthodontic intrusion, or extraction of these teeth if they are hopeless.

13. T
 RUE OR FALSE: Buccolingually, the implant should be placed under the central fossa of the
future prosthesis. This is true for both screw-retained and cement-retained implant prostheses.

True: Endosseous root form implants distribute occlusal load best when it is applied in an axial
direction. In the posterior region of the mouth, occlusion should be directed toward the center/
long axis of the implant, so the center of the implant (and therefore the pilot hole) should be
located under the central fossa of the future implant crown.

90
14. TRUE OR FALSE: Positioning of the implant in the anterior region depends on the type of the
prosthesis to be fabricated (screw-retained versus cement-retained). In the case of a cement-
retained prosthesis, the center of the implant (the pilot hole) should be located under the
cingulum of the future implant crown. However, if a screw-retained prosthesis is desired, the
center of the implant should be located under the incisal edge of the future crown.

False: Unlike the buccolingual positioning in the posterior region (where the implant is always
under the central fossa of the implant crown), the positioning of the implant in the anterior
region depends on the type of prosthesis to be fabricated (screw-retained versus cement-
retained). In the case of a cement-retained prosthesis, the center of the implant (the pilot hole)
should be located under the incisal edge of the future implant crown. This is because the cement-
retained abutment is large (necessary for retention), and thus placing the implant under the cingu-
lum would make the implant crown very bulky. However, if a screw-retained prosthesis is desired,
the center of the implant should be located under the cingulum of the future crown. This is because
the access for the abutment screw should not compromise the esthetics of the implant prosthesis.

15. Surgical solutions for alveolar ridge width deficiency include all of the following EXCEPT:
a. Block grafting
b. Guided bone regeneration
c. Distraction osteogenesis
d. Alveoplasty
e. Ridge expansion using the split-cortical technique

c: Distraction osteogenesis

16. In order to minimize the sloughing of the mucosal flap during the postoperative healing
period following extensive alveoplasty or the removal of a palatal tori, the operator should
do which of the following?
a. Place mild pressure on the incision line for some time after suturing.
b. Ensure primary closure.
c. Trim away any friable or macerated tissues during closure.
d. Ask the patient to abstain from using the interim prosthesis for a week.

c: Trim away any friable or macerated tissues during closure

17. Buccolingually, the natural teeth are perpendicular to the curve of:
a. Spee
b. Wilson

b: Wilson

18. Mesiodistally, the natural teeth are perpendicular to the curve of:
a. Spee
b. Wilson

a: Spee

91
4 SURGICAL PROCEDURES AND COMPLICATIONS

19. W
 hen placing multiple implants, what surgical tool can you use to align the implants parallel
to each other?
a. Force direction indicator
b. Drill extender
c. Guide drill

a: A force direction indicator (or parallel pin) can be used to help in paralleling the implants to
each other during osteotomy formation.

20. W
 hat is hypoesthesia?
a. Abnormal sensation
b. Increased sensation
c. Reduced sensation
d. Painful sensation

c: Reduced sensation

21. W
 hat is hyperesthesia?
a. Abnormal sensation
b. Increased sensation
c. Reduced sensation
d. Painful sensation

b: Increased sensation

22. What is dysesthesia?


a. Abnormal sensation
b. Increased sensation
c. Reduced sensation
d. Painful sensation

d: Painful (or unpleasant) sensation

23. What is paresthesia?


a. Abnormal sensation
b. Increased sensation
c. Reduced sensation
d. Painful sensation

a: Abnormal sensation

92
24. What is the safe distance you should keep between the tip of the drill and the superior bor-
der of the inferior alveolar nerve or the mental nerve?
a. 1 mm
b. 2 mm
c. 3 mm
d. 4 mm

b: Maintaining a safe distance of at least 2 mm between the apical end of the drill and the supe-
rior aspect of the inferior alveolar canal or the mental nerve will minimize the possibility of nerve
injury during osteotomy preparation.

25. TRUE OR FALSE: Implant drills are usually up to 1 mm longer than the length printed on them.

True: Implant manufacturers do not include the length of the cutting tip of the drills in their
measurements. These tips can be up to a full millimeter long and can cause damage to nerves if
the operator does not account for them.

26. All of the followings steps can be useful in minimizing the possibility of injury to the inferior
alveolar nerve during implant surgery EXCEPT:
a. Obtaining a panoramic radiograph before surgery
b. Using drill stoppers when available
c. Using computer-generated depth-controlled surgical guides
d. Maintaining a safe distance from the nerve when drilling

a: Panoramic radiographs are not reliable. Before surgery, the implant surgeon should obtain a
computed tomography (CT) scan and determine the exact distance available between the superior
border of the inferior alveolar canal and the crestal bone.

27. To prevent lingual nerve injury during surgeries in the posterior mandible,
the operator should:
a. Avoid cutting the lingual nerve, and the distal releasing incision in the retromolar (triangle)
pad area should be 30 degrees toward the buccal (not straight), because the nerve might
be laying on the retromolar pad.
b. Carefully and gently reflect the lingual side of the flap in the posterior mandible.
c. Avoid lingual releasing incisions.
d. a and b
e. All of the above

e: All of the above

28. TRUE OR FALSE: When performing alveoplasty to gain a wider crest, the operator should
evaluate the shape of the alveolar ridge on a CT scan to ensure that the narrow crest is
limited to the coronal portion of the ridge and not affecting the entire ridge.

True: Often once the flap is reflected, the operator notices an irregular, sharp, and/or too-
narrow alveolar ridge (arch form alveolar crest). In these scenarios, it is recommended to
perform an alveoplasty procedure before the placement of implants to make the alveolar
ridge smoother and to gain a wider plateau to accommodate enough width for the desired

93
4 SURGICAL PROCEDURES AND COMPLICATIONS

implants. However, in some cases, alveoplasty does not help in achieving these goals
because the ridge is narrow along its entire length. The figure below at left shows a cross-
sectional CT scan of an anterior maxilla in which alveoplasty would not be beneficial in gaining
more width at the crestal bone level for implant insertion (advanced resorption). The figure on
the right shows a different type of resorption (pyramidal) for which alveoplasty would make a
difference in the width at the crest for implant insertion.

29. W
 hat tool should you use to correct the curvature of a socket before placing an implant?
a. Dense bone drill
b. Lindemann drill
c. Tapping drill

b: The Lindemann drill is a side-cutting drill and is used to correct an angled osteotomy or curved
sockets before complete formation of the implant osteotomy.

30. Placement of an implant too close to a tooth can:


a. Impinge on its blood supply.
b. Impinge on its nerve supply.
c. Overheat the surrounding bone during osteotomy preparation.
d. b and c only
e. All of the above

e: If the implant is too close to a tooth, it might damage it by impinging on its blood supply or
its innervation or by overheating the bone around it during the osteotomy, causing the tooth to
become nonvital due to irreversible pulpal damage. In this case, the tooth will need endodontic
therapy, an apicoectomy, or even extraction, and the implant also might need to be removed.

31. T
 RUE OR FALSE: If pulpal damage to the adjacent tooth is suspected after implant
placement, endodontic therapy should be initiated, and the implant should be removed
if it penetrated the root of the affected tooth.

 rue: Patients with teeth damaged via implant placement complain about severe pain, swelling, and
T
temperature sensitivity soon after the implant placement or even a full month or more later. Once
a tooth becomes nonvital, it will react slightly or strongly to percussion but will give no response
to thermal and electric pulp testing. A radiograph will reveal a radiolucency at the apex of the root
within a short time following the damage via implant placement. In this case, the tooth will need
endodontic therapy, an apicoectomy, or even extraction, and the implant also might need to be
removed. The implant should certainly be removed if it penetrated the root of the affected tooth.

94
32. Damage to adjacent teeth can be avoided during implant placement by following all of these
steps EXCEPT:
a. Carefully assessing the space of the edentulous area on CT images
b. Taking a periapical radiograph with the parallel pin in the osteotomy site immediately after
the pilot drill to adjust the position and angulation of the osteotomy as early as possible
c. Using orthodontic therapy to make the edentulous space bigger if it is too narrow
d. Using a lower speed when drilling to generate less heat at the osteotomy site
e. Using computer-generated surgical guides to direct the osteotomies in proper directions
when the teeth are in close proximity to the planned implants

d: The speed of the drill does not influence the heat-production level as long as adequate irri-
gation is provided.

33. TRUE OR FALSE: Both acute and chronic infections in the implant osteotomy site present a
contraindication for implant placement.

False: All acute infections in or near the planned implant site must be treated before implant
placement. Implant placement should be delayed 2 to 4 months following treatment for an acute
infection, waiting until all symptoms have disappeared. Chronic infection on the contralateral
side does not present a contraindication for implant placement as long as the infection can be
completely removed before the implant is placed.

Acute infection. Symptoms include


tenderness, redness, swelling, suppu-
ration, and possibly the presence of
a fistula.

34. TRUE OR FALSE: A retained root tip at the implant site may serve as a source of infection,
which may lead to an implant periapical lesion or retrograde peri-implantitis.

True: If a retained root tip is suspected in the insertion site prior to implant placement, a CT scan
should be obtained to navigate its exact location and size. The root tip should then be removed
and a guided bone regeneration grafting technique performed to treat the area. Implant place-
ment can proceed 2 to 4 months after the removal of the retained root tip. In some cases, implant
placement can be performed at the same time as removal of the retained root tip.

35. TRUE OR FALSE: To keep the heat production low at the implant osteotomy site,
the operator should drill with a speed less than 1,200 rpm.

False: The speed of the drill does not affect the heat production as long as proper irrigation and
other conditions are met (see question #36).

95
4 SURGICAL PROCEDURES AND COMPLICATIONS

36. T
 o keep the heat production low during osteotomy preparation, the operator must do all of
the following EXCEPT:
a. Use sharp drills.
b. Use a sequential drilling technique.
c. Use an in-and-out motion during drilling.
d. Use an adequate amount of coolant.
e. Drill at a speed between 800 and 1,200 rpm.

e: The speed of the drill does not affect the heat production as long as proper irrigation and the
other conditions listed here are met.

37. W
 hich of the following is a symptom of a failed implant due to bone overheating?
a. Pain
b. Inflammation
c. Radiolucency on a periapical radiograph
d. Bacteria around the implant
e. Necrotic bone around the implant
f. All of the above except a
g. All of the above

g: The microscopic picture of a failed implant due to overheating of the surgical site has the
following features: (1) presence of bone sequestra, (2) no regeneration of the peri-implant bone,
(3) presence of an inflammatory infiltrate in the gap between bone and implant, (4) no organization
of the peri-implant bone clot, (5) presence of compact and mature bone around the implant, and
(6) presence of bacteria around the implant and the necrotic bone.

38. F
 ailed implants due to bone overheating are treated by which of the following?
a. Prescribing systemic antibiotics only
b. Performing a bone grafting procedure (guided bone regeneration)
c. Removing the implant

c: The implant should be removed as well as the necrotic bone. The patient should be put
on antibiotics and prescribed anti-inflammatory and pain medications, and the area should be
monitored for proper healing. Bone grafting and implant placement can be reattempted after
the bone heals (within 3 to 4 months).

39. S
 tripping of the implant site can occur in:
a. D
 ense bone when the clinician attempts to seat the implant deeper than the
prepared osteotomy site.
b. Soft bone when the clinician attempts to seat the implant deeper than the prepared
osteotomy site.

a: During insertion of the implant, the torque will reach high levels and then suddenly become
very low, indicating that stripping has occurred. If this happens, the implant should be removed.

96
40. TRUE OR FALSE: Displacement of implants into the sinus cavity can occur at any time
postoperatively if the bone under the sinuses has poor density and there is minimal
alveolar bony height.

True: Complete displacement of dental implants into the maxillary sinus can occur during implant
placement or as a late postoperative complication. Guler and Delilbasi reported a case in which
an implant migrated into the sinus cavity 8 years after placement. (Source: Guler N, Delilbasi C.
Ectopic dental implants in the maxillary sinus. Quintessence Int 2007;38:e238–e239.) This risk is
higher when implants are being placed simultaneously with sinus augmentation due to the usual
poor bone quality/density and lack of sufficient bone volume for proper stability under the sinus
when the sinus elevation is performed.

41. If an implant migrates into the maxillary sinus:


a. It does not have to be removed but must be kept under observation.
b. It should be removed.

b: If the implant migrates into the maxillary sinus, it should be removed in order to avoid sinus
pathology.

42. In the non-esthetic zone, the implant platform should be placed how far beneath the
gingival level?
a. 1–2 mm
b. 2–3 mm
c. 3–4 mm

b: For an optimal soft tissue seal in the non-esthetic zone, the implant platform should be placed
2 to 3 mm under the free gingival level.

43. In the esthetic zone, the implant platform should be placed how far beneath the
gingival level?
a. 1–2 mm
b. 2–3 mm
c. 3–4 mm

c: For an optimal soft tissue seal in the esthetic zone, the implant platform should be placed 3 to
4 mm under the free gingival level.

97
4 SURGICAL PROCEDURES AND COMPLICATIONS

44. For the best papillary fill potential, what should be the distance from the interproximal
contact of the restoration to the crestal bone?
a. < 3 mm
b. < 4 mm
c. < 5 mm
d. < 6 mm

c: < 5 mm

Bone height

Distance < 5 mm
from contact point
> 5 mm
to bone

A distance greater than 5 mm will result in a black triangle.

45. Thick gingival biotype exhibits all of the following characteristics EXCEPT:
a. Dense tissue with a large zone of attached gingiva
b. Short papillae
c. Thick osseous forms
d. Reacts to periodontal disease with rapid bone loss and gingival recession

d: Thick gingival biotype reacts to periodontal disease with bone loss and pocket formation.

46. Thin biotype exhibits all of the following characteristics EXCEPT:


a. Thin tissue with minimal amount of attached gingiva
b. A periodontal probe is visible through the gingival tissues
c. Thin osseous forms with dehiscence and fenestration
d. Reacts to periodontal disease with bone loss and pocket formation

d: Thin gingival biotype reacts to periodontal disease with rapid bone loss and gingival recession.
The table below summarizes the differences between thick and thin biotypes. Knowledge of these
differences should help the implant dentist choose the proper strategies for the management of
different periodontal clinical scenarios for more predictable outcomes.

98
Thick versus thin tissue biotypes
Feature Thick biotype Thin biotype
Tissue quality Dense, with a large zone of Thin, with minimal attached gingiva
attached gingiva
Periodontal probing Probe not visible through gingiva Probe visible through gingiva
Papilla architecture Short Long
Gingival topography Flat Scalloped
Osseous form Thick Thin, with dehiscence and fenestration
Course of periodontal Bone loss and pocket formation Rapid bone loss and gingival recession
disease
Pattern of postsurgery Minimal ridge atrophy with Ridge resorption and unpredictable
healing predictable soft tissue contours soft tissue contours

47. Complications associated with implant placement in an overly supracrestal position (shallow
placement) include which of the following?
a. Possible exposure of the cover screw
b. Poor emergence profile of the implant prosthesis
c. Increased crown height
d. Exposure of the abutment to the implant crown margin
e. Possible implant body exposure
f. a, b, c, and d
g. a, b, d, and e
h. All of the above

g: Decreased crown height is a usual side effect of shallow implant placement.

48. When using a tissue punch to remove soft tissue for the purpose of flapless implant
placement, it is important to use a tissue punch that:
a. Has the same diameter as the implant.
b. Is slightly narrower than the implant diameter.
c. Is slightly wider than the implant diameter.

b: If a tissue punch is used, it should be slightly narrower than the implant’s diameter for better
peri-implant tissue healing, a shorter junctional epithelium, and less crestal bone loss.

49. Markers of implant-related mandibular fracture include all of the following EXCEPT:
a. Pain
b. Swelling (with or without fluctuation)
c. Presence or absence of mobility in the mandible
d. Change in occlusion
e. A history of trauma

e: One of the indicators of implant-related mandibular fracture is fracture without a history of trauma.

99
4 SURGICAL PROCEDURES AND COMPLICATIONS

50. In the anterior mandible, what is the minimum bone height to prevent bone fracture during
or after multiple implant placement?
a. > 8 mm
b. > 10 mm
c. > 12 mm
d. > 14 mm

b: Mandibular fracture is a rare complication and can occur with the insertion or removal of
endosseous implants in severely resorbed mandibular bone. This complication is most likely to
occur in very atrophic edentulous mandibles with bone height less than 10 mm. Placement of
multiple (four or more) implants or large implants in an atrophic mandible may further weaken
the jaw and lead to a fracture. Patients should be warned of the potential for this complication.

51. T
 RUE OR FALSE: Mandibular fractures due to implant placement can occur during or after
implant surgery.

True: Mandibular fractures can occur during or after implant surgery due to stress concentrations
in weakened areas, such as the osteotomy site or the site of a removed implant in an already
compromised mandible. Also, the site of an implant that has not yet osseointegrated acts as a
site of tensile stress concentration; with repeated functional forces, this can lead to a spontaneous
fracture without trauma.

52. T
 o prevent implant neck fracture during insertion, the operator must:
a. Not use narrow-platform implants in dense bone.
b. Precisely prepare the osteotomy site.
c. Use a tapping drill.

b: When placing implants (especially narrow-platform implants in a dense bone like D1 or D2
types), care should be taken to prepare the osteotomy site precisely so as not to put too much
torque stress on the implant neck during placement.

53. T
 o prevent compression necrosis, what is the maximum recommended torque you should
attain upon the placement of an implant in dense bone?
a. 35–40 Ncm
b. 45–50 Ncm
c. 60–65 Ncm

b: An initial torque of about 20 Ncm is usually adequate for achieving osseointegration if all other
healing factors are met, including an adequate healing period, a low-trauma surgical technique,
no micromovement during healing, no gap between the implant and the walls of the osteotomy
(precise preparation), and no implant surface contamination by organic or inorganic materials.
With the immediate loading technique, however, this initial torque must be increased to withstand
the micromovement and stress applied to the implant in the critical early stages after placement
through the immediate provisional prosthesis. Most of the literature refers to an optimum inser-
tion torque of about 35 to 45 Ncm for immediately loaded implants. Even if immediate loading
is not desired, the operator should try to achieve higher than 20 Ncm of initial torque, because
this leads to a shorter healing period and an increased chance of successful osseointegration.

100
54. What method should you use to measure implant stability a few months after implant placement?
a. Resonance frequency analysis (Osstell machine)
b. Reverse torque method (ratchet)
c. Percussion test (periodontal test machine)

a: The reverse torque method and percussion test might affect sound osseointegration, so they
are not recommended for use beyond the day of placement. If a measurement of the implant
stability is desired, then the use of a resonance frequency analysis instrument is recommended.

55. Which of the following conditions might contribute to low initial stability upon implant placement?
a. Low bone density at the implant site
b. Overpreparation
c. Use of dense bone drills or tapping drills when they are not indicated
d. Imprecise drilling
e. All of the above

e: All of the above

56. TRUE OR FALSE: Pain is not usually a typical feature of tissue emphysema induced by
dental procedures.

True: Tissue emphysema is identified by swelling that is coincident with dental treatment. The
onset of the condition is almost always immediate. If the onset is delayed (hours after therapy),
the cause of the emphysema is usually coughing or sneezing that results in the entry of air into
the oral wound. In most cases, palpation of the skin produces crepitus (crackling sensation) as
the gas is pushed through the tissue. Pain is not a usual feature.

57. Differential diagnosis for tissue emphysema includes:


a. Angioedema
b. Soft tissue infection
c. Hematoma
d. a and b only
e. All of the above

e: All of the above

58. Steps to prevent soft tissue emphysema include all of the following EXCEPT:
a. Using surgical handpieces that vent the air away from the surgical field
b. Not extending mucoperiosteal flaps onto the lingual aspect of the mandibular third molar
area or to the medial side of the vertical ramus
c. Avoiding use of an air-water syringe in areas of soft tissue and periodontal infections
d. Using high-speed drills in the pulp chamber during endodontic therapy

d: Drilling in the pulp chamber during endodontic therapy should be performed with low-speed drills.

101
4 SURGICAL PROCEDURES AND COMPLICATIONS

59. T
 RUE OR FALSE: Management of tissue emphysema induced by dental procedures includes
antibiotics, mild analgesic therapy, observation, and referral to a physician.

True: Most cases of tissue emphysema are mild. Treatment usually consists of broad-spectrum
antibiotic therapy (due to the possible introduction of bacteria into the tissue with the compressed
air), mild analgesic therapy, reassurance, and observation. Patients should also be warned against
forceful sneezing, blowing of the nose, or restrained coughing. Symptoms usually subside in 3 to
10 days; however, consultation with a physician is necessary to rule out further complications. If
the patient reports any respiratory distress, he or she must be referred to a hospital immediately
for close observation and high-dose intravenous antibiotics.

60. W
 hich of the following conditions must be met to prevent the incision line from
reopening postoperatively?
a. The incision line closure must be tension free.
b. Proper long-lasting suturing material must be used.
c. Proper postoperative instructions must be given.
d. The patient must abstain from smoking.
e. A properly designed interim prosthesis must be fabricated.
f. All of the above

f: Incision line reopening can occur soon after surgery and can compromise the final results of
the procedure by increasing the chances of infection to the surgical area and/or affecting the
postoperative soft tissue esthetic outcome. All of these conditions must be met to minimize the
possibility of incision line reopening.

61. T
 RUE OR FALSE: If the incision line reopening is minor, no attempt should be made to close it.

True: If the reopening is minor, the incision line will be closed by secondary intention. If the
granulation process extends more than 2 weeks, it can be encouraged by trimming the epithelial
margins of the wound with coarse diamonds.

62. T
 RUE OR FALSE: Wound healing by secondary intention goes through inflammatory,
proliferative, and remodeling phases.

False: Wound healing by secondary intention goes through hemorrhagic, proliferative, and
remodeling phases. The following table summarizes the phases of healing by primary inten-
tion and secondary intention. (Source: Siervo S. Suturing Techniques in Oral Surgery. Chicago:
Quintessence, 2008:211.)

102
Phases of healing
Phase Duration Characteristics
Inflammation 3 to 7 days Increased blood vessel

Primary intention
permeability; presence of
leukocytes
Proliferative 3 to 12 days Contraction of wound area;
neodeposition of fibrinogen
Remodeling 12 days to Reorganization of new tissue
months
Hemorrhagic Up to 15 days Presence of polymorpho­
Secondary intention nucleate cells and macro-
phages; migration of
epithelial cells
Proliferative Up to 20 days Formation of granulation
tissue
Remodeling Months Scarring

63. TRUE OR FALSE: Mucositis refers to inflammation and ulceration in the soft tissues around an
implant; it is a reversible host response to periodontal pathogens.

True

64. Signs of mucositis include which of the following?


a. Bleeding on probing
b. Redness and swelling of soft tissue
c. Slight loss of supporting bone around the implant
d. a and b
e. b and c
f. All of the above

d: Mucositis does not include loss of supporting bone around the implant.

65. TRUE OR FALSE: Peri-implantitis refers to the destructive inflammatory process affecting
both soft and hard tissue surrounding a dental implant; it is more severe than mucositis and
leads to bone loss.

True: Bone loss without mobility must present (among other signs) for the diagnosis of peri-implantitis.

103
4 SURGICAL PROCEDURES AND COMPLICATIONS

66. S
 igns of peri-implantitis include which of the following?
a. Craterlike bone defect(s)
b. Bleeding and/or suppuration on probing
c. Implant mobility
d. > 4 mm of peri-implant depth probing
e. a, b, and d
f. b, c, and d
g. a, c, and d
h. All of the above

e: Bone loss without mobility must present (among other signs) for the diagnosis of peri-implantitis.

67. Risk factors for peri-implantitis include which of the following?


a. Poor oral hygiene
b. History of periodontal disease
c. Cigarette smoking
d. Inadequate adherence to supportive periodontal treatment
e. a, b, and d
f. b, c, and d
g. a, c, and d
h. All of the above

h: All of the above

68. TRUE OR FALSE: Nonsurgical mechanical debridement is effective in controlling peri-implantitis.

False: Although nonsurgical mechanical debridement may be effective in controlling mucositis


lesions, this intervention alone seems to have limited efficacy at peri-implantitis sites.
Peri-implantitis lesions commonly require surgical intervention.

69. T
 RUE OR FALSE: When treating mucositis, the outcome of nonsurgical therapy may be
improved by adjunctive measures (eg, local antibiotics, air abrasive devices, laser applications).

True

70. TRUE OR FALSE: Retrograde peri-implantitis is the term given to describe a radiolucency
around the most apical part of an osseointegrated implant that usually develops a few
months after surgical insertion of the implant.

True

104
71. Etiologic factors for an implant periapical lesion or retrograde peri-implantitis include all of
the following EXCEPT:
a. Necrosis from excessive compression of the bone during creation of the osteotomy
b. Infection from scar tissue of an impacted tooth
c. Contamination from adjacent teeth with obvious endodontic pathology
d. Infection in sites of previous tooth loss due to periodontitis
e. Necrosis from excessive heating of bone during creation of the osteotomy
f. Apical entrapment of gingival epithelial cells during the surgical insertion

a: Necrosis from excessive compression of the bone during creation of the osteotomy.

72. TRUE OR FALSE: Radiation causes endarteritis (inflammation of the tunica intima of an
artery), leading to tissue hypoxia, hypocellularity, and hypovascularity, which may lead to
chronic nonhealing wounds.

True: Irradiated bone may not be capable of proper remodeling due to damage to osteoclasts
that occurred before these vascular alterations. Also, there is an increased risk of osteoradione-
crosis in irradiated bone.

73. TRUE OR FALSE: Titanium implants should be removed before radiotherapy.

 alse: Removal within 1 month before irradiation could be dangerous, possibly resulting in osteo-
F
radionecrosis. The removal of implants by trephine can cause a significant trauma to the bone. On
the other hand, when abutments and titanium bars were left in place during irradiation, several
patients showed soft tissue complications (ie, skin dehiscence) as a consequence of the back-
scatter irradiation effects. (Sources: Granström G, Tjellström A, Albrektsson T. Postimplantation
irradiation for head and neck cancer treatment. Int J Oral Maxillofac Implants 1993;8:495–501. /
Ihde S, Kopp S, Gundlach K, Konstantinovic VS. Effects of radiation therapy on craniofacial and
dental implants: A review of the literature. Oral Surg Oral Med Oral Pathol Oral Radiol Endod
2009;107:56–65.) Therefore, all prostheses, frameworks, and abutments should be removed prior
to irradiation, while implants should be left in situ but covered with skin or mucosa.

74. TRUE OR FALSE: Implants can osseointegrate into irradiated facial bones.

True: Titanium implants can osseointegrate into the facial bones even in highly irradiated tissues.
However, there is greater loss of implants over time in previously irradiated bone. Gentle surgical
technique during implant placement, intensive cooling of the bone, and consequent saving of the
blood vessels in the implant area can minimize the possibilities of osteoradionecrosis in irradiated
bone. Implant loss can also be prevented by the adjunctive use of hyperbaric oxygen.

75. What is the recommended waiting period for placing implants in irradiated bone?
a. 2 years
b. 3 years
c. 4 years
d. 5 years

b: Higher rates of implant loss in previously irradiated bone are usually reported within the first
3 years after implant surgery. (Source: Granström G, Tjellström A, Albrektsson T. Postimplantation
irradiation for head and neck cancer treatment. Int J Oral Maxillofac Implants 1993;8:495–501.)

105
4 SURGICAL PROCEDURES AND COMPLICATIONS

76. TRUE OR FALSE: Hyperbaric oxygen therapy (HBO) plays a positive role in revitalizing
irradiated bone.

 rue: Several studies describe the positive role of HBO in revitalizing irradiated bone by stimulating
T
angiogenesis, leading to improved success rates for dental implants in irradiated bone. HBO was
shown to increase the removal torque necessary to unscrew implants in both the controls and
the irradiated animals. (Source: Nilsson P, Albrektsson T, Granström G, Röckert HO. The effect
of hyperbaric oxygen treatment on bone regeneration. An experimental study using the bone
harvest chamber in the rabbit. Int J Oral Maxillofac Implants 1988;3:43–48.)
The effects of HBO on titanium implant osseointegration seem to be mainly the result of
increased bone formation in the titanium-bone interface. In a study of the effects of HBO on
the strength of the bone–titanium implant interface in irradiated and nonirradiated bone, HBO
delivered at 2.8 ATA (2-hour daily treatments for 21 days) increased the biomechanical force
necessary to unscrew the implants by 44% in irradiated bone and 22% in nonirradiated bone.
(Source: Johnsson K, Hansson A, Granström G, Jacobsson M, Turesson I. The effects of hyperbaric
oxygenation on bone-titanium implant interface strength with and without preceding irradiation.
Int J Oral Maxillofac Implants 1993;8:415–419.)

77. W
 hen performing sinus floor elevation through the osteotomy site, what is the minimum
recommended alveolar bony height under the sinus floor?
a. 1.0 mm
b. 3.0 mm
c. 5.0 mm
d. 7.0 mm

c: The bone density under the sinus floor in the posterior maxilla is usually very low. Therefore,
5.0 mm of alveolar bony height is recommended; performing the procedure with less height than
this will not provide reliable results.

78. D
 uring sinus elevation through the osteotomy site, you perforated the sinus floor, but there
is 9 mm of available alveolar bone and you are planning to place a 10-mm-long implant. If
you proceed, the implant is going to extrude into the sinus by 1 mm. What should you do?
a. Abort the procedure.
b. Place a collagen wound dressing and then place the implant.
c. Place the implant without placing any bone grafting or membrane material.

c: Although you should do your best not to perforate the sinus floor, it has been reported in the
literature that implants perforating the sinus membrane and penetrating 1 mm into the sinus do
not have reduced survival rates. (Sources: Agliardi EL, Tetè S, Romeo D, Malchiodi L, Gherlone E.
Immediate function of partial fixed rehabilitation with axial and tilted implants having intrasinus
insertion. J Craniofac Surg 2014;25:851–855. / Si MS, Zhuang LF, Gu YX, Mo JJ, Qiao SC, Lai HC.
Osteotome sinus floor elevation with or without grafting: A 3-year randomized controlled clinical
trial. J Clin Periodontol 2013;40:396–403. / Pal US, Sharma NK, Singh RK, et al. Direct vs. indirect
sinus lift procedure: A comparison. Natl J Maxillofac Surg 2012;3:31–37.)

79. T
 RUE OR FALSE: Sinus augmentation through the lateral window approach can be
performed after using panoramic radiography only as a diagnostic tool.

False: Soft tissue pathologies and certain anatomical structures cannot be seen on a panoramic
radiograph; a CT scan is required to visualize them. Any procedure performed by the implant
surgeon must always be based on the best diagnostic tools available. It is reckless to perform
this procedure without a CT scan.

106
80. You encountered bleeding while performing the bony window osteotomy during sinus
augmentation. What artery was damaged?
a. The maxillary artery
b. The infraorbital artery
c. The superior posterior alveolar artery
d. The anterior superior alveolar artery
e. Could be c or d
f. None of the above

f: Hemorrhage during bony window osteotomy usually arises from damage to the intraosseous
anastomosis between the posterior superior alveolar artery and the infraorbital artery, both of
which are branches of the maxillary artery. The anastomosis that causes bleeding is often small in
size, and hemostasis should be allowed to occur naturally, perhaps facilitated by the application of
either light pressure with gauze or bone wax. An electrocoagulator is not recommended because
it may cause membrane necrosis.

Gasserian
ganglion Vidian nerve
Maxillary nerve (V2)

Infraorbital
artery
and nerve
Medial superior
alveolar nerve
(V2)
Auriculo- Posterior superior
temporal alveolar nerve (V2)
nerve (V3)
Maxillary
Inferior sinus
alveolar nerve
Intraosseous
Lingual anastomosis
nerve
Posterior
Maxillary superior Palatine nerve
artery alveolar
artery Extraosseous anastomosis

81. What is the safest tool to use when performing the lateral window osteotomy to prevent
perforation of the sinus membrane?
a. Carbide bur
b. Diamond bur
c. Piezotome tip (Piezosurgery)
d. Any of the above if you are careful

c: While you certainly have to be careful with any tool, the Piezotome tip carries the lowest risk
of perforating the sinus membrane. One study indicated that perforation can occur in up to
56% of cases in which rotary instruments are used and in as few as 3.8% in which piezoelectric
instruments are used. (Sources: Wallace SS, Mazor Z, Froum SJ, Cho SC, Tarnow DP. Schneider-
ian membrane perforation rate during sinus elevation using piezosurgery: Clinical results of 100
consecutive cases. Int J Periodontics Restorative Dent 2007;27:413–419. / Jensen OT, Shulman
LB, Block MS, Iacono VJ. Report of the sinus consensus conference of 1996. Int J Oral Maxillofac
Implants 1998;13[suppl]:11–45.)

107
4 SURGICAL PROCEDURES AND COMPLICATIONS

82. F
 actors that affect sinus membrane perforation during membrane elevation include which of
the following?
a. The size of the access window
b. Use of a carbide bur
c. The width of the sinus buccolingually
d. The presence of maxillary septa
e. a and d
f. All of the above

f: All of the above

83. A
 fter preparing the window osteotomy, you discovered that it was too small or too distal.
What tool should you use to safely enlarge the size of the window?
a. Diamond bur but without irrigation
b. Piezotome tip
c. Kerrison rongeur
d. Any of the above

c: You must use a Kerrison rongeur to enlarge the window size to minimize the potential for
membrane perforation.

84. T
 RUE OR FALSE: You successfully elevated the sinus membrane and are now ready to place
the bone graft material. There was no perforation noted in the sinus membrane, but you
should still place a membrane in the sinus before introducing any bone graft material.

True: The author recommends using Oratape (Salvin) in cases of small perforations as well as in
cases of no perforation to protect the sinus membrane from possible sharp fragments of bone
grafting material. Oratape is a repair device that serves as an adhesive when moistened with
blood. Healing is not affected when it is used with small perforations because of the abundance
of overfolded, accordion-like tissues and the rapidity of epithelial proliferation.

108
85. You successfully elevated the sinus membrane and are now ready to place the bone graft
material. You noticed a small to medium-size perforation (under 10 mm in diameter) in the
sinus membrane. What should you do?
a. Nothing. This requires no treatment, because elevation of the membrane may obliterate the
defect as a result of folding of the membrane upon itself.
b. Place collagen wound dressing like Oratape.
c. Place a long-lasting collagen membrane like Bio-Gide (Geistlich).

c: In the case of one or more small to medium-size perforations, the surgeon should place a
long-lasting absorbable collagen membrane to repair the perforation(s) and to prevent the bone
graft materials from invading the sinus cavity.

86. How do you manage extensive sinus membrane perforation?


a. Abort the procedure.
b. Repair the perforation using the Loma Linda pouch technique.
c. Repair the perforation using a pedicled portion of the buccal fat pad as a membrane.
d. Any of the above

d: Any of the above

87. TRUE OR FALSE: When performing lateral window sinus elevation, the sinus membrane
should be elevated off the medial wall.

True: Complete elevation of the sinus membrane off the anterior and medial walls of the sinus
to the desired height is important to prevent re-pneumatization of the sinus, promote proper
drainage, and expose the graft material to greater bone surface area. Incomplete elevation will
lead to partial augmentation, with subsequent bone graft volume loss and possible complications
with implant placement.

a b c d
(a) Partial sinus membrane elevation can lead to (b) an incomplete graft placement, which can cause (c) re-pneumatization
and volume loss of the graft and (d) imperfect implant positioning in the grafted sinus.

109
4 SURGICAL PROCEDURES AND COMPLICATIONS

88. During the sinus elevation procedure, you recognize that you went too far with the elevation.
What should you do next?
a. Nothing; wait for a few minutes and the sinus membrane will drop from the breathing pressure.
b. Gently pull the membrane down with smooth-tip pliers.
c. Abort the procedure; a reattempt can be made in 2 to 3 months.

c: Excessive elevation of the membrane, which can lead to excessive filling of the sinus, can be
prevented with gentle elevation and meticulous manipulation of the sinus membrane and by
taking measurements with a periodontal probe or ruler to avoid overfilling of the sinus cavity.
The best way to handle this complication is to abort the procedure rather than overfill the sinus.
The membrane will gradually relapse toward the floor of the sinus over a period of several weeks,
and then another attempt can be made. Overfilling can lead to membrane tear and improper
sinus drainage, with subsequent graft material loss. If this occurs, removal of all the graft material
is indicated.

89. E
 tiology of acute graft infection and sinusitis after sinus augmentation includes which of
the following?
a. Preexisting sinus disease
b. Contamination intrasurgically from bacteria present in the sinus or the oral cavity
c. Infection from extracted teeth that penetrates the sinus floor
d. Perforation of the sinus membrane postoperatively
e. Significant migration of graft material into the sinus during surgery
f. Ostium obstruction due to overpacked graft material or postoperative edema
g. Poor aseptic conditions
h. Secondary to delayed healing as a result of incision line reopening
i. a, b, c, and d
j. All of the above

j: All of the above

90. What is aspergillosis?


a. Bacterial infection
b. Viral infection
c. Fungal infection

c: Aspergillosis is a fungal infection caused by Aspergillus spores. A ball of fungus fibers, blood
clots, and white blood cells may form in the lungs or sinuses. Symptoms are similar to those of
chronic sinusitis, but the infection is difficult to resolve and can become invasive, resembling
malignancy. Development of aspergillosis is most often related to endodontic filling material but
has also been observed following sinus grafting. Treatment consists of antifungal therapy and
surgery by an otolaryngologist.

91. T
 RUE OR FALSE: Progressive implant loading can help prevent implant failure in poor-density
bone and in the augmented sinus.

True: Progressive loading gradually converts less mineralized bone into bone more capable of
bearing masticatory forces.

110
92. Bisphosphonates are used to treat which of the following bone conditions?
a. Paget disease
b. Bone malignancies
c. Osteoporosis
d. a and c
e. All of the above

e: All of the above

93. TRUE OR FALSE: Bisphosphonate-related osteonecrosis of the jaw (BRONJ) is a potentially


harmful side effect of bisphosphonate therapy.

 rue: BRONJ is a potentially harmful side effect of bisphosphonate therapies. According to recent
T
studies, the best-known triggering event of this severe condition is dentoalveolar surgery. (Sources:
de Papp A, Bone HG, Caulfield MP, et al. A cross-sectional study of bone turnover markers in
healthy premenopausal women. Bone 2007;40:1222–1230. / Edwards BJ, Hellstein JW, Jacobsen
PL. Updated recommendations for managing the care of patients receiving oral bisphospho-
nate therapy: An advisory statement from the American Dental Association Council on Scientific
Affairs. J Am Dent Assoc 2008;139:1674–1677 [erratum 2009;140:522].) A vast array of extrinsic
and intrinsic factors such as duration of therapy, smoking, and surgical procedures also affect the
onset of BRONJ. BRONJ affects the mandible more than the maxilla, implicating an abolition of
osteoclast-mediated bone resorption rather than antiangiogenesis.

94. After how many years of continuous bisphosphonate treatment does BRONJ occur?
a. 1 year
b. 2 to 3 years
c. 4 years

b: Most cases of BRONJ occur after 2 to 3 years of continuous treatment at higher dosages
(70 mg/week) or 5 years at lower dosages (35 mg/week). Bisphosphonates have a tendency to
accumulate in osseous tissue over time. Dental surgical procedures and local trauma may lead to
the release of high concentrations of these accumulated chemicals from the bone. High levels of
bisphosphonates are toxic to epithelial tissues and may cause failure in wound healing and closure,
thus increasing the susceptibility of tissues to infection. Toxicity to epithelial tissues may lead to
osteonecrosis of the jaw. Alendronate has been found to be toxic to gastrointestinal epithelial
tissues at a concentration of 30 nmol, while risedronate has been known to be toxic at 10 nmol.

95. TRUE OR FALSE: Bisphosphonates do not affect the oral flora.

False: Bisphosphonates have been known to cause a shift in the balance of the normal oral flora.
Invasion of bacteria into bone causes osteocyte death and the formation of bacterial biofilms,
resulting in pocket formation, bone disintegration, and bone loss. Bisphosphonate use impedes
oral cavity host defenses and helps establish a niche for Actinomyces within the bone when they
are subsequently introduced. Most affected patients show disruption in the mucous membrane
lining of the oral cavity as a result of dental procedures, trauma, oncologic surgery, or dental
diseases. Bisphosphonate inhibition of the life cycle of keratinocytes further aggravates mucosal
breakdown and prevents repair. Previous disruption of bone may facilitate infection. Actinomyces
are thought to then infect living bone without secondarily infecting already necrotic bone.
Timely recognition of actinomycosis is critical. Without appropriate treatment, recovery is unlikely.
High dosages of antimicrobials and a prolonged course of antibiotics are needed for management
of the condition. It is customary for these patients to undergo 2 to 6 weeks of intravenous therapy,
followed by oral therapy for 6 to 12 months for more serious infections.

111
4 SURGICAL PROCEDURES AND COMPLICATIONS

96. Risk factors for BRONJ include which of the following?


a. Dentoalveolar trauma
b. Denture use
c. Poor oral hygiene
d. Smoking
e. a, b, and c
f. All of the above

e: A history of dentoalveolar trauma, denture use, and poor oral hygiene are risk factors for
BRONJ. Ruggiero et al reported 63 cases of BRONJ from their oral surgery practice over a span of
2 years, and 86% of these patients had undergone a previous dental procedure that did not heal.
(Source: Ruggiero S, Gralow J, Marx RE, et al. Practical guidelines for the prevention, diagnosis,
and treatment of osteonecrosis of the jaw in patients with cancer. J Oncol Pract 2006;2:7–14.)

97. Preventive measures for BRONJ include which of the following?


a. Maintenance of good oral hygiene to prevent the necessity of dental surgery
b. Checkup every 4 to 6 months to detect early signs of BRONJ
c. Use of antibiotics whenever necessary
d. Selecting less traumatic dental procedures
e. Suspending bisphosphonate therapy for 3 to 4 months until the surgical site has healed
f. All of the above

f: All of the above

98. W
 hich of these tests can be used to predict the susceptibility of patients to
developing BRONJ?
a. Bone turnover markers (BTMs)
b. BRD
c. Bone mineral density (BMD)
d. OSC
e. a and b
f. All of the above

f : Currently, the medical and dental disciplines are unable to accurately predict which patients may
be susceptible to developing BRONJ following an oral surgical procedure. However, the use of
BTMs and BMD studies provide some indication of the patient’s overall bone metabolic stability.
If BTMs such as the patient serum cross-linked N-terminal telopeptide and serum bone-specific
alkaline phosphatase values are within an acceptable range, the implant dentist should consult
with the prescribing physician and discuss if a bisphosphonate therapy holiday would be accept-
able (if the patient has been on bisphosphonates for 2 or more years). If any of the values are out
of range, this may indicate that the bisphosphonate therapy is ineffective, and therefore other
treatment modalities (eg, teriparatide) may be indicated. Repeating these tests annually provides
guidance as to when antiresorptive bisphosphonate therapy should be reinstituted.
If the BTMs are within normal range, the implant surgeon may have reason to believe that the
patient can successfully undergo surgery. However, all patients who have recently taken a bisphos-
phonate should be advised that the risk of BRONJ exists. The patient may be at a greater risk of
developing BRONJ if the BTMs are not within the acceptable range, and it may be advisable to
delay implant surgery.

112
99. Treatment for BRONJ includes all of the following EXCEPT:
a. Systemic antibiotics
b. Painkillers
c. Anti-inflammatory medication
d. Antibacterial mouthwash
e. Debridement of the surgical site

c : Patients showing signs of BRONJ should be treated with systemic antibiotics, oral rinses (0.12%
chlorhexidine gluconate or minocycline hydrochloride), pain control, and limited debridement.
Microbial cultures may help identify infectious microorganisms that require antibiotics and anti-
septic mouthwash.
In advanced cases and more severe stages of BRONJ, long-term intravenous antibiotic treatment
may be necessary with aggressive surgical debridement. A penicillin and metronidazole combi-
nation is useful in patients with refractory and recurrent infections. Azithromycin or a quinolone
may be used instead of penicillin in allergic patients.

100. TRUE OR FALSE: If a patient taking an anticoagulant or antiplatelet agent is to undergo


a surgical procedure, it is important to discontinue use of the anticoagulant or
antiplatelet medication.

False: For many years this was thought to be the case, but this strategy is no longer acceptable.
Disproportionate activation of coagulation or inhibition of anticoagulant mechanisms may result
in hypercoagulability and thrombosis. Thrombosis formation is of greater clinical concern in
terms of morbidity and mortality than all of the hemorrhagic disorders combined. Thrombosis
can be induced by injury to a blood vessel wall, alterations in blood flow, and changes in the
composition of blood.

101. Guidelines for dental treatment of patients receiving anticoagulants/antiplatelet agents


include which of the following?
a. Identify the reason the patient is receiving anticoagulant therapy.
b. Know the laboratory tests used to assess anticoagulation levels.
c. Evaluate the potential risk versus benefit of altering the drug regimen.
d. Be familiar with local methods of obtaining hemostasis both intraoperatively
and postoperatively.
e. Consult with the patient’s prescribing physician and inform him or her about the
type of dental treatment planned and investigate whether there is a need to alter
the anticoagulant regimen.
f. All of the above except e
g. All of the above

g: All of the above

113
4 SURGICAL PROCEDURES AND COMPLICATIONS

102. W
 hat is the INR (international normalized ratio) value at or under which anticoagulant
therapy may continue?
a. 2.0
b. 3.0
c. 4.0
d. 5.0

c : Several studies have evaluated the adjustment of oral anticoagulant levels before dental proce-
dures. Devani et al concluded that because of the difficulty in predicting the decrease in INR value
in any given patient, the risk of experiencing a thromboembolism overrides the risk of experiencing
excessive postoperative bleeding with tooth removal. (Source: Devani P, Lavery KM, Howell CJ.
Dental extractions in patients on warfarin: Is alteration of anticoagulant regime necessary? Br J
Oral Maxillofac Surg 1998;36:107–111.)
The literature supports the idea that bleeding observed from dental surgery in patients with
INR values of less than 4.0 is manageable with local measures and that it is not necessary or
recommended that anticoagulant (eg, warfarin) therapy be interrupted. Local hemostatic measures
may include collagen plugs, powder, or sheets; gelatin sponges with sutures; soaked sponges,
mouthrinse, oral tablets, or intravenous forms of tranexamic acid; vasoconstrictors in local anes-
thetic; and atraumatic surgical techniques.
If the INR value is beyond the acceptable range of ≤ 4.0 and it is deemed necessary to provide
dental implant treatment, then the prescribing physician should be consulted to determine what
the appropriate INR target value should be. Ideally an INR value should be obtained on the day
of the dental procedure, although the INR obtained within the previous 4 weeks may be accept-
able. However, if any changes in warfarin therapy or in other medications were instituted since
the previous INR, a new INR value should be obtained.

103. T
 RUE OR FALSE: Aspirin use should be discontinued for dental implant procedures in
patients receiving low doses (81 mg) or in those who are on a single daily dose of 325 mg.

False: Aspirin, the prototype antiplatelet drug, is the least expensive, most often used, and best-
known drug in this group. Aspirin’s maximal antithrombotic effect is achieved at dosages much
lower than those required for other actions of aspirin. Higher dosages of aspirin do not prove
to be more effective; in fact, higher dosages may potentially be less effective. The antiplatelet
actions of other salicylates and nonsteroidal anti-inflammatory drugs are reversible, and these
agents have a shorter duration of platelet-inhibitory action.
Although there is long-standing concern on the part of dental practitioners about the possibility
of prolonged bleeding during and after invasive dental procedures in patients taking aspirin, a
prospective study of single-tooth extraction on patients randomized to aspirin versus a placebo
failed to show a statistically significant difference in postoperative bleeding.(Source: Valerin MA,
Brennan MT, Noll JL, et al. Relationship between aspirin use and postoperative bleeding from
dental extractions in a healthy population. Oral Surg Oral Med Oral Pathol Oral Radiol Endod
2006;102:326.) Aspirin use should not be discontinued for dental implant procedures in patients
receiving low doses (81 mg) of aspirin or a single, conventional dose of 325 mg daily. If the
patient has recently or traditionally taken several adult aspirin tablets daily, the implant dentist in
consultation with the patient’s physician can consider evaluating platelet function with a platelet
function analyzer or an Ivy bleeding time test. Rarely is there significant bleeding during or after
oral surgical procedures unless this bleeding time is greater than 20 minutes. The risk of excessive
bleeding with aspirin is increased by the concurrent use of anticoagulants or alcohol and conditions
such as advanced age, liver disease, and coexisting coagulopathies.

114
104. TRUE OR FALSE: Nonsteroidal anti-inflammatory drugs (NSAIDs) such as ibuprofen and
indomethacin should be discontinued for dental implant procedures because they significantly
affect platelet activity and thus can contribute to excessive bleeding during surgery.

 alse: NSAIDs have limited clinical use as antiplatelet agents. Salsalate and COX-2 inhibitors such
F
as celecoxib do not significantly affect platelet activity when used at a therapeutic dosage. When
these are used alone, it is not necessary to discontinue therapy prior to dental implant procedures.
Excessive bleeding, if observed, should be controlled by local measures. If the NSAID is being
taken for analgesic purposes and not antiplatelet effects, the implant dentist may discontinue it
for three half-lives of the NSAID to allow sufficient elimination of the drug and thereby the return
of normal platelet function. The risk of excessive bleeding with NSAIDs is increased by the simul-
taneous use of anticoagulants or alcohol and circumstances such as advanced age, liver disease,
and coexisting coagulopathies.

105. The most common bacteria causing acute sinusitis (not from an odontogenic source) include
all of the following EXCEPT:
a. Haemophilus influenzae
b. Moraxella catarrhalis
c. Escherichia coli
d. Streptococcus pneumoniae

c : Acute, uncomplicated sinusitis in otherwise healthy adults is most frequently caused by S pneu-
moniae, M catarrhalis, or H influenzae. E coli is rarely isolated in these patients.

106. Which of the following is NOT a common symptom of acute sinusitis?


a. Dental pain
b. Facial numbness
c. Fever
d. Foul nasal drainage

 : While facial numbness can occur with acute sinusitis, it is unusual and should prompt additional
b
workup for invasive processes (such as tumors or invasive fungal sinusitis). The most frequent
symptoms of acute sinusitis include fever, facial/dental pain, congestion, foul nasal drainage, and
other symptoms consistent with upper respiratory infection.

107. What is the most common incident preceding an episode of acute bacterial sinusitis?
a. A viral upper respiratory infection
b. An allergy exposure
c. Dental work
d. Facial trauma

 : Most episodes of acute sinusitis evolve from the ostial obstruction and ciliary dysfunction caused
a
by a viral upper respiratory infection. The symptoms of acute bacterial sinusitis tend to localize and
increase about the time patients expect their cold to disappear. While allergy exposures, dental
work, and other causes of inflammation can lead to acute sinusitis, they are much less common.

115
4 SURGICAL PROCEDURES AND COMPLICATIONS

108. What is the average diameter of the natural ostium of the maxillary sinus?
a. 1.8 mm
b. 2.4 mm
c. 3.1 mm
d. 4.6 mm

b: The natural ostium of the maxillary sinus averages 2.4 mm in diameter (although there is wide
variation between individuals) and therefore requires relatively little mucosal inflammation to
obstruct it.

109. First-line antibiotic therapy for acute bacterial sinusitis may include which of the following?
a. Amoxicillin
b. Amoxicillin clavulanate
c. Doxycycline
d. Clindamycin
e. a or b
f. Any of the above

 : According to the 2015 Clinical Practice Guidelines from the American Academy of Otolaryngology,
e
first-line antibiotic therapy for acute bacterial sinusitis should be amoxicillin, with or without clavulanate.
If the patient has a penicillin allergy, doxycycline or a respiratory quinolone is recommended.

110. The best way to distinguish viral sinusitis from bacterial sinusitis is:
a. The presence of dental pain
b. The presence of fever
c. Length of symptoms
d. Severity of symptoms

c: Viral sinusitis can be difficult to tell apart from bacterial sinusitis. In the absence of a culture of
the sinus drainage, the length of symptoms (10 days is a rough cutoff point) is the best indicator
of which type of organism is at fault. All the symptoms of acute bacterial sinusitis can be present
in viral infections, and severity is difficult to judge.

111. Odontogenic sinusitis is responsible for up to what percent of acute sinus infections?
a. 10%
b. 20%
c. 30%
d. 50%

 : While dental sources of sinus infection are rare in the population a physician sees, they are much
a
more likely in those patients seeking dental care. An odontogenic source should be considered in
those patients with a history positive for odontogenic infection or dentoalveolar surgery or who
are resistant to standard sinusitis therapy.

116
112. Which of the following is NOT true of sinusitis of odontogenic origin?
a. It is caused by a mix of aerobic and anaerobic bacteria.
b. It is caused by the same bacteria as other forms of acute sinusitis.
c. The most common organisms include anaerobic streptococci, Bacteroides, Proteus,
and coliform bacilli.
d. There is a preponderance of anaerobic bacteria in these infections.

b: Odontogenic sinusitis is typically a mixed aerobic-anaerobic infection, with anaerobes (includ-
ing anaerobic streptococci, Bacteroides, Proteus, and coliform bacilli) predominating. The most
common bacteria found in “standard” acute sinusitis include S pneumoniae, M catarrhalis, and
H influenzae.

113. Which of the following is true regarding management of odontogenic sinusitis?


a. It typically clears within a week on appropriate antibiotics.
b. There is no need to remove odontogenic foreign bodies.
c. Surgical management of dental abscesses is required.
d. Oroantral communications should not be addressed until infection is cleared.

c : Resolution of odontogenic sinus infections requires treatment of both the sinus infection and the
dental site of origin. In addition to antibiotics covering both respiratory and dental pathogens, this
may include drainage of dental abscesses, removal of odontogenic foreign bodies, and closure
of oroantral communications.

114. Predisposing factors to acute sinusitis after sinus grafting include all of the following EXCEPT:
a. Smoking
b. Membrane perforation
c. Preexisting sinus disease
d. Female sex

d: Multiple studies have demonstrated the association of smoking with postoperative sinusitis.
(Sources: White LC, Kazi AA, Jang DW, Gurrola J, Kountakis SE. The effect of smoking on quality of
life following sinus surgery: 10-year follow-up. ORL J Otorhinolaryngol Relat Spec 2015;77:39–43.
/ Briggs RD, Wright ST, Cordes S, Calhoun KH. Smoking in chronic rhinosinusitis: A predictor of
poor long-term outcome after endoscopic sinus surgery. Laryngoscope 2004;114:126–128. / Das
S, Becker AM, Perakis H, Prosser JD, Kountakis SE. The effects of smoking on short-term quality
of life outcomes in sinus surgery. Laryngoscope 2007;117:2229–2232.) Membrane perforation
has likewise been shown to be associated with sinusitis. (Sources: Oh E, Kraut RA. Effect of sinus
membrane perforation on dental implant integration: A retrospective study on 128 patients.
Implant Dent 2011;20:13–19. / Nolan PJ, Freeman K, Kraut RA. Correlation between Schneiderian
membrane perforation and sinus lift graft outcome: A retrospective evaluation of 359 augmented
sites. J Oral Maxillofac Surg 2014;72:47–52. / Kim YK, Hwang JY, Yun PY. Relationship between
prognosis of dental implants and maxillary sinusitis associated with the sinus elevation proce-
dure. Int J Oral Maxillofac Implants 2013;28:178–183.) Preexisting sinus inflammatory disease is
also a risk factor for postoperative infection. (Source: Anavi Y, Allon DM, Avishai G, Calderon S.
Complications of maxillary sinus augmentations in a selective series of patients. Oral Surg Oral
Med Oral Pathol Oral Radiol Endod 2008;106:34–38.)

117
4 SURGICAL PROCEDURES AND COMPLICATIONS

115. Which of the following is NOT required to diagnose chronic sinusitis?


a. More than 12 weeks of symptoms
b. Imaging consistent with chronic inflammatory changes
c. Presence of nasal polyps on imaging or examination
d. Inflammatory changes on intranasal examination

c: Chronic sinusitis is diagnosed with a combination of at least 12 weeks of symptoms, imaging
(usually CT) consistent with inflammatory changes, and signs of inflammation on anterior rhinos-
copy or nasal endoscopy. Chronic sinusitis is recognized to have two general phenotypes: with
nasal polyps and without nasal polyps.

116. Frequent symptoms of chronic sinusitis include which of the following?


a. Fever
b. Severe facial pain
c. Hyposmia
d. Isolated severe headache

c: The most frequent symptoms of chronic sinusitis include congestion, facial pressure, postnasal
drainage, nasal obstruction, fatigue, and hyposmia (decreased sense of smell). Less common
symptoms include headache, facial pain, dental pain, rhinorrhea (runny nose), and visual changes.
Isolated severe headache, severe facial pain, or fever is almost never found in chronic sinusitis.

117. Which of the following is the best clinical examination technique to diagnose chronic sinusitis?
a. Nasal endoscopy
b. Anterior rhinoscopy
c. Percussion of the sinuses
d. Transillumination of the sinuses

a: Fiberoptic nasal endoscopy allows visualization of the recesses of the nasal cavity and the sinus
drainage pathways. Anterior rhinoscopy (looking in the nostrils with an otoscope or a headlight) may
be able to detect more anterior disease, such as polyps extending toward the nostrils. Percussion
and transillumination of the sinuses are of very little value in the diagnosis of chronic sinusitis.

118. W
 hich of the following is the preferred diagnostic imaging modality in the diagnosis of
chronic sinusitis?
a. Plain radiographs, including Caldwell and Waters views
b. MRI
c. CT with contrast
d. CT without contrast

d: CT without contrast. MRI is better than CT for soft tissue changes but does not show the fine
bony detail needed for the sinuses and tends to overemphasize soft tissue changes. Plain radio-
graphs also do not show the level of bony or soft tissue detail needed and are largely archaic in
the diagnosis of chronic sinusitis. Contrast is not helpful in sinus CT scans. Cone beam CT (CBCT)
scans work well for diagnosis of chronic sinusitis, but the limited plate sizes on dental CBCT
machines may not show all of the maxillary sinuses (or the other paranasal sinuses).

118
119. Dome-shaped opacities arising from the floor of the maxillary sinus may be which of
the following?
a. Pseudocysts
b. Retention cysts
c. Mucoceles
d. a or b

d: Dome-shaped opacities commonly arise from the floor of the maxillary sinus. These are
noninflammatory and can be either pseudocysts (subperiosteal) or retention cysts (epithelium-
lined cysts). Neither of these must be addressed prior to sinus grafting unless they are very large.
Mucoceles are obstructed areas within a sinus (either part or all of it) commonly caused by surgery
or trauma and must be surgically addressed before grafting. It is a misnomer to refer to dome-
shaped opacities as mucoceles.

120. When would a dome-shaped opacity within a maxillary sinus need to be addressed prior
to grafting?
a. When elevating the cyst would cause it to block the ostium of the sinus.
b. When the cyst arises from any other surface than the floor of the sinus.
c. Dome-shaped opacities should always be addressed prior to surgery.
d. Dome-shaped opacities never need to be addressed prior to surgery.

a: While the vast majority of noninflammatory dome-shaped opacities/cysts in the maxillary sinus
do not need to be addressed prior to grafting, when elevating these cysts with the sinus membrane
would cause them to obstruct the natural ostium of the sinus, they should be addressed prior to
surgery. A rough rule is that when a cyst arising from the floor of the sinus fills more than half of the
sinus, it should be assessed for removal. While aspiration of the cyst at the time of grafting may
provide a temporary resolution, the concern is that the cyst will re-form and create an obstruction;
therefore, referral of these large cysts to an otolaryngologist is recommended.

121. All of the following criteria are useful for distinguishing inflammatory soft tissue
changes from noninflammatory soft tissue changes of the maxillary sinus membrane
on a CT scan EXCEPT:
a. The thickening extends throughout the entire sinus
b. The absolute thickness of the membrane in millimeters
c. The shape of the mucosal change
d. The presence or absence of diffuse thickening in the other sinuses

b: While much is often made of the absolute thickness of the mucosal membrane, using this
finding alone is relatively unhelpful. Inflammatory changes of the sinus mucosa tend to be diffuse
throughout the sinus, often seen in other sinuses as well. Noninflammatory changes (such as
dome-shaped opacities/sinus cysts) tend to be raised, rounded, and surrounded by normal areas
within the sinus.

119
4 SURGICAL PROCEDURES AND COMPLICATIONS

122. W
 hich of the following has NOT been shown to have a role in the medical management of
chronic sinusitis?
a. Oral steroids
b. Herbal supplements
c. Nasal steroid sprays
d. Antibiotics

b: Chronic sinusitis is primarily a disease of inflammation; therefore, anti-inflammatory cortico-


steroids are frequently used in the medical management of this disease. Bacterial infections
secondary to obstruction and ciliary dysfunction resulting from inflammation may occur, requiring
antibiotics. While complementary and alternative treatments for chronic sinusitis are popular, due
to the intractability of the disease, none of them has been shown to be effective.

123. C
 ommon short-term adverse effects of oral corticosteroids in the management of chronic
sinusitis include which of the following?
a. Buffalo hump
b. Hirsutism
c. Emotional instability
d. Osteoporosis

c: Depending on the dose and course length, oral corticosteroids may have adverse effects
in many patients. The most frequently reported are moodiness, difficulty sleeping, indigestion,
and weight gain. Long-term adverse effects are seen in patients requiring these drugs for months
and years at a time and include thinning of the bones and skin as well as suppression of the
hypothalamic-pituitary-adrenal axis, leading to Cushingoid habitus.

124. The underlying etiology of chronic sinusitis is:


a. Allergic
b. Infectious
c. Autoimmune
d. Unknown

d: While many different mechanisms have been proposed as the cause of chronic sinusitis, none
has been proven and debate continues. Only about half of chronic sinusitis patients prove to
have allergy upon testing. While immune system reactions to inhaled fungi and damage caused
by bacterial superantigens have been proposed, neither of these infectious etiologies has been
conclusively proven.

125. FESS is an acronym for which of the following?


a. Functional endoscopic sinus surgery
b. Fiberoptic endonasal sinus surgery
c. Fiberoptic endonasal sinusitis section
d. Found entirely substandard

a: In the early days of sinus surgery, the entire lining of the sinus would often be removed with a
curette, requiring extensive regrowth of mucosa. The cilia in this regrown mucosa were fewer in
number and less effective than the native cilia. The idea of “functional” endoscopic sinus surgery
is to remove only minimal amounts of the sinus membrane, with the goal of allowing the sinus
to recover and function normally.

120
126. Advantages of balloon sinuplasty over FESS may include all of the following EXCEPT:
a. Quicker recovery
b. More effective for advanced disease
c. May be done in a clinic setting
d. Lower costs to the health care system

b: The increasing popularity of using a balloon catheter to dilate the natural ostium of the maxil-
lary, frontal, or sphenoidal sinuses is likely due to its promise of less mucosal disruption and thus
a quicker, less painful recovery. It can be done under local anesthesia in an office situation in
many cases and thus provides lower overall costs to the health care system. Because it does not
remove diseased tissues but only dilates openings, it is incapable of dealing with advanced sinus
disease, such as extensive polyps.

127. The lesion marked with the star in this CT is a retention cyst. For which of the following
reasons would referral to an otolaryngologist be indicated?

a. The cyst extends into the upper part of the sinus.


b. There are inflammatory changes in the sinus.
c. There may be inflammatory changes in the contralateral sinus.
d. All of the above

d: While this clearly appears to be a noninflammatory retention cyst (based on its shape), it is
large enough that elevation may lift it into the drainage pathway of the sinus. Furthermore, there
are inflammatory changes elsewhere in the sinus, possibly due to obstruction from the cyst. The
contralateral sinus has some diffuse thickening that extends over a significant portion of the sinus,
possibly indicating a tendency for inflammation in the sinuses.

121
4 SURGICAL PROCEDURES AND COMPLICATIONS

128. What is the most likely disease process shown on this CT scan?

a. Sinus cancer
b. Acute sinusitis
c. Allergic fungal sinusitis
d. Viral upper respiratory infection

c: In allergic fungal sinusitis, the mucosa of the nose and sinuses react to inhaled fungal spores
with extreme swelling, forming nasal polyps that in turn trap the fungal elements and lead to
further swelling. It is classically unilateral, although both sides may be involved and often feature
areas of mixed densities, representing inspissated debris. There is sometimes dramatic expansile
erosion of the bone separating the sinuses from the orbit or skull base.

129. T
 he symptoms of a patient with the lesion indicated by the white arrow may include which of
the following?

a. Epistaxis
b. A whistling sound with nasal breathing
c. Mucus crusting
d. All of the above

d: A perforated nasal septum may cause nosebleeds (epistaxis), mucus crusting, and obstruction
and a whistling sound when breathing through the nose. The most common cause of septal
perforation is likely surgery (including cautery for epistaxis), although trauma and intranasal drug
use (both prescription and illicit) may also be to blame. A spontaneous septal perforation without
any of the above history should lead to a workup for autoimmune or granulomatous disease.

122
130. What does this CT suggest about the left maxillary sinus?

a. It has a bacterial infection.


b. It has undergone FESS.
c. There is ciliary dyskinesia.
d. It has a fungal ball.

 : The left maxillary sinus has undergone FESS as evidenced by the missing bone (uncinate process
b
and medial maxillary sinus wall) near its communication with the nose. There is no significant
mucosal change or other findings to suggest infection or dysfunction of the cilia.

131. A patient presents with complaints of right maxillary dental pain. On questioning, he admits
to additional symptoms of congestion, postnasal drainage, and low-grade fever. CT imaging
is obtained. What is the most likely diagnosis?

a. Acute sinusitis
b. Chronic sinusitis
c. Fungal sinusitis
d. Dental abscess

a: Acute sinusitis frequently evolves out of a viral upper respiratory infection (common cold) and
presents with fever, facial/dental pain, purulent sinus drainage, and nasal congestion. The CT
image demonstrates an air-fluid level consistent with fluid (pus) within the sinus; bubbles within
the fluid suggest a somewhat viscous fluid (also consistent with pus). In addition, there are signs
of inflammation in the contralateral sinus, although no air-fluid level is demonstrated. Chronic
sinusitis rarely presents with fever or pain.

123
4 SURGICAL PROCEDURES AND COMPLICATIONS

132. W
 hat features on the axial CT image suggest that this is a malignant process?

a. Posterior wall erosion


b. Medial wall and nasal septum erosion
c. Lack of inflammatory changes in the other visualized sinuses
d. All of the above

d: Malignant lesions can often be distinguished from benign lesions even on noncontrast CT
scans. The erosion of the posterior maxillary wall (white arrow) is an obvious area of aggressive
behavior, as is the missing medial wall and apparent invasion of the nasal septum. The lack of
inflammation in the other sinuses suggests that this is not an inflammatory process. Benign lesions
tend to expand or push the bony walls of the sinus rather than erode through them. This patient
needs additional imaging (CT with contrast and MRI) and expedited referral to an otolaryngologist.

133. C
 T imaging in an asymptomatic patient reveals a maxillary sinus that is completely opacified.
All other visualized sinuses are clear. The opacified sinus demonstrates mixed densities
throughout. After referral to an otolaryngologist, the patient undergoes FESS. When the
sinus is opened, thick peanut butter–consistency material is found to be filling the sinus.
After removal, the sinus lining appears healthy. What is the diagnosis?

a. Invasive fungal sinusitis


b. Allergic fungal sinusitis
c. Fungal ball
d. Sinus cancer

c : Fungal balls usually occur in otherwise healthy patients. Unlike invasive fungal sinusitis, there is no
evidence of invasion into the mucosa and the patient does not have a compromised immune system.
Fungal balls are usually isolated to one sinus, unlike allergic fungal sinusitis, which tends to affect multi-
ple sinuses and features extensive nasal polyps. Sinus cancer tends to appear as necrotic soft tissue
with obvious mucosal and bony invasion. The high-density areas in fungal sinusitis (both fungal balls
and allergic fungal sinusitis) are thought to be caused by heavy metals, such as iron and manganese,
rather than calcium. Complete physical removal of the fungal ball via FESS is usually curative.

124
134. A potential graft patient complains of nasal obstruction, so you look up his nose with an
otoscope. What are you looking at?

a. Nasal polyps
b. Inverted papilloma
c. Squamous cell carcinoma
d. A foreign body

a: Nasal polyps are benign inflammatory changes seen in some patients with chronic sinusitis.
They are pale, watery-appearing soft tissue masses that are insensate and often appear to be
extending from the middle meatus. By contrast, both benign (inverted papilloma) and malignant
(squamous cell carcinoma) tumors of the nose and sinuses tend to be more opaque, red, and
may bleed easily. Nasal polyps have no malignant potential and are usually managed with corti-
costeroids or removal via FESS when symptomatic.

135. Which of the following is the most common malignancy of the maxillary sinus?
a. Adenocarcinoma
b. Adenoid cystic carcinoma
c. Squamous cell carcinoma
d. Olfactory neuroblastoma

c : Arising from the mucosa, squamous cell carcinoma is usually treated with combined surgery and
chemoradiation. Adenocarcinoma is classically associated with the woodworking trade. Adenoid
cystic carcinoma is known for its tendency to spread along nerves and its late recurrence. Olfactory
neuroblastoma, also known as esthesioneuroblastoma, arises from the olfactory mucosa high
within the nasal cavity and only invades the maxillary sinus when there is extensive spread.

136. What is the most common location of origin for benign inverted papillomas of the
paranasal sinuses?
a. Frontal sinus
b. Ethmoidal sinus
c. Sphenoidal sinus
d. Maxillary sinus

 : Inverted papillomas are benign tumors that most commonly arise from the lateral nasal wall. About
d
54% begin in the maxillary sinus, 32% in the ethmoidal sinuses, and relatively few in the frontal or
sphenoidal sinuses. They are referred to as “inverted” because microscopically they demonstrate
invagination of the epithelium into the underlying stroma without invasion through the basement
membrane. Although benign, they can be locally aggressive, with frequent recurrence after surgery
and a reported 5% to 15% malignant transformation rate into squamous cell carcinoma.

125
4 SURGICAL PROCEDURES AND COMPLICATIONS

137. A
 patient who has undergone a left lateral window sinus elevation procedure returns to the
clinic 1 week postoperatively with complaints of decreased sensation over the left cheek and
upper lip. What is the most appropriate management?
a. Refer the patient immediately to an otolaryngologist for microscopic infraorbital nerve repair.
b. Refer the patient to a neurologist for electromyography.
c. Start the patient on gabapentin.
d. Reassure the patient and observe.

d: Despite meticulous technique and careful, blunt elevation of the soft tissues, injury to the
infraorbital nerve (second division of cranial nerve V) can occur. If the injury is mild (neuropraxia—
caused by compression or traction of the nerve), sensation typically returns within a month. More
severe injury (axonotmesis) may take weeks to months to resolve. Complete transection of the
nerve (neurotmesis) may require surgery, if recognized quickly, but prognosis is poor. The patient
in this scenario has only decreased sensation and is not completely numb, so observation and
reassurance is the best option.

138. A
 patient who underwent a sinus graft procedure 3 days ago calls to complain of sneezing
out blood with “little white pieces” in it from the ipsilateral nostril. What is the most
appropriate management?
a. Immediate graft removal though a lateral window approach
b. CT scan in 1 month
c. Nasal packing
d. Immediate referral to an otolaryngologist

b: Graft material that has extruded into the sinus cavity may be due to unrecognized membrane
perforation or overfilling. While the graft material may be heavy enough that the cilia of the sinus
membrane are unable to clear it out of the maxillary sinus and into the nose, there is no reason for
immediate intervention. Explain what has happened to the patient, wait for things to heal and the
process to stabilize, and repeat the CT in 4 to 6 weeks. If graft material persists within the sinus
cavity, consider referral to an otolaryngologist for possible endoscopic removal.

139. W
 hich of the following would NOT be an indication for preoperative referral to an
otolaryngologist for maxillary sinus evaluation?
a. Severe unilateral episodic facial pain with a normal CBCT
b. A history of prior FESS, nasal polyps, or chronic sinusitis
c. Inflammatory changes on preoperative CBCT
d. Foul nasal drainage unresponsive to antibiotics

a: A history of sinus problems or surgery, suspicious imaging, or findings on examination consis-
tent with sinus disease should result in consultation with an otolaryngologist, which may prevent
complications. It is less than ideal to have your patient’s first encounter with an otolaryngologist
be after you have operated.

126
140. TRUE OR FALSE: Radiosurgery is an atraumatic method of cutting and coagulating soft
tissue, without the postoperative pain and tissue destruction of electrocautery. The cutting
effect is performed without manual pressure and does not crush tissue cells.

True: The cutting effect, known as electrosection, results from heat generated by the tissues’
resistance to the radiofrequency wave, which is applied with a fine wire called a surgical electrode.
This generated heat then disintegrates among the cells in the path of the waves, essentially
vaporizing them. This causes the tissue to split apart as if cut with a knife but without the trauma
to the surrounding tissues and postoperative pain.
The lack of trauma associated with electrosection results in better tissue healing. An equally
significant advantage of radiosurgery is its sterilizing effect. Finally, it helps to eliminate the
postoperative sequelae (pain, swelling, infection, trismus, and postsurgical shock from excessive
blood loss) that are often experienced following manual instrumentation. Radiosurgery therefore
facilitates, accelerates, and improves surgical procedures.

141. TRUE OR FALSE: Radiosurgery can create tissue damage by facilitating the accumulation of
heat in the tissue to the point where excessive dehydration occurs and the tissue is destroyed.

True: Preventing the accumulation of such heat is the basic objective of the electrosurgical
technique. The accumulation of lateral heat in tissue depends on multiple factors—the amount
of time that the electrode contacts the tissue, the intensity of the power, the electrode size, the
nature of the wave, and the frequency—as indicated in the following formula:

Time of contact between electrode and tissue 


Intensity of power  Electrode size  Nature of wave
Lateral heat  
Frequency

142. The radiosurgical waveforms include which of the following?


a. Fully filtered wave
b. Fully rectified wave
c. Partially rectified wave
d. Spark-gap wave
e. a, b, and c
f. All of the above

f: All of the above

143. TRUE OR FALSE: The fully filtered waveform is a pure continuous flow of high-frequency
waves and provides a microsmooth cutting flow.

 rue: Under many clinical conditions, this continuous, nonpulsating flow of waves is advantageous,
T
producing the least amount of lateral heat and tissue destruction.

144. TRUE OR FALSE: The fully rectified waveform produces a small but perceptible pulsating effect,
which can, under certain circumstances, slightly increase the efficiency of the cutting effect.

False: This waveform does indeed produce a minute pulsating effect, but it slightly reduces the
efficiency of the cutting effect.

127
4 SURGICAL PROCEDURES AND COMPLICATIONS

145. T
 RUE OR FALSE: The partially rectified waveform is an intermittent flow of the
high-frequency waves producing hemostasis.

True: This waveform is very effective in sealing off hemorrhages up to 1⁄16 inch in diameter,
eliminating the need to tie them off. It is also recommended for the coagulation of blood vessels.

146. T
 RUE OR FALSE: The spark-gap wave (fulguration) is a mutated electronic current that has
been strengthened to simulate the effects of the Oudin current.

 alse: The spark-gap wave is a mutated electronic current that has been weakened to simulate the
F
effects of the Oudin current. This current produces a powerful dehydrating effect on the tissues.

147. T
 RUE OR FALSE: Radiosurgery should not be used on or by anyone who wears a pacemaker
without first consulting their physician to ensure that the pacemaker is protected and not
affected by high-frequency interference.

True

148. T
 RUE OR FALSE: When using radiosurgery, the tissue to be cut should be dry.

False: The tissue to be cut should be moist. If it is too dry, surface charring will occur. Excessively
dry tissue can most easily be moistened with wet gauze.

149. T
 RUE OR FALSE: During the actual cutting with radiosurgical electrodes, it is important
to use a smooth, uninterrupted motion with even and light pressure. The movement should
be slow.

False: It is indeed important to use a smooth, uninterrupted motion with even and light pressure.
However, the movement should not be too slow. If it is, the buildup of lateral heat in the tissue
may cause charring, followed by necrosis and sloughing.

128
PHARMACOLOGY 5

The population requiring dental implants is increasing due


to improved dental IQs and an increased life span. Older
patients often present with multiple pathologies and complex
pharmacologic treatments, and many pathologies and medi-
cations affect dental treatment. Therefore, clinicians must
have an in-depth knowledge of the patient’s medical history
and current ability to undergo complex treatments. The phar-
macology questions included in this chapter are intended
to help clinicians evaluate their knowledge in this discipline.

129
5 PHARMACOLOGY

1. What is the main purpose of surgical antibiotic prophylaxis?


a. To prevent infectious complications brought about by the surgical procedure
b. To protect against errors of omission or commission during the surgical procedure
c. To reduce the oral in situ bacterial count
d. To create a bacteria-free surgical site

a: The primary reason for administering presurgical antibiotics is to prevent the complication of
an infection that results from the introduction of bacteria during the surgical procedure.

2. What is the proper duration of surgical prophylaxis to minimize the risk of serious adverse
effects, the development of antibiotic resistance, and the proliferation of resistant bacteria?
a. The shortest possible duration (usually one preoperative dose)
b. A 3-day course of antibiotics
c. A 5-day course of antibiotics
d. A 7-day course of antibiotics

a: Surgical prophylaxis should have a high blood and tissue concentration during the surgical
procedure, and the antibiotic should be out of the system 24 hours after the last suture is placed.
Antibiotic blood and tissue levels beyond the 24-hour mark may lead to untoward side effects,
formation of immunoglobulin E antibodies to the antibiotic, development of resistant strains of
bacteria, and the overgrowth of nonsusceptible bacteria.

3. Which of the following statements is NOT true?


a. Pharmacodynamics is the action of a drug on the body (receptor interactions, dose-response
occurrences, mechanisms of action and toxic effects).
b. Pharmacokinetics is the action of the body on the drug (absorption, distribution, metabolism,
and elimination).
c. Permeation is the movement of a drug molecule into and within the biologic environment.
d. Drugs that bind to receptors with covalent bonds usually have reversible actions.

d: Some drugs do bind to receptors with covalent bonds, but once these bonds are made, the
drug effect is irreversible. Weaker electrostatic bonds, like those between cation and anions, are
much weaker and therefore usually reversible.

4. Which route of drug administration results in the highest blood level?


a. Oral
b. Intravenous
c. Intramuscular
d. Topical

b: Intravenous injection provides complete absorption of the drug. The bioavailability of the drug
to the body is 100%. The other routes of administration have factors that may interfere or alter
the absorption of the drug.

130
5. Which of the following statements is most correct regarding the first-pass effect on drugs
administered orally?
a. A significant amount of the drug is metabolized in the blood.
b. A significant amount of the drug is metabolized in the gut wall before it reaches the
systemic circulation.
c. A significant amount of the drug is metabolized in the portal circulation before it reaches
the systemic circulation.
d. A significant amount of the drug is metabolized in the gut wall, portal circulation, and liver
before it reaches the systemic circulation.

 : Drugs given orally are subject to absorption, which is often times slower than parenteral routes.
d
Oral drugs are therefore subject to the first-pass metabolism as they pass through the gut wall,
enter into the portal circulation, and pass through the liver. Thus, many drugs have low bioavail-
ability when administered orally.

6. Which of the following is true regarding the sublingual route of administration?


a. D
 irect absorption into the systemic venous circulation occurs, therefore bypassing the hepatic
portal circuit and first-pass metabolism.
b. Absorption is directly into the arterial circulation; therefore, the mechanism of action is rapid.
c. Drug distribution is usually limited and therefore not acceptable for drugs needing to cross
the blood-brain barrier.
d. Compound drugs are usually metabolized in part by the saliva; therefore, the sublingual route
of administration is a preferred route of administration for prodrugs.

a: The sublingual route provides for drug absorption directly into the venous circulation. Once
the drug has been distributed throughout the body, it will eventually be subject to the hepatic
portal circulation and liver metabolism.

7. A pharmacologic antagonist is:


a. A drug that activates a specific receptor upon binding.
b. A drug that binds to a receptor but does not activate it.
c. A drug that counters the effects of another drug by binding to a different receptor and
causing an opposing effect.
d. A drug that counters the effects of another drug by binding the drug and preventing
its action.

b: A drug that counters the effects of another drug by binding to a different receptor and caus-
ing an opposing effect is known as a physiologic antagonist. A drug that counters the effects of
another drug by binding the drug and preventing its action is known as a chemical antagonist.

8. A drug’s maximal efficacy refers to:


a. The maximal effectiveness of a drug when administered intravenously.
b. The quantity of a drug needed to produce the maximal effect.
c. The maximal effect a drug can produce.
d. The maximal amount of a drug that should be administered.

c: Efficacy is the maximal effect (Emax) a drug can produce regardless of dose. Drugs that are
full agonists produce the maximal effect; partial agonists have a lower Emax than full agonists.

131
5 PHARMACOLOGY

9. Drug potency refers to:


a. The amount of a drug needed to produce the desired effect.
b. How well a drug can produce the desired effect.
c. How the drug ranks in efficacy to comparable drugs in the same class.
d. The toxicity of a drug.

 : The desired effect is usually 50% of the maximal effect, and the dose causing this effect is referred
a
to as the EC50. Potency is in large part determined by the affinity the receptor has for the drug.

10. Inert binding sites are:


a. Receptors that exist for a drug that will induce a less-than-full antagonist response.
b. Endogenous receptor molecules that bind the drug without inducing any of the drug’s
desired effects.
c. Receptors that will not bind the drug.
d. Endogenous molecules that bind the drug but induce an antagonistic effect.

 : Inert binding sites are components of endogenous molecules that bind a drug without initiating
b
events leading to any of the drug’s effects. For example, plasma-binding sites play a significant
role in buffering a drug’s free concentration. Bound drug interferes with the free drug available
to drive the concentration gradient that allows for diffusion of the drug. Two plasma proteins that
bind drugs are albumin and orosomucoid (1-acid glycoprotein).

11. Which of the following antibiotics (at the concentration listed) can be mixed with autogenous,
allogeneic, or xenogeneic bone grafting materials and not affect the proliferation or
metabolic activity of primary human osteoblast (PHO) cells?
a. Lincomycin 300 mg per 2 mL
b. Clindamycin 300 mg per 2 mL
c. Cefazolin 2 g per 10 mL
d. Erythromycin 1,000 mg per 2 mL

a: Lincomycin does not interfere with PHO proliferation, nor does it induce cytotoxicity at any
concentration. Clindamycin and erythromycin demonstrate PHO cytotoxicity at the concentrations
listed. Cefazolin decreases proliferation of PHOs at the concentration listed.

12. W
 hich of the following antibiotics would be preferred for mixing with autogenous,
allogeneic, or xenogeneic bone grafting materials? Consideration should be given to the
ability of the antibiotic to inhibit anaerobic bacterial growth.
a. Cefazolin
b. Azithromycin
c. Lincomycin
d. Amoxicillin

c: Lincomycin has the greatest inhibitory action toward anaerobic bacteria of the choices given.
Because the bone graft will have no systemic circulation and will be in a state of hypoxia during
the initial healing period, anaerobic bacteria are a greater concern than aerobic bacteria.

132
13. The cumulative interceptive supportive therapy (CIST) protocol for the treatment of
peri-implant infections consists of which of the following steps in the order listed?
a. Antibiotic therapy, antiseptic therapy, mechanical debridement, and regenerative or resective
surgery
b. Antiseptic therapy, antibiotic therapy, mechanical debridement, and regenerative or resective
surgery
c. Mechanical debridement, antiseptic therapy, antibiotic therapy, and regenerative or resective
surgery
d. Localized anti-infective/glucocorticosteroid therapy, antibiotic therapy, mechanical debride-
ment, and regenerative or resective surgery

c : In 1988, the University of Bern School of Dental Medicine demonstrated that peri-implant infec-
tions can be successfully treated with four sequential steps: (1) mechanical debridement, (2) anti-
septic therapy, (3) antibiotic therapy, and (4) regenerative or resective surgery. (Sources: Mombelli
A, Lang NP. The diagnosis and treatment of peri-implantitis. Periodontol 2000 1998;17:63–76. /
Lang NP, Wilson TG, Corbet EF. Biological complications with dental implants: Their prevention,
diagnosis and treatment. Clin Oral Implants Res 2000;11[suppl 1]:146–155.)

14. According to the CIST protocol, localized antiseptic and systemic antibiotic therapy are
indicated for peri-implant infections that present with plaque deposits, bleeding on probing,
evidence of bone loss on radiographs, and pocket depths greater than:
a. 4 mm
b. 5 mm
c. 6 mm
d. 7 mm

b: 5 mm

15. When treating peri-implant infections, systemic antibiotic therapy should be directed toward
the reduction of which type of bacteria?
a. Gram-negative anaerobes
b. Gram-negative aerobes
c. Gram-positive anaerobes
d. Gram-positive aerobes

a: Gram-negative anaerobes

16. What is the preferred systemic antibiotic for a peri-implant infection?


a. Amoxicillin
b. Azithromycin metronidazole
c. Clindamycin
d. Metronidazole

d: Of the antibiotics listed, metronidazole possesses the greatest activity against the type of
bacteria implicated in peri-implant infections, gram-negative anaerobes.

133
5 PHARMACOLOGY

17. S
 ystemic antibiotics will not penetrate the biofilm on a dental implant. Therefore, which of
the following is necessary prior to initiating systemic antibiotic therapy?
a. Mechanical debridement
b. Antiseptic therapy
c. Regenerative or resective surgery
d. None of the above

 : Because systemically administered antibiotics will not penetrate the biofilm on a dental implant,
a
it is necessary to thoroughly debride the biofilm on the surface of the implant prior to initiating
systemic antibiotic therapy.

18. S
 ystemic antibiotic therapy used in the treatment of peri-implant infections should be
continued for a minimum of how many days?
a. 7 days
b. 10 days
c. 14 days
d. 21 days

b: Mombelli and Lang suggest that when metronidazole is used as a single agent, the therapy
should be prescribed as 250 mg three times a day for a minimum of 10 days. (Source: Mombelli
A, Lang NP. The diagnosis and treatment of peri-implantitis. Periodontol 2000 1998;17:63–76.)

19. T
 he administration of local antibiotics delivered through a localized device has emerged as
a suitable therapy for peri-implant infections. The delivery device must release the drug at a
concentration sufficient to penetrate the submucosal biofilm over how many days?
a. 3–5
b. 5–7
c. 7–10
d. 12–14

c: The controlled-delivery device must guarantee adequate release kinetics to ensure efficacious
results. The antibiotic must remain at the site of desired action for a minimum of 7 to 10 days and
deliver a concentration sufficient to infiltrate the submucosal biofilm.

20. W
 hen administered orally, low-dose doxycycline (Periostat, CollaGenex) produces its effect
by which of the following mechanisms?
a. Interfering with bacterial cell wall synthesis
b. Interfering with DNA gyrase involved in bacterial proliferation
c. Interfering with the body’s production of matrix metalloproteinases (MMPs)
d. Interfering with the body’s production of prostaglandin E2 (PGE2)

c: Low-dose doxycycline is an MMP inhibitor that decreases laminin-5 levels. MMPs are anti-
inflammatory, but they also induce the fragmentation of epithelial cell–derived adhesion protein
(laminin-5). Laminin-5 is known to induce the apical migration of epithelial cells during the devel-
opment of periodontal pockets. Therefore, by inhibiting MMPs and laminin-5 production, doxy-
cycline decreases the tendency for apical migration of epithelial cells and further development
of periodontal pockets.

134
21. In the repair of bone, mitogenic and cell-differentiation growth factors are known to act on
osteoblast precursors. These osteoblast precursor cells are derived:
a. P rimarily from the adjacent periosteum and secondarily from the bone marrow, small blood
vessels, and fibroblasts.
b. Primarily from the adjacent vital bone and secondarily from the dense granules of platelets.
c. Primarily from extravascular blood that fills the surgical site and secondarily from the
bone marrow.
d. Primarily from the inflammatory cells that migrate into the area and secondarily from the
surrounding blood supply.

 : The primary source for mitogenic and cell-differentiation growth factors is the inner surface of
a
the periosteum (cambium layer); secondary sources include the existing bone marrow cells, small
blood vessels, and fibroblasts. The alpha granules, not the dense granules, found in platelets do
provide some mitogenic and cell-differentiation growth factors but not the osteoblast precursor cells.

22. Traditional nonsteroidal anti-inflammatory drugs (tNSAIDs) inhibit cyclooxygenase 1 and


2 (COX-1 and COX-2). Which of the following drugs is not a COX-1 and COX-2 inhibitor?
a. Ibuprofen
b. Celecoxib
c. Naproxen
d. Flurbiprofen

b: Celecoxib is a selective COX-2 inhibitor.

23. Which of the following prostaglandins has a role in early bone healing or formation?
a. PGE2
b. PGI2
c. PGE1
d. None of the above

a: PGE2 has been detected at elevated levels in the early stages of bone repair. PGE1 has vasodi-
lation properties, and PGI2 (prostacyclin) has a role in improved renal function but not bone repair.

24. Which tNSAID has been reported to enhance bone growth or has been referred to as
“bone sparing”?
a. Ibuprofen
b. Celecoxib
c. Naproxen
d. Flurbiprofen

d: Li et al (1989) reported that flurbiprofen enhanced bone growth in animals or was at least
bone sparing. Subcutaneous injections induced anabolic changes in rapidly growing long bone
by reducing osteoclast activity. Cortical bone mass was increased by stimulating periosteal bone
growth, and cancellous bone mass was increased by depressing trabecular bone resorption without
altering bone formation. Jeffcoat et al (1995) found that patients taking flurbiprofen 100 mg twice
a day experienced approximately half the bone loss when compared to low-dose flurbiprofen or
placebo groups. No other NSAIDs have demonstrated this bone-sparing effect. (Sources: Li XJ,
Jee WS, Li YL. Flurbiprofen enhances growth and cancellous and cortical bone accumulation in
rapidly growing long bone. Bone 1989;10:35–44. / Jeffcoat MK, Reddy MS, Wang IC, Meuninghoff
LA, Farmer JB, Koth DL. The effect of systemic flurbiprofen on bone supporting dental implants.
J Am Dent Assoc 1995;126:305–311.)

135
5 PHARMACOLOGY

25. Which of the following is NOT a selective COX-2 inhibitor?


a. Metoprolol (Lopressor, Novartis)
b. Valdecoxib (Bextra, Pfizer)
c. Meloxicam (Mobic, Boehringer Ingelheim)
d. Celecoxib (Celebrex, Pfizer)

a: Metoprolol is a specific 2 inhibitor used to treat angina and high blood pressure. Note that
Bextra was withdrawn from the market in 2007.

26. For patients who present with a history of taking bisphosphonates or monoclonal
antibodies (for the treatment of osteopenia or osteoporosis), the best way to evaluate if the
pharmacologic agents have affected bone strength is by examining which of the following?
a. Bone mineral density (BMD) and bone quality (bone turnover markers [BTMs])
b. BMD and cortical bone thickness on a CT scan
c. BMD and percentage of cancellous bone relative to cortical bone
d. BMD and Hounsfield units (HUs) measurement on a CT scan

a: The best method for determining bone strength is by examining BMD via a dual-energy x-ray
absorptiometry (DXA) scan and obtaining BTMs (serum C-terminal telopeptide [CTX], serum
N-terminal telopeptide [NTX], and serum bone-specific alkaline phosphatase [sBSAP]). BMD is
expressed as grams of mineral per area or volume and is defined by the individual peak bone
density and the resorption rate in relation to the peak. Bone quality can be determined by features
of the bone matrix such as the microarchitecture, bone turnover, microdamage accumulation,
degree of calcification, and collagen content. BTMs provide indications of changes in bone quality
earlier than BMD values. To assess bone quality, BTMs must be measured.

27. S
 uffixes provided in drug names can provide clues for determining the drug class.
The suffix “azole” indicates which drug class?
a. Azole-type antifungal or antibacterial-antiparasitic drug
b. Local anesthetic
c. Serotonin 5-HT1B/1D agonist antimigraine drug
d. Angiotensin II receptor blocker

a: The suffix “azole” indicates an azole-type antifungal (eg, fluconazole) or an antibacterial-
antiparasitic drug (eg, metronidazole). The suffix “caine” indicates a local anesthetic (eg, lidocaine),
“triptan” a serotonin 5-HT1B/1D agonist antimigraine drug (eg, sumatriptan), and “sartan” an
angiotensin II receptor blocker (eg, losartan).

28. Which drug class is indicated by the suffix “coxib”?


a. Dihydropyridine calcium channel blocker
b. Monocolonal antibody
c. Selective COX-2 inhibitor
d. Angiotensin-converting enzyme (ACE) inhibitor

c: The suffix “coxib” indicates a selective COX-2 inhibitor (eg, celecoxib). The suffix “dipine”
indicates a dihydropyridine calcium channel blocker (eg, nifedipine), “mab” a monocolonal
antibody (eg, infliximab), and “pril” or “prilat” an ACE inhibitor (eg, captopril).

136
29. Which drug class is indicated by the suffix “olol”?
a. 1-adrenergic receptor blocker
b. -adrenergic receptor blocker
c. -adrenergic receptor blocker that also blocks the 1-adrenergic receptor
d. Quaternary ammonium compound used as a peripheral competitive skeletal muscle relaxer

b: The suffix “olol” indicates a -adrenergic receptor blocker (eg, propranolol). The suffix
“osin” indicates an 1-adrenergic receptor blocker (eg, prazosin), “ilol” or “alol” a -adrenergic
receptor blocker that also blocks the 1-adrenergic receptor (eg, carvedilol, labetalol), and
“onium” or “urium” a quaternary ammonium compound used as a peripheral competitive skeletal
muscle relaxer (eg, pancuronium, atracurium).

30. Which drug class is indicated by the suffix “pam” or “lam”?


a. HMG CoA reductase inhibitor antilipid drug
b. Antiviral drug
c. 1-adrenergic receptor blocker
d. Benzodiazepine antianxiety agent or sedative-hypnotic drug

d: The suffix “pam” or “lam” indicates a benzodiazepine antianxiety agent or sedative-
hypnotic drug (eg, diazepam, triazolam). The suffix “statin” indicates an HMG CoA reductase
inhibitor antilipid drug (eg, lovastatin), “vir” an antiviral drug (eg, acyclovir), and “osin” an
1-adrenergic receptor blocker (eg, prazosin).

31. Local anesthetics produce their principal numbing effect by which mechanism of action?
a. Block neuronal K+ channels
b. Block neuronal Na+ channels
c. Block neuronal Cl– channels
d. Block neuronal Ca2+ channels

 : Conventional local anesthetics directly interact with and block neuronal Na+ channels to inhibit
b
nerve conduction. Due to similarities between Na+, Ca2+, and K+ voltage-gated ion channels, there
is some inhibition of specific K+ and Ca2+ currents, which may lead to blockage of nociception.

32. Failure to obtain suitable pain relief with a single or multiple local anesthetic injection(s)
in inflamed tissue may be due to which of the following?
a. Inflammation decreases the pH at the operative site and thereby interferes with the diffusion
of the local anesthetic into the axolemma of the nerve.
b. Increased blood flow in the area carries the local anesthetic away from the area.
c. Decreased effectiveness of the added catecholamine (epinephrine) results in less
vasoconstriction to retain the local anesthetic at the intended site.
d. All of the above

 : Additionally, changes in the Na+ channel number, function, or type may interfere with the local
d
anesthetic’s ability to block nerve conduction. Changes in the expression or activity of the ion
channels may also have an effect.

137
5 PHARMACOLOGY

33. A
 1.8-mL carpule containing 2% lidocaine with epinephrine 1:100,000 would have how many
milligrams of lidocaine in it?
a. 1.8 mg
b. 18 mg
c. 3.6 mg
d. 36 mg

d: The 2% is a weight/volume (w/v) percentage, so a 2% lidocaine solution contains 2 g of lido-


caine per 100 mL. You can use a ratio to determine the quantity of lidocaine in a 1.8-mL carpule:
n grams / 1.8 mL = 2 g / 100 mL. This gives 100n = 3.6, or n = 0.036 g (36 mg).

34. A
 1.8-mL carpule containing 2% lidocaine and epinephrine 1:100,000 would have how many
micrograms (μg) of epinephrine in it?
a. 1.8 μg
b. 18 μg
c. 3.6 μg
d. 36 μg

b: The 1:100,000 is a w/v ratio, so a 1:100,000 solution contains 1 g (1,000 mg) of epinephrine
in 100,000 mL. You can use a ratio to determine the quantity of epinephrine in a 1.8-mL carpule:
n milligrams of epinephrine / 1.8 mL = 1,000 mg / 100,000 mL. This gives 100n = 1.8 (reduced
from 100,000n = 1,800), or n = 0.018 mg (18 μg).

35. A
 1.8-mL carpule containing 4% articaine and epinephrine 1:200,000 would have how many
micrograms (μg) of epinephrine in it?
a. 9 μg
b. 18 μg
c. 3.6 μg
d. 36 μg

a: A 1:200,000 solution contains 1 g (1,000 mg) of epinephrine in 200,000 mL. You can use a ratio
to determine the quantity of epinephrine in a 1.8-mL carpule: n milligrams / 1.8 mL = 1,000 mg
/ 200,000 mL. This gives 200n = 1.8 (reduced from 200,000n = 1,800), or n = 0.009 mg (9 μg).

36. A 1.8-mL carpule containing 2% lidocaine and epinephrine 1:50,000 would have how many
micrograms (μg) of epinephrine in it?
a. 18 μg
b. 180 μg
c. 36 μg
d. 360 μg

c: A 1:50,000 solution contains 1 g (1,000 mg) of epinephrine in 50,000 mL. You can use a ratio
to determine the quantity of epinephrine in a 1.8-mL carpule: n milligrams / 1.8 mL = 1,000 mg /
50,000 mL. This gives 50n = 1.8 (reduced from 50,000n = 1,800), or n = 0.036 mg (36 μg).

138
37. A 1.8-mL carpule containing 4% articaine would have how many milligrams of articaine in it?
a. 18 mg
b. 36 mg
c. 72 mg
d. 720 mg

c: A 4% articaine solution contains 4 g of articaine per 100 mL. You can use a ratio to determine
the quantity of articaine in a 1.8-mL carpule: n grams / 1.8 mL = 4 g / 100 mL. This gives 100n =
7.2, or n = 0.072 g (72 mg).

38. A 1.8-mL carpule containing 0.5% bupivacaine would have how many milligrams of
bupivacaine in it?
a. 0.9 mg
b. 9 mg
c. 18 mg
d. 36 mg

b: A 0.5% bupivacaine solution contains 0.5 g of bupivacaine per 100 mL. You can use a ratio to
determine the quantity of bupivacaine in a 1.8-mL carpule: n grams / 1.8 mL = 0.5 g / 100 mL.
This gives 100n = 0.9, or n = 0.009 g (9 mg).

39. The recommended maximum dose for lidocaine (with epinephrine 1:100,000) is always
the smaller of two values: 7 mg/kg or up to a maximum dose of 500 mg. Therefore, what is
the maximum dose that should be given to a 146-lb woman without cardiac disease?
(Note: Round to the nearest tenth.)
a. 260 mg
b. 315 mg
c. 385 mg
d. 465 mg

d: Calculation is as follows: The patient weighs 146 lb. To convert this to kg, divide by
2.2 (1 kg = 2.2 lb). The patient weighs 66.4 kg. Multiply this by 7 mg/kg (maximum lidocaine dose
is 7 mg/kg) to get a maximum dose of 464.8 mg (or 465 mg).

40. If the total recommended safe dose for lidocaine for a 146-lb woman is 465 mg
(146 lbs ÷ 2.2 lbs/kg = 66.4 kg × 7 mg/kg = 464.8 mg), how many 1.8-mL carpules of
2% lidocaine with epinephrine 1:100,000 does this represent?
a. 10
b. 11
c. 12
d. 13

d: Calculation is as follows: The total dose that can be given is 464.8 mg (rounded to 465 mg).
If each carpule of 2% lidocaine with epinephrine 1:100,000 contains 36 mg of lidocaine (see
question #33), then you can use the ratio 36 mg / 1 carpule = 465 mg / n carpules. This gives
36n = 465, or n = 12.92 (13 carpules).

139
5 PHARMACOLOGY

41. The epinephrine in a dental local anesthesia carpule is known to activate which receptor?
a. a1
b. b1
c. b2
d. All of the above

 : Epinephrine will activate all three receptors listed. Activation of a1 will result in vasoconstriction.
d
Activation of b1 will produce an increased heart rate and increased cardiac force of contraction.
Activation of b2 will produce vasodilation and bronchodilation.

42. Allergic reactions to local anesthetics are rare. However, some individuals are more susceptible
to local anesthetics that contain epinephrine. These individuals who are more susceptible to
such allergic reactions have a history of:
a. Rheumatoid arthritis
b. Systemic lupus erythematosus
c. Sjögren’s syndrome autoimmune disease
d. Asthma

d: Asthmatic patients oftentimes cannot tolerate sulfites, bisulfites, and metabisulfite preserva-
tives. Metabisulfite preservatives are used as antioxidants in local anesthetic dental carpules that
include epinephrine. Originally it was thought that the reaction had an immunologic basis, but
more recent thinking suggests that those individuals who have a reaction are hyperreactive to
sulfites that are inhaled or ingested but not injected.

43. W
 hen administering local anesthetics by injection, it is important to aspirate the syringe to
ensure that the local anesthetic is not being injected intravascularly. It is well known that the
maximum safe dose for lidocaine is 7 mg/kg, but what is the toxic dose when lidocaine is
injected intravascularly?
a. 1 mg
b. 10 mg
c. 100 mg
d. 1,000 mg

b: Patients can experience a toxic reaction to amide local anesthetics at doses lower than
expected. Such reactions may be the result of an abnormal or idiosyncratic reaction. They
can be the result of anxiety or inadvertent intravascular injection. The convulsant dose of
lidocaine when injected into the vertebral artery is 10 mg. Therefore, it is important to aspirate
the syringe when injecting; if the aspiration is positive, the needle must be removed and a new
dental local anesthestic carpule used for the intended injection.

140
44. The seizures produced by local anesthesia are normally self-limiting. However, when the
seizures are intense or prolonged, then hypoxia becomes possible, and it is necessary to
administer a benzodiazepine to stop the seizure. The most effective route of administration
is intravenously. Which of the following dosages would be appropriate to administer to
control an ongoing local anesthesia–induced grand mal seizure without producing significant
depressive effects on ventilation or circulation?
a. Intravenous diazepam (0.01–0.03 mg/kg) or intravenous midazolam (0.003–0.01 mg/kg)
b. Intravenous diazepam (0.1–0.3 mg/kg) or intravenous midazolam (0.03–0.1 mg/kg)
c. Intravenous diazepam (0.8 mg/kg) or intravenous midazolam (0.5 mg/kg)
d. Intravenous diazepam (1 mg/kg) or intravenous midazolam (0.3 mg/kg)

b: Convulsions are usually self-limiting, and the only treatment normally required is to support
ventilation and protect the patient from harming his or her body during the seizure. However,
should the seizure become a status seizure or very intense, then a benzodiazepine should
be administered intravenously at a recommended dose of 0.1–0.3 mg/kg for diazepam or
0.03–0.1 mg/kg for midazolam.

45. Which of the following medications, when administered preoperatively, can increase the
likelihood of seizures when local anesthesia is administered?
a. Meperidine
b. Diazepam
c. Midazolam
d. Diphenhydramine

a: Many medications have been administered prophylactically to prevent seizures induced by
local anesthesia. Diazepam and midazolam both reduce the likelihood of local anesthesia–
induced seizures. Meperidine and promethazine are known to increase this likelihood.

46. The accidental injection of a local anesthetic into a neuron may result in nerve damage.
This nerve damage can be attributed to which of the following? (MULTIPLE ANSWERS)
a. Undiluted local anesthetic (particularly 4% concentrations)
b. Allergic reaction
c. Hydrostatic pressure of the injection
d. Direct nerve injury
e. All of the above

 , c, d: High-concentration solutions are more likely to cause a long-lasting or permanent nerve


a
injury when administered as an inferior alveolar nerve block. Unsheathed neurons exposed to 4%
concentrations experience an irreversible increase in intracellular Ca2+ and necrotic cell death.

47. Most local anesthetic–related deaths are the result of:


a. Tissue anoxia
b. Bradycardia
c. Increased systolic blood pressure
d. Pulmonary edema

 : Most deaths related to local anesthesia administration are the result of tissue anoxia. Therefore,
a
support of ventilation is of paramount importance. Circulation must also be supported due to
arterial hypotension via intravenous fluids and sympathomimetic agents. CPR is necessary when
cardiac function is impaired.

141
5 PHARMACOLOGY

48. W
 hen a patient reports allergies to all conventional local anesthetics, which of the following
is a suitable alternative?
a. Promethazine 2% with epinephrine 1:50,000
b. Diphenhydramine 1% with epinephrine 1:100,000
c. Chlorpromazine 2% with epinephrine 1:200,000
d. Meperidine 3% with epinephrine 1:100,000

b: Diphenhydramine 1% with epinephrine 1:100,000 has been used as a local anesthetic with
some success. Other antihistamines have been used as well. Drug allergy screening is an alterna-
tive approach. However, sensitivity testing is not very reliable and may be potentially dangerous.
It should be noted that a regimen of intracutaneous injections under properly supervised medical
conditions has proven to be successful in identifying local anesthetic solutions that can be used
safely. (Source: deShazo RD, Nelson HS. An approach to the patient with a history of local anes-
thetic hypersensitivity: Experience with 90 patients. J Allergy Clin Immunol 1979;63:387–394.)

49. W
 hich of the following enzymes primarily metabolizes the exogenous epinephrine that is
administered with a local anesthetic solution?
a. Monoamine oxidase-A (MAO-A)
b. Monoamine oxidase-B (MAO-B)
c. Catechol-O-methyl transferase (COMT)
d. Phospholipase A2

c: Patients who take COMT inhibitors (entacapone, tolcapone) for Parkinson disease may experi-
ence prolonged numbness due to the inhibition of COMT and therefore a reduced metabolism
rate of the epinephrine.

50. W
 hich of the following generic to trade name conversions is NOT correct?
a. Lidocaine = Xylocaine (AstraZeneca)
b. Mepivacaine = Carbocaine (Hospira)
c. Bupivacaine = Marcaine (Hospira)
d. Prilocaine = Duranest (AstraZeneca)

d: Prilocaine is Citanest (AstraZeneca). It is an amide local anesthetic that comes in a 4% concen-
tration without epinephrine or with epinephrine 1:200,000. Duranest (generic etidocaine) is a
long-acting amide local anesthetic. It comes in a 1% concentration without epinephrine or 1% or
1.5% with epinephrine 1:200,000.

51. W
 hat is the mechanism of action of b-lactam antibiotics (eg, penicillin, amoxicillin)?
a. Alteration in cell membrane integrity
b. Inhibition of ribosomal protein synthesis
c. Cell wall synthesis inhibitors
d. Inhibition of nucleic acid synthesis

c : Bacterial cell walls are comprised of alternating peptidoglycan units of N-acetyl-D-glucosamine


and N-acetylmuramic acid. The units are cross-linked with short peptides. b-lactam antibiotics
interfere with the cross-linking of the units. Polymyxin-B interferes with bacterial cell membrane
integrity. Macrolide antibiotics (erythromycin and clindamycin) inhibit bacterial ribosomal protein
synthesis by binding to the 50S ribosomal subunit. Metronidazole and fluoroquinolones inhibit
nucleic acid synthesis. Sulfonamides and trimethoprim inhibit bacterial folic acid synthesis.

142
52. When a patient with no known drug allergies presents with a swelling that is of less than
24 hours’ duration, which of the following should be prescribed once the swelling is incised
and drained (if possible)?
a. Amoxicillin 1 g initially and then 500 mg every 8 hours for 7 to 10 days, or cephalexin 1 g
initially and then 500 mg every 6 hours for 7 to 10 days
b. Ciprofloxacin 1 g initially and then 500 mg every 12 hours for 10 days
c. Metronidazole 500 mg initially and then 250 mg every 8 hours for 14 days
d. Clindamycin 300 mg initially and then 150 mg every 6 hours for 7 or 10 days

d: You should start with a b-lactam antibiotic that has activity against gram-positive aerobic and
gram-negative anaerobic bacteria. When possible, it is best to obtain a culture prior to initiating
antibiotic therapy. It is always desirable to reduce the inoculum size of the infecting organism(s)
by surgical drainage. Naturally, if an incision and drainage are not possible, go directly to the
antibiotic and/or make a referral to the appropriate specialist.

53. Which of the following is a true statement regarding the appropriate prescribing of antibiotics?
a. Prolonged antibiotic therapy obliterates resistant bacteria.
b. Prolonged antibiotic therapy is essential to prevent oral rebound infections.
c. Prolonged antibiotic therapy can be used to prevent an infection.
d. None of the above

d: These are all misconceptions with no scientific validity.

54. What is the cornerstone of the therapeutic approach to treating an abscess?


a. Administration of high-dose antibiotics for a minimum of 10 days
b. Surgical drainage
c. Application of moist heat to the site of infection
d. Surgical removal of the offending tooth, implant, or object

b: Cunha and Ortega (1995) have stated that patients with abscesses will not be healed
with antibiotics alone. Surgical drainage is required for complete healing. (Source: Cunha BA,
Ortega AM. Antibiotic failure. Med Clin North Am 1995;79:633–672.)

55. For the pregnant patient, which antibiotics are contraindicated? (MULTIPLE ANSWERS)
a. Amoxicillin
b. Tetracyclines
c. Erythromycin estolate
d. Metronidazole
e. Clindamycin
f. All of the above

 , c, d: Tetracyclines are contraindicated because of their tooth-staining effects and hepatotoxicity.


b
Erythromycin estolate is contraindicated because it can induce cholestatic hepatitis. Metronidazole
is contraindicated because it has an effect on DNA synthesis and therefore is suspected of having
teratogenic, mutagenic, or carcinogenic effects. The same is true of fluoroquinolones.

143
5 PHARMACOLOGY

56. W
 hat is the antibiotic of choice for patients who acquired Clostridium difficile–associated
colitis (CDAC) or pseudomembranous colitis (PMC) from antibiotic therapy?
a. Vancomycin 125 mg orally four times a day for 10 days
b. Ciprofloxacin 500 mg twice a day for 10 days
c. Azithromycin 500 mg daily for 5 days
d. Metronidazole 250 mg four times a day or 500 mg three times a day

d: Vancomycin is reserved for cases that are refractory to metronidazole. Azithromycin and
ciprofloxacin are not indicated for the treatment of CDAC or PMC.

57. Which of the following statements is true regarding cephalosporins for patients with a history
of a mild skin reaction to penicillin?
a. It is reasonably safe to give cephalosporins to these patients.
b. If a patient has an allergic reaction to one generation of cephalosporins, it is okay to give
that patient a different generation without fear of an allergic reaction.
c. These patients should not be given a cephalosporin.
d. Patients who develop diarrhea with penicillin should not be given a first-generation
cephalosporin.

 : It has been generally agreed that patients who have just a mild skin reaction or a positive penicilloyl-
a
polylysine skin test to penicillin can be given a cephalosporin. Penicillin-allergic patients have a
fourfold greater risk of allergy to cephalosporins than non–penicillin allergic patients. However,
penicillin-allergic patients have a fourfold greater risk of allergy to ALL medications, and there are
no reliable skin tests to detect a cephalosporin allergy or standardized desensitization processes.

58. W
 hich of the following macrolides does NOT affect the liver microsomes?
a. Azithromycin
b. Clarithromycin
c. Erythromycin freebase
d. Erythromycin estolate

a: Erythromycin and clarithromycin inhibit liver microsomal enzyme drug-metabolizing pathways.
Azithromycin and dirithromycin do not have an effect on the liver microsomes.

59. W
 hich class of antibiotics can increase the myopathy and rhabdomyolysis that can occur with
statin anticholesterol drugs?
a. Macrolides
b. Tetracyclines
c. Fluoroquinolones/quinolones
d. b-lactams

a: Macrolides

144
60. All macrolide antibiotics can significantly reduce cardiac glycoside metabolism in the
gastrointestinal tract, resulting in toxic levels of digitalis. How many days of simultaneous
administration does it take to reach a toxic state?
a. 1 day
b. 3 days
c. 5 days
d. 7 days

b: It only takes 3 days of simultaneous digoxin and macrolide antibiotic administration to reach a
state of digitalis toxicity. The macrolide antibiotics can interfere with Eggerthella lenta’s metabolism
of the cardiac glycoside. Eggerthella lenta normally metabolizes 30% to 40% of the drug. Once
its population has been reduced by the macrolide, there is more of the intact cardiac glycoside
to be absorbed. Digoxin or digitalis has a narrow therapeutic index, and if the absorbed drug is
increased by 30% to 40%, the available digoxin or digitalis approaches the toxic range.

61. Patients who are taking antitumor medications are susceptible to which of the following
oral conditions?
a. Stomatitis
b. Rapid caries progression and periodontal bone loss
c. Hemorrhage
d. Acute and chronic infection
e. All of the above

e: This includes all antineoplastic chemotherapeutic drugs (alkylating agents, antimetabolites,
vinca alkaloids, antibiotics, hormone agonists and antagonists, enzymes, platinum complexes,
DNA demethylation agents, biologic response modifiers, protectants, or combinations of these).

62. Which of the following analgesics has a ceiling effect for analgesia?
a. Hydrocodone
b. Oxycodone
c. Codeine
d. Ibuprofen

d: All NSAIDs, including ibuprofen, have a ceiling effect for analgesia; the other drugs listed here
are opioid narcotics, which do not have a ceiling analgesic effect. Opioids possess unlimited effi-
cacy for pain relief. Provided that large-enough opioid doses are given to the patient, analgesia
will result at some point. However, the dose may be so excessive that unwanted or life-threatening
side effects may occur.

63. Which of the following analgesics does NOT produce tolerance or physical dependence?
a. Hydrocodone
b. Oxycodone
c. Codeine
d. Ibuprofen

d: Over time, opioid narcotics will produce tolerance and physical dependence.

145
5 PHARMACOLOGY

64. If a patient does not receive the desired pain relief from one NSAID, it would be acceptable
to do which of the following?
a. Switch the patient to a different NSAID.
b. Add acetaminophen to the analgesic regimen.
c. Add a narcotic-acetaminophen combination.
d. Any of the above

d: Any of the above

65. W
 hich of the following would be the drug of choice for a patient experiencing an angina
pectoris attack?
a. Oxygen
b. Nitroglycerin
c. Albuterol inhaler
d. Aspirin

b: Nitroglycerin relaxes vascular smooth muscles, which results in dilation of systemic venous and
arterial vessels. This in turn leads to a reduction in venous return and systemic vascular resistance.
All of these actions result in reduced myocardial work and oxygen consumption. The administration
of supplemental oxygen would also be of benefit. If the angina attack appears to be progressing
into a myocardial infarction, the administration of 325 mg aspirin would be beneficial, provided
that aspirin is not contraindicated for the patient.

66. W
 hich of the following is a contraindication for the administration of aspirin to a patient
undergoing a myocardial infarction?
a. The patient is already taking 81 mg of aspirin daily for the prevention of stroke or
myocardial infarction.
b. The patient has an aspirin intolerance and suffers an asthmatic attack when he or she
takes aspirin.
c. The patient takes Celebrex 200 mg twice a day.
d. The patient has a history of developing a rash after taking ibuprofen 600 mg four times a
day for 30 days.

b: Aspirin 325 mg should be given to a patient experiencing an evolving myocardial infarction.
The antiplatelet effect of aspirin will decrease the myocardial ischemia. Contraindications to
administering aspirin are (1) patients who have aspirin intolerance and (2) patients with a severe
bleeding disorder.

67. W
 hich of the following devices should NOT be used to administer oxygen to a patient who is
spontaneously breathing without difficulty?
a. Bag-valve-mask (BVM)
b. Nasal cannula
c. Full face mask
d. Nasal hood

a: The nasal cannula, full face mask, and nasal hood can be used to administer oxygen to a
patient who is spontaneously breathing without difficulty. The BVM should be used for the patient
who is either not spontaneously breathing or having difficulty breathing. These patients require
positive-pressure ventilation.

146
68. Which of the following medications would NOT be acceptable to give an unconscious patient
in the dental office with a history of insulin-dependent diabetes mellitus?
a. Insulin subcutaneously
b. Glucagon intramuscularly or intravenously
c. 50% glucose intravenously
d. All of the above

a: If a diabetic patient is unconscious in the dental office, you should suspect an insulin-
carbohydrate imbalance. If the patient is conscious, then providing him or her with an oral glucose
form would be acceptable. However, if the patient is unconscious, then the glucose must be
given intravenously, or glucagon via intramuscular or intravenous injection should be given to
stimulate the release of glucose from glycogen in the liver. The administration of insulin during
an emergency in the dental setting is never a good choice. You should always assume that in a
diabetic emergency, the patient is suffering from hypoglycemia, not hyperglycemia.

69. Acute bronchospasm in the dental office should be treated with which of the following?
a. Beclomethasone oral inhaler
b. Epinephrine 1 mg subcutaneously
c. Fluticasone inhaler
d. Albuterol inhaler

d: Albuterol is a b2-adrenergic agonist that will produce bronchial smooth muscle relaxation. It
will also inhibit the release of chemical mediators normally released during hypersensitivity reac-
tions. Beclomethasone and fluticasone inhalers are glucocorticosteroid inhalers and are used by
asthmatics for maintenance. Epinephrine 1 mg is the dose used for cardiac arrest; the dose for
bronchospasm would be 0.3 to 0.5 mg. The initial treatment for bronchospasm should be the
use of an albuterol inhaler; if the patient is unconscious or unable to cooperate with the use of
an inhaler, then the subcutaneous injection of epinephrine 0.3 to 0.5 mg would be acceptable.

70. Epinephrine, when given at resuscitative doses, provides which of the following benefits?
a. Increased myocardial and cerebral blood flow
b. Increased systemic vascular resistance and arterial blood pressure
c. Increased heart rate and myocardial contractility
d. All of the above

d: Epinephrine increases vascular resistance and arterial blood pressure due to the activation of
a receptors causing vasoconstriction. It increases heart rate and myocardial contractility due to b1
receptor activation. These effects in turn increase myocardial and cerebral blood flow. Epinephrine
also activates b2 receptors, which induce bronchodilation.

147
5 PHARMACOLOGY

71. A
 patient who is taking propranolol (a nonspecific b1/b2 blocker) and is administered
epinephrine in conjunction with the local anesthetic may experience which of the following?
a. Hypotensive crisis with decreased heart rate (bradycardia)
b. Hypotensive crisis with increased heart rate (tachycardia)
c. Hypertensive crisis with decreased heart rate (bradycardia)
d. Hypertensive crisis with increased heart rate (tachycardia)

c: Epinephrine will attempt to activate b, b1, and b2 receptors. Activation of b receptors causes
vasoconstriction, activation of b1 receptors causes an increased heart rate and increased myocar-
dial force of contraction, and activation of b2 receptors causes vasodilation and bronchodilation.
However, for the patient who is taking propranolol, both b1 and b2 receptors are blocked. With
the b1-receptor blockade, there will be no increase in heart rate or increase in myocardial force
of contraction. With the b2-receptor blockade, there will be no vasodilation or bronchodilation.
However, the b receptor–induced vasoconstriction would still occur and, unopposed, could lead
to a possible hypertensive crisis. The heart rate would decrease in response to the increased
blood pressure (Frank-Starling law). The net result would be a hypertensive crisis with a decrease
in heart rate (bradycardia).

72. W
 hat is the best way to treat the medical emergency resulting from question #71?
a. Administer atropine 0.4 mg sublingually every 5 minutes until the heart rate returns to
baseline; if the heart rate is not responding after two doses, emergency medical services
(EMS) should be activated.
b. Administer nitroglycerine 0.4 mg every 3 to 5 minutes until the blood pressure returns to
baseline; if the blood pressure is not responding after two doses, EMS should be activated.
c. Administer oxygen and perform a carotid massage until the blood pressure returns to base-
line; if the heart rate and blood pressure are not responding after 10 minutes, EMS should
be activated.
d. Administer nitrous oxide to induce vasodilation until the blood pressure returns to baseline;
if the blood pressure is not responding within 10 minutes, EMS should be activated.

b: The nitroglycerine will induce vasodilation and in doing so will decrease the blood pressure.
Atropine is an anticholinergic drug, and administration of it would remove the parasympathetic
slowing effect on the heart rate, thereby increasing the heart rate and the cardiac output, making
the hypertensive crisis even more severe. Administering oxygen would be good, but it would not
affect the blood pressure. Carotid massage would also not affect the blood pressure. Nitrous oxide
would induce vasodilation but most likely not to a degree that would remove the hypertensive
crisis, although it may decrease the blood pressure some. (Source: Gandy W. Severe epineph-
rine-propranolol interaction. Ann Emerg Med 1989;18:98–99.)

73. C
 lonidine is a centrally acting a2 receptor agonist that is known to cause xerostomia.
Long-term xerostomia may result in which of the following?
a. Oral candidiasis and dental caries
b. Gingival hyperplasia
c. Mucosal sloughing
d. Gingival erosion

a: Oral candidiasis and dental caries

148
74. A patient taking a monoamine oxidase (MAO) inhibitor will have one of the two pathways for
metabolism of exogenous epinephrine removed. MAO is one pathway for the metabolism of
epinephrine. What is the other?
a. COMT
b. Uptake into presynaptic neurons
c. Cytochrome P450 enzymes
d. Pseudocholinesterase in the blood

a: Exogenous epinephrine is metabolized principally by COMT. MAO is a secondary pathway for
the metabolism of exogenous epinephrine. Exogenous epinephrine is not appreciably metabolized
by uptake into presynaptic neurons, and cytochrome P450 enzymes and pseudocholinesterase
are not involved in the metabolism of epinephrine.

75. Prazosin (Minipress, Pfizer) is an a1 adrenergic blocking agent used to treat hypertension.
Its mechanism of action is to block a1 receptors on vascular smooth muscles, resulting in
vasodilation. Orthostatic hypotension is a possible side effect. Which of the following
statements makes sense regarding patients taking these agents?
a. The patient will appear to be lethargic.
b. The patient’s short-term memory will be compromised.
c. Gingival bleeding is to be expected.
d. When dismissing the patient, the dental chair should be brought to the upright position slowly.

 : For these patients, transitioning from the supine to the upright position often results in syncope
d
due to orthostatic hypotension. The clinician and staff should be aware of this and therefore
upright the dental chair slowly. Drugs that contain both a receptor– and a receptor–blocking
activity (caredilol [Coreg, GlaxoSmithKline] and labetalol [Trandate, Prometheus]) are prone to
the same side effect.

76. A patient reports that he is taking the MAO inhibitor isocarboxazid (Marplan, Validus).
How long should the patient be off the Marplan before you prescribe hydrocodone
with acetaminophen?
a. 1 week
b. 2 weeks
c. 3 weeks
d. 4 weeks

 : The combination of hydrocodone and Marplan may lead to an increase in respiratory depression,
b
decreased blood pressure, syncope, coma, and death. Patients should discontinue Marplan at least
2 weeks prior to beginning an opioid. The interaction can be far more severe with meperidine.

149
5 PHARMACOLOGY

77. A
 patient taking fluoxetine (Prozac, Eli Lilly) has just undergone an extensive dental implant
procedure. You anticipate that the patient will experience moderate to severe postoperative
pain. Which of the following pain medications is contraindicated?
a. Ibuprofen
b. Naproxen
c. Oxycodone
d. Tramadol

c: Oxycodone, hydrocodone, and codeine are prodrugs that must be metabolized by the liver
enzyme cytochrome P450 2D6 to their oxy-, hydro-, or morphine active forms. Selective serotonin
reuptake inhibitors (SSRIs; eg, Prozac) inhibit the release of cytochrome P450 2D6 enzyme and
therefore prevent the conversion of the inactive oxycodone prodrug to the active oxymorphone
form. Patients taking SSRIs will have very little analgesic benefit when prescribed opioids such as
oxycodone, hydrocodone, or codeine. (Source: Schneider JP, Miller A. Oxycodone to oxymorphone
metabolism. Practical Pain Management. http://www.practicalpainmanagement.com/treatments/
pharmacological/opioids/oxycodone-oxymorphone-metabolism. Accessed 21 April 2016.)

78. A
 40-year-old man gives a history of chronic alcoholism. How would this patient most likely
respond to a preoperative dose of oral benzodiazepine for oral-anxiolytic sedation?
a. He would most likely have an additive effect of the alcohol and benzodiazepine, resulting in
significant respiratory depression.
b. He would most likely have a synergistic effect of the alcohol and benzodiazepine, resulting in
significant respiratory depression.
c. He would most likely be resistant to the CNS depressant effects of the benzodiazepine.
d. He would most likely have a paradoxical reaction and become excited.

c: Individuals with a history of chronic alcohol or barbiturate abuse oftentimes are resistant to the
CNS depressant effects of benzodiazepines. A paradoxical reaction is possible, but most likely
the patient would be resistant to the benzodiazepine effects.

79. A
 healthy 77-year-old woman requests that she be sedated for her dental implant procedure
that is expected to last 60 minutes. She takes no medications. Which of the following would
be an appropriate intravenous conscious sedation protocol?
a. Titrate midazolam in 0.5-mg doses until the desired effect is achieved.
b. Titrate midazolam in 1-mg doses until a total of 5 mg, and then administer fentanyl in 50-µg
doses to a total of 150 µg.
c. Titrate fentanyl in 50-µg doses to a total of 150 µg, and then administer midazolam in 1-mg
doses to a total of 5 mg.
d. Titrate diazepam in 2.5-mg doses to a total dose of 20 mg.

a: The patient’s oxygen saturation, blood pressure, pulse, respirations, and responses should
be monitored throughout the procedure. Narcotics should be avoided if at all possible. Elderly
patients (> 65 years) are far more susceptible to the CNS depressant effects of benzodiazepines
and narcotics.

150
80. Flumazenil is the only FDA-cleared benzodiazepine receptor antagonist. It will reverse the
CNS depressant effects of benzodiazepines. Which of the following statements is true?
a. Flumazenil has no intrinsic activity of its own and will not produce an effect if a
benzodiazepine is not occupying a receptor.
b. Flumazenil is known as an antagonist with activity to reverse benzodiazepine effects,
but it may actually act as a CNS stimulant and result in seizures.
c. Flumazenil will reverse the CNS effects of barbiturates and benzodiazepines.
d. Flumazenil will irreversibly bind a benzodiazepine receptor until such time that the
flumazenil is metabolized; at that time, the bond to the receptor is cleaved.

a: Benzodiazepine antagonists have no intrinsic activity of their own and do not reverse the basal
constitutive benzodiazepine receptor activity. The other three choices are distractors and are false.

81. What is an appropriate one-time dose of oral triazolam (Halcion, Pfizer) for a
70-year-old, 182-lb man who takes no other medications and is planning to undergo
a dental implant procedure?
a. 0.125 mg
b. 0.25 mg
c. 0.375 mg
d. 0.5 mg

a: 0.125 mg

82. What is an appropriate one-time dose of oral alprazolam (Xanax, Pfizer) for a healthy
35-year-old, 170-lb woman who takes no other medications and is anxious about her dental
implant appointment?
a. 0.25–0.75 mg
b. 0.75–2.0 mg
c. 2–4 mg
d. 4–6 mg

b: 0.75–2.0 mg. As always, the lowest possible dose to achieve the desired relief of anxiety
should be used.

83. What is an appropriate one-time dose of oral diazepam (Valium, Roche) for a healthy
50-year-old, 220-lb man who takes no other medications and has anxiety about his dental
implant procedure?
a. 0.5–4.0 mg
b. 4–20 mg
c. 20–30 mg
d. 30–40 mg

b: 4–20 mg. As always, the lowest possible dose to achieve the desired relief of anxiety should
be used.

151
5 PHARMACOLOGY

84. W
 hich of the following analgesics is known to be effective as a preemptive analgesic when
given preoperatively to prevent postoperative pain?
a. Acetaminophen
b. Codeine
c. Ibuprofen
d. Tramadol

c: Ibuprofen has been shown to be effective for preemptive analgesia. The other analgesics listed
have not been proven to have this property.

85. W
 hich of the following analgesics does NOT possess anti-inflammatory properties?
a. Aspirin
b. Acetaminophen
c. Celecoxib
d. Naproxen

 : Acetaminophen appears to have very little systemic anti-inflammatory activity. Acetaminophen’s


b
activity occurs primarily in areas of low peroxide formation, such as the hypothalamus.

86. A
 t what dose (taken in a short time period) does acetaminophen induce hepatotoxicity?
a. 4 g
b. 6 g
c. 8 g
d. 10 g

d: 10 g

87. In patients who have taken large quantities of acetaminophen, either alone or in combination
with an opioid, over several days for oral pain, what is an oral sign of liver toxicity?
a. Gingival hyperalgesia
b. Black tongue
c. Intraoral bleeding
d. Gingival hyperplasia

c: Patients who have taken suprapharmacologic doses of acetaminophen for several days may
demonstrate intraoral bleeding due to liver injury. The liver is responsible for synthesizing many
blood coagulation factors, and if the liver cells are damaged, the synthesis of these coagulation
factors may be impaired.

152
88. Your dental implant patient has routinely consumed four to six alcoholic beverages each
evening after work over a 6-year period. He has built up a tolerance to the systemic effects
of the alcohol and never drives or interacts with others after consuming it. He says, “I simply
drink so I can fall asleep.” Naturally, you do not prescribe an opioid for him. He also says that
ibuprofen never works for him. You decide to prescribe acetaminophen for him. Which of the
following would be the appropriate protocol?
a. Have the patient take 1 g of acetaminophen every 4 to 6 hours and stop all alcohol consumption.
b. Have the patient take 0.5 g of acetaminophen every 4 to 6 hours and stop all alcohol consumption.
c. Have the patient take 1 g of acetaminophen every 4 to 6 hours and continue to drink the
alcohol as he normally does.
d. Have the patient take 0.5 g of acetaminophen every 6 hours and continue to drink the alcohol
as he normally does.

d: The liver metabolizes acetaminophen. Hepatotoxicity can occur from chronic alcohol (etha-
nol) consumption as well as high-dose acetaminophen consumption. The recommended acet-
aminophen dose for patients who routinely consume ethanol is 0.5 g of acetaminophen every
6 hours (a total of 2 g/24 hours). Acetaminophen is metabolized in part by the toxic metabolite
N-acetyl-p-benzoquinoneimine (NAPQI). NAPQI is produced in the liver by the cytochrome P450
enzyme 2E1 (CYP2E1). NAPQI is detoxified under normal conditions by the hepatic enzyme gluta-
thione. Ethanol is also metabolized by CYP2E1. Chronic alcohol ingestion increases the hepatic
production of CYP2E1. For patients who are prescribed acetaminophen and told to discontinue
their normal alcohol consumption abruptly, the only substrate for the excessive alcohol-induced
CYP2E1 is the acetaminophen. The excessive CYP2E1 metabolizes the acetaminophen into the
toxic metabolite NAPQI. This results in excessive concentrations of NAPQI. The existing hepatic
levels of glutathione are unable to detoxify the excessive NAPQI. The hepatic levels of toxic
NAPQI results in irreversible hepatic damage.
Therefore, when prescribing acetaminophen to patients who chronically consume alcohol, it is
important that they be instructed to continue their normal ethanol consumption pattern. Acet-
aminophen with an opioid should not be prescribed in these situations, because the patients
will not be able to discontinue the alcohol. The administration of opioids in the patient who is
consuming alcohol may result in severe CNS depression. (Source: Gómez-Moreno G, Guardia J,
Cutando A. Interaction of paracetamol in chronic alcoholic patients. Importance for odontologists.
Med Oral Patol Oral Cir Bucal 2008;13:E235–E238.)

89. A 35-year-old woman needs an implant placed in the position of the maxillary left lateral
incisor. The available bone and space between the maxillary left central incisor and canine
will only allow for placement of a 3.2-mm-wide implant. The patient also has a history of
smoking 1 pack of cigarettes a day for the past 10 years. Which of the following medications
would contribute to an increased risk of failure in this clinical scenario?
a. Elavil (AstraZeneca); amitriptyline: tricyclic antidepressant
b. Wellbutrin (Valeant); bupropion: selective norepinephrine reuptake inhibitor
c. Zoloft (Pfizer); sertraline: SSRI
d. Norvasc (Pfizer); amlodipine: calcium channel blocker

c: SSRIs have been shown to have a decreased success rate, particularly in cases that require
≤ 4-mm-wide implants and include a history of smoking. It has been hypothesized that SSRIs may
interfere with dental implant osseointegration. The other drugs listed have not been implicated in
increased implant failures. (Source: Wu X, Al-Abedalla K, Rastikerdar E, et al. Selective serotonin
reuptake inhibitors and the risk of osseointegrated implant failure: A cohort study. J Dent Res
2014;93:1054–1061.)

153
5 PHARMACOLOGY

90. A
 73-year-old man requires postoperative analgesics. He currently takes Lotensin (Novartis;
benazepril) to control his hypertension. Benazepril is an ACE inhibitor. ACE inhibitors block
the ability of angiotensin II and aldosterone to constrict blood vessels. Which of the follow-
ing analgesics may interact with the benazepril and reduce its effectiveness?
a. Acetaminophen
b. Ibuprofen
c. Tramadol
d. All of the above

b: Most NSAIDs (including ibuprofen) can interfere with the action of ACE inhibitors. ACE inhib-
itors produce vasodilation and decrease blood pressure by inhibiting production of angiotensin
II and aldosterone. Angiotensin II normally inactivates bradykinin, but if the patient is taking an
ACE inhibitor, bradykinin is not broken down. The still-present bradykinin further enhances the
ACE inhibitor–induced vasodilation. Bradykinin also induces the release of prostaglandins, which
further decreases blood pressure. NSAIDs block the formation and release of prostaglandins.
Therefore, NSAIDs interfere with the action of ACE inhibitors and can make the blood pressure
increase. The other analgesic options are not known to interfere with the antihypertensive effects
of ACE inhibitors. The use of NSAIDs (excepting sulindac [Clinoril, Merck]) in patients who are
taking antihypertensive medication (excepting calcium channel blockers) is a relative contraindi-
cation. (Source: Kalafutova S, Juraskova B, Vlcek J. The impact of combinations of non-steroidal
anti-inflammatory drugs and anti-hypertensive agents on blood pressure. Adv Clin Exp Med
2014;23:993–1000.)

 1. You realize midway through surgery that there has been a constant slow oozing of blood
9
from small arterioles, veins, and capillaries. The vessels are such that they cannot be ligated.
Which of the following is a suitable technique to control this bleeding? (MULTIPLE ANSWERS)
a. Apply pressure using sterile surgical gauze packs.
b. Apply pressure using sterile surgical gauze packs that have been saturated with local anes-
thetic preparations containing epinephrine.
c. Apply a collagen sponge to the site.
d. All of the above

d: All clinicians who perform oral surgical procedures should have a minimum of six methods
available for controlling bleeding.

92. W
 hat is the mechanism of action by which gelatin sponges control bleeding?
a. They release hemostatic agents that will induce blood coagulation.
b. They trap platelets and red blood cells, resulting in platelet activation.
c. They induce thrombin formation.
d. They act as a plug to seal off bleeding vessels.

b: Gelatin sponges act as a matrix for collecting platelets and red blood cells, which leads to
platelet activation and clot formation.

154
93. Which of the following preparations used to induce coagulation is known to delay healing
and interfere with bone formation and epithelialization?
a. Gelatin sponges
b. Bovine collagen sponges
c. Porcine collagen sponges
d. Oxidized cellulose

d: Denatured cellulose sponges act as physical plugs and chemical hemostatic agents. These
sponges release cellulosic acid, which denatures hemoglobin. The hemoglobin breakdown prod-
ucts plug the bleeding vessels. Cellulose sponges should be removed once the bleeding has
been controlled because they will interfere with the healing process.

94. Which of the following activates the greatest percentage of platelets and is the preferred
intrasocket dressing to control bleeding?
a. Collagen sponge
b. Gelatin sponges
c. Oxidized cellulose sponge
d. Oxidized regenerated cellulose sponge

a: The collagen sponge accelerates the aggregation of platelets and forms a physical barrier.

95. Which of the following is the most effective method to control bleeding in a patient who has
a coagulation deficiency or is taking oral anticoagulants?
a. Monsel’s solution (20% ferric subsulfate)
b. 8% zinc chloride
c. Tranexamic acid
d. Topical thrombin

d: Topical thrombin applied via a matrix (such as gelatin) acts as a hemostatic. It is particularly
effective in patients who have a coagulation deficiency or are taking oral anticoagulants. Thrombin
should never be administered intravenously because it can cause extensive thrombosis and even
death. Monsel’s solution and 8% zinc chloride are astringents. Tranexamic acid is an antifibrinolytic.

96. Which of the following blood concentrates could be applied to a site to help control bleeding?
a. Platelet-poor plasma (PPP)
b. Buffy coat platelet-rich plasma (BC-PRP)
c. Platelet-rich fibrin (PRF)
d. All of the above

 : PPP has a very low platelet concentration, but it has multiple clotting factors contained within it
d
that will aid in clot formation. BC-PRP and PRF have a high concentration of platelets and therefore
aid in coagulation. (Source: Davis VL, Abukabda AB, Radio NM, et al. Platelet-rich preparations
to improve healing. Part II: Platelet activation and enrichment, leukocyte inclusion, and other
selection criteria. J Oral Implantol 2014;40:511–521.)

155
5 PHARMACOLOGY

97. Which of the following medications will stimulate bone formation?


a. Atorvastatin (Lipitor, Pfizer)
b. Ibuprofen (Motrin, Johnson & Johnson)
c. Methotrexate (Trexall, Teva)
d. Phenytoin (Dilantin, Pfizer)

a: Atorvastatin is known to increase bone morphogenetic protein 2 release via osteoblasts and
depress osteoclast differentiation. Ibuprofen (and other NSAIDs) inhibit COX-1 and COX-2 enzymes
that lead to the formation of PGE2, which is known to stimulate bone formation. Methotrexate at
the doses used to treat cancer interferes with osteoblast proliferation. At the lower doses used
to treat rheumatoid arthritis, methotrexate may not have an effect on bone. Antiepileptic agents
such as phenytoin affect the cytochrome P450 enzyme pathway, which in turn affects the vitamin
D–PTH–calcitonin axis and subsequent bone formation. (Source: Goodman SB, Jiranek, W, Petrow
E, Yasko AW. The effects of medications on bone. J Am Acad Orthop Surg 2007;15:450–460.)

98. A
 n implant patient is taking dabigatran etexilate (Pradaxa, Boehringer Ingelheim).
Dabigatran etexilate is a specific, reversible direct thrombin inhibitor that is absorbed in
an inactive form and then converted by cytochrome P450 enzymes into its active form,
dabigatran. This dabigatran then binds with the active site on free and clot-bound thrombin
so it cannot transform fibrinogen into fibrin. Multiple studies have demonstrated that the risk
of hemorrhage for patients taking dabigatran etexilate is similar to that for patients taking
warfarin when the international normalized ratio (INR) value ranges between 2 and 3. What
is the most prudent pharmacologic protocol for treating patients taking this medication?
a. Discontinue dabigatran etexilate 3 days prior to the dental implant procedure, and restart the
medication 2 days postoperatively.
b. Discontinue dabigatran etexilate 1 day prior to the dental implant procedure, and restart the
medication 1 day postoperatively.
c. Have the patient skip the dabigatran etexilate dose the day of the implant surgery but take all
other doses as prescribed.
d. Do not discontinue the dabigatran etexilate.

d: The dabigatran etexilate should not be discontinued because if the drug is stopped, the risk of
a cardiovascular event is greater than the risk of hemorrhage from staying on the drug. Recently,
a reversal agent for dabigatran was cleared by the FDA (idarucizumab). Note that you must have
multiple methods available to control localized bleeding in these patients, and you should only
perform reasonable surgeries. (Source: Costantinides F, Rizzo R, Pascazio L, Maglione M. Manag-
ing patients taking novel oral anticoagulants [NOAs] in dentistry: A discussion paper on clinical
implications. BMC Oral Health 2016;16:5.)

99. R
 ivaroxaban (Xarelto, Janssen) is a reversible direct factor Xa inhibitor. It prevents the
activation of prothrombin (factor II). It is indicated for the prevention of deep venous
thrombosis following elective hip or knee replacement surgery. What is the most prudent
pharmacologic protocol for treating patients taking this medication?
a. Discontinue rivaroxaban 3 days prior to the dental implant procedure, and restart the
medication 2 days postoperatively.
b. Discontinue rivaroxaban 1 day prior to the dental implant procedure, and restart the
medication 1 day postoperatively.
c. Have the patient skip the rivaroxaban dose the day of the implant surgery but take all
other doses as prescribed.
d. Do not discontinue the rivaroxaban.

 : There have been no clinical trials supporting specific measures to control hemorrhage following
d
dental surgery on patients taking rivaroxaban. Current information suggests that discontinuing rivar-
oxaban is not necessary. The risk of a cardiovascular event from stopping rivaroxaban is greater than
the risk of hemorrhage from staying on the drug. Note that you must have multiple methods available
to control localized bleeding in these patients, and you should only perform reasonable surgeries.

156
100. Apixaban (Eliquis, Bristol-Myers Squibb) is a direct anticoagulant that is a reversible factor
Xa inhibitor. It prevents the activation of prothrombin (factor II). There is no reversal agent
for this medication. What is the most prudent pharmacologic protocol for treating patients
taking this medication?
a. Discontinue apixaban 3 days prior to the dental implant procedure, and restart the medication
2 days postoperatively.
b. Discontinue apixaban 1 day prior to the dental implant procedure, and restart the medication
1 day postoperatively.
c. Have the patient skip the apixaban dose the day of the implant surgery but take all other
doses as prescribed.
d. Do not discontinue the apixaban.

 : There have been no clinical trials supporting specific measures to control hemorrhage following
d
dental surgery on patients taking apixaban. Current information suggests that discontinuing apix-
aban is not necessary. The risk of a cardiovascular event from stopping apixaban is greater than the
risk of hemorrhage from staying on the drug. Note that you must have multiple methods available
to control localized bleeding in these patients, and you should only perform reasonable surgeries.

101. Your patient is concerned about how much local anesthetic you are using. How many
milliliters of 1% lidocaine with 1:100,000 epinephrine could you infiltrate in a 60-kg woman
before toxicity?
a. 12 mL
b. 18 mL
c. 26 mL
d. 42 mL

d: The toxic dose for lidocaine with epinephrine is 7 mg/kg (see question #39), so the maxi-
mum dose for this patient is 420 mg (60 kg  7 mg/kg). Each milliliter of 1% lidocaine contains
10 mg of lidocaine (1 g per 100 mL), so this patient can have 42 mL (420 / 10). Lidocaine is a
fast-acting amide with a duration of action from 1 to 3 hours. The addition of epinephrine causes
local vasoconstriction, preventing rapid loss of the anesthetic due to the excellent vascular supply
of the mucosa. The maximum lidocaine dose without epinephrine is 5 mg/kg.

102. What is the advantage of using bupivacaine over lidocaine?


a. Quicker onset of action
b. Longer-lasting anesthesia
c. Ability to give more volume without toxicity
d. Can be used in patients with sensitivity to amide anesthetics

b: While the anesthetic effect of lidocaine lasts from 1 to 3 hours, that of bupivacaine lasts from
3 to 10 hours. However, its onset of action is not as rapid as that of lidocaine, and it has a lower
toxic dose (3 mg/kg). Both lidocaine and bupivacaine are amide anesthetics.

157
5 PHARMACOLOGY

103. Why might adding sodium bicarbonate to a local anesthetic be desirable?


a. Less tissue toxicity
b. Spares motor neurons
c. More unionized anesthetic available
d. Stings less due to lower pH

c: Local anesthetics work by blocking voltage-gated sodium channels on the axonal membrane
to prevent propagation of the action potential. There are three factors influencing anesthetic
activity: (1) Lipid solubility correlates with potency, (2) ionization limits penetration and onset of
action, and (3) protein binding ability is the main influence on duration of action. Adding sodium
bicarbonate results in more unionized drug being available, with consequent improvement in
time to onset of action.

104. Why do we taper courses of oral corticosteroids?


a. To allow the HPA axis to recover
b. To prevent rebound effect
c. To limit the severity of short-term side effects
d. To control costs

a: The hypothalamic-pituitary-adrenal (HPA) axis is suppressed through administration of exoge-


nous corticosteroids. While abrupt cessation of short courses (less than a week) of oral steroids is
unlikely to result in insufficiency symptoms, longer courses are generally tapered to prevent this.

105. What is the most common side effect of nasal steroid sprays?
a. Cataracts
b. HPA suppression
c. Acne
d. Epistaxis

d: The most common side effects of nasal steroid sprays are local irritation and epistaxis (nose-
bleeds). Systemic absorption of these drugs is extremely limited and is generally only a concern
in patients who are subject to a variety of corticosteroid inputs (eg, inhaled steroids for asthma,
steroid creams for skin conditions, steroid eye drops, and oral corticosteroids).

106. Which of the following is NOT true regarding clindamycin?


a. It has been associated with CDAC.
b. It binds to the 50S subunit of the bacterial ribosome.
c. It is most effective against aerobic gram-positive cocci.
d. It has no effect on gram-negative species.

d: Clindamycin interferes with bacterial protein synthesis by binding to the 50S subunit of
the bacterial ribosome. While it is most effective against aerobic gram-positive cocci, such as
Staphylococcus and Streptococcus, it also has activity against anaerobic gram-negative rods, such
as Bacteroides and Fusobacterium. The most notable adverse effect of clindamycin is infectious
colitis and diarrhea.

158
107. Which of the following is true regarding amoxicillin?
a. Amoxicillin is ineffective against gram-positive bacteria.
b. Amoxicillin works through inhibition of the synthesis of bacterial cell walls.
c. Amoxicillin is ineffective against gram-negative bacteria.
d. Amoxicillin is poorly absorbed compared with other aminopenicillins.

 : Amoxicillin is a moderate-spectrum b-lactam antibiotic in the aminopenicillin family. Compared


b
with other drugs in this class, it is well absorbed when taken orally. It works through inhibition
of cross-linkage between the linear peptidoglycan polymer chains in the cell walls of both
gram-positive and gram-negative bacteria.

108. Why is clavulanic acid often combined with amoxicillin?


a. To increase gram-negative coverage
b. To decrease cost, as clavulanic acid is cheaper
c. To prevent drug degradation by b-lactamase–producing bacteria
d. To reduce the incidence of gastrointestinal adverse effects

c: Amoxicillin is susceptible to degradation by b-lactamase–producing bacteria. Clavulanic acid


is a b-lactamase inhibitor. Gastrointestinal side effects are the most common adverse effects of
both amoxicillin and amoxicillin-clavulanate.

109. Chlorhexidine mouthwash is advocated by some surgeons for perioperative use in


implant procedures. Which of the following is NOT true regarding this medication?
a. It works through binding of its cation to cell walls.
b. It is effective against gram-positive bacteria, gram-negative bacteria, aerobes, anaerobes,
and yeasts.
c. Prolonged use can stain teeth and gingiva.
d. It is highly effective against viruses.

d: Chlorhexidine solutions are used for their antibacterial effects, accomplished through bind-
ing of the positively charged chlorhexidine cation to the negatively charged bacterial cell wall.
Chlorhexidine is highly effective against gram-positive bacteria and, at higher concentrations,
gram-negative bacteria and fungi; but it is probably ineffective against viruses. Prolonged use
can be associated with staining of teeth and gingiva.

110. Which of the following is true regarding tetracycline?


a. Its mechanism of action is through inhibition of bacterial protein synthesis.
b. It is active against a broad spectrum of bacteria.
c. It is thought to have high bioavailability in the gingival sulcus.
d. All of the above

d: Tetracyclines have been available since the 1950s and have activity against a wide variety of
bacteria, including gram-positive bacteria, gram-negative rods, and anaerobes. They are also
thought to have high bioavailability in the gingival sulcus, making them a popular choice for
treating implant infections. Downsides include frequent bacterial resistance and phototoxicity.

159
5 PHARMACOLOGY

111. A
 potential surgical candidate is taking warfarin due to chronic atrial fibrillation. How does
this drug decrease blood coagulation?
a. Through inhibition of vitamin K epoxide reductase.
b. Through direct blockage of the action of vitamin K.
c. Through activating antithrombin III.
d. The mechanism of action is unknown.

a: Warfarin inhibits vitamin K reductase, resulting in decreased vitamin K recycling, gradually
depleting active vitamin K. The onset of action requires 2 to 3 days before active clotting factors
are depleted, and the duration of action after cessation of therapy is 2 to 5 days. Warfarin activity
is monitored through blood testing for the INR. Heparin, another anticoagulant, binds to and
activates antithrombin III, which in turn inactivates thrombin.

112. A
 potential surgical candidate is taking 81 mg aspirin daily as a precaution against heart
attack or stroke. For how many days does this patient need to stop the aspirin in order to
eliminate the platelet-inhibiting function of the drug?
a. 1–2 days
b. 3–4 days
c. 5–6 days
d. 7–10 days

 : While many surgeons may not feel it necessary to stop a low-dose aspirin regimen before dental
d
surgery, it is advisable because it generally takes 7 to 10 days for the platelet-inhibiting functions
of the drug to clinically disappear. After a single dose of aspirin, platelet cyclooxygenase activity
recovers by approximately 10% per day as platelet turnover increases.

113. W
 hat is the most important drug in the management of an anaphylactic episode?
a. Diphenhydramine
b. Epinephrine
c. Albuterol
d. Prednisone

 : The first and most important drug to give a patient suspected to be in anaphylaxis is epinephrine.
b
This is given through subcutaneous injection of 0.3 mg (0.15 mg in children) via either a 23-gauge
needle or an autoinjector device. The injection is often given into the anterolateral thigh muscle
and can be repeated after 5 minutes, if necessary. Antihistamines (such as diphenhydramine)
and corticosteroids (such as prednisone) may be helpful to prevent a late-phase reaction hours
later, but they are not of much help in initial therapy. Albuterol can be used for wheezing in an
anaphylactic reaction, but is not nearly as effective or important as epinephrine.

114. W
 hich of the following drugs is the most prevalent allergy-causing medication?
a. Penicillin G
b. Clindamycin
c. Trimethoprim-sulfamethoxazole
d. Ciprofloxacin

a: While each of these antibiotics can cause adverse reactions, penicillin is the most preva-
lent allergy-causing medication, with approximately 10% of patients reporting being penicillin-
allergic. Of this group, however, 90% are not actually allergic, and serious allergic reactions only
occur in about 0.03%. Mild to moderate allergic reactions are more common and include skin rash
or hives, itching, gastrointestinal symptoms, and wheezing. Management of mild symptoms include
cessation of the offending drug, administration of an antihistamine such as diphenhydramine, and
observation. More serious reactions should be referred to the emergency room.

160
115. An 82-year-old woman has undergone implant placement with perioperative broad-
spectrum antibiotic coverage. She now presents with complaints of pain in her throat,
mouth, and tongue and mild dysphagia. Examination reveals areas of white patches and
plaques on the tongue and oral mucous membranes, as well as areas of erythema. First-line
therapy for this problem might include which of the following?
a. Clotrimazole troches
b. Miconazole mucoadhesive buccal tablets
c. Nystatin suspension
d. Any of the above

 : Oropharyngeal candidiasis, also known as “thrush,” is a fungal infection that occurs when there
d
is an overgrowth of the yeast Candida. This yeast is normally present on the mucous membranes
in small amounts, but when the environment becomes unbalanced (often as a consequence of
antibacterial use), the yeasts can multiply and cause symptoms. For mild disease, clotrimazole
troches (10 mg five times daily), miconazole mucoadhesive buccal tablets (50 mg daily), or nystatin
suspension (100,000 U/mL, 5 mL four times daily) for 7 to 14 days should be effective. For more
advanced disease, oral fluconazole or other drugs may be necessary.

161
BIOMECHANICS 6

Any clinician preparing to venture into the world of implant


dentistry soon discovers that biomechanics is an essential
subject. This chapter focuses on topics such as the appropri-
ate number of implants, the significance of implant angulation
with respect to the occlusal plane, the clinical significance of
misfit of metal frameworks, and the deleterious effects of
leverage, torsion, and occlusal overload, among other factors.

163
6 BIOMECHANICS

1. According to Wolff’s law:


a. Bone will remodel in response to stress.
b. Bone will model in response to stress.
c. Stress below the physiologic average will cause bone apposition to exceed bone resorption.
d. Stress above the physiologic average (heavy load or force) will result in a net loss of bone mass.

a: If the stress is within the physiologic range, equilibrium is reached and resorption equals
apposition. Stress above the physiologic average (heavy load or force) will cause apposition to
exceed resorption, as the bone buttresses itself against the increased load (hence the bone mass
increase in professional athletes and the increase in torus formation in bruxers). Stress beneath
the physiologic average will result in a net loss of bone mass (hence the atrophy seen in the limbs
of paralysis victims, the bone density loss found in astronauts after being in zero gravity, and the
resorption seen in edentulous ridges). In summary, form follows function.

2. Stress is calculated as the:


a. Functional area (A) divided by the force (F), or S = A/F
b. Force (F) divided by the functional area (A), or S = F/A
c. Force (F) multiplied by the functional area (A), or S = FA

b: Stress is determined by the magnitude of force divided by the functional area over which it is
applied, or S = F /A.

3. Strain is calculated as the:


a. Change in length divided by the original length.
b. Change in length multiplied by the original length.
c. Change in length subtracted from the original length.

a: Strain is the change (deformation) of length of a material divided by the original length, or
strain = change in length / original length. The higher the stress, the greater the strain. The density
of alveolar bone evolves as a result of mechanical deformation from strain.

4. Shear force:
a. Pushes masses toward each other.
b. Pulls objects apart.
c. Causes sliding forces.

c : Shear forces cause sliding forces. Both tensile and shear forces tend to distract the bone-
implant interface. These interfaces, cements, retention screws, and implant components all
accommodate greater compressive forces than tensile or shear forces. Compressive forces
push masses toward each other. This type of force tends to maintain the integrity of a bone-
implant interface. Compressive forces should typically be dominant in prosthetic occlusion. Tensile
forces pull objects apart.

164
5. Moment load or torque is calculated as the:
a. Force divided by the distance.
b. Force multiplied by the distance.
c. Force multiplied by the functional area.

 : Moment load (torque) is a force (or offset load) at a point that tends to produce rotation or bending
b
around that point. Moment load is quite destructive to the implant system. The magnitude of the
moment (M) is calculated as the force (F) multiplied by the perpendicular distance (d) or the moment
arm (lever arm), which runs from the point of interest to the line of action of the force: M = F × d.
On axial force, stress will be well distributed around the implant cross section. If the force
applied is in a transverse direction relative to the implant axis, it will result in a bending moment.
In this case, only a few threads will counteract the load, leading to higher stress levels in both
the implant and the bone.

(a) Well-distributed stress on the


implant and bone when chewing
forces are applied along the long axis
of the implant. (b) Higher stress levels
on the implant and bone when chew-
ing forces are applied in a transverse a b
direction relative to the implant axis.

6. A misfitting framework (nonpassive fit) can:


a. Cause loads on implants even before any biting force is applied.
b. Double the loads on implants when force is applied.
c. Reduce the loads on implants when force is applied.

a: Forces develop on all abutments as a result of misfit at only one abutment. Each abutment is
therefore loaded by constant forces before any biting forces are even applied. Subsequent biting
forces on the prosthesis can add to the forces on each abutment, which can cause problems with
the hardware and bone.

Gap (misfit) between


screw and abutment Prosthesis

Abutments

a
Bone
As screw is tightened, prosthesis
bends (exaggerated)

Free-body diagram
Prosthesis

(a) Nonpassive fit at one of the abut-


ments. (b) Forcing the framework to Abutments
fit by applying force on the misfitted c
abutment. (c) Constant forces would
load each abutment due to the misfit.

165
6 BIOMECHANICS

7. Which of the following factors influence(s) fatigue failure in implant dentistry?


a. Choice of biomaterial
b. Geometry of the structure
c. Force magnitude
d. Number of cycles
e. c and d only
f. All of the above

f: All of the above. Four factors influence fatigue failure in implant dentistry: (1) choice of bio-
material, (2) geometry of the structure, (3) force magnitude, and (4) number of cycles.

• Choice of biomaterial: The type of alloy and its specific modulus of elasticity (rigidity).
• Geometry of the structure: The thickness of the material. A material two times as thick will
be 16 times as strong.
• Force magnitude: At high stress, only a few cycles of loading can be tolerated before fracture
can occur.
• Number of cycles: Even one cycle of force of sufficient magnitude can cause a fracture (eg,
karate blow to a piece of wood). However, repeated strikes of lower force magnitude will also
fracture the object (eg, a wire coat hanger will break after a few cycles of bending because
of fatigue, not because the last bend was more forceful).

The endurance limit is the stress level below which an implant biomaterial can be loaded indefi-
nitely. To reduce fatigue failure, you should at a minimum reduce stress, increase the surface area (the
number and size of the implants), eliminate parafunctional habits, and reduce any occlusal contacts.

Stress

Failure

Endurance limit

Cycles
The endurance limit in the fatigue curve.

8. Which of the following are signs of occlusal overload?


a. Fracture of prosthetic components
b. Screw loosening
c. Mobility
d. Radiolucency around the implant
e. Loss of crestal alveolar bone
f. a and b
g. a, b, and e
h. All of the above

 : Fracture of prosthetic components, screw loosening, mobility, radiolucency around the implant,
h
and loss of crestal alveolar bone are all signs of occlusal overload.

166
9. Loosening of the prosthetic screw can be caused by which of the following?
a. Occlusal overload
b. Excessive leverage
c. Torsion
d. Parafunctional habits
e. Poor framework fit
f. Beam flexion
g. Poor machining tolerances of components
h. Inadequate tautness of the retaining screws
i. a, b, d, and h
j. All of the above

j: All of the above

10. All fixed partial dentures flex to various degrees under occlusal load. Flexion of the beam
(pontic) is dependent on which of the following factors?
a. Geometric design of the pontic (the beam)
b. Type of alloy
c. Length of the span
d. b and c
e. All of the above

e: All of the above

11. TRUE OR FALSE: The cross-sectional design of the interproximal connectors influences the
amount of flex of the pontic (the beam), the width being more important than the height.

False: The cross-sectional design of the interproximal connectors does influence the amount of
flex, but the height is more important than the width. Doubling the width doubles the strength,
whereas doubling the height has an eightfold strengthening effect.
Second premolar

Second premolar
First premolar

First premolar
Second molar

Second molar
First molar

First molar
Canine

Canine

Pontic design with short connectors. Pontic design with tall connectors for improved rigidity.

167
6 BIOMECHANICS

12. T
 RUE OR FALSE: The beam will flex to the cube power relative to additional length.

True: If a three-unit bridge has a value of x, the addition of one pontic to create a four-unit
prosthesis will double the pontic length and allow flexion of 8x. The addition of two pontics to
create a five-unit prosthesis with triple the pontic length will allow flexion of 27x.

a
2x

8x

3x

27x
c

13. Potential biomechanical problems of off-axial loading include:


a. Fracture of the restoration
b. Retaining screw fracture
c. Abutment fracture
d. Implant body fracture
e. Osseous distraction by unfavorable loading
f. A ridge lap pontic design that is less than optimal for oral hygiene
g. a, b, and c
h. All of the above

 : Endosseous root form implants distribute occlusal load best in an axial direction. Off-axis loading
h
can lead to all of the problems listed: (1) fracture of the restoration, (2) retaining screw fracture,
(3) abutment fracture, (4) implant body fracture, (5) osseous distraction by unfavorable loading,
and (6) a ridge lap pontic design that is less than optimal for oral hygiene.

1
Cheek

3
4 Lever
function
Tongue

168
14. TRUE OR FALSE: Buccolingual implant inclination can occur in situations where the prosthetic
reconstruction is offset relative to the implant head. Up to 15 degrees in tilt is acceptable;
however, as the implant inclination approaches or exceeds 25 degrees, it is recommended
that the implant be removed or the implant placement aborted.

True: Buccolingual inclination will introduce a bending moment to the implant. The support-
ing bone is severely compromised in load transmission when the implant inclination exceeds
15 degrees.

15º 30º

15. TRUE OR FALSE: A mesiodistal implant inclination in a multiple-implant case does not lead to
increased loading, as a bending in the plane defined by the long axis of the prosthesis and
the implant direction will be counteracted by the superstructure.

True: The anchorage of the implant can sometimes be improved by tilting the implant
to utilize dense bone structure remote from the intended implant position. The All-on-4 technique,
for example, uses this biomechanical principle. As long as the position of the implant head is
unaltered, a limited inclination of the implant itself has minor influence on the load transfer to
the implant.

169
6 BIOMECHANICS

16. W
 hich of the following factors play(s) a role in protecting the implant against occlusal
overload in the long term?
a. Bone density
b. Implant surface area (length, width, and design)
c. Implant surface
d. Parafunctional habits
e. Prosthetic design
f. a, b, and c
g. All of the above

g: All of the above

17. T
 RUE OR FALSE: It is important to center the occlusal contact on the implant prosthesis
because otherwise contacts during lateral excursion will lead to a high bending moment.

True: A higher cusp inclination leads to a higher magnitude of the transverse force components,
and the more lateral the excursive contact, the greater the leverage and the greater the bending
moment. Centering the occlusal contact counteracts this effect. This is why it is important to
observe the patient for excessive occlusal wear, a history of crown fractures, and bruxism and
other parafunctions.

The occlusal contact must be centered


along the long axis of the implant to
minimize the damaging effect of the
bending moment that results from
contacts on cusp inclination.

18. T
 RUE OR FALSE: When three implants are to be placed next to each other, they should
be placed in a straight line because the straighter the alignment, the lower the potential
bending of the implants.

False: Implants placed in a straight line allow for bending around a single axis, which imparts
less desirable stress distribution to the vertical stack and to the supporting bone; the straighter
the alignment, the greater the potential bending of the implants. A single axis will always pass
through two implants. Three or more implants in an offset configuration to each other creates a
much more stable geometry. As a consequence, there is less of a damaging bending moment
and more favorable axial loading.

170
A straight-line configuration does not provide When implants are positioned on a curved line (full-arch resto-
the compensating nature of an offset implant ration), any potential bending around the line combining any
to counteract a transverse force. two implants will effectively be counteracted by axial forces of
the implants that are offset relative to that line.

19. Placement of implants using the tripod configuration can reduce the stress on the implants by:
a. 30%
b. 50%
c. 70%
d. The stress level is not affected by the tripod configuration.

b: It can be estimated that the stress will be reduced approximately 50% by the tripod configu-
ration compared with the straight-line configuration for the same number of implants.

The ideal situation is a slightly curved A tripod configuration allows the load
configuration with the middle implant response to bending forces to be predom-
offset at least 2 to 3 mm in the buccolin- inantly axial, minimizing the stress level.
gual direction.

171
6 BIOMECHANICS

20. T
 RUE OR FALSE: The tripod placement should be determined by the positioning of the
implant bodies.

 alse: The tripod placement should be determined by where the implant heads pierce the anchor-
F
ing bone. A slight inclination of the implants may be useful in achieving such a placement.

The implant heads define the position


of the prosthesis support in the bone.

21. T
 RUE OR FALSE: All dental implants are loaded the same way as natural teeth—that is,
by compressive forces directed along the long axis of the implant.

False: When an implant is replacing a single tooth, then it is usually (but certainly not exclusively)
exposed to chewing forces that are compressive. However, not all implants are used as single-tooth
replacements. Instead, it is more common to employ four, five, or six implants as abutments to
support a partial or full-arch implant-supported prosthesis. In these clinical scenarios, the dental
implants will be subjected to tensile as well as compressive forces. The tensile forces will exceed
the value of the biting force on the prosthesis by two or three times, owing to the leverage effects.
Also, there can be moments applied on implants.
The figure below shows a simplified model of the jaw as a class 3 lever. The fulcrum is at the
condyle (C). The forces from the muscles of mastication (M1, M2) contract during mastication
and cause the mandible to close, producing a biting force (F) on a bolus of food at a certain
location in the arch and a reaction force (J) at the temporomandibular joint. F acts on the food
particles and in turn on the occlusal surface of the restoration connected to the tooth or implant.
The result is a force with a direction perpendicular to the plane of occlusion. In reality, this is an
oversimplification because, owing to the inclined occlusal surfaces of the crown, food typically
does not make contact on the crown with contact forces perfectly parallel to the long axis of the
tooth or implant.

172
J
M1

M2 F

In reality, the direction of a biting force is not perfectly parallel to the long axis of the tooth or
implant. A force (F) acting on the pontic at point B may be resolved into components directed
along the x, y, and z axes. F(z) tends to compress or push the implant into bone, while the lateral
forces F(y) and F(x) tend to push the implant sideways and tip it around a point. In most instances,
the axial component of biting force on a single tooth or implant is the largest.

x
z

Point B

173
6 BIOMECHANICS

22. T
 RUE OR FALSE: Teeth and implants often have to support moments as well as forces.

 rue: A moment (also called torque) tends to produce rotation of a rigid body. The units for moment
T
are force multiplied by distance (Ncm). Both natural teeth and implants can experience moments.
The best example of a moment in dentistry is in orthodontics. In the figure below, a force (F)
applied to the crown of a tooth at a distance (d) from the center of rotation (CR) produces a
moment (Fd) on the tooth and a tipping (rotation) of the tooth around the CR.

Orthodontic
bracket

F F

d
CR CR
MCR= Fd

a b

A freestanding implant is loaded by a purely vertical force (F), but F acts eccentrically (ie, at
point C, which is not along the line of action of the implant but at a distance [d]) and produces
a tendency for the implant to tip or rotate in a counterclockwise direction (sense). With respect
to point B, force F acts through a perpendicular distance (d) and produces a counterclockwise
moment of magnitude Fd. Note two significant points: (1) The bone-implant interface must supply
the counterbalancing moment to keep the implant in static equilibrium, and (2) the implant hard-
ware (eg, the screw) must be able to withstand the moment without failing. However, both the
interface and the implant hardware have their limits.

C MB = Fd

174
23. TRUE OR FALSE: When considering biting forces (P) on implants (F1 and F2) supporting
a prosthesis with cantilever, these forces increase rapidly as the interimplant spacing (b)
increases and the cantilever length (a) increases.

Free-body diagram
P
P a Q b
a b
Prosthesis

y
F2 F1
x F2 F1

2 1 Fixtures
Bone
Bone

2 1

True: When a biting force (P) is applied to the prosthesis, the forces on the implants
(F1 and F2) exceed P, depending on the ratio of a to b per the following formulas:

F1 = (a/b)  P
F2 = (1  a/b)  P

For instance, suppose the cantilever length is 20 mm and the interimplant spacing is
10 mm, creating an a/b ratio of 2; in this case, the forces on the implants would be 2P and 3P.
If P is 250 Ncm, the forces would be 500 N and 750 N on implants 1 and 2, respectively.

24. TRUE OR FALSE: In the diagram in question #23 above, the force on implant 2 is tensile
while the force on implant 1 is compressive.

False: The force on implant 2 is compressive, tending to push it into the bone, while the force
on implant 1 is tensile, tending to pull it out of the bone. The force on the prosthesis from each
implant is equal and opposite to the force on each implant from the prosthesis.

25. TRUE OR FALSE: In this diagram, the force on the cantilever is distributed equally onto all
four implants.

These implants are


not considered

4 3
2
1 Tensile force
on implant
b

a Compressive
force on implant

Bite force

False: In this clinical scenario, most forces are applied onto implants 1 and 2 only. It is okay to
ignore the remaining implants in the distribution. The two implants nearest the loading point do
most of the work.

175
6 BIOMECHANICS

26. W
 hich of the following factors must be considered when predicting the forces on each
abutment in a group of four to six implants supporting a prosthesis?
a. Mechanical properties of the prosthesis
b. Location of the implants along the arch
c. Spacing between each implant
d. Mechanical properties of the implants
e. Geometric properties of the implants
f. Location of the biting force applied to the prosthesis
g. Strength of the biting force
h. Bone density
i. Patient’s age
j. a, d, g, and h
k. All but i
l. All of the above

l: All of the above

27. In this diagram showing six implants supporting a fixed prosthesis, if we assume that the
bone and the prosthesis are rigid (Skalak model), and if a vertical biting force of 111 N is
applied at the point where  =10 degrees (a case of cantilever loading), how will the forces
be distributed among the six implants?

Vertical load
P

1 10º
6
5 2
4 3
Estimated
neutral axis
of implants

a. Implants 1, 2, and 6 experience compressive forces while implants 3, 4, and 5


experience tensile forces.
b. Implants 1 and 2 experience compressive forces while implants 3, 4, 5, and 6
experience tensile forces.
c. Implant 1 experiences compressive forces while implants 2, 3, 4, 5 and 6
experience tensile forces.
d. Implants 1 and 6 experience compressive forces while implants 2, 3, 4, and 5
experience tensile forces.
e. Each implant experiences the same amount of force (111 N divided by six).

a: The distalmost implants nearest the load (1 and 2) experience compressive forces (negative
value), as does implant 6, on the other side of the arch. Meanwhile, the anterior implants (3, 4,
and 5) experience tensile forces (positive value). The results can be understood by recognizing
that the implants are exerting forces on the prosthesis to keep it from tipping distally and to the
side under the action of the vertical downward force on the prosthesis.
Note also that for the applied force magnitude of 111 N, the load on each implant is
less than 111 N, except for implant 1, nearest the loading point; this implant’s load is about
–148 N (the negative sign indicates compression), or about 1.3 times the load on the prosthesis.
It is important to note here that clinically the forces on the abutments are not predicted exactly
by the Skalak model (where bone and the prosthesis are “infinitely rigid”), because the prosthesis
and implants in the bone are somewhat deformable.

176
28. TRUE OR FALSE: A flexible prosthesis produces a more uniform distribution of forces among
its supporting abutments than a rigid prosthesis.

 alse: In the Skalak model explained in question #27, when the prosthesis framework was replaced
F
by a somewhat more flexible U-shaped steel prosthesis, minor discrepancies began to show up
between the measured forces and those predicted by the original rigid prosthesis. Therefore, it
appears that as the mechanical stiffness of the prosthesis decreases, the forces increase on the
abutments closest to the point where the prosthesis was loaded. In the limit of an extremely flex-
ible, soft prosthesis (ie, acrylic), it can be expected that all of the load applied to the prosthesis
would be concentrated on the implants closest to the loading point.
If the goal is to produce an even distribution of forces among the abutments, one would be
better off with a stiffer rather than a more flexible prosthesis and with biting forces applied as
close as possible to the midline.

29. TRUE OR FALSE: More implants are always better.

False: It is hard to come up with firm recommendations regarding the number of implants for
fixed prostheses for different clinical situations; many factors should be considered for fully and
partially edentulous arches.
If we compare the stress on four implants versus the stress on six implants with all other factors
being equal, then of course it is better to employ six implants instead of four. However, in some
situations, four implants might be as good as or even better than six implants. Factors that influence
the stress level include the number of implants, the size of the implants (diameter and length), the
bone density, the location of the implants, the spread of the implants, the type of the opposing
dentition, the vertical height/cantilever of the prosthesis, the strength of the chewing forces, the
direction of the chewing/load forces, and prosthetic factors such as the size of the occlusal table
of the implant prosthesis and the existence of a long pontic or a cantilever.
To put these factors into a clinical perspective, for a 75-year-old patient with good bone density
and minimal alveolar bone height deficiency, four implants (in the positions of the canines and first
molars) are sufficient to support a prosthesis without cantilever opposing a removable denture.
But in a patient with poor bone density who is still in his forties (meaning he has stronger chewing
muscles) to support a prosthesis with a cantilever opposing natural teeth, six implants are better
(all implants are placed in between the two mental foramina due to posterior height deficiency
over the inferior alveolar nerve).
When we take all of these factors into consideration, the statement is false.

177
6 BIOMECHANICS

30. T
 RUE OR FALSE: When a force is directed perpendicular to the occlusal plane, the angulation
of an implant with respect to the occlusal plane makes a big difference in the stresses in the
bone around the implant.

True: It is true that the implant’s angulation affects the stresses in the bone around the implant;
however, this is usually the case with single implants. In multiple-implant cases, the force distri-
bution depends mainly on the patient’s biting characteristics, the number and spacing of the
implants, and the nature of the prosthesis, and not so much on the angulation that any one
implant makes with the occlusal plane.
In the following figure, reminiscent of that shown in question #23, implant 1 has a 30-degree
inclination to the occlusal plane. There are significant lateral components on the implant, but the
analysis works out the same as the scenario in question #23, where both implants are perpen-
dicular to the occlusal plane.

Free-body diagram
P
P a b
a b
Prosthesis

y
F2 F1
x
F2 F1
Fixtures
Bone
2 1
2 1
Bone
30º

31. P
 lacement of tilted single implants is not an acceptable practice. What is the maximum
acceptable tilt?
a. 10 degrees or less
b. 15 degrees or less
c. 20 degrees or less
d. 25 degrees or less

b: If it is not possible to place an implant with minimal buccolingual tilt (15 degrees or less),
then the implant site should be aborted and the implant placed in a different location, or the
implant placement should be delayed and the area grafted (using techniques such as guided
bone regeneration, block grafting, ridge splitting, etc) so it can accept an implant in the optimal
buccolingual angulation after healing.

178
32. TRUE OR FALSE: It is permissible to place implants tilted up to 30 degrees mesiodistally
when the restoration is a fixed, multiple-unit implant-supported prosthesis.

True: The anchorage of the implant can sometimes be improved by tilting the implant to
utilize dense bone structure remote from the intended implant position. In some cases,
mesiodistally tilted implants can be used as an alternative to vertical ridge augmentation or
sinus graft surgery in patients with general or local contraindications (ie, general medical
problems, pathologies of the maxillary sinus, or advanced age). Limited mesiodistal inclination
in a multiple-implant case does not lead to increased loading and has minor influence on the
load transfer to the implant, as a bending in the plane defined by the long axis of the prosthesis
and the implant direction will be counteracted by the prosthetic superstructure. The survival of
mesiodistally tilted implants in multiple-unit cases has been reported in the literature with encour-
aging success rates of 93% to 97.5%.

33. TRUE OR FALSE: Flexion is caused by the attachment of internal pterygoid muscles on the
medial ramus of the mandible.

 rue: Concerns exist with full-arch rigid fixed prostheses as to the clinical effects of flexion and torsion
T
with implants. Flexion occurs in the area of the mental foramen and increases distal to it. Maximum
flexion of the mandible in the molar area is 0.62 mm during protrusion and opening. The flexion
measures from 0.8 mm at the first molar to 1.5 mm in the ramus area. Component loosening and/or
fracture or osseous damage may result over time. There is no flexion on the maxilla, so it is better to
construct a one-piece restoration, especially with bar overdentures so the prosthesis will not rock.

Internal
pterygoid
muscle

Masseter
muscle

179
6 BIOMECHANICS

34. T
 RUE OR FALSE: Torsion is caused by the temporalis muscle attachments with forceful contraction.

 alse: Torsion is caused by the masseter muscle attachments with forceful contraction (see illustra-
F
tion in question #33). It is more powerful than flexure. Because of this, parafunctional bruxism and
clenching may cause more problems than repeated jaw opening. During torsion, the mandibular
body twists on the working side and bends in the parasagittal plane on the balancing side during
the power stroke of both mastication and unilateral molar biting.

180
IMPLANT 7
PROSTHODONTICS

This chapter focuses on prosthodontics and dental implant


restorations. Success is highly reliant on a variety of factors.
Many fractures, failures, and unplanned situations can easily
be avoided with proper planning. Specific insights into
numerous factors affecting the success of a treatment are
provided. These factors range from treatment planning and
occlusion to material selection and prosthetic design.

181
7 IMPLANT PROSTHODONTICS

1. Airborne-particle abrading (sandblasting) zirconia crowns:


a. Makes them stronger.
b. Makes them weaker.
c. Makes bonding stronger.
d. Makes no difference.

b: Implant-supported zirconia crowns can suffer catastrophic failures and fractures. Airborne-
particle abrading the internal or fitting surface of these restorations with 50-micron Al2O3 has
been shown to weaken these restorations.

2. According to the 2010 Baldissara et al study on zirconia crowns:


a. Lava crowns block soft tissue color completely.
b. Lava crowns make teeth too dark.
c. Lava crowns at 0.5-mm coping thickness are translucent.
d. Lava crowns at 1-mm coping thickness are translucent.

c: When restoring implant restorations with zirconia crowns, you must carefully control the coping
thickness. This is particularly important when a titanium abutment is to be used in the anterior
dentition. Otherwise, the definitive restoration will appear more gray than planned. (Source:
Baldissara P, Llukacej A, Ciocca L, Valandro FL, Scotti R. Translucency of zirconia copings made
with different CAD/CAM systems. J Prosthet Dent 2010;104:6–12.)

3. When restoring anterior implant restorations, a flatter anterior guidance angle means:
a. Weak front teeth
b. Translucent front teeth
c. Lower muscle force
d. Higher muscle force
e. None of the above

c: Flatter or shallow anterior guidance translates to less muscle force. The anterior implant-
supported restorations will have a better chance of survival because they will not be mechan-
ically overloaded. (Source: Williamson EH, Lundquist D. Anterior guidance: Its effect on
electromyographic activity of the temporal and masseter muscles. J Prosthet Dent 1983;49:
816–823.)

4. Clinical signs of loss of posterior support include which of the following?


a. Periodontal ligament widening
b. Fremitus
c. Fractured anterior teeth
d. Drifting of anterior teeth
e. All of the above

e: When restoring the anterior dentition with implant-supported restorations, you must evaluate
whether the posterior occlusion can support the vertical loads that are to be exerted onto the
anterior teeth. If this support is not present, the patient may be diagnosed as having a loss of
posterior support. This condition can compromise anterior implant-supported restorations.

182
5. Who invented the roll flap technique?
a. Al-Faraje
b. Abrams
c. Ingber
d. Glauser

c : Abraham Ingber was the first person to describe the roll flap technique. It is considered a simple
and trusted method of increasing keratinized tissue around dental implants prior to restoration.

6. When considering placement of a submerged implant, you must aim to do which of the following?
a. Increase soft tissue volume
b. Remove GBR material (pins, membranes)
c. Facilitate any corrections (surface texture, tattoos, scars)
d. All of the above

d: All of the above

7. TRUE OR FALSE: (1) Connective tissue grafts can be taken from the tuberosity area.
(2) This type of connective tissue grafting is soft and not resistant. 
a. Both 1 and 2 are true.
b. 1 is true and 2 is false.
c. 1 is false and 2 is true.
d. Both 1 and 2 are false.

b: 1 is true and 2 is false.

8. Adding soft tissue volume at the time of stage-two surgery may include which of the
following techniques?
a. Mini roll flap
b. Palacci technique
c. Proper restorative contours
d. a and b
e. All of the above

e: All of the above

9. Which of the following statements about hyperbaric oxygen (HBO) therapy is true?
a. H
 BO therapy is the inhalation of 100% oxygen at an elevated pressure of at least
1.5 atmospheres absolute (150 kPa), typically 2 to 3 atmospheres absolute (200–300 kPa).
b. HBO therapy is the inhalation of 100% oxygen at an elevated pressure of up to
5 atmospheres absolute (>500 kPa), typically 2 to 3 atmospheres absolute (200–300 kPa).
c. HBO therapy is the inhalation of 70% oxygen at an elevated pressure of at least
1.5 atmospheres absolute (150 kPa), typically 2 to 3 atmospheres absolute (200–300 kPa).
d. HBO therapy is the inhalation of 50% oxygen at an elevated pressure of up to
5 atmospheres absolute (>500 kPa), typically 2 to 3 atmospheres absolute (200–300 kPa).

a: An HBO chamber is a medical device designed to deliver high levels of oxygen that cannot be
provided by any other means. These devices are typically used in patients whose jaws have been
irradiated and require dental implant therapy. In these patients, hypoxia (deficiency in oxygen
reaching the tissues) results from the ischemia caused by radiation.

183
7 IMPLANT PROSTHODONTICS

10. In a histomorphometric study of bone reactions to titanium implants in irradiated bone and
the effect of HBO, Johnsson et al (1999) found that:
a. Minimal doses of radiation increase bone-to-implant contact in the mandible.
b. Radiation reduces the capacity for osseointegration of titanium implants. HBO treatment may
improve bone formation and has positive effects on bone maturation after radiation.
c. Radiation increases the capacity for osseointegration of titanium implants. HBO treatment
may reduce bone formation and has positive effects on bone maturation after radiation.
d. Neither radiation nor HBO treatment has any effect on bone formation and osseointegration.

b: Radiation reduces the capacity for osseointegration of titanium implants. HBO treatment may
improve bone formation and has positive effects on bone maturation after radiation. (Source:
Johnsson AA, Sawaii T, Jacobsson M, Granström G, Turesson I. A histomorphometric study of bone
reactions to titanium implants in irradiated bone and the effect of hyperbaric oxygen treatment.
Int J Oral Maxillofac Implants 1999;14:699–706.)

11. W
 hen evaluating patients who have received radiation to the jaws, at what dosage (Gy) does
the risk of implant failure increase?
a. Above 10
b. Above 25
c. Above 30
d. Above 50
e. Above 100

 : Lower radiation doses (less than 50 Gy) are significantly associated with improved implant survival
d
compared with higher doses (over 50 Gy). A review article noted that no failures were observed
with radiation doses less than 45 Gy. (Source: Shugaa-Addin B, Al-Shamiri HM, Al-Maweri S, Tarakji
B. The effect of radiotherapy on survival of dental implants in head and neck cancer patients. J
Clin Exp Dent 2016;8:e194–e200.) Studies comparing implants placed at various intervals before
or after irradiation did not show significant differences in survival rates and could not make a
strong recommendation for treatment.

12. W
 hich of the following are risk factors for implant failure due to peri-implantitis?
a. History of treated periodontitis
b. Smoking
c. Obesity
d. a and b
e. All of the above

d: Following an extensive literature search, Heitz-Mayfield and Huynh-Ba concluded in 2009 that
there is an increased risk of peri-implantitis for smokers compared with nonsmokers (reported
odds ratio from 3.6 to 4.6). The combined effect of patients who smoked and had a history of
periodontal treatment increased the chance of peri-implant bone loss and implant failure. (Source:
Heitz-Mayfield LJ, Huynh-Ba G. History of treated periodontitis and smoking as risks for implant
therapy. Int J Oral Maxillofac Implants 2009;24[suppl]:39–68.)

184
13. When planning full-arch implant-supported fixed detachable cantilevered restorations,
load-bearing capacity is maximized by which of the following?
a. A curvilinear arrangement of implants across the arch
b. A low anteroposterior (AP) spread of implants
c. a and b
d. None of the above

 : When restoring an edentulous arch, load-bearing capacity can be maximized by placing implants
a
in a curved arrangement with a large AP spread. This will have a positive biomechanical effect
and ensure longevity of the implants and restorations.

14. When planning full-arch implant-supported fixed detachable cantilevered restorations,


a prudent clinician must do which of the following?
a. Reduce cantilever length as much as possible.
b. Increase AP spread of implants.
c. Select the longest posterior implants possible.
d. Design the implant arrangement with a curvilinear approach.
e. All of the above

 : All of these parameters will provide a biomechanical advantage and warrant longevity. However,
e
keeping the cantilever length to a minimum will have the best positive impact on longevity.
Cantilevers significantly magnify occlusal loads.

15. What is the recommended occlusal guideline for the All-on-4 prosthesis for immediate loading?
a. Bilateral balanced occlusion
b. Balanced occlusion
c. Bilateral working-side contacts
d. Open anterior contacts
e. Even occlusal contacts on anterior teeth and light contacts on posterior teeth

 : During immediate loading, occlusal forces on posterior teeth are kept to a minimum. This tech-
e
nique avoids overloading the posterior implants, as they are closer to the muscles of mastication.

16. Which of the following factors can influence the accuracy of the master cast?
a. Force placed during insertion of the impression tray
b. Type of implants used
c. Number of implants
d. Arrangement of implants
e. a and c
f. None of the above

f: The accuracy of the master cast is affected when a stone with high expansion is used. Correct
measurements for the stone are critical in replicating the correct implant position on the stone
cast. Splinting the impression copings prior to impression-making stabilizes them and ensures
the creation of a more accurate stone cast.

185
7 IMPLANT PROSTHODONTICS

17. P
 rior to placement of angled multiple-unit abutments on tilted implants, which of the
following procedures must be performed?
a. Testing of implant torque levels
b. Implant stability quotient (ISQ) testing
c. Using a bone mill to remove bony interferences around the implant platform
d. Suturing
e. None of the above

c: Typically, the distal aspect of the distalmost implant will need removal of bony interferences
as the implant is submerged deeper in this region. It is prudent and efficient to perform this
procedure around all of the implants.

18. In the maxillary arch, you may want to do which of the following to achieve primary stability
of the implants?
a. Underprepare the osteotomy site.
b. Use implants with an aggressive thread design that is tapered.
c. Use bone graft material.
d. a and b
e. All of the above

 : The maxilla normally has softer bone than the mandible, so underpreparation and use of implants
d
with a more aggressive thread design will increase the likelihood of achieving primary stability.

19. Benefits of tilting implants for a graftless solution include which of the following?
a. Reduction in cantilever length
b. Bicortical fixation
c. Avoidance of vital anatomical structures
d. a and c
e. All of the above

d: The main advantages of tilting posterior implants are to increase the AP spread, reduce the
cantilever length, and avoid the maxillary sinus and mental nerve.

20. T
 he amount of vertical alveolar reduction in the edentulous patient receiving a graftless
immediate load provisional restoration depends on which of the following?
a. Lip support
b. Gingival display
c. Material used for the definitive restoration
d. Emergence and pontic surface of the prosthesis
e. a and c
f. All of the above

f: All of these factors will influence the amount of bone reduction needed for a functional and
esthetic prosthesis. Hiding the horizontal transitional line is an essential part of the planning for
this kind of appliance. It is imperative that the patient does not display the junction between the
pink restorative material and the ridge during a high smile.

186
21. When evaluating a patient for a graftless implant solution in the maxilla, the following
zones of available bone have been identified: zone 1, incisive bone; zone 2, premolar
regions; zone 3, molar regions; zone 4, zygoma. If a patient presents with adequate bone
only in zone 4 bilaterally, where would be the ideal positions to place implants?
a. Two axial implants in zone 1 and two axial implants in zone 3
b. Two axial implants in zone 1 and two tilted implants in zone 2
c. Two axial implants in zone 1 and two zygomatic implants
d. Four zygomatic implants
e. None of the above
f. All of the above

 : Bilateral quad zygomatic implants are an excellent service for the patient with a severely atrophic
d
maxilla. These implants can be immediately loaded and a fixed provisional restoration provided.

22. What is the minimum amount of vertical restorative space needed for a fixed implant-
supported metal and acrylic resin prosthesis?
a. 7 mm
b. 17 mm
c. 27 mm
d. 37 mm
e. It depends on other factors

b: In order to ensure the strength of the framework, pink acrylic resin, and denture teeth, a total
of 17 mm in height is always required. A tooth length of 11 mm, minimum acrylic resin thickness
of 4 mm, and metal framework height of 3 to 4 mm will be necessary for adequate strength.

23. Which of the following can cause excessive gingival display?


a. Short and hyperactive upper lip
b. Altered passive eruption
c. Dentoalveolar extrusion
d. Vertical maxillary excess
e. All of the above
f. None of the above

e: It is imperative to accurately diagnose the causes of excessive gingival display. This is particu-
larly important when performing a full-arch maxillary restoration. The correct diagnosis will have
a huge bearing on the course of treatment.

24. Which of the following is true?


a. An average upper lip length for a female in her early 20s is 22 mm long.
b. A maxillary central incisor is on average 10.8 mm long.
c. A maxillary central incisor display in repose should show no teeth at all.
d. a and b
e. All of the above

d: When the lips are in repose, the maxillary central incisor display should be no less than 1 mm;
otherwise, the patient may appear edentulous at rest. There is a very delicate balance between
lip length and incisor length and position that must be visualized prior to implant placement. This
can be tested during the wax try-in phase.

187
7 IMPLANT PROSTHODONTICS

25. How is the AP spread of implants measured?


a. AP spread is the distance between all of the anterior and posterior implants divided by 2.
b. AP spread is measured as the distance between two parallel lines, one drawn through the
center of the most anterior implant and the other drawn between the most posterior implants.
c. AP spread is measured by deducting the distance between the most anterior implant and the
retromolar pads on both sides and then dividing that number by 2.
d. AP spread is the distance between the most anterior part of the most anterior implant and the
most posterior part of the most posterior implant.
e. None of the above

b: The AP spread provides very relevant information as to how much cantilever would be safe
in a posterior direction bilaterally.

26. R
 isk factors for All-on-4 implant treatment include which of the following?
a. Severe lack of lip support
b. A very high smile line
c. Too much bone volume
d. Long upper lip
e. a and b
f. All of the above

e: Esthetic failure can result in treatment failure due to patient dissatisfaction. A severe lack of
lip support could mean that a flange and a removable overdenture would be the acceptable
treatment for a small group of patients. A high smile line may mean that the junction between
the patient’s ridge and the pink restorative material is visible. This is almost impossible to hide.

27. Which pontic design is the most unhygienic and should be avoided?
a. Hygienic
b. Ridge lap
c. Modified ridge lap
d. None of the above

b: A ridge lap pontic is almost impossible for the patient to clean due to lack of access. This may
result in biologic failure and soft tissue inflammation and irritation.

28. W
 hich occlusal scheme would best suit a patient receiving a maxillary complete denture
opposing a mandibular implant-supported fixed detachable cantilevered prosthesis?
a. Canine guidance
b. Group function
c. Balanced occlusion
d. a and b
e. Functionally generated path

c: A balanced occlusion, which means that all teeth contact in all excursive movements, would be
the best solution for this treatment. Excursive movements will not dislodge the maxillary denture
or break the posterior palatal seal.

188
29. Which occlusal scheme would best suit a patient receiving maxillary and mandibular
implant-supported fixed detachable cantilevered prostheses?
a. Canine guidance
b. Group function
c. Balanced occlusion
d. a and b
e. Functionally generated path

e: In this situation, balanced occlusion will create destructive occlusal forces that will damage
the restorations. It would be best to design the occlusion so that any nonworking side contacts
are eliminated.

30. In their investigation of interimplant bone height as a function of proximity of adjacent
implants, Tarnow et al (2003) showed that:
a. A definite loss of 1.0 to 1.5 mm in papillary height occurs with any adjacent implant.
b. A probable loss of 0.5 to 1.0 mm in papillary height occurs with any adjacent implant.
c. Adjacent implants maintain the bone in its original position.
d. It makes no difference how close the implants are to each other as long as the drilling
sequence is properly followed.

a: Papillary loss is inevitable in these situations. This becomes a high-risk esthetic problem for
patients with a high smile line or when implants are placed in a unilateral situation. (Source:
Tarnow D, Elian N, Fletcher P, et al. Vertical distance from the crest of bone to the height of the
interproximal papilla between adjacent implants. J Periodontol 2003;74:1785–1788.)

31. TRUE OR FALSE: Any maxillary anterior implant restoration starts with the correct
placement of the maxillary incisal edge position. (1) The vertical position of the incisal edge
is determined by the desired level of tooth display. (2) The horizontal position is determined
by the “S” sound.
a. Only 1 is true.
b. Only 2 is true.
c. Both 1 and 2 are true.
d. 1 is true, and 2 is sometimes true.
e. Neither 1 nor 2 is true.

c: One additional factor is the angulation of the maxillary incisal edge, which is determined by
the “F” sound. It would be ideal if the maxillary incisors touch the vermilion border when the
patient pronounces this fricative.

189
7 IMPLANT PROSTHODONTICS

32. Cantor et al (1969) showed that:


a. The mandible deviates more severely following radical neck dissection.
b. Significant masticatory function can be restored with dentures for mandibulectomy patients.
c. Lowering the palatal vault with a prosthesis can aid speech where tongue mobility is restricted.
d. Speech therapy is the most important factor in improving speech for partial glossectomy
patients.
e. a and c
f. a and d
g. c and d
h. b, c, and d

c : By lowering the palatal vault for these patients, the tongue will have a better chance of creating
a seal for pronunciation. (Source: Cantor R, Curtis TA, Shipp T, Beumer J 3rd, Vogel BS. Maxillary
speech prostheses for mandibular surgical defects. J Prosthet Dent 1969;22:253–260.)

33. M
 urray et al (1980) showed that the rate of osteoradionecrosis in patients undergoing
postradiation extractions was closest to:
a. 5%
b. 10%
c. 25%
d. 45%
e. 75%

e: These figures approached 100%. It is prudent to perform extractions prior to any radiotherapy.
(Source: Murray C, Herson J, Daly T, Zimmerman S. Radiation necrosis of the mandible: A 10-year
study. Int J Radiat Oncol Biol Phys 1980;6:549–553.)

34. Osteoradionecrosis:
a. Occurs most often in the mandible.
b. Occurs in about 10% to 15% of patients undergoing preradiation dental extractions.
c. Is best treated by long-term oral antibiotics.
d. Eventually requires resection of the infected, nonvital bony segment to control the infection in
half of cases.
e. a and b
f. a and c
g. a and d
h. b and c
i. b and d
j. c and d

g: Osteoradionecrosis is a serious condition that should be avoided at all costs. Any tooth that
has a potential for failure should be proactively removed prior to any radiotherapy. 

190
35. Implants used to retain maxillary obturators:
a. Have a low success rate when positioned in the incisive bone.
b. Have a very low success rate in irradiated patients.
c. Are very useful on the defect side in the cut portion of the zygoma.
d. Are more predictable in combination with a retentive defect.
e. Have performed well in the maxillary tuberosity once osseointegrated and under favorable
biomechanical conditions.
f. a, b, and d
g. b, d, and e
h. a, c, and d
i. c, d, and e
j. d and e

g: Treatment of the irradiated patient generally has a less favorable outcome compared with
nonirradiated patients, but all efforts must be made to improve quality of life. Patients must be
warned of limitations of treatment at the outset.

36. What is the principle difficulty encountered when using implants in free fibula bone grafts?
a. Debulking the tissues at the implant sites
b. Achieving initial fixation of the implants
c. Achieving proper implant position and angulation
d. Achieving a reasonable success rate

a: Once the soft tissues have been debulked, the rest of the treatment can proceed fairly simply
because the graft has a high bony volume.

37. Which of the following statements is true?


a. Tilting dental implants can increase bone loss.
b. Tilting dental implants can increase stress concentration around implants.
c. Tilting dental implants can increase the chance of component fracture.
d. a and b
e. b and c
f. All of the above

f: However, many studies are now available that clearly show that as long as cross-arch stability
and rigidity of the prosthesis are established, tilted implants can actually be beneficial. (Source:
Duyck J, Van Oosterwyck H, Vander Sloten J, De Cooman M, Puers R, Naert I. Magnitude and
distribution of occlusal forces on oral implants supporting fixed prostheses: An in vivo study. Clin
Oral Implants Res 2000;11:465–475.)

38. Removable implant-retained prostheses can be classified as all of the following EXCEPT:
a. Tissue supported
b. Tissue supported and implant retained
c. Implant supported and implant retained
d. Screw retained and supported

d: The screw should never be relied upon to retain a prosthesis, and the screw should never be
kept or loaded under tension. Compressive forces on the screw are favored.

191
7 IMPLANT PROSTHODONTICS

39. A
 ll of the following are indications for a maxillary bar overdenture EXCEPT:
a. Lack of lip support
b. High esthetic needs
c. Poor AP spread of implants
d. Major financial restraints

c: A poor AP spread of implants is a contraindication for a bar overdenture. At least a moderate
spread of implants in the anterior maxilla is desired for the best outcomes.

40. H
 ow many implants must be used to retain a complete maxillary denture with the
palate removed?
a. One
b. Two
c. Three
d. Four
e. None of the above

d: Four well-distributed implants must be used in the maxilla before it is safe to use a maxillary
denture with an open palate.

41. W
 hen a tissue-supported and implant-retained maxillary denture is to be fabricated,
what is the minimum restorative space required in the anterior maxilla?
a. 5 mm
b. 10 mm
c. 15 mm
d. 20 mm
e. 25 mm

c: 15 mm will allow adequate space for the attachments, bar, and acrylic resin.

42. W
 hen a tissue-supported mandibular overdenture (retained by two anterior implants and
Locator attachments) is to be fabricated, what is the minimum restorative space required?
a. 5 mm
b. 10 mm
c. 15 mm
d. 20 mm
e. 25 mm

b: 10 mm will allow adequate space for the Locator attachments and housing as well as an
adequate minimum thickness of 4 mm for acrylic resin strength.

43. In order to safely restore a maxillary overdenture with an open palate, all of the following
rules must be respected EXCEPT:
a. Long distal implants
b. Good AP spread of implants
c. Short anterior teeth
d. Resilient attachments on distal areas

c: Choices a, b, and d are critical for long-term success of a maxillary overdenture with rotational
features built in.

192
44. All of the following are classic features of a removable fully implant-supported and -retained
maxillary overdenture EXCEPT:
a. These restorations do not rotate and are fully implant supported.
b. There is some soft tissue support at the back.
c. Lip support is the main indication for making these restorations today.
d. Six to eight implants are ideal.
e. Frictional latches are used for retention.

 : These restorations are essentially fixed, but the patient can remove them. The only valid reason
b
for using them today is if the patient has a severe lack of lip support that cannot be provided with
a screw-retained restoration.

45. When using a Hader bar for a maxillary overdenture, all of the following rules must be
followed EXCEPT:
a. Two anterior implants must be 15 to 20 mm apart.
b. Only one Hader must be used.
c. The bar must be circular in cross section.
d. Hader bars must be coupled with ERA attachments posteriorly.

b: The Hader works best when two are used next to each other on the anterior bar. The bar must
be at least 15 mm long to accommodate two 7-mm attachments in the denture.

46. TRUE OR FALSE: (1) Any prosthesis with a Hader bar will allow rotational movements.
(2) Rotational movements are dangerous for overdentures.
a. Both 1 and 2 are true.
b. 1 is true and 2 is false.
c. 1 is false and 2 is true.
d. Both 1 and 2 are false.

b: Hader bars are essentially designed to allow rotational movements. These movements are
not destructive as long as the whole system is designed to allow full rotational movement of the
prosthesis.

47. According to Williams et al (2001), which system provides the best retention when making
a rotational overdenture?
a. Two Hader clips
b. Four Hader clips
c. One Hader clip with ERA attachment
d. Three ZAAG (Zest Dental Solutions) attachments on the bar
e. Four ZAAG attachments

c: The combination of Hader clips and ERA attachments provides the best retention. This is also
the best system to allow rotational movement of the prosthesis. (Source: Williams BH, Ochiai KT,
Hojo S, Nishimura R, Caputo AA. Retention of maxillary implant overdenture bars of different
designs. J Prosthet Dent 2001;86:603–607.)

193
7 IMPLANT PROSTHODONTICS

48. W
 hich of the following are commonly seen with implant overdentures?
(MULTIPLE ANSWERS)
a. Low implant failure rates
b. High maintenance requirement
c. High satisfaction and function
d. Large number of acrylic resin fractures
e. Large number of attachment fractures

 , b, c: Implant overdentures generally have been shown to have low failure rates and high levels
a
of patient satisfaction. However, they are associated with many maintenance issues. The patient
must be made aware of these prior to treatment.

49. D
 isadvantages of a cement-retained implant prosthesis include all of the following
EXCEPT that:
a. Potential cement is left in the pocket.
b. It is harder to retrieve.
c. After cementation, alarm signals such as screw loosening are difficult to identify.
d. It is more expensive to fabricate than a screw-retained prosthesis.
e. If the cement dissolves or breaks on one abutment, it might not be detected and might lead
to an unfavorable distribution of force.

d: A cement-retained implant prosthesis is less expensive to fabricate than a screw-retained


prosthesis.

50. D
 isadvantages of a screw-retained implant prosthesis include all of the following
EXCEPT that:
a. Esthetically, it is less acceptable than a cement-retained prosthesis.
b. It is bulky on the palatal side when placed in the anterior region.
c. It is more expensive to fabricate than a cement-retained prosthesis.
d. Biomechanically, it is less sound than a cement-retained prosthesis in the anterior region
because the forces of occlusion are not directed toward the long axis of the implant.
e. It has a lower risk of framework nonpassive fit (especially in multiple-unit cases) than a
cement-retained prosthesis.

e: A screw-retained implant prosthesis has a higher risk of framework nonpassive fit (especially in
multiple-unit cases) than a cement-retained prosthesis.

51. W
 hat is the minimum distance that must be kept between two natural crowns in order to
place a 4-mm-diameter implant between them?
a. 4.5 mm
b. 5.0 mm
c. 5.5 mm
d. 7.0 mm

d: The dimensions of an implant are critical to consider during implant selection. For optimal
papillary esthetics around implants, a minimum of 1.5 mm of space must be maintained on either
side of the implant (ideally 2 mm). A greater distance between the tooth and the implant means
better blood supply and bone support for the interproximal papilla and, consequently, better
esthetic results.

194
52. The fundamental rule is that an implant should be placed in the center of the available space
mesiodistally. In situations where this rule cannot be followed, you place the implant slightly:
a. Distally
b. Mesially

a: Implant placement slightly toward the distal provides some esthetic advantages, because the
distal papillae will be hidden by the buccal contour of the tooth restoration.

53. Implant angulation is probably the most difficult factor to visualize during surgery. One
of the best ways to visualize it is to virtually divide the buccal contour of the tooth to be
replaced or the adjacent ones into thirds: incisal, middle, and gingival. The long access of the
implant should be directed as closely as possible to the:
a. Gingival third
b. Middle third
c. Incisal third

c: The long access of the implant should be directed as closely as possible to the incisal edge as
an extension of the crown. This resembles the morphology of natural anterior teeth and thereby
facilitates the creation of a natural emergence profile.

54. What is the minimum thickness required for zirconia abutments to be able to withstand
occlusal functional loads without fracture?
a. 0.5 mm
b. 0.8 mm
c. 1.0 mm
d. 1.5 mm

b: 0.8 mm

55. TRUE OR FALSE: (1) Submerging roots in the anterior maxilla is dangerous. (2) Submerging
roots has the benefit of maintaining the original hard and soft tissue ridge contours in the
anterior maxilla. (3) Both vital and nonvital roots can be submerged.
a. All three statements are true.
b. 1 is false, 2 is true, and 3 is true.
c. 1 is true, 2 is false, and 3 is true.
d. 1 is true, 2 is true, and 3 is false.
e. All three statements are false.

b: As long as roots that are to be submerged are not infected, it is safe to provide this type of
treatment. 

56. TRUE OR FALSE: An open tray is used for the transfer impression, while a closed tray is used
for the pickup impression.

False: Open tray impressions are pickup impressions, and closed tray impressions are known as
transfer impressions. They can both produce accurate results with proper handling.

195
7 IMPLANT PROSTHODONTICS

57. All of the following are parameters used to select abutments EXCEPT:
a. Implant diameter and platform width
b. Bone density
c. Space available
d. Type of restoration
e. Implant trajectory

b: Bone type and density play no part in abutment selection.

58. W
 hat is the maximum manufacturer-recommended torque for a regular platform abutment?
a. 10 Ncm
b. 20 Ncm
c. 32 Ncm
d. 45 Ncm
e. Any of the above

c: It is imperative not to overtorque screws for implants. This may result in plastic deformation
of the screw and fracture.

59. T
 RUE OR FALSE: Implants only have an intraoral application.

False: Dental implants can be used for many nondental and maxillofacial prosthetics. Prosthetic
ears, noses, and eyes may be attached with extraoral implants.

60. Who developed the blade implant?


a. Linkow (1966)
b. Hodosh (1975)
c. Scialom (1962)
d. Brånemark (1965)

 : Leonard Linkow designed the blade implant. Unfortunately, these implants had a high failure rate,
a
and failure was typically catastrophic. Blade implants were the reason why the dental community
was hesitant in accepting osseointegrated implants.

61. T
 RUE OR FALSE: A radiographic stent can be used as a surgical stent.

True: The stent should ideally be converted to a surgical guide. This is achieved by removing the
radiographic marker and creating a channel wide enough for implant drills to be inserted through
it during placement.

196
62. It is important to verify the seating of any component connected to the implant intraorally via:
a. Pressing hard against the screw while you are connecting the components
b. Visual verification
c. Tactile sense
d. Radiographic confirmation

d: Radiographic evaluation is the most objective method to verify seating of the prosthesis.
Correct angulation of the x-ray unit and film is essential. A vertical bitewing would provide the
correct angle. Also, the clinician must make sure that the implant threads are sharp and clearly
visible on the radiograph.

63. Implants vary in the types of abutment connection they have. Which of the following is NOT
a type of abutment connection?
a. External connection with hex
b. Morse taper
c. Ball attachment
d. Internal connection with hex

c: A ball attachment is not an abutment connection for an implant. The ball attachment is made
as an abutment connection for an implant-retained denture.

64. Ideally, surgical stents should be:


a. Stable and retentive
b. Sterilized
c. Transparent
d. All of the above

d: All of the above

65. After seating the prosthetic abutments, what should you do next?
a. Jaw relation records
b. Final impressions
c. Preliminary impressions using alginate
d. Radiographs to verify proper seating of prosthetic abutments

d: You must ensure that the abutment is fully seated before proceeding to the next step. If the
abutment is not seated (seating can only be verified objectively with a radiograph), all future
steps will be futile.

197
7 IMPLANT PROSTHODONTICS

66. TRUE OR FALSE: (1) The anterior mandible most often presents with type 1 highly cortical bone
suitable for immediate loading. (2) The most predictable area for dental implant placement is in
the anterior mandible because there are no anatomical areas that can be damaged.
a. 1 is true and 2 is false.
b. 1 is false and 2 is true.
c. Both 1 and 2 are true.
d. Both 1 and 2 are false.

a: The anterior mandible typically presents with high-density, highly cortical bone. With proper
surgical management, it is almost guaranteed that immediate loading with a provisional restoration
will proceed in this area without any complications.

67. T
 RUE OR FALSE: One of the most important keys to being successful with implant treatment
is communication among the entire team.

True: Good communication also includes proper systems in place for ordering components,
planning the procedure, selecting the number and distribution of dental implants, and obtaining
the proper consent for treatment.

68. K
 eys to prosthetic and surgical implant success during immediate loading include which of
the following?
a. Sterile technique under operating room conditions
b. Controlled rotational drill speeds and controlled torque
c. Copious cooling irrigation
d. High torque values
e. Supragingival cement margins during immediate loading
f. All of the above

f: High success rates during immediate loading require a combined surgical and prosthetic effort
with a high level of attention to detail. Failure is often caused by a combination of errors. Metic-
ulous attention to detail in all areas is necessary for predictable outcomes.

69. T
 RUE OR FALSE: Tapered impression posts are routinely used for the open tray
impression technique.

False: Open tray impression copings or posts must have retentive features built in. This allows
a very stable situation during pickup as well as manipulation later during the laboratory phases.
Closed tray impression posts should be tapered because they are designed to pop in and out
of the impression.

70. TRUE OR FALSE: Stock trays and a closed tray impression technique are strongly recommended
when making impressions of misaligned implants (divergent angles greater than 30 degrees).

False: When making impressions of full-arch cases, splinted and impression copings with an open
custom tray achieve the most accurate replication of the implant positions. (Source: Kim JH, Kim
KR, Kim S. Critical appraisal of implant impression accuracies: A systematic review. J Prosthet
Dent 2015;114:185.e1–192.e1.)

198
71. TRUE OR FALSE: All cemented single-unit implant-supported crowns require the use of a
hexed abutment to prevent rotation and provide proper alignment of the abutment over
the implant.

True: For single units, the internal features of the abutment prevent rotation. For splinted units in
a screw-retained situation, the reciprocal effect of the splinted implants prevents rotation.

72. TRUE OR FALSE: The ideal and simplest treatment for a single-unit implant-supported
restoration is a screw-retained prosthesis.

 rue: In prosthodontics, simplicity and retrievability are keys to success. A screw-retained restoration
T
satisfies this. In addition, the lack of a cement junction reduces the chances of cement-related
periodontal issues.

73. Abutment selection for the implant-supported fixed partial denture (FPD) is dependent
on all of the following EXCEPT:
a. Cement- versus screw-retained restoration
b. Resin versus porcelain-fused-to-metal (PFM) restoration
c. The interocclusal space available for the restoration
d. Single- versus multiple-unit restoration
e. The relative angle of the long axes of all supporting fixtures

b: Resin versus PFM restoration. For simplicity and retrievability purposes, a screw-retained resto-
ration would be ideal in these situations. However, this is not always possible due to implant
angulation and depth. A cement-retained restoration may need to be utilized to bypass depth
and angle complications.

74. The UCLA abutment is recommended in which of the following conditions?


a. Single-tooth replacement
b. Cement-retained multiple-unit FPD
c. Screw-retained multiple-unit FPD
d. When the space between the implant and the opposing occlusion is less than 7 mm
e. All of the above

 : The UCLA abutment has been the workhorse for implant dentistry and can be used in many situa-
e
tions. However, since the advent of computer-aided design (CAD)/computer-assisted manufacturing
(CAM) technology, this system is used less often because it requires casting onto the abutment.

199
7 IMPLANT PROSTHODONTICS

75. Which of the following statements is NOT true?


a. When planning an implant FPD for a bruxer, additional implants are recommended.
b. Occlusal contact should be lighter on an implant-supported FPD than on the natural dentition.
c. Lateral forces are well tolerated by implants.
d. The diameter of an implant plays a role in the resistance of lateral forces.
e. a and b
f. c and d
g. All of the above

f: Implant biomechanics play an important role in implant surgery and prosthetic design. The
number, location, and size of implants should be planned by both the surgeon and the prosthetic
dentist for optimal design. Implant surgery is a prosthetically driven discipline.

76. W
 hich bar shape does NOT allow for free rotational movement of the retained prosthesis?
a. Round
b. Oval
c. Parallel-sided

c: Typically, parallel-sided bars are designed to avoid movement and rotation. So the clinician
should decide early in the treatment whether the definitive restoration will have rotational features
or not. Parallel-sided bars are normally used for implant-supported and -retained restorations
with no movement at all.

77. W
 hat is the minimal distance needed between two implants (center to center) to
accommodate one Hader clip for the prosthesis? (Assume an implant diameter of 4 mm.)
a. 7 mm
b. 12 mm
c. 20 mm
d. 24 mm

b: A minimum distance of 12 mm allows for the following: 7 mm for the diameter of the Hader
clip, 4 mm for the implant diameter, and 0.5 mm of clearance on each side of the clip.

78. C
 orrect implant bar design/arrangement includes which of the following?
a. Sufficient distance between the implants
b. Placement directly over the ridge
c. Parallel occlusal plane that is perpendicular to the midline
d. Sufficient gap distance to the soft tissue to allow for hygiene
e. All of the above

e: Proper prosthetic design should be incorporated into implant planning and placement with
a removable implant-retained and tissue-supported prosthesis. This will ensure proper function
and correct rotational movements with slower wear of the components.

200
79. When making a preliminary impression of a mandibular arch containing dental implants
at abutment level, what are the components and materials required to fabricate a
diagnostic cast?
a. Abutment-level impression copings (open or closed tray systems)
b. Standard abutment replicas
c. Properly selected impression trays (ideally custom trays)
d. All of the above

d: For full-arch restorations, open tray pickup impressions have been shown to be more accurate.
And if these components are connected together prior to the pickup, an accurate and stable
impression can be made. A custom tray reduces impression bulk and potential shrinkage and
increases patient comfort and efficiency.

80. What would best describe an implant that is clinically successful in terms of its connection
with the adjacent bone?
a. Healthy periodontal ligament
b. Direct bone-to-implant contact
c. Fibrous encapsulation
d. None of the above

b: One of the main premises for osseointegration is direct bone-to-implant contact similar to an
ankylosis situation. Fibrous encapsulation means failure in implant dentistry. Periodontal ligament
is only found around teeth and should not interpose implants and bone.

81. Maintenance care for an implant patient consists of:


a. Oral hygiene education for the patient
b. Calculus and plaque removal
c. Patient compliance
d. All of the above

d: Patient aftercare is equally important as the treatment provided by the clinician. Failure to
comply with hygiene may result in a higher rate of peri-implantitis and failure.

82. During a typical maintenance visit, which of the following should you do?
a. Remove supragingival plaque only around the implants.
b. Remove supragingival and subgingival plaque around the implants and the remaining natural
teeth and decontaminate the implants with iodine.
c. Remove only subgingival plaque around the implants and the remaining natural teeth.
d. None of the above

b: Supra- and subgingival cleaning must be performed.

201
7 IMPLANT PROSTHODONTICS

83. To achieve good home care, which of the following will the patient need?
a. Soft toothbrush
b. Interdental brush
c. Dental floss
d. Waterpik
e. All of the above

e: All of the above

84. What does “biologic seal” refer to?


a. The gingival attachment between the gingiva and the implant
b. The area apical to the osseointegrated portion of the implant
c. a and b
d. None of the above

a: This is a particularly important concept for the restorative dentist because certain materials
(titanium and zirconia) are conducive to maintaining this seal. A flat and concave emergence
profile will also help to maintain the seal in the long term.

85. T
 RUE OR FALSE: When placing an implant in a maxillary central incisor position to support a
cemented restoration, the implant head should be directed slightly to the buccal.

 rue: Cemented restorations require a different implant trajectory from screw-retained restorations.
T
For a cemented restoration, the surgeon typically aims to have the trajectory through the incisal
edge of the restoration.

86. Site development for implant placement can include which of the following?
a. Soft tissue grafting
b. Orthodontic tooth movement
c. Guided tissue regeneration
d. All of the above

 : In contemporary implant dentistry, site development may happen continuously and throughout
d
the surgical part of the treatment. Both hard and soft tissue development are commonly used for
a more stable long-term result.

87. T
 o predictably produce an interproximal papilla between an implant and a natural tooth, how
far does the crest of bone need to be from the contact point of the definitive restoration?
a. 3 mm
b. 5 mm
c. 7 mm
d. 9 mm

b: As soon as this distance increases beyond 5 mm, the chance of producing a papilla drops
significantly. Prosthetically creating a long contact point may help disguise a missing or shortened
papilla. (Source: Tarnow DP, Magner AW, Fletcher P. The effect of the distance from the contact
point to the crest of bone on the presence or absence of the interproximal dental papilla. J Peri-
odontol 1992;63:995–996.)

202
88. Which of the following describes tissue molding around an implant restoration?
a. The response of the peri-implant tissue to a provisional restoration, resulting in a soft tissue
fill interproximally
b. The placement of a resorbable membrane to bring tissue height to the neck of the implant
c. Placing a connective tissue graft to increase the amount of soft tissue
d. All of the above

d: Achieving a correct and stable soft tissue position and contour around an implant requires
combined efforts from the surgeon and the restorative dentist.

89. Which word describes the ultimate implant restoration in the esthetic zone?
a. Beautiful
b. Functional
c. Harmonious
d. All of the above

 : A harmonious esthetic situation should not be confused with a symmetric situation. Often some
d
movement and harmony is much superior to a symmetric arrangement. Patients will need to be
educated about this concept; too much symmetry may make the smile appear “fake.”

90. Which of the following structures is difficult to reconstruct in the oral cavity?
a. Gingival margin
b. Attached gingiva
c. Alveolar bone
d. Gingival papilla

d: Once a papilla is lost, it is very difficult to recreate surgically. However, there are many prostho-
dontic techniques that can disguise the fact that the papilla is missing as well as ceramic techniques
to give the illusion of a papilla.

91. Which of the following factors is important in the success of ridge augmentation procedures?
a. Use of a space maker
b. Immobility of the barrier
c. Isolation of the defect with the barrier
d. Barrier exposure
e. All of above

e: All of the above

92. What does “emergence profile” refer to?


a. Length of the implant
b. Soft tissue and restorative interface
c. Form and shape of the implant
d. Surface roughness of the implant

 : The facial aspect of maxillary central and lateral incisors in a patient with a high smile line should
b
have an emergence profile that is either flat or concave but smooth. This should be tested with
a provisional restoration. The exact emergence profile can be copied with a customized pickup
impression coping, which replicates the provisional restoration.

203
7 IMPLANT PROSTHODONTICS

93. Which of the following is an advantage of wide-body implants?


a. They provide better initial stability in posterior areas.
b. They increase the metal-to-bone contact.
c. They are closer to the diameter of posterior teeth.
d. All of the above

 : Wide-body implants are useful as long as they are placed in adequate bone volume with at least
d
1 mm of bone buccal and lingual to the implant. Prosthetically, the width of the implant allows
the restorative dentist to make a restoration that is more natural in emergence and contours and
minimizes any black triangles.

94. According to Parel and Sullivan, implant heads in the esthetic zone should be placed:
a. 3 mm subgingivally
b. 2 mm coronal to the adjacent cementoenamel junction (CEJ)
c. Level with the adjacent CEJ
d. 2 mm apical to the adjacent CEJ
e. 3 mm apical to the adjacent CEJ

e: This depth and reference point are more predictable and can create a healthier and more
esthetic emergence profile. (Source: Parel SM, Sullivan DY. Esthetics and Osseointegration. Dallas:
Osseointegration Seminars Inc, 1989.)

95. A wide-diameter implant has which of the following advantages?


a. It requires less torque to insert.
b. It is more affordable.
c. It provides a better crown-to-implant ratio.
d. It presents a greater surface area for potential bone interface.

 : One indication for placing wide-diameter implants is when primary stability cannot be achieved
d
with narrower implants. As long as adequate bone volume is present, the narrower implant can
be backed out and a wider implant placed without any further drilling. This technique will create
higher torque.

96. O
 nce an abutment is connected in the esthetic zone, how long should you wait before
removing it for an ideal outcome?
a. 6 weeks
b. 8 weeks
c. 12 weeks
d. 16 weeks
e. It should never be removed.

e: The best outcomes in terms of stability of the gingival crest are achieved when the abutment
is not removed at all. The concept of “one abutment–one time” promotes the stability of the
gingival crest.

204
97. Esthetic limitations for implant-supported prostheses frequently associated with screw access
chambers can be avoided by:
a. Careful treatment planning and coordination of surgical and restorative phases of treatment.
b. Planning for the use of a cement-retained prosthesis.
c. Using an angulated abutment system.
d. All of the above

b: A cement-retained restoration may be the ideal prosthetic solution in areas where the occlusal
aspect is directly visible (eg, mandibular premolar area). However, newer all-ceramic restorations
make it very simple to close the access hole esthetically with composite resin, increasing the
scope for screw-retained restorations.

98. Epithelial healing around an implant is completed how long after implant exposure?
a. 1–3 weeks
b. 2–5 weeks
c. 5–8 weeks
d. 8–12 weeks
e. 16–20 weeks

a: The restorative dentist can make final implant-level impressions after this period.

99. TRUE OR FALSE: The divisions of available bone (A to D) were first described by
Misch and Judy.

True: This classification is commonly used for surgical and radiologic communication in implant
dentistry. (Source: Misch CE, Judy KW. Classification of partially edentulous arches for implant
dentistry. Int J Oral Implantol 1987;4:7–13.)

100. What tooth positions are ideal for placement of two root form dental implants to retain a
mandibular overdenture?
a. Central incisors
b. Lateral incisors
c. Canines
d. Molars
e. All of the above

c : Placing these implants in canine sites offers many advantages in terms of stability, biomechanics,
and hygiene. There is always plenty of bone in these areas, and the prosthesis can rotate easily.

205
7 IMPLANT PROSTHODONTICS

101. W
 hat tooth positions are ideal for placement of four dental implants to retain a mandibular
rotational overdenture?
a. Central incisors and molars
b. Lateral incisors and canines
c. Canines and molars
d. Molars only
e. Four implants should never be placed to retain a mandibular rotational overdenture.

e: Placing more than two implants will prevent the prosthesis from rotating, cause breakage
of the acrylic resin, and require a high level of maintenance. If four implants are to be placed,
then they should have good AP spread and be connected with a bar to make an implant-
retained and tissue-supported prosthesis.

102. The ability of implant materials to increase preosteoblast differentiation is called:


a. Osseointegration
b. Osteoinduction
c. Osteoreduction
d. Osteoclastogenesis
e. Osteoconduction

b: Osteoinduction is the process by which osteogenesis is induced and is responsible for bone
healing. In this phenomenon, immature cells are recruited and stimulated to develop into preos-
teoblasts, which then turn into osteoblasts to form new bone.

103. Dental implant therapy should never be considered in a:


a. Diabetic patient
b. Patient who had a myocardial infarction
c. Patient with controlled thyroid disease
d. Pregnant patient
e. Hypersensitive patient

d: Implant dentistry requires radiography, and this should be delayed until after pregnancy.

104. P
 atients who are discovered to be anemic during the preoperative implant appointment:
a. Must never be treated with implants.
b. Must be evaluated by their primary physician.
c. Should be allowed to receive implants without any further evaluation.
d. Are probably actively and acutely bleeding.
e. None of the above

b: Anemic patients are at greater risk of death and other complications during surgery, so they
should always be evaluated by their primary physician prior to any dental surgery.

206
105. TRUE OR FALSE: (1) According to Tarnow et al (2003), an interdental papilla between an
implant and a natural tooth in the esthetic zone has an average height of 4.5 mm. (2) The
papilla between two implants in the same area has an average height of 3.5 mm.
a. Both 1 and 2 are true.
b. 1 is true and 2 is false.
c. 1 is false and 2 is true.
d. Both 1 and 2 are false.

 : With an ideal papilla height, the emergence profile may be reduced to create a slender prosthetic
a
rise and provide an acceptable shape. As the papillary height is reduced, it becomes necessary to
control the outline by moving the line angles closer together with a high level of polish. This gives
the illusion of a narrower tooth in a wider space and with a shorter papilla. (Source: Tarnow D,
Elian N, Fletcher P, et al. Vertical distance from the crest of bone to the height of the interproximal
papilla between adjacent implants. J Periodontol 2003;74:1785–1788.)

106. If the restorative implant dentist is faced with a wide space and short papilla, what can be
done to give the illusion of a natural and harmonious appearance when compared with the
contralateral side?
a. Displace the line angles medially.
b. Highlight the new line angles to a high polish.
c. Displace the proximal contacts toward the palate.
d. Flatten the mesial triangle to create some light deflection.
e. Maintain a convex facial surface.
f. None of the above
g. All of the above

g: The ceramist and dentist can redirect light and create illusions to compensate for the loss of
the papilla.

107. TRUE OR FALSE: (1) According to Garber and Salama, the gingival outline of the maxillary
incisors should follow the upper lip. (2) Disharmony occurs with a curved gingival outline
against a flat upper lip when the patient smiles.
a. Both 1 and 2 are true.
b. 1 is true and 2 is false.
c. 1 is false and 2 is true.
d. Both 1 and 2 are false.

a: In a patient with a high smile line, particular attention should be given to the relationship of
the upper lip to the gingival outline for optimal esthetic results. (Source: Garber DA, Salama MA.
The aesthetic smile: Diagnosis and treatment. Periodontol 2000 1996;11:18–28.)

108. TRUE OR FALSE: (1) The progression of the gingival margins is considered normal when the
gingival margin of the lateral incisor is 1.0 to 1.5 mm coronal to the gingival margins of the
central incisor and canine on each side. (2) It is considered unesthetic when the gingival mar-
gin of the lateral incisor is too apical.
a. Both 1 and 2 are true.
b. 1 is true and 2 is false.
c. 1 is false and 2 is true.
d. Both 1 and 2 are false.

a: A prudent clinician must carefully evaluate the gingival margins because often the gingival
margins of the central incisors are too coronal (due to supereruption). Intrusion of these teeth will
correct the problem prior to implant placement.

207
7 IMPLANT PROSTHODONTICS

109. T
 RUE OR FALSE: When performing implant site development using orthodontic extrusion:
(1) Overextrusion is considered a good and safe option. (2) Orthodontic vertical extrusion
followed by extraction prior to implant placement is an integral part of hard and soft tissue
development.
a. Both 1 and 2 are true.
b. 1 is true 2 is false.
c. 1 is false and 2 is true.
d. Both 1 and 2 are false.

c: Overextrusion simply obscures clinical landmarks, making the implant procedure more of a
subtractive surgical process.

110. T
 RUE OR FALSE: (1) Achieving ideal soft tissue form between adjacent implants is very
predictable. (2) When extracting multiple anterior maxillary teeth, the best scenario is to
perform immediate implant placement into each extraction socket.
a. Both 1 and 2 are true.
b. 1 is true and 2 is false.
c. 1 is false and 2 is true.
d. Both 1 and 2 are false.

d: Placing implants next to each other creates the largest esthetic challenges. It is irregular to
achieve ideal soft tissue around adjacent implants. This is because concurrent multiple tooth loss
results in the loss of the facial bone plate and flattening of the interproximal bone scalloping.

111. W
 hen increasing the patient’s vertical dimension in a full-mouth implant reconstruction,
which sound can be affected with too much vertical opening?
a. “P”
b. “V”
c. “S”
d. “D”
e. “R”

c: If this has not resolved within 2 to 4 weeks, the vertical dimension should be reduced until the
patient’s speech is back to normal.

112. T
 RUE OR FALSE: (1) Root banking has been used effectively to maintain ridges in the hori-
zontal direction. (2) Root banking can only be performed on nonvital teeth.
a. Both 1 and 2 are true.
b. 1 is true and 2 is false.
c. 1 is false and 2 is true.
d. Both 1 and 2 are false.

b: Root banking can be used to bury both vital and nonvital teeth in pontic sites next to implants.
Vital exposed pulp tissue can form anastomoses with the overlying soft tissues after burial and
remain vital. In 2007, Salama et al showed that this technique is useful in maintaining the ridge in
the horizontal dimension. (Source: Salama M, Ishikawa T, Salama H, Funato A, Garber D. Advan-
tages of the root submergence technique for pontic site development in esthetic implant therapy.
Int J Periodontics Restorative Dent 2007;27:521–527.)

208
113. In histologically comparing the functional loading capacity of an implant versus a natural
tooth, Ogiso et al (1994) found that:
a. Hyperocclusion caused dental pain.
b. Hyperocclusion caused tooth intrusion and bone thickening around the implant.
c. Hyperocclusion caused tooth extrusion and bone loss around the implant.
d. None of the above

b: The clinician must be very careful in managing the occlusion when restoring dental implants
because tooth movement can cause complicated situations. (Source: Ogiso M, Tabata T, Kuo
PT, Borgese D. A histologic comparison of the functional loading capacity of an occluded dense
apatite implant and the natural dentition. J Prosthet Dent 1994;71:581–588.)

114. In the study by Ogiso et al, when two opposing implant-supported restorations were put
into high occlusion:
a. The patient experienced pain.
b. Bone thickened around both implants.
c. Bone loss occurred around both implants.
d. None of the above

c: Opposing implants placed into high occlusion will cause bone loss around both implants.

115. Which mandibular implant overdenture arrangement is the simplest and most cost-effective
and has the highest level of patient satisfaction?
a. Four implants and a bar
b. Two implants and a bar
c. Two Locators and two implants
d. Four Locators

c: In a prospective randomized crossover trial, Burns et al showed that the simplest technique
for an overdenture in the mandible included two implants and two Locators, and this technique
had the same level of perceived stability and retention as the other groups as well as the highest
level of patient satisfaction. (Source: Burns DR, Unger JW, Coffey JP, Waldrop TC, Elswick RK Jr.
Randomized, prospective, clinical evaluation of prosthodontic modalities for mandibular implant
overdenture treatment. J Prosthet Dent 2011;106:12–22.)

116. Why are bar-supported and -retained nonrotational implant overdentures so unpopular today?
a. They are complicated to fabricate.
b. They have a high rate of acrylic resin fracture.
c. They have a high maintenance requirement as the attachments wear or break.
d. All of the above

d: These restorations have lost popularity due to fatigue and failure of various components and
their high level of maintenance. The only indication today would be for lip support, which can
only be achieved with a flange.

209
7 IMPLANT PROSTHODONTICS

117. T
 RUE OR FALSE: (1) Angled abutments can provide angle correction of an implant trajectory
of up to 35 degrees. (2) Angled abutments may be a poor choice for shallow implants in the
anterior dentition.
a. Both 1 and 2 are true.
b. 1 is true and 2 is false.
c. 1 is false and 2 is true.
d. Both 1 and 2 are false.

 : Angled abutments can correct significant implant trajectory problems. However, when implants
a
are too shallow in highly esthetic areas, the angled abutment collar may become visible and may
also encroach on the restorative space.

118. Full-arch zirconia restorations are contraindicated when:


a. There is limited posterior restorative space.
b. Long posterior cantilevers are present.
c. Esthetics is of primary concern.
d. The patient is elderly.
e. a and b
f. c and d
g. All of the above

e: When the posterior restorative space is limited and a cantilever is present, the risk of the
cantilevered segment fracturing with fatigue increases. This is because of the higher forces closer
to the hinge axis and thinner material cross section.

119. T
 RUE OR FALSE: Pure titanium is a highly reactive metal and forms a surface oxide as soon
as it is exposed to the atmosphere.

True: Osseointegration is a dynamic process in which surface features of the implant play a part
in controlling molecular and cellular behavior. While osseointegration has been observed using
different materials, it is most often used to describe the reaction of bone tissues to titanium oxide.

120. W
 hich of the following statements is true about a fixed metal and acrylic resin prosthesis
supported by four to six implants?
a. These restorations are safer when used in the maxillary arch.
b. These restorations are safer when used in the mandibular arch.
c. These restorations are contraindicated when the restorative space is less than 15 mm.
d. These restorations can be used with any amount of restorative space.
e. b and c
f. All of the above

e: In a study by Davis et al (2003), it was shown that the maxillary anterior teeth for this type of
prosthesis have a high failure rate. (Source: Davis DM, Packer ME, Watson RM. Maintenance
requirements of implant-supported fixed prostheses opposed by implant-supported fixed pros-
theses, natural teeth, or complete dentures: A 5-year retrospective study. Int J Prosthodont
2003;16:521–523.) When the restorative space is below 15 mm, these restorations have a tendency
to fracture because the acrylic resin is weak in thinner sections. The acrylic resin interface must
be at least 4 mm thick for adequate strength.

210
121. TRUE OR FALSE: (1) Immediate loading of implants should be safe for torque levels of
20 Ncm or more. (2) Immediate loading of implants is safer in the posterior maxilla than
the anterior mandible.
a. Both 1 and 2 are true.
b. 1 is true and 2 is false.
c. 1 is false and 2 is true.
d. Both 1 and 2 are false.

d: Immediate loading of implants should be safe for torque levels of 35 Ncm or more. Immediate
loading of implants is safer in the anterior mandible than the posterior maxilla.

122. Which is the strongest restoration for full-mouth implant reconstruction?


a. Titanium framework and acrylic resin teeth
b. Titanium framework and stacked porcelain
c. Milled base metal framework and stacked porcelain
d. All of the above are equally strong

c: Milled base metal alloys are an excellent choice for full-arch restorations when the restorative
space is limited and cantilevers are present. They bond well to ceramic, they fit well, and they
are strong even when restoration width is thin.

123. Which zirconia implant connection is the strongest (Kim et al, 2013)?
a. Zirconia interface
b. Friction-fit titanium connection and zirconia crown
c. Bonded titanium connection and zirconia crown
d. All are equally strong

c: In a study by Kim et al (2013), a universal testing machine evaluated load to failure. Titanium
inserts bonded to zirconia crowns had the highest load to fracture and performed the best. Bond-
ing between the zirconia and titanium was achieved with Panavia (Kuraray) resin cement. (Source:
Kim JS, Raigrodski AJ, Flinn BD, Rubenstein JE, Chung KH, Mancl LA. In vitro assessment of
three types of zirconia implant abutments under static load. J Prosthet Dent 2013;109:255–263.)

124. Which is the correct order for bite force (least to most)? (1) Complete dentures. (2) Implant-
retained overdenture. (3) Natural teeth. (4) Implant-supported and -retained fixed prosthesis.
a. 1, 2, 3, 4
b. 1, 3, 2, 4
c. 1, 2, 4, 3
d. 4, 3, 2, 1
e. 4, 3, 1, 2
f. None of the above

a: Transitioning a patient from a complete denture to an implant-supported fixed prosthesis


results in a dramatic increase in maximum bite force. Complete denture wearers are unable to
exert similar forces due to the limitations of pain in the denture-bearing soft tissues.

211
7 IMPLANT PROSTHODONTICS

125. In a classic study, Monteith (1984) looked at the management of forces on mandibular
distal-extension prostheses and evaluated various design concepts. What did he conclude?
a. The soft tissues provide adequate resilience for a stable prosthesis.
b. Teeth, implants, and the soft tissues move apically under occlusal load to the same degree.
c. Teeth and implants move apically under occlusal load to the same degree.
d. Although teeth, implants, and the soft tissues move apically to the same degree, dentures are
better because they provide lip support.
e. Implant-supported prostheses provide a significant advantage in terms of support.

e: Implants only move 4 to 5 microns under occlusal loads as opposed to 500 microns for a partial
distal-extension denture. This provides a significant advantage in terms of comfort and support.
(Monteith BD. Management of loading forces on mandibular distal-extension prostheses. Part I:
Evaluation of concepts for design. J Prosthet Dent 1984;52:673–681.)

126. Which of the following implant-supported restorations has the added advantage of retrievability?
a. Cement-retained restorations
b. Screw-retained restorations
c. A combination screw- and cement-retained prosthesis
d. All of the above
e. None of the above

b: Retrievability is an advantage for servicing, replacing, or salvaging the restoration. A true
screw-retained restoration always provides this advantage for safety.

127. W
 hen considering long-term maintenance of the implant-supported prosthesis, all of the
following should be considered EXCEPT:
a. Loosening of the retaining screw
b. Porcelain fracture
c. Abutment fracture
d. A need to modify the prosthesis through the loss of an abutment
e. Discoloration of the ceramic

e: A screw-retained restoration allows retrievability and avoids long-term complications.

128. W
 hich of the following is an added advantage of implant-supported prostheses compared to
removable partial dentures?
a. They somewhat protect the remaining dentition and surrounding soft tissues.
b. Patients wearing removable partial dentures often exhibit movement of the remaining teeth.
c. Patients wearing removable partial dentures often exhibit greater plaque retention and in-
creased bleeding index and bone loss.
d. Removable partial denture abutment teeth are lost significantly more often than implant-
supported prosthesis abutment teeth in the long term.
e. All of the above

e: Aquilino et al found a 56% 10-year survival rate for removable partial dentures, compared with
92% for fixed partial dentures. (Source: Aquilino SA, Shugars DA, Bader JD, White BA. Ten-year
survival rates of teeth adjacent to treated and untreated posterior bounded edentulous spaces. J
Prosthet Dent 2001;85:455–460.) As a whole, implant-supported restorations provide a significant
health advantage over removable partial dentures.

212
129. The predictability of the outcome of an implant restoration in the posterior dentition
depends on which of the following?
a. Available space
b. Implant number and position(s)
c. Occlusal considerations
d. Type of prosthesis
e. Overall treatment plan
f. All of the above

f: When treatment planning an implant-supported prosthesis in the posterior dentition, all of
these factors must be considered.

130. When prosthetically evaluating the mesiodistal space required for implant restorations,
one must ensure all of the following EXCEPT:
a. The implant must be at least 1.5 mm away from the adjacent tooth.
b. The implant must be at least 3 mm away from the adjacent tooth.
c. The implant must be at least 3 mm away from the adjacent implant.
d. A wider-diameter implant should be selected for posterior implants.

b: The mesiodistal space depends on the type of tooth being replaced (molar or premolar) as
well as the number of teeth. A diagnostic wax-up often helps with this decision-making process.

131. What are the average heights of the maxillary first premolar, second premolar, and first
molar, respectively, at their contact points?
a. 7.1 mm, 6.6 mm, and 10.4 mm
b. 8.1 mm, 9.1 mm, and 10.1 mm
c. 9.1 mm, 9.1 mm, and 10.1 mm
d. 4.8 mm, 4.7 mm, and 7.9 mm
e. 4.2 mm, 6.2 mm, and 8.2 mm
f. None of the above

 : It is essential to know these figures for proper prosthetic treatment planning. Otherwise, hygiene,
a
esthetic, and form and emergence issues will arise.

132. What are the average heights of the maxillary first premolar, second premolar, and first
molar, respectively, at the CEJ?
a. 7.1 mm, 6.6 mm, and 10.4 mm
b. 8.1 mm, 9.1 mm, and 10.1 mm
c. 9.1 mm, 9.1 mm, and 10.1 mm
d. 4.8 mm, 4.7 mm, and 7.9 mm
e. 4.2 mm, 6.2 mm, and 8.2 mm
f. None of the above

d: A critical point to understand is that the natural tooth narrows towards the CEJ, so implant
neck sizes should be selected accordingly. It would be a mistake to use wide-platform implants
in areas where the neck of the natural tooth is narrower than the implant neck.

213
7 IMPLANT PROSTHODONTICS

133. H
 ow much space is required for placement of two 4-mm-diameter implants in an edentulous
bounded ridge (teeth on either side)?
a. 8 mm
b. 10 mm
c. 12 mm
d. 14 mm
e. 16 mm

d: This allows 3 mm between the implants and a 1.5-mm interproximal space for each adjacent
tooth.

134. When replacing molars and premolars with implants, what is the recommended space
requirement between the residual ridge and the opposing occlusion (restorative space)?
a. 4 mm
b. 6 mm
c. 8 mm
d. 10 mm
e. 12 mm

d: These teeth are 10 mm tall on average. If there is inadequate space, one may consider ridge
reduction or preparation, orthodontic intrusion, and removal of the opposing tooth. The absolute
minimum requirement is 7 mm.

135. T
 RUE OR FALSE: (1) Short implants do not last as long as longer implants. (2) Short
implants show more crestal bone loss. (3) The crown-to-implant ratio is an important
factor for implant survival.
a. 1, 2, and 3 are true.
b. 1, 2, and 3 are false.
c. Only 1 is true.
d. Only 2 is true.
e. Only 3 is true.

b: In a study by ten Bruggenkate et al (1998), shorter implants (6 to 8 mm) lasted just as long as
longer implants (10 to 12 mm) and did not show increased crestal bone loss despite the unfavor-
able crown-to-implant ratio. (Source: ten Bruggenkate CM, Asikainen P, Foitzik C, Krekeler G,
Sutter F. Short [6-mm] nonsubmerged dental implants: Results of a multicenter clinical trial of 1
to 7 years. Int J Oral Maxillofac Implants 1998;13:791–798.)

136. W
 hen planning the placement and restoration of posterior implants, all of the following are
critical factors EXCEPT:
a. The implants should be placed so that the projection of the fixture is contained within the
confines of the crown.
b. The screw access hole should be positioned toward the functional cusps.
c. The posterior mandibular implants should be placed so that the exit angle of the screw access
hole points toward the inner incline of the palatal cusp.
d. The posterior maxillary implants should be placed so that the exit angle of the screw access
hole points toward the inner incline of the buccal cusp.

 : The screw access hole should be positioned toward the center of the occlusal table. This allows
b
adequate bulk and thickness all around the hole.

214
137. All of the following are predictable ways of reducing occlusal forces on implants EXCEPT:
a. Splinting
b. Making even occlusal contacts
c. Increasing the cantilever length
d. Reducing the cantilever length
e. Ensuring shallow posterior cuspal inclination

c: Cantilevers create destructive forces on implants. They should be kept to a minimum as much
as possible. Splinting multiple posterior implant restorations creates excellent force distribution.

138. All of the following are advantages of splinting implant restorations EXCEPT:
a. The patient can still floss.
b. Forces are better distributed.
c. There is no time wasted checking proximal contacts.
d. Delivery is easy.
e. Screw loosening is less likely.

a: The only real disadvantage of splinting implant restorations is that patients cannot floss their
teeth. Although patients seldom complain about this, it should be discussed prior to the start
of treatment.

139. When restoring a tooth with a screw-retained restoration, which tooth is most likely to have
a high chance of esthetic failure due to the visibility of the screw access hole?
a. Maxillary incisor
b. Mandibular incisor
c. Maxillary premolar
d. Mandibular premolar
e. Maxillary canine

d: Mandibular premolars have the highest chance of patient dissatisfaction. These teeth have
small occlusal surfaces, so a screw access hole can take up the entire occlusal surface and create
an esthetic concern for the patient.

140. When using custom implant abutments, it is essential to:


a. Design circular abutments at the gingival level.
b. Design scalloped abutments to follow soft tissue contours.
c. Design a white-color abutment for better esthetics.
d. All of the above
e. None of the above

b: Circular abutments often end up too deep in some areas and too shallow in others. This may
create biologic or esthetic issues, respectively. Placing a cement margin too deep will make it
virtually impossible to clear excess cement.

215
7 IMPLANT PROSTHODONTICS

141. T
 he predictability of the esthetic outcome of an implant restoration depends on which of the
following factors?
a. Patient selection and smile line
b. Tooth position
c. Adjacent root position
d. Biotype of the periodontium and tooth shape
e. Bony anatomy of the implant site
f. Implant position
g. All of the above

g: Sometimes certain factors have an overriding effect.

142. W
 hat percentage of patients have thin and scalloped periodontium in the anterior dentition?
a. 5%
b. 15%
c. 25%
d. 35%
e. 45%

b: Only 15% of patients have a delicate soft tissue architecture. These are the cases where careful
management and grafting should be coupled with refined prosthodontic planning and treatment.

143. W
 hat are the advantages of implant-retained overdentures compared with fixed full-arch
restorations? (MULTIPLE ANSWERS)
a. Cost
b. Hygiene
c. Ease of fabrication
d. Lip support
e. Patient satisfaction
f. Maintenance level
g. All of the above

 , b, c, d, e: Implant overdentures are a relatively inexpensive and effective method to treat


a
edentulous patients. However, these restorations do require a significant amount of maintenance,
which the patient should be made aware of prior to the start of treatment.

144. Which of the following combinations creates the highest level of esthetic risk?
a. High scalloping, triangular teeth, high smile line, and thin soft tissues
b. Low scalloping, triangular teeth, high smile line, and thin soft tissues
c. Low scalloping, square teeth, high smile line, and thin soft tissues
d. Low scalloping, square teeth, low smile line, and thick soft tissues

a: It is essential to evaluate the soft tissue and smile line at presentation and not just look at the
bony architecture. The soft tissue presentation has a huge bearing on the esthetic predictability
of the case.

216
145. Which of the following is key to implant esthetics?
a. Bone preservation and/or regeneration
b. Implant surface design and position
c. Soft tissue thickness and quality
d. Prosthetic soft tissue support
e. Restorative material selection and emergence from soft tissues
f. All of the above

f: A complex and delicate interplay among these factors allows the clinician to create natural
esthetics.

146. Many studies have evaluated the efficacy and success of short implants. All of the following
are true EXCEPT:
a. 3.75  7–mm implants had a success rate of 90.3%.
b. 4  7–mm implants had a success rate of 92.8%.
c. 55% of all failures occurred before prosthetic connection.
d. The main failure was attributed to bone quality.
e. Short implants are a very poor choice in all situations.

e: According to a comprehensive study by das Neves et al (2006), short implants are an excellent
solution in situations where bone quality is good but volume and height are limited. (Source:
das Neves FD, Fones D, Bernardes SR, do Prado CJ, Neto AJF. Short implants: An analysis of
longitudinal studies. Int J Oral Maxillofac Implants 2006;21:86–93.)

147. Which of the following has been shown to create the most accurate impression for full-arch
implant reconstruction?
a. Not splinting components before impression-making
b. Splinting impression copings
c. Using polyether impression material
d. Using silicone impression material
e. All of the above

b: For full-arch reconstructions, particularly if the angles are more than 25 degrees divergent,
splinting impression copings prior to impression-making allows for a more accurate impression of
the spatial position of the components. (Source: Tsagkalidis G, Tortopidis D, Mpikos P, Kaisarlis G,
Koidis P. Accuracy of 3 different impression techniques for internal connection angulated implants.
J Prosthet Dent 2015;114:517–523.)

148. Which of the following is true regarding zirconia full-arch restorations (Venezia et al 2015)?
a. Zirconia full-arch restorations have a high failure rate.
b. Zirconia should never be used in full-arch restorations.
c. Zirconia full-arch restorations perform very well with minimal facial layering.
d. Zirconia full-arch restorations perform very well with substantial facial layering.
e. There are better materials on the market.

c : Zirconia full-arch restorations are easy to design, inexpensive, and appear to have high strength
and survival. (Source: Venezia P, Torsello F, Cavalcanti R, D’Amato S. Retrospective analysis of
26 complete-arch implant-supported monolithic zirconia prostheses with feldspathic porcelain
veneering limited to the facial surface. J Prosthet Dent 2015;114:506–512.)

217
7 IMPLANT PROSTHODONTICS

149. W
 hich of the following are advantages of laser-welded alloy frameworks over
cast frameworks:
a. Laser welding has better fit.
b. Laser welding has more passive fit.
c. Laser welding permits less vertical misfit.
d. Laser welding ensures more favorable stress distribution to the I-beam framework for
four implants.
e. All of the above

 : Laser welding is an excellent system to create passive, well-fitting frameworks. Casting generally
e
has high shrinkage and distortion levels.

150. T
 RUE OR FALSE: (1) A microtoughened and threaded implant surface maintains increased crestal
bone. (2) A straight or reversed implant neck design does NOT preserve more crestal bone.
a. Both 1 and 2 are true.
b. Both 1 and 2 are false.
c. 1 is true and 2 is false.
d. 1 is false and 2 is true.

c: It is important to read the literature, because an idea that may sound logical may prove not to
be a viable clinical treatment option.

151. T
 RUE OR FALSE: (1) A convergent, narrow abutment design produces more stable and
tighter peri-implant mucosa. (2) A concave and negative abutment profile on the facial areas
produces more stable and tighter peri-implant mucosa.
a. Both 1 and 2 are true.
b. Both 1 and 2 are false.
c. 1 is true and 2 is false.
d. 1 is false and 2 true.

 : It is essential that the abutment shape and emergence are tested prior to the final delivery. This
a
will allow a more predictable outcome in the long term. A concave facial profile is conducive to
maintaining a stable soft tissue height on the facial aspect of an anterior tooth.

152. W
 hat is the minimum number of posterior occluding units required to maintain a
stable occlusion?
a. 4
b. 6
c. 8
d. 10
e. 12

c: Eight posterior occluding units is the minimum requirement in a symmetric situation.
A premolar tooth is one unit, and a molar tooth is two units.

218
153. Which of the following are required for immediate placement of implants in the esthetic
zone? (MULTIPLE ANSWERS)
a. No infection
b. Intact bone walls
c. Thin soft tissues
d. Good implant stability
e. Atraumatic occlusion
f. All of the above

 , b, d, e: If the patient has thin soft tissues, or if any of these factors is not present, a prudent
a
clinician would graft the area and instruct the patient to return for implant placement.

154. TRUE OR FALSE: (1) It is more difficult to maintain a papilla between two adjacent implants
than between an implant and an adjacent tooth. (2) The height of the papilla between two
adjacent implants is unpredictable.
a. Both 1 and 2 are true.
b. Both 1 and 2 are false.
c. 1 is true and 2 is false.
d. 1 is false and 2 is true.

a: As soon as the supracrestal fibers (which are designed to hold a papilla in the correct position)
disappear, the papilla will begin to disappear. The longer the edentulous span, the higher this risk.

155. Which occlusal classification will place the most destructive occlusal force on an anterior
maxillary implant restoration?
a. Class I
b. Class II, division 1
c. Class II, division 2
d. Class III

b: A deep vertical overlap with minimal horizontal overlap creates severe destructive forces on
anterior implant restorations and can create screw loosening and fractures.

156. TRUE OR FALSE: In a prospective endoscopic study on cemented implant restorations:


(1) Excess cement was associated with signs of peri-implantitis in the majority (81%) of cases.
(2) Clinical and endoscopic signs of the disease were absent for 74% of the test implants
after removal of the excess cement.
a. Both 1 and 2 are true.
b. Both 1 and 2 are false.
c. 1 is true and 2 is false.
d. 1 is false and 2 is true.

a: Excess cement is most likely the main cause of peri-implantitis. Prosthetic abutment design
and cementation techniques will have a huge impact on the gingival response to the restoration.
(Source: Renvert S, Quirynen M. Risk indicators for peri-implantitis. A narrative review. Clin Oral
Implants Res 2015;26[suppl 11]:15–44.)

219
7 IMPLANT PROSTHODONTICS

157. T
 RUE OR FALSE: (1) The gingival index and periodontal response around cemented implant
restorations worsen with time. (2) The gingival response around screw-retained restorations
worsens with time.
a. Both 1 and 2 are true.
b. Both 1 and 2 are false.
c. 1 is true and 2 is false.
d. 1 is false and 2 is true.

c: It has been clearly demonstrated that screw-retained restorations have a much better gingi-
val response in the short and long term than cement-retained restorations. (Source: Renvert S,
Quirynen M. Risk indicators for peri-implantitis. A narrative review. Clin Oral Implants Res
2015;26[suppl 11]:15–44.)

158. W
 hat is the most common concern when constructing a metal-ceramic full-arch restoration
made with base metal alloys?
a. Bonding of the ceramic to the alloy
b. The cost of the restoration
c. The color of the definitive restoration
d. The shape of the restoration
e. The strength of the framework

a: Traditionally, base metal alloys did not bond to ceramic very well, and this resulted in
delamination problems. The advent of newer milled frameworks has minimized these problems,
and base metal frameworks have gained popularity.

159. M
 illed base metal implant frameworks have:
a. Poor fit on the implant.
b. Poor bonding of the framework to the ceramic material.
c. A high level of corrosion.
d. a and c
e. b and c
f. All of the above
g. None of the above

 : Base metal alloys, when milled rather than cast, have excellent properties relating to fit, bonding,
g
and corrosion. They are a good choice when space is limited because they have high strength in
smaller diameters. Also, the cost factor is very attractive to patients.

160. A
 ll of the following are good rules when cementing implant restorations EXCEPT:
a. Cement margins should be at least 2 mm subgingival for esthetics.
b. Cements that can protect teeth can actually damage implant surfaces.
c. It is wise to use radiopaque cements.
d. Implant cements should not be shaded to blend in with the gingiva.

a: Cement margins should be no more than 0.5 mm subgingival. This ensures that excess cement
can be easily removed.

220
161. Which of the following rules must be followed when immediately loading implants in the
anterior maxilla of a partially dentate patient?
a. The tooth must have protrusive contacts.
b. The implant must have at least 15 Ncm of torque on placement.
c. Interproximal contacts must be relieved.
d. A hard diet must be followed to strengthen the implant.
e. All of the above

c: Interproximal contacts must be removed because micromovement can occur as the adjacent
teeth move during function. This can result in loss of osseointegration.

162. Who discovered immediate loading on dental implants?


a. Brånemark
b. Al-Faraje
c. Schnitman
d. Tarnow
e. Chee

c: Paul A. Schnitman was the first person to write about immediate loading and the successful
osseointegration of these implants, which were only placed to hold a provisional restoration during
function. (Schnitman PA, Wohrle PS, Rubenstein JE. Immediate fixed interim prostheses supported
by two-stage threaded implants: Methodology and results. J Oral Implantol 1990;16:96–105.)

163. New screw technology allows reangulating screw access holes up to “x” degrees from the
original path and in a “y” radius. What are the values for x and y?
a. x = 10, y = 180
b. x = 25, y = 180
c. x = 25, y = 360
d. x = 45, y = 360
e. None of the above

c: Cemented restorations can be avoided with the help of new driver and screw technology.

164. What is the correct definition of osseointegration?


a. A process whereby clinically asymptomatic rigid fixation of alloplastic materials is achieved
and maintained in bone during functional loading
b. 100% bone-to-implant contact around the entire circumference of the implant
c. 80% bone-to-implant contact around the entire circumference of the implant
d. 60% bone-to-implant contact around the entire circumference of the implant
e. The formation of a direct interface between an implant and bone

a: However, this definition is constantly changing as our understanding of the process evolves.

221
7 IMPLANT PROSTHODONTICS

165. A
 ll of the following are strategies to reduce bone loss around the implant neck EXCEPT:
a. Altering the implant neck design
b. Altering the implant surface characteristics
c. Platform switching
d. Using a one-piece implant
e. Using the widest implant possible

e: Many strategies have been developed to maintain bone levels around the implant neck. Wider
implants have a higher chance of bone loss around the neck because they may have inadequate
surrounding bone volumes.

166. W
 hich of the following theories could explain why platform switching works?
a. Biomechanical theory
b. Biologic width theory
c. Inflammation at the IAJ theory
d. None of the above
e. All of the above

e: Platform switching refers to the inward horizontal repositioning of the implant-abutment junc-
tion (IAJ) so that it is positioned away from the outer edge of the implant and adjacent bone.
All three mechanical and biologic theories have been proposed, but none has yet been proven.

167. W
 hich of the following is a criticism of the external hex implant connection?
a. It has limited effectiveness when subjected to off-axis loads.
b. Under high occlusal loads, the external hex connection may facilitate micromovement of the
abutment, thus causing instability of the joint, which may result in abutment screw loosening
or even fatigue fracture.
c. Eccentric lateral force may cause more rotation displacement than centric force.
d. All of the above
e. None of the above

d: The external hex is still a fantastic choice for full-arch rehabilitations from a restorative point of
view, providing many restorative options.

168. T
 he internal connection implant has all of the following advantages EXCEPT:
a. It is a stable and secure connection.
b. The walls of the narrow implant are thick enough for use in load-bearing areas.
c. It has higher resistance to bending forces.
d. It has improved load distribution in the connection area.

b: Narrow internal connection implants, particularly those made with commercially pure titanium,
pose a risk for fracture. Using a wider-diameter implant reduces this risk.

222
169. What is a preload?
a. It is what the technician does to the implant crown on the stone cast.
b. It is something that is done in laboratory studies to see if the crown breaks.
c. It is something that is done in clinical studies to see if the crown breaks.
d. It is when an implant screw is tightened by applying torque that develops a force within
the screw.
e. None of the above

d: A preload is essential in implant screw technology. As the screw is tightened, it elongates,
producing tension. Elastic recovery of the screw pulls the two parts together, creating a clamping
force. The preload force in the screw, from elongation to elastic recovery, is equal in magnitude
to the clamping force.

170. Considering the average skeletal growth of children, what are the guidelines for
implant placement? (MULTIPLE ANSWERS)
a. Implants should not be placed in boys under the age of 18 years.
b. Implants should not be placed in girls under the age of 16 years.
c. Skeletal growth never continues into the second decade of life.
d. All of the above

a, b: Growth in children has a large range and standard deviation. There are no absolute rules,
and a prudent clinician will need to investigate and monitor each child’s growth before implants
are placed. In some people, growth can continue well into their 20s.

171. Which of the following can cause screw loosening?


a. Off-axis occlusal contacts
b. Lateral excursive contacts
c. Interproximal contacts between natural teeth and implant restorations
d. Protrusive contacts
e. Parafunctional forces
f. Nonpassive frameworks that attach to the implants
g. All of the above

g: The external load quickly erodes the preload, causing shaking and micromovement that can
result in screw loosening. One must appreciate potential forces that will be exerted on the screw
joint in order to minimize the potential for screw loosening or fractures.

172. Implant screws can be affected by which of the following?


a. Thermal changes
b. Magnitude of occlusal forces
c. Cantilever length
d. Occlusal scheme
e. All of the above
f. None of the above

 : Chewing patterns as well as all of these factors can fatigue the screw joint connection, poten-
e
tially resulting in screw loosening. To avoid bending of the joint, the screw must be appropriately
resilient (elastic with a high-yield strength). Screw strength is a byproduct of its design and material.

223
BONE GRAFTING 8

Bone grafting has become an integral part of implant


therapy, as many patients present with insufficient bone
for implant stability. This chapter addresses the different
types of bone grafts available, the indications for grafting,
when to use membranes or meshes for protection, graft
harvesting, and several other relevant concerns.

225
8 BONE GRAFTING

1. A bone graft is defined as:


a. A living tissue capable of promoting bone healing that is transplanted into a bone defect
either alone or in combination with other materials.
b. An osseous tissue taken from a natural source and transplanted into a bone defect of a mem-
ber of the same species either alone or in combination with other materials.
c. A natural or synthetic material that does not contain viable cells but that can promote bone
healing when placed into a bone defect either alone or in combination with other materials.
d. A tissue taken from another species that does not contain viable cells and can induce bone
formation when placed into a bone defect either alone or in combination with other materials.

a: A bone graft is defined as a living tissue that is transplanted into a bone defect, either alone or
in combination with other materials, to promote bone healing. (Source: Zhao R, Yang R, Cooper
PR, Khurshid Z, Shavandi A, Ratnayake J. Bone grafts and substitutes in dentistry: A review of
current trends and developments. Molecules 2021;26:3007.)

2. A bone substitute is defined as:


a. A synthetic polymer that can induce bone formation when placed into a bone defect either
alone or in combination with other bone formation compatible materials.
b. Bone material taken from a different species that can induce bone formation when placed into
a bone defect either alone or in combination with other bone formation compatible materials.
c. A protein taken from a different species that can induce bone formation when placed into a
bone defect either alone or in combination with other bone formation compatible materials.
d. A natural or synthetic material that can achieve bone formation when placed into a bone de-
fect either alone or in combination with other bone formation compatible materials.

d: A bone substitute is defined as a natural or synthetic material that can achieve bone forma-
tion when placed into a bone defect either alone or in combination with other bone formation
compatible materials. It often contains a mineralized bone matrix without viable cells. (Source:
Zhao R, Yang R, Cooper PR, Khurshid Z, Shavandi A, Ratnayake J. Bone grafts and substitutes in
dentistry: A review of current trends and developments. Molecules 2021;26:3007.)

3. W
 hat is the estimated percentage of dental implants that require a bone graft to meet the
minimum bone volume requirements for successful placement?
a. 25%
b. 35%
c. 50%
d. 75%

c: 50%

4. What are the disadvantages of autografts? (MULTIPLE ANSWERS)


a. Lack of availability of sufficient graft tissue volume
b. Pain and morbidity associated with donor site
c. Need for two operative sites
d. Insufficient cancellous bone

a, b, c: All are disadvantages of autografts.

226
5. What are the disadvantages of allografts? (MULTIPLE ANSWERS)
a. Less consistent results
b. A risk of disease transfer, albeit finite
c. The structural integrity and mechanical strength of the bone is decreased by up to 50% due to
allograft processing.
d. Slower incorporation

a, b, c, d: All are disadvantages of allografts.

6. Osteogenesis refers to:


a. The ability of a bone grafting material to create a bioactive scaffold on which host cells can
develop. This structure facilitates the travel of vessels, osteoblasts, and host progenitor cells
into the interconnected osteomatrix.
b. The development of de novo bone via osteoblasts or progenitor cells existing within the
grafting material.
c. The recruitment of host stem cells into the grafting site, where localized proteins and other
factors stimulate the differentiation of osteoblasts from stem cells. Various growth factors
affect this process, including platelet-derived growth factors (PDGFs), transforming growth
factor beta (TGF-b), and fibroblast growth factors (FGFs).
d. The capability of a grafting material to bond chemically to the surface of the native bone in
the absence of intervening fibrous tissue.

b: Osteogenesis refers to the development of de novo bone via osteoblasts or progenitor cells
existing within the grafting material.

7. Osseointegration refers to:


a. The ability of a bone grafting material to create a bioactive scaffold on which host cells can
develop. This structure facilitates the travel of vessels, osteoblasts, and host progenitor cells
into the interconnected osteomatrix.
b. The development of de novo bone via osteoblasts or progenitor cells existing within the
grafting material.
c. The recruitment of host stem cells into the grafting site, where localized proteins and other
factors stimulate the formation of osteoblasts from stem cells. Various growth factors affect
this process, including PDGFs, TGF-b, and FGFs.
d. The capability of a grafting material to bond chemically to the surface of the native bone in
the absence of intervening fibrous tissue.

d: Osseointegration refers to the capability of a grafting material to bond chemically to the surface
of the native bone in the absence of intervening fibrous tissue.

227
8 BONE GRAFTING

8. Osteoinduction refers to:


a. The ability of a bone grafting material to create a bioactive scaffold on which host cells can
develop. This structure facilitates the travel of vessels, osteoblasts, and host progenitor cells
into the interconnected osteomatrix.
b. The development of de novo bone via osteoblasts or progenitor cells existing within the
grafting material.
c. The recruitment of host stem cells into the grafting site, where localized proteins and other
factors stimulate the formation of osteoblasts from stem cells. Various growth factors affect
this process, including PDGFs, TGF-, and FGFs.
d. The capability of a grafting material to bond chemically to the surface of the native bone in
the absence of intervening fibrous tissue.

c: Osteoinduction refers to the recruitment of host stem cells into the grafting site, where localized
proteins and other factors stimulate the formation of osteoblasts from stem cells.

9. Osteoconduction refers to:


a. The ability of a bone grafting material to create a bioactive scaffold on which host cells can
develop. This structure facilitates the travel of vessels, osteoblasts, and host progenitor cells
into the interconnected osteomatrix.
b. The development of de novo bone via osteoblasts or progenitor cells existing within the
grafting material.
c. The recruitment of host stem cells into the grafting site, where localized proteins and other
factors stimulate the formation of osteoblasts from stem cells. Various growth factors affect
this process, including PDGFs, TGF-b, and FGFs.
d. The capability of a grafting material to bond chemically to the surface of the native bone in
the absence of intervening fibrous tissue.

a: Osteoconduction refers to the ability of a bone grafting material to create a bioactive scaffold
on which host cells can develop.

10. W
 hen bone grafting materials or bone substitutes are placed in fresh extractions sockets,
they can perform which of the following? (MULTIPLE ANSWERS)
a. Prevent cells from the surrounding soft tissue from entering the extraction socket
b. Provide a scaffold for osteogenesis
c. Prevent the collapse of ancillary membranes
d. Maintain the space for bone formation during healing

b, c, d: All are actions that bone grafting materials and bone substitutes placed in fresh extraction
sockets can perform.

228
11. What are the differences between high-density polytetrafluoroethylene (d-PTFE) membranes
and expanded PTFE (e-PTFE) membranes? (MULTIPLE ANSWERS)
a. The original first-generation PTFE membranes were e-PTFE, meaning there were macropores
in the membrane; e-PTFE membranes are more difficult to remove because soft tissue tends
to grow into the macropores.
b. First generation e-PTFE membranes require primary closure to prevent bacterial contamina-
tion; therefore, e-PTFE membranes cannot be left exposed.
c. Second-generation PTFE membranes are d-PTFE membranes that have a submicron pore size
(0.21 microns) that prevents bacterial infiltration into the grafted site. A major advantage to
d-PTFE membranes is that they can be left exposed (although the exposed portions should be
kept clean as they are susceptible to plaque accumulation).
d. d-PTFE membranes block soft tissue ingrowth and the permeation of nutrients into the graft.

a, b, c, d: First-generation e-PTFE membranes require primary closure because the pore sizes
are large enough to allow for bacterial invasion, albeit not soft tissue invasion. The second-
generation membranes (d-PTFE) are denser with smaller pores that prevent bacterial invasion
and soft tissue ingrowth, and thus can be left exposed. However, neither the first-generation
(e-PTFE) or the second-generation (d-PTFE) allow for nutrients to cross the membrane and get
into the underlying graft.

12. What are the differences between the newer dense fibril matrix e-PTFE membranes and
regular e-PTFE and d-PTFE membranes? (MULTIPLE ANSWERS)
a. Dense fibril matrix e-PTFE barrier membranes are composed of compact (dense) PTFE fibril
matrices. They do not require primary closure and can be left exposed.
b. Dense fibril matrix e-PTFE barrier membranes allow nutrients to permeate the barrier.
c. Dense fibril matrix e-PTFE barrier membranes (1) block bacterial invasion and (2) stop soft
tissue ingrowth.
d. Because of the fibril configuration, dense fibril matrix e-PTFE barrier membranes cannot be
left exposed.

a, b, c: The third-generation PTFE membranes (dense fibril matrix e-PTFE) have the advantages
of blocking bacterial invasion and the ingrowth of soft tissue. They also allow the permeation of
nutrients into the graft material. They can be left exposed, which makes d an incorrect answer.

13. Alveolar ridge socket preservation procedures are performed to: (MULTIPLE ANSWERS)
a. Prevent necrosis of gingival papillae
b. Minimize the amount of horizontal ridge resorption
c. Minimize the amount of vertical ridge resorption
d. Prevent loss of keratinized attached gingival tissue on the buccal (labial) aspect of the ridge

b, c, d: All are goals of alveolar ridge socket preservation procedures.

14. When the resulting extraction socket has moderate buccal bone destruction
(≥ 2 to < 5 mm bone loss), which of the following are true? (MULTIPLE ANSWERS)
a. Ridge preservation is needed to prevent the collapse of the lingual bony plate.
b. Ridge preservation is needed to address potential buccal soft tissue migration into the socket.
c. Ridge preservation is necessary to prevent alveolar osteitis.
d. Ridge preservation is beneficial for proper bone regeneration.

b, d: Both statements are true.

229
8 BONE GRAFTING

15. W
 hen the resulting extraction socket has severe buccal bone destruction (≥ 5 mm bone loss),
which of the following must be considered? (MULTIPLE ANSWERS)
a. With most of the buccal wall gone, the clinician must consider preserving the space necessary
for new bone formation.
b. A buccal flap raised apical to the mucogingival junction.
c. Debriding the socket of all soft tissue and infection.
d. Placement of a tack or suture-secured, nonmobile resorbable membrane.

a, b, c, d: All must be considered.

16. C
 omplications experienced with titanium mesh–guided bone grafting procedures include:
(MULTIPLE ANSWERS)
a. Early exposure of the titanium mesh
b. Decreased thickness of gingival phenotype
c. Lingual tissue recession
d. Infection (major or minor)

a, d: Both are complications associated with titanium mesh–guided bone grafting procedures.

17. P
 iezoelectric surgery allows for selective cutting—the cutting of bone but not soft tissues.
This minimizes damage to vital structures, such as surrounding nerves, blood vessels, and
soft tissues. The reason selective cutting works is because:
a. Soft tissue cutting requires a frequency of 50,000 Hz, and the piezoelectric device uses a
frequency of 20,000 to 30,000 Hz for selective cutting of hard tissues such as bone.
b. Cutting soft tissue with a sonic instrument is not possible at any frequency.
c. Ultrasonic instruments are not “sharp” like static-cutting steel instrumentation.
d. The cavitation effect of ultrasonic vibrations causes a decompression that prevents soft
tissues from being cut.

a: Piezoelectric devices are operated at frequencies lower than that needed to cut soft tissue.

18. C
 ontraindications for sinus elevation grafting procedures include which of the following?
(MULTIPLE ANSWERS)
a. Poor oral hygiene
b. Presence of bony septum
c. Patency of the osteomeatal complex (OMC)
d. Sinus pathology that includes untreated acute or chronic sinusitis

a, d: Both are contraindications for sinus elevation grafting procedures.

19. W
 hen performing the harvest of a ramus graft, the maximum thickness of the graft should
be no more than:
a. 25% of the mandibular ramus thickness
b. 40% of the mandibular ramus thickness
c. 50% of the mandibular ramus thickness
d. 75% of the mandibular ramus thickness

c: The maximum thickness of the graft should be no more than 50% of the mandibular ramus thickness.

230
20. The primary anatomical structure of concern when retrieving a ramus graft is:
a. The lingual nerve as it approaches the crest of the mandibular posterior ridge
b. The buccal nerve as it crosses over the deepest portion of the vertical ramus
c. The inferior alveolar nerve as it traverses through the ramus and body of the mandible
d. The facial artery as it crosses the inferior portion of the mandible at the site of the first molar

c: The inferior alveolar nerve as it traverses through the ramus and body of the mandible.

21. The negative aspect(s) of the ramus donor graft include which of the following?
(MULTIPLE ANSWERS)
a. Retrieved bone generally has low vascularity.
b. A relatively small volume of bone is retrieved.
c. Postoperative muscle trismus is common.
d. There is potential for postoperative lip numbness.

b, d: Both are negative aspects of ramus bone grafts.

22. Benefits of utilizing the chin (symphysis) as an intraoral harvest donor graft site include:
(MULTIPLE ANSWERS)
a. A greater volume of bone is obtained than with a ramus graft.
b. Ease of access.
c. A greater volume of cancellous bone is obtained than with a ramus graft.
d. Less risk of postoperative chin numbness than with the ramus graft.

a, b: Both are benefits of using the chin symphysis as an intraoral harvest donor graft site.

23. Taking bone grafts from the posterior iliac crest (PIC) has which of the following
advantages? (MULTIPLE ANSWERS)
a. A large volume (150 mL) of cortico-cancellous bone can be obtained.
b. Reflection of only two muscles (gluteus medius and gluteus maximus) is necessary to access
the graft site.
c. Bone stock retrieved is osteogenic, osteoinductive, and osteoconductive.
d. Once grafted, the bone is resistant to peri-implantitis.

a, c: Both are advantages of taking bone grafts from the PIC.

24. The anterior iliac crest (AIC) is a traditional extraoral harvest site for intraoral bone grafting.
Which of the following are advantages of using the AIC as a donor graft? (MULTIPLE ANSWERS)
a. A suitable volume of cortical bone (up to 5 cm by 5 cm) can be retrieved.
b. 30 to 50 mL of cancellous bone can be harvested.
c. The anterior ilium has an abundance of osteoprogenitor cells, which facilitate bone growth
and revascularization at the receptor site.
d. Reflection of only two muscles (gluteus medius and gluteus maximus) is necessary to access
the graft site.

a, b, c: All are advantages of using the AIC as a donor graft.

231
8 BONE GRAFTING

25. Inferior alveolar nerve lateralization (IANL) is a technique that allows for implant placement
in the severely resorbed mandible without the use of bone grafting. However, the technique
does have significant disadvantages. Which of the following are disadvantages of IANL?
(MULTIPLE ANSWERS)
a. Possible permanent neurosensory dysfunction
b. Mandibular fracture
c. Complicated prosthetic emergence profile(s) with excessive interarch spaces
d. Narrow implant diameters required in cases with horizontal defects

a, b, c, d: All are disadvantages of IANL.

26. T
 he use of recombinant morphogenetic proteins, such as recombinant human bone
morphogenetic protein 2 (rhBMP-2), guides the modulation and differentiation of
mesenchymal stem cells into osteoblasts. The advantages of using rhBMP-2 include
which of the following? (MULTIPLE ANSWERS)
a. It reduces the sigma for bone formation.
b. It enhances allograft success.
c. It induces the differentiation of stem cells native to the allograft.
d. It reduces the need for using autogenous bone and thus decreases surgical time.

b, d: Both are advantages of using rhBMP-2.

27. Free gingival grafts (FGGs) are often used to increase the zone of keratinized gingival tissue
around dental implants. What are the main disadvantages of using FGGs? (MULTIPLE ANSWERS)
a. There is poor esthetic color incorporation of the graft with the proximal soft tissues.
b. FGGs often chronically interfere with normal tactile sensations.
c. Characteristics of the donor site tissue are maintained, which can be unacceptable in the
esthetic zone.
d. FGGs can be subject to sloughing with time.

a, c: Both are disadvantages of using FGGs.

28. S
 ubepithelial connective tissue grafts (SCTGs) are frequently used for (1) the management of
recession and soft tissue defects and (2) augmenting alveolar ridge contours. What are some
advantages of using SCTGs? (MULTIPLE ANSWERS)
a. Blood supply is of little concern.
b. Implant sites augmented with SCTGs at the time of implant placement have superior esthetics
and thicker peri-implant tissues.
c. Some exposed implant coverage can be achieved when a combination epithelialized-
subepithelial connective tissue graft is utilized.
d. The procedure is technically easier than an FGG.

b, c: Both are advantages of using SCTGs.

232
29. An alternative to autogenous bone grafting is to use AIC bone marrow aspirate to
deliver autogenous stem cells to the site. The stem cells will differentiate into bone-forming
osteoblasts. This technique normally provides excellent results with few complications.
However, minor complications can occur. Possible complications of using bone marrow
aspirate include: (MULTIPLE ANSWERS)
a. Deep, drawing pain when the bone marrow aspirate is withdrawn.
b. Soreness at the site of perforation through the iliac crest.
c. If the patient’s bone marrow has transformed from hematopoietic active “red marrow”
to mostly yellow “fatty marrow,” minimal stem cells are retrieved.
d. If the aspirating needle is poorly positioned, a “dry tap” can occur, in which no aspirate
is retrieved.

a, b, c, d: All are possible complications of using bone marrow aspirate.

30. Using bone marrow aspirate to obtain mesenchymal stem cells is an emerging technique.
Advantages of this technique include which of the following? (MULTIPLE ANSWERS)
a. Aspirated autogenous bone marrow can be used in conjunction with allografts, thus
stimulating osteoinduction.
b. The need for a second open surgical site to retrieve stem cells from an autogenous bone graft
is eliminated.
c. When large quantities of stem cells are retrieved, this technique may be superior to
autogenous bone grafting.
d. In comparison to retrieving extraoral autogenous bone, this procedure is relatively simple
and can be performed on an outpatient basis.

a, b, c, d: All are advantages of using bone marrow aspirate to obtain mesenchymal stem cells.

233
ZYGOMATIC 9
AND PTERYGOID
IMPLANTS

When there is insufficient maxillary bone and grafting is


not a predictable option, zygomatic or pterygoid implants
can be used to anchor into the zygomatic bone and the
pterygoid plate of the sphenoid bone. This chapter reviews the
anatomy of these structures and discusses the advantages,
contraindications, complications, and ideal placement of
zygomatic and pterygoid implants.

235
9 ZYGOMATIC AND PTERYGOID IMPLANTS

1. TRUE OR FALSE: The zygomatic bone has three surfaces—maxillary, temporal, and orbital.

False: The three surfaces of the zygomatic bone are the lateral, temporal, and orbital surfaces.

2. TRUE OR FALSE: The zygomatic bone contains three foramina.

True: The three foramina of the zygomatic bone are the zygomaticofacial, zygomatico-orbital,
and zygomaticotemporal foramina.

3. All of the following muscles attach to the zygomatic bone EXCEPT:


a. Zygomaticus major
b. Zygomaticus minor
c. Masseter
d. Temporal
e. Levator labii superioris

d: The temporal muscle does not attach to the zygomatic bone.

4. The temporomandibular joint (TMJ) is located in the:


a. Temporal fossa
b. Infratemporal fossa

b: The TMJ is located in the infratemporal fossa.

5. Advantages of zygomatic implants include:


a. Elimination of the need to perform sinus elevation surgery. Although lateral window sinus
elevation is a valid treatment modality, it comes with certain contraindications and
complications and adds to the treatment time.
b. Decreased morbidity.
c. Increased success rate of restoring the posterior maxilla with dental implants with bicortical
stabilization in this soft bone region.
d. Shortened treatment time.
e. They may be used to treat patients with partial or total edentulism.
f. All of the above except e
g. All of the above

g: All of the above

236
6. Absolute contraindications for zygomatic implant therapy include all of the following EXCEPT:
a. Acute sinusitis
b. Chronic sinusitis
c. Zygoma pathology
d. Malignant systemic disease

b: Chronic sinusitis is not an absolute contraindication for zygomatic implant therapy.

7. TRUE OR FALSE: For full-arch reconstruction in the maxilla, at least two regular implants
in the anterior maxilla must assist the zygomatic implants, and the prosthesis must have a
proper design and passive fit.

True: Compared to standard implants, zygomatic implants have an increased tendency to bend
under horizontal loads due to increased implant length (30 to 52.5 mm) and limited bone support
at the maxillary alveolar crest. Accordingly, for full-arch reconstruction in the maxilla, at least two
regular implants in the anterior maxilla must assist the zygomatic implants, and the prosthesis
must have a proper design and passive fit.

8. TRUE OR FALSE: Forces that cause bending moments (lateral loads) are most unfavorable
for implants as they can jeopardize the long-term stability of the implants and the implant-
supported restoration. Occlusal forces to the zygomatic implants are mainly supported by
the zygomatic bone, as there is less than 1 mm of residual crestal maxillary bone at the
implant platform in most cases.

True

9. TRUE OR FALSE: Zygomatic implants do not have to be splinted with other implants.

False: Zygomatic implants should be splinted with other implants. Splinting ensures distribution
of functional loads across all implants in the prosthesis.

10. TRUE OR FALSE: The original Brånemark zygomatic implant technique often led to implant
placement with problematic prosthetic consequences.

True: Following the original intrasinus implant path protocol in order to reach the zygomatic
buttress will often lead to palatal implant placement with problematic prosthetic consequences.
This original technique does not take the anatomy of the zygoma into consideration.

237
9 ZYGOMATIC AND PTERYGOID IMPLANTS

11. T
 RUE OR FALSE: The zygoma anatomy–guided approach (ZAGA) to zygomatic implant
placement entails placing the head of the implant where the prosthetic teeth will be located
without forcing an intrasinus implant path every time.

True: The ZAGA approach allows for implant emergence that is close to the top of the crest and not
too palatal. Taking anatomy and biomechanics into consideration for optimal prosthetic design leads
to less bulky constructions, which is not only beneficial from a hygiene point of view but is also more
comfortable for the patient. The ZAGA technique is also beneficial for soft tissue health because the
maxillary sinus/palate is not perforated near the implant head. When there is adequate alveolar bone,
the implant path will be either completely or partially intrasinus (Figs a and b). However, when there
is not enough alveolar bone, the preparation is made buccally to the crest in the shape of a canal
with no perforation at the sinus floor level (Fig c). Where there is severe resorption, the zygomatic
implant is placed lateral to the maxillary wall to directly perforate the zygomatic buttress (Fig d).

a b

A B
Complete intrasinus path Partial intrasinus path
Complete intra-sinus path Partial intra-sinus path

c d

(a) Complete intrasinus path. C (b) D


Partial intrasinus path. (c) Extrasinus
path. (d) Extramaxillary path. Extrasinus path Extramaxillary path
Extra-sinus path Extra-maxillary path

12. S
 oft tissue dehiscence is usually a major concern with the extrasinus zygomatic implant path
technique. To avoid soft tissue dehiscence:
a. Make the midcrestal incision palatal to the midcrest.
b. Create a U-shaped osteotomy on the alveolar bone.
c. Use a prophylactic connective tissue (CT) graft to increase the quality of the coverage around
the implant head.
d. Use a superior-constrictor advancement-rotation flap (SCARF).
e. All of the above except d
f. All of the above

f: All of the above

238
13. Advantages of pterygoid implants include:
a. Elimination of the need to perform sinus elevation surgery
b. Increased success rate of restoring the posterior maxilla with dental implants by providing
bicortical stabilization in this soft bone region
c. Elimination of distal cantilever
d. Decreased morbidity
e. Shortened treatment time
f. All of the above

f: All of the above

14. TRUE OR FALSE: The ideal placement of pterygoid implants is through the pyramidal process
of the palatine bone and the lateral pterygoid process of the sphenoid bone.

False: The ideal placement of a pterygoid implant is through the pyramidal process of the palatine
bone and the medial pterygoid process of the sphenoid bone.

15. TRUE OR FALSE: The entry point on the crest for the placement of pterygoid implants is
usually 10 mm from the distalmost border of the maxilla and within the buccal third of the
crestal ridge of the tuberosity. The degree of maxillary sinus pneumatization determines
anteroposterior insertion and angulation of the implant.

True

16. What is the mesial inclination of pterygoid implants?


a. 25 to 35 degrees
b. 35 to 55 degrees
c. 55 to 65 degrees

b: The mesial inclination of pterygoid implants is 35 to 55 degrees (with an average of 45 degrees).


The computed tomography occlusal view of the maxilla is used to determine the optimal angle
of implant placement to obtain a fixed implant for possible immediate function. The pterygoid
hamulus process on the medial pterygoid plate is palpable in the oropharynx, and the pterygoid
implant is placed laterally to this landmark.

17. What is the vertical angulation of pterygoid implants?


a. 10 degrees
b. 25 degrees
c. 45 degrees

c: The vertical angulation of pterygoid implants is usually around 45 degrees but may be up to
70 degrees as the entry point moves distally to avoid the sinus.

239
9 ZYGOMATIC AND PTERYGOID IMPLANTS

18. T
 RUE OR FALSE: Significantly underprepping the pterygoid implant osteotomy may lead to
pterygoid plate bone fracture and/or failed osteotomy engagement because the implant
might deviate from the osteotomy.

True

19. T
 RUE OR FALSE: Pterygoid implants should be splinted with other implants.

True: The splinting effect results in distribution of the functional loads across all implants in the
prosthesis.

20. P
 terygoid implant complications include:
a. Trismus
b. Bleeding
c. Implant migration into the maxillary sinus
d. Implant migration into the infratemporal fossa
e. Pain
f. All of the above

f: All of the above

240
BLOOD 10
CONCENTRATES AND
GROWTH FACTORS

Blood concentrates and growth factors speed healing


and have been found to improve outcomes of implant
surgery. This chapter reviews the rationale and process
for fabrication of blood concentrates in-office as well as the
commercial products available on the market.

241
10 BLOOD CONCENTRATES AND GROWTH FACTORS

1. The component of whole blood that is directly responsible for healing of injured tissues is:
a. White blood cells (WBCs)
b. Red blood cells (RBCs)
c. Platelets
d. Plasma

c: All cells play a role in healing injured tissue, but platelets play an immediate role because they
are responsible for controlling the immediate issue of bleeding and then the release of growth
factors that will in turn activate receptors that will direct chemotactic and mitogenic activities to
other cells necessary for healing. WBCs help prevent and fight infection. RBCs are responsible
for carrying oxygen from the lungs to all tissues. Plasma distributes nutrients, removes waste
products, and prevents infection.

2. Following centrifugation, the top layer formed in a yellow-top vacutainer tube containing
trisodium citrate and whole blood is:
a. Platelet-rich plasma (PRP)
b. Platelet-rich serum (PRS)
c. Platelet-poor plasma (PPP)
d. Platelet-poor serum (PPS)

c: The top layer is PPP, containing fat globules, coagulating factors, and proteins. Plasma is the
liquid portion of blood and makes up approximately 55% of whole blood. The remaining 45% of
the collected whole blood is (1) RBCs, which have the greatest mass and are driven to the bottom
of the tube; (2) WBCs, which have less mass and settle on top of the RBCs; and (3) platelets,
which have even less mass and settle on top of the WBCs. The top layer is referred to as PPP
because the contents of whole blood that have greater mass (platelets, WBCs, and RBCs) are
driven toward the bottom of the tube during the centrifugation process, thus leaving the plasma,
with very few (if any) platelets, at the top of the tube. The desired growth factors are contained
within the b-granules of platelets; therefore because PPP contains few platelets, it contains few
growth factors.

3. Following centrifugation, the bottom layer formed in a yellow-top vacutainer tube containing
trisodium citrate and whole blood is:
a. WBCs
b. RBCs
c. Plasma
d. Platelets

b: RBCs are found at the bottom of the centrifuged whole blood in a yellow-top collection tube
because they have the greatest mass of the suspended blood contents. The WBCs have less mass
and are found settled on top of the RBCs. The platelets have less mass than the WBCs and RBCs
and therefore are found settled on top of the WBCs.

242
4. Following centrifugation, the middle layer formed in a yellow-top vacutainer tube containing
trisodium citrate and whole blood is:
a. RBCs
b. WBCs and platelets
c. RBCs and platelets
d. WBCs and RBCs

b: The middle part of a centrifuged yellow-top vacutainer tube with whole blood contains buffy
coat–platelet-rich plasma (BC-PRP). BC-PRP contains both WBCs and platelets, with the platelets
settled on top of the WBCs. The bottom layer of the tube is concentrated RBCs. Options c and d
are not possible because the centrifugation process separates the contents of those combinations.

5. Trisodium citrate is:


a. An anticoagulant
b. A hemostatic agent
c. An antiplatelet agent
d. Prothrombin

a: Trisodium citrate is an anticoagulant. The citrate binds (chelates) the ionized calcium circulating
in the plasma of whole blood. Ionized calcium is a needed cofactor for (1) platelet aggregation and
(2) formation of plasma fibrin in both the intrinsic and extrinsic coagulation pathways. Once the
citrate has chelated the ionized calcium, the calcium is no longer available for the clotting cascade.

6. The red-top vacutainer tube is used to:


a. Fabricate leukocyte- and platelet-rich fibrin (L-PRF)
b. Fabricate injectable platelet-rich fibrin (I-PRF)
c. Fabricate a platelet-rich blood clot
d. Fabricate leukocyte-rich serum

a: The red-top glass vacutainer tube induces the formation of a blood clot or fibrin clot because
the silica contained within the glass activates clotting. Plastic red-top vacutainer tubes have a
spray-on silica coating to induce the blood clot or fibrin clot formation. If either the glass red-top
or plastic red-top tubes are not centrifuged immediately, a blood clot will form. This blood clot
will contain WBCs, RBCs, and platelets. However, if the tubes are centrifuged immediately, then
the RBCs will be separated and driven to the bottom of the tube. This will result in a fibrin clot
(ie, platelet-rich fibrin—PRF). Most of the WBCs and platelets will be trapped in the fibrin clot.

7. The current recommended centrifugation time for simultaneous centrifugation of yellow-top


and red-top tubes containing whole blood is:
a. 300 to 400 rpm for 5 to 7 minutes
b. 500 to 700 rpm for 7 to 9 minutes
c. 2,100 to 2,500 rpm for 9 to 11 minutes
d. 2,700 to 3,100 rpm for 10 to 12 minutes

d: Rutkowski et al demonstrated that the highest platelet and WBC concentration in yellow-top
tubes will be achieved at 2,700 to 3,100 rpm for 10 to 12 minutes in a fixed-rotor centrifuge. The
same settings will provide sufficient PRF membranes. Therefore, this speed and time allows for the
simultaneous centrifugation of both the yellow-top and red-top vacutainer tubes. Multiple studies
have examined slower speeds, which may benefit the formation of PRF membranes, although
clinical significance has not been studied. (Source: Rutkowski JL, Thomas JM, Bering CL, et al.
Analysis of a rapid, simple, and inexpensive technique used to obtain platelet-rich plasma for use
in clinical practice. J Oral Implantol 2008;34:25–33.)

243
10 BLOOD CONCENTRATES AND GROWTH FACTORS

8. On days 1 to 3 following application of BC-PRP and PRF, which product releases the most
growth factors at the operative site?
a. BC-PRP
b. PRF
c. The release of growth factors is equal for BC-PRP and PRF for days 1 to 3.
d. The release of growth factors is zero, as neither product is active until day 4.

a: Studies have demonstrated that BC-PRP releases more growth factors of dental relevance on
days 1 to 3. BC-PRP provides an immediate and stronger release of growth factors, whereas PRF
releases its growth factors gradually and over a longer time. (Sources: Kobayashi E, Flückiger L,
Fujioka-Kobayashi M, et al. Comparative release of growth factors from PRP, PRF, and advanced-
PRF. Clin Oral Investig 2016;20:2353–2360. / He L, Lin Y, Hu X, Zhang Y, Wu H. A comparative
study of platelet-rich fibrin (PRF) and platelet-rich plasma (PRP) on the effect of proliferation and
differentiation of rat osteoblasts in vitro. Oral Surg Oral Med Oral Pathol Oral Radiol Endod
2009;108:707–713.)

9. On days 4 to 7 following the application of BC-PRP and PRF, which product releases the most
growth factors at the operative site?
a. BC-PRP
b. PRF
c. The release of growth factors is equal for BC-PRP and PRF.
d. The release of growth factors is zero, as neither product is active on days 4 to 7.

c: For days 4 to 7, BC-PRP and PRF are essentially equal in growth factor release. BC-PRP provides
an immediate and stronger early release of growth factors, and PRF releases growth factors
gradually and for a longer time.

10. O
 n days 8 to 21 following the application of BC-PRP and PRF, which product releases the
most growth factors at the operative site?
a. BC-PRP
b. PRF
c. The release of growth factors is equal for BC-PRP and PRF.
d. The release of growth factors is zero, as neither product remains active after day 6.

b: On days 8 to 21, PRF releases more growth factors. BC-PRP provides an earlier and stronger
release of growth factors, but PRF releases growth factors gradually and for a longer time.

11. T
 he yellow-top tube is used to create:
a. BC-PRP
b. I-PRF
c. L-PRF
d. Advanced PRF (A-PRF)

a: The yellow-top tube contains an anticoagulant (trisodium citrate), so it is used to create BC-PRP.
The anticoagulant keeps the whole blood from forming a blood clot or a fibrin clot. BC-PRP is a
viscous liquid, and for it to take effect at the surgical site, it must be placed on a carrier to retain it
at the site. BC-PRP can be mixed with bone grafting material(s) or placed on a collagen membrane
to keep it at the surgical site. If it is not retained by a carrier at the site, it will flow away and not
demonstrate a local effect.

244
12. Which color glass tube top contains no coatings or additives?
a. Red
b. Yellow
c. Gold
d. White

a: The yellow-top tubes contain trisodium citrate, an anticoagulant, as a liquid additive. The gold-
top tubes (not used in dentistry) contain a clot activator and serum separating gel. The white-top
tubes are coated with spray-dried dipotassium ethylenediaminetetraacetic acid (K2 EDTA) on
the inner wall. EDTA acts as an anticoagulant by binding the calcium ions and interrupting the
clotting of the blood sample. The red-top glass tubes contain no coatings or additives. The glass
of the tube contains silica, which initiates the formation of the fibrin clot when centrifuged. If the
red-top tube is not centrifuged immediately, a blood clot will result. The difference between the
fibrin clot and the blood clot is that the fibrin clot contains few (if any) RBCs, whereas the blood
clot contains a fibrin clot with RBCs trapped within. Plastic red-top tubes do not inherently contain
silica, so their inner walls are spray-coated with silica to initiate the formation of the fibrin clot.

13. Which growth factor remains in a latent form when contained in the a-granules of the
platelets and immediately following release?
a. Platelet-derived growth factor AA (PDGF-AA)
b. Vascular endothelial growth factor (VEGF)
c. Epidermal growth factor (EGF)
d. Transforming growth factor beta 1 (TGF-b1)

d: PDGF-AA, VEGF, and EGF are in an active form when released from platelets. TGF-b1 is in a
latent form when stored in platelets and released. It remains in a latent form until it is activated
by proteases—matrix metalloproteinase-2 (MMP-2), thrombospondin-1 (TSP1), and TSP2.

14. The half-life of active (not latent) TGF-1 is:


a. 30 seconds
b. 60 seconds
c. 90 seconds
d. 6 hours

c: The half-life of active TGF-b1 has been reported to be from 90 to 180 seconds. (Sources:
Wakefield LM, Winokur TS, Hollands RS, Christopherson K, Levinson AD, Sporn MB. Recombinant
latent transforming growth factor beta 1 has a longer plasma half-life in rats than active transform-
ing growth factor beta 1, and a different tissue distribution. J Clin Invest 1990;86:1976–1984. /
Rutkowski JL, Thomas JM, Bering CL, et al. Analysis of a rapid, simple, and inexpensive technique
used to obtain platelet-rich plasma for use in clinical practice. J Oral Implantol 2008;34:25–33.)

15. How long is BC-PRP stable when left in a yellow-top glass vacutainer tube at room temperature?
a. 1 hour
b. 1.5 hours
c. 3 hours
d. 6 hours

d: Studies have demonstrated that at room temperature, the platelets and growth factors contained
within BC-PRP remain stable and at consistent concentrations for up to 6 hours following centrif-
ugation. It may remain stable longer, but the study only extended to 6 hours. (Source: Rutkowski
JL, Thomas JM, Bering CL, et al. Analysis of a rapid, simple, and inexpensive technique used to
obtain platelet-rich plasma for use in clinical practice. J Oral Implantol 2008;34:25–33.)

245
10 BLOOD CONCENTRATES AND GROWTH FACTORS

16. When collecting multiple vacutainers of whole blood, what should the order of draw be?
a. Collect all red-top tubes first, then all yellow-top tubes.
b. Collect all yellow-top tubes first, then all red-top tubes.
c. Collect tubes alternately, but start with red-top tubes, then yellow-top, then red-top, etc.
d. Collect tubes alternately, but start with yellow-top tubes, then red-top, then yellow-top, etc.

b: All yellow-top tubes should be collected first because they contain the anticoagulant trisodium
citrate, which will prevent the formation of a blood clot. Once collected, yellow-top tubes should
be inverted gently 12 times before centrifuging. This will ensure the mixing of the anticoagulant
with the collected whole blood. Red-top tubes should be collected closest to the time of centrif-
ugation. If the red-top tubes are not centrifuged immediately, a blood clot will begin to form
that will not have the stability or function of the intended fibrin clot. Do not invert red-top tubes
because the silica contained in the glass tubes or the sprayed-on silica coating in plastic tubes
will activate the platelets to initiate the formation of a blood clot.

17. Ideally, how soon after collecting whole blood in a red-top vacutainer tube should it
be centrifuged?
a. Immediately
b. 5 minutes
c. 10 minutes
d. 15 minutes

a: If the red-top tubes are not centrifuged immediately, a blood clot will begin to form before
a fibrin clot can be initiated. This is true of all red-top tubes, whether they are glass or plastic.

18. W
 hen processing the contents of the red-top vacutainer tube, for how long is the fibrin clot
placed in the compressor box?
a. 2 minutes
b. 3 minutes
c. 4 minutes
d. 5 minutes

d: Current protocol calls for a 5-minute compression time. While using a shorter compression time
with greater pressure is acceptable, it is believed that longer compression time with consistent
pressure probably leads to more even fibrin clot formation due to less disruption of the fibril
formation within the fibrin clot.

19. O
 nce a PRF membrane is processed, how can it be stored until the clinician is ready to use it?
a. Store the PRF membrane in the serum found below the tray in the PRF compressor box.
b. Store the PRF membrane in the PPP created from centrifugation of the yellow-top vacutainer tube.
c. Store the PRF membrane in the RBC concentrate created from centrifugation of the yellow-
top vacutainer tube.
d. The PRF membrane should not be stored in anything but rather left to sit in the tray of the
PRF compressor box (with the lid on the box).

a: The liquid top layer of a centrifuged yellow-top tube is plasma (albeit PPP). The liquid expressed
from the PRF fibrin clot is serum. Plasma contains fibrinogen, and serum does not. The fibrinogen
in the yellow-top tube is not used to form a clot and remains unused in the plasma at the top of
the tube. The fibrinogen in the red-top tube was used to make the PRF clot, so it is not present
in the liquid that is expressed from the fibrin clot. Serum is less viscous and will flow, whereas
plasma has some viscosity and does not flow as easily. The serum expressed from the PRF clot is
an ideal hydrating medium for keeping the fibrin clot from shrinking.

246
20. Once the yellow-top vacutainer tube is centrifuged, the dotted line on the BC-PRP label
is placed:
a. 2 mm below the interface of the RBCs and the buffy coat
b. 2 mm below the interface of the PPP and the buffy coat
c. 2 mm above the interface of the PPP and the buffy coat
d. At the interface of the RBCs and the buffy coat

d: The dotted line of the label is to be placed at the interface of the RBC layer and the buffy coat
layer (WBCs and platelets). The larger space to the solid line (3 mm above the dotted line) is directed
toward the top of the vacutainer tube. The smaller space (2 mm) between the dotted line and the
lower solid line is directed toward the bottom of the vacutainer tube. This will result in approximately
500 µL of BC-PRP, which provides the ideal concentration of growth factors and volume of liquid for
wetting either 1 mL of bone graft material or a 12 mm by 15 mm collagen membrane.

21. Periosteum is a connective tissue that covers the bone surface and is a repository for pluripotent
cells that promote bone healing and formation. How many layers does the periosteum have?
a. One layer—a fibrous, dense, irregular connective tissue that contains fibroblasts that
differentiate mesenchymal stem cells into osteoblasts
b. Two layers—a fibrous layer and a layer next to the bone called the cambium layer that
contains osteoprogenitor cells that differentiate into osteoblasts
c. One layer—the cambium (or osteogenic) layer that contains autogenous growth factors
d. Three layers—the inner adhesive layer that makes the connective tissue adhere to the bone, the
middle cambium layer containing growth factors, and the outer fibrous layer containing fibroblasts

b: There are two layers to the periosteum. The outer layer is fibrous, dense, irregular connective
tissue that contains fibroblasts, and the inner layer that lies next to the bone is known as the
cambium layer or osteogenic layer. The cambium layer contains autogenous growth factors and
has a high concentration of progenitor cells that differentiate into osteoblasts.

22. Which osteoinductive recombinant growth factor has US Food and Drug Administration
(FDA) approval for use in dentistry?
a. Recombinant human bone morphogenetic protein 2 (rhBMP-2)
b. rhBMP-3
c. rhBMP-4
d. rhBMP-7

a: The only FDA-approved osteoinductive bone substitute growth factor is rhBMP-2. The other
options are used for research purposes only.

247
10 BLOOD CONCENTRATES AND GROWTH FACTORS

23. The use of rhBMP-2 (Infuse Bone Graft, Medtronic) can be related to side effects that are
dependent upon which of the following? (MULTIPLE ANSWERS)
a. Dosage
b. Time of administration
c. Carrier used to deliver the recombinant protein
d. Site of administration

a, c: Side effects are related to dosage and method of delivery. Studies and clinical use have shown
that dosage greatly influences clinical side effects of rhBMP-2, including induced inflammation,
osteoclast activation, induction of adipogenesis, local reactions, graft failure, infections, wound
healing complications, and cancer. Additionally, the carrier type and its presence are of great
importance in reducing the incidence of side effects. The rhBMP-2 must be retained at the surgi-
cal site by a carrier or it will be free to leave the area and move systemically, activating receptors
elsewhere in the body, which may result in side effects. Infuse Bone Graft is only available as a
solution applied to an absorbable collagen sponge (ACS). Over time, the ACS releases the protein
into the surgical site. The ACS also functions as a scaffold to assist in the growth of new bone.
Because rhBMP-2 is only administered at the time of surgery, option b is not correct. Also,
there have been no reports that the site of administration in the oral cavity influences side effects.
(Sources: Park SY, Kim KH, Kim S, Lee YM, Seol YJ. BMP-2 gene delivery-based bone regeneration
in dentistry. Pharmaceutics 2019;11:393. / Herford AS, Stoffella E, Stanford CM. Bone grafts and
bone substitute materials. In: Torabinejad M, Sabeti MA, Goodacre CJ (eds). Principles and Practice
of Single Implant and Restoration. Philadelphia: Saunders, 2014:75–86. / James AW, LaChaud G,
Shen J, et al. A review of the clinical side effects of bone morphogenetic protein-2. Tissue Eng Part
B Rev 2016;22:284–297. / Woo EJ. Adverse events reported after the use of recombinant human
bone morphogenetic protein 2. J Oral Maxillofac Surg 2012;70:765–767.)

24. T
 he FDA has approved Infuse Bone Graft for which of the following dental procedures?
(MULTIPLE ANSWERS)
a. Alveolar cleft repair
b. Horizontal augmentation of completely edentulous mandibular ridge
c. Sinus augmentation
d. Localized alveolar ridge augmentation for defects associated with extraction sockets

c, d: The FDA has only given approval for use of Infuse Bone Graft as an alternative to autogenous
bone graft for sinus augmentations and for localized alveolar ridge augmentations for defects
associated with extraction sockets.

25. B
 ased on clinical studies, clinicians can expect which of the following when rhBMP-2 with
ACS is used for alveolar ridge or maxillary sinus augmentation at the proper dose and
with the appropriate application process? (MULTIPLE ANSWERS)
a. Minimal side effects
b. Clinically significant bone formation for maxillary sinus augmentation without consistent
differences between various rhBMP-2 concentrations
c. Effectively maintained alveolar ridge height and enhanced alveolar ridge width when
applied properly to an extraction socket at the time of tooth removal
d. Significant bone loss at 25% of administration sites

a, b, c: In a human study, Freitas et al found that side effects were minimal and results were
clinically significant when rhBMP-2/ACS was used in the proper dose and as directed. Results
were clinically significant even at lower doses.

248
26. Which of the following statements are true of enamel matrix derivative
(EMD; Emdogain, Straumann)? (MULTIPLE ANSWERS)
a. It is an equine derivative xenogeneic regenerative material comprised of enamel matrix
proteins that are significant in root development.
b. It is a protein extract of porcine fetal enamel matrix that is responsible for the induction
of migration, proliferation, differentiation, adhesion, and mineralization of cells in
periodontal tissues.
c. It is an allograft protein extract used to induce in situ protein formation.
d. It induces the stimulation of transforming growth factor beta 1 (TGF-b1), BMP-2, BMP-7,
platelet-derived growth factor AB (PDGF-AB), vascular endothelial growth factor (VEGF),
fibroblast growth factor 2 (FGF-2), insulin-like growth factor 1 (IGF-1), epidermal growth
factor (EGF), osteopontin (OPN), and alkaline phosphatase protein synthesis.

b, d: EMD is a porcine derivative comprised of proteins responsible for root development. It is


also responsible for stimulating the release of multiple proteins significant for bone formation.

27. Clinicians can expect which of the following to be true regarding the effect of EMD on bone
regeneration as an adjunctive treatment for peri-implant diseases? (MULTIPLE ANSWERS)
a. EMD is safe when added to bone grafting materials.
b. The addition of EMD to bone grafting materials will result in significantly improved
clinical results.
c. The addition of EMD to bone grafting materials will not produce results inferior to
control protocols.
d. EMD improves bone-to-implant contact (BIC) when used with immediately placed implants.

a, c, d: Clinical studies have found inconsistent results with the addition of EMD to bone grafting
materials in implant dentistry. However, studies have confirmed that EMD is a safe alternative,
and its results were not inferior to study controls. Additionally, EMD does improve BIC when
used with immediately placed implants. (Source: Alberti A, Francetti L, Taschieri S, Corbella S.
The applications of enamel matrix derivative in implant dentistry: A narrative review. Materials
(Basel) 2021;14:3045.)

28. Which type of stem cells have the greatest utility and potential in implant dentistry?
a. Embryonic stem cells (ESCs)
b. Adult epithelial stem cells
c. Adult neural stem cells
d. Adult mesenchymal stem cells (MSCs)

d: Adult MSCs are of importance in regenerative medicine because they are capable of tissue
repair, tissue growth, cell substitution, and wound healing. MSCs can differentiate into osteo-
progenitor cells and thus enhance bone graft materials.

249
10 BLOOD CONCENTRATES AND GROWTH FACTORS

29. M
 SCs are thought to be either hypoimmunogenic (immune privileged) or immune evasive.
This property allows for:
a. Transplantation of the cells across major histocompatibility barriers and creation of
off-the-shelf therapy supplies created from cultured MSCs.
b. Retrieval of stem cells from the pulp of an extracted third molar and cryopreserving them
for eventual use by the same individual.
c. Transplantation of MSCs from one species to another (animal donor origin to human receptor).
d. Removal of stem cells from a bone marrow aspirate to be used in the same individual.

a: Because MSCs are either hypoimmunogenic or (more likely) immune evasive, they can be taken
from one individual and then cultured, cryopreserved, and eventually used in another individual
of the same species. Stem cells retrieved from extracted third molars or bone marrow aspirates
and later used in that same individual do not present an immunogenic issue. (Source: Ankrum
JA, Ong JF, Karp JM. Mesenchymal stem cells: Immune evasive, not immune privileged. Nat
Biotechnol 2014;32:252–260.)

30. T
 RUE OR FALSE: When attempting to perform a predictable tissue-engineered bone graft
using MSCs, it is necessary to have a scaffold present.

True: MSCs will stimulate bone-forming cells to proliferate and signal for further differentiation,
but there must be a scaffold present for the de novo bone to develop to the desired shape
and volume.

31. W
 hen using banked, off-the-shelf MSCs such as alloOss Plus (Ace Southern), 1 mL of the
grafting material containing the stem cells takes how long to thaw?
a. 15 to 20 minutes
b. 30 to 60 minutes
c. 90 to 120 minutes
d. 180 to 240 minutes

a: For a 1-mL package, thawing will take 15 to 20 minutes (20 to 25 minutes for a 2-mL package).
The product is shipped to the dental clinic on dry ice and must be kept cryopreserved until
ready for thawing. It must be used within 24 hours of arrival unless other long-term packaging
is provided. Once thawed, the product is a putty-like matrix combination of mesenchymal stem
cells and mineralized/demineralized allograft that must be used within 2 hours. The product is
osteoconductive, osteoinductive, osteogenic, and angiogenic.

32. G
 EM 21S (Lynch Biologics) is a commercially available product used to enhance bone
grafting results. What is GEM 21S?
a. rhPDGF-AA with allograft mineralized bone
b. rhPDGF with porcine xenograft
c. rhVEGF with beta-tricalcium phosphate (b-TCP)
d. rhPDGF-BB with b-TCP

d: GEM 21S is rhPDGF-BB, with b-TCP as a scaffolding material. The product was originally available
as just rhPDGF-BB without a scaffolding material. The scaffolding material was added to enhance
de novo bone formation. The product has been shown to improve the ingrowth of regenerative
stem cells into bone grafts within 72 hours. (Sources: Singh P, Suresh DK. Clinical evaluation of
GEM 21S(®) and a collagen membrane with a coronally advanced flap as a root coverage proce-
dure in the treatment of gingival recession defects: A comparative study. J Indian Soc Periodontol
2012;16:577–583. / Lynch Biologics website. https://www.lynchbiologics.com/gem-21s/. Accessed
10 November 2022.)

250
33. GEM 21S is supplied in what form?
a. b-TCP in particulate form with a separate syringe containing rhPDGF-BB
b. b-TCP in putty form with a separate syringe containing rhPDGF-BB
c. Powder containing both b-TCP and rhPDGF-BB
d. Putty containing both b-TCP and rhPDGF-BB

a: Gem 21S is a growth factor–enhanced matrix that is provided in two parts. One part is b-TCP
in particulate form, and the second part is a syringe containing rhPDGF-BB as a solution.

34. Dental dehydrated human amnion-chorion allograft membranes (ACM; BioXclude, Snoasis
Medical) are derived from human placental amniotic membranes and have two sides. ACMs
are rich in many of the same growth factors found in the a-granules of platelets. The chorion
side contains more growth factors than the amnion side. Which of the following statements
regarding these membranes are true? (MULTIPLE ANSWERS)
a. Amnion is filled with amniotic fluid and surrounds the embryo.
b. Amnion surrounds the chorion.
c. Chorion is filled with amniotic fluid and surrounds the embryo.
d. Chorion surrounds the amnion.

a, d: The side that rests next to the fetus is the amnion side, containing amniotic fluid. The chorion
covers the amnion layer. The chorion has two layers—an outer trophoblast layer and an inner layer
known as the mesoderm. AmnioExcite (Salvin Regenerative) contains not only the amnion and
chorion layers but also the trophoblast layer, which is thought to improve handling. The chorion
contributes 82% of the total growth factors with the exception of EGF, 20% of which is found in
the chorion layer and the remaining 80% in the amnion layer.

35. Ground tooth dentin has been studied as a grafting alternative to traditional bone grafting
materials. This is clinically possible because tooth dentin contains BMPs that induce de novo
bone formation without causing inflammation. Clinically, the steps in preparing the dentin
for bone grafting are as follows: (1) tooth extraction, (2) cleaning the tooth with a surgical
bur to remove all the periodontal ligament and enamel, (3) cleaning and drying the remain-
ing tooth, (4) grinding the tooth, and (5) placing the ground dentin in the extraction socket.
What is the ideal particle size for ground dentin when using it as an alternative to grafting
material for socket grafting?
a. 150 to 300 µm
b. 300 to 1,200 µm
c. 1,200 to 1,500 µm
d. 1,500 to 2,500 µm

b: A mixture of tooth dentin particle sizes between 300 and 1,200 µm should be used for grafting
material. It has been shown that particles of this size provide interparticulate spacing (allowing
for the ingrowth of blood vessels) and act as a scaffold for new bone formation. Employing a
variety of particle sizes prevents over-condensing the particles and compressing the interparticle
spacing, which would prevent the ingrowth of blood vessels. (Sources: Cervera-Maillo JM, Morales-
Schwarz D, Morales-Melendez H, Mahesh L, Calvo-Guirado JL. Autologous tooth dentin graft: A
retrospective study in humans. Medicina (Kaunas) 2021;58:56. / Pallesen L, Schou S, Aaboe M,
Hjørting-Hansen E, Nattestad A, Melsen F. Influence of particle size of autogenous bone grafts
on the early stages of bone regeneration: A histologic and stereologic study in rabbit calvarium.
Int J Oral Maxillofac Implants 2002;17:498–506. / Straumann Biomaterials Product Portfolio.
https://www.straumann.com/content/dam/mediacenter/straumann/en/documents/brochure/
product-information/490.561-en_low.pdf. Accessed 10 November 2022.)

251
BIBLIOGRAPHY
Chapter 1
Abubaker AO, Benson KJ. Oral and Maxillofacial Surgery Secrets, ed 2. St Louis: Mosby, 2007.
Aframian DJ, Lalla RV, Peterson DE. Management of dental patients taking common hemostasis-altering
medications. Oral Surg Oral Med Oral Pathol Oral Radiol Endod 2007;103(suppl 45):1–11.
American Heart Association. Infective Endocarditis. http://www.heart.org/HEARTORG/Conditions/
CongenitalHeartDefects/TheImpactofCongenitalHeartDefects/Infective-Endocarditis_UCM_307108_
Article.jsp#.Vxjs7atX--I. Accessed 21 April 2016.
American Society of Anesthesiologists. ASA Physical Status Classification System. https://www.asahq.
org/resources/clinical-information/asa-physical-status-classification-system. Accessed 20 April 2016.
Bagheri SC. Clinical Review of Oral and Maxillofacial Surgery: A Case-Based Approach, ed 2. St Louis:
Mosby, 2014.
Brady WF, Martinoff JT. Validity of health history data collected from dental patients and patient perception
of health status. J Am Dent Assoc 1980;101:642–645.
Brennan MT, Kent ML, Fox PC, Norton HJ, Lockhart PB. The impact of oral disease and nonsurgical treat-
ment on bacteremia in children. J Am Dent Assoc 2007;138:80–85.
Brusch JL. Infective Endocarditis. http://emedicine.medscape.com/article/216650-overview. Accessed
19 April 2016.
Budtz-Jørgensen E. Prosthodontics for the Elderly: Diagnosis and Treatment. Chicago: Quintessence, 1999.
Chi AC, Neville BW, Krayer JW, Gonsalves WC. Oral manifestations of systemic disease. Am Fam Physician
2010;82:1381–1388.
Chung TT, Grossman A, Clark AJL. Adrenal insufficiency. In: Jameson JL, De Groot LJ (eds). Endocrinology:
Adult and Pediatric, ed 6. Philadelphia: Saunders, 2010:1853–1863.
Collet JP, Himbet F, Steg PG. Myocardial infarction after aspirin cessation in stable coronary artery disease
patients. Int J Cardiol 2000;76:257–258.
Durack DT, Beeson PB. Experimental bacterial endocarditis. II. Survival of a bacteria in endocardial vege-
tations. Br J Exp Pathol 1972;53:50–53.
Fanning NF, Walters TD, Fox AJ, Symons SP. Association between calcification of the cervical carotid artery
bifurcation and white matter ischemia. AJNR Am J Neuroradiol 2006;27:378–383.
Flack JM, Adekola B. Blood pressure and the new ACC/AHA hypertension guidelines. Trends Cardiovasc
Med 2020;30:160–164.
Hersh EV, Moore PA. Three serious drug interactions that every dentist should know about. Compend
Contin Educ Dent 2015;36:408–413.

253
BIBLIOGRAPHY

Kahn MA, Hall JM. The ADA Practical Guide to Soft Tissue Oral Disease. Hoboken, NJ: Wiley-Blackwell, 2014.
Kandel ER, Schwartz JH, Jessell TM, Siegelbaum SA, Hudspeth AJ. Principles of Neural Science, ed 5.
New York: McGraw-Hill, 2012:318–319.
Kostis WJ, Shetty M, Chowdhury YS, Kostis JB. ACE inhibitor-induced angioedema: A review. Curr Hyper-
tens Rep 2018;20:55.
Lam D, Laskin D (eds). Oral and Maxillofacial Surgery Review: A Study Guide. Chicago: Quintessence, 2015.
Lavelle CLB. Applied Oral Physiology, ed 2. London: John Wright & Sons, 1988.
Little JW, Falace D, Miller C, Rhodus NL. Dental Management of the Medically Compromised Patient,
ed 7. St Louis: Mosby, 2008:240–250.
Little JW, Miller CS, Rhodus NL. Little and Falace’s Dental Management of the Medically Compromised
Patient, ed 9. St Louis: Elsevier, 2018.
Lockhart PB (ed). Oral Medicine and Medically Complex Patients, ed 6. Hoboken, NJ: Wiley-Blackwell, 2013.
Lucas VS, Lytra V, Hassan T, Tatham H, Wilson M, Roberts GJ. Comparison of lysis filtration and an auto-
mated blood culture system (BACTEC) for detection, quantification, and identification of odontogenic
bacteremia in children. J Clin Microbiol 2002;40:3416–3420.
Mayo Clinic. Factor V Leiden. http://www.mayoclinic.org/diseases-conditions/factor-v-leiden/basics/
symptoms/con-20032637. Accessed 20 April 2016.
Mergenhagen KA, Wattengel BA, Skelly MK, Clark CM, Russo TA. Fact versus fiction: A review of the
evidence behind alcohol and antibiotic interactions. Antimicrob Agents Chemother 2020;64:e02167-19.
Minneman KP, Wecker L. Brody’s Human Pharmacology: Molecular to Clinical, ed 4. St Louis: Mosby, 2006.
Mougeot FKB, Saunders SE, Brennan MT, Lockhart PB. Associations between bacteremia from oral sources
and distant-site infections: Tooth brushing versus single tooth extraction. Oral Surg Oral Med Oral
Pathol Oral Radiol 2015;119:430–435.
Napeñas JJ, Hong CH, Brennan MT, Furney SL, Fox PC, Lockhart PB. The frequency of bleeding compli-
cations after invasive dental treatment in patients receiving single and dual antiplatelet therapy. J Am
Dent Assoc 2009;140:690–695.
Neville BW, Damm DD, White DK, Waldron CA. Color Atlas of Clinical Oral Pathology. Philadelphia: Lea
& Febiger, 1990.
Ogle OE, Dym H, Weinstock RJ (eds). Medical Emergencies in Dental Practice. Chicago: Quintessence, 2016.
Patton LL, Glick M (eds). The ADA Practical Guide to Patients with Medical Conditions, ed 2. Hoboken,
NJ: Wiley-Blackwell, 2016.
Regezi JA, Sciubba JJ. Oral Pathology: Clinical-Pathologic Correlations, ed 2. Philadelphia: Saunders, 1993.
Rhodus NL, Little JW. Dental management of the patient with cardiac arrhythmias: An update. Oral Surg
Oral Med Oral Pathol Oral Radiol Endod 2003;96:659–668.
Roberts HW, Redding SW. Coronary artery stents: Review and patient-management recommendations.
J Am Dent Assoc 2000;131:797–801.
Roerecke M, Kaczorowski J, Myers MG. Comparing automated office blood pressure readings with other
methods of blood pressure measurement for identifying patients with possible hypertension: A system-
atic review and meta-analysis. JAMA Intern Med 2019;179:351–362.
Salam S, Yusuf H, Milosevic A. Bleeding after dental extractions in patients taking warfarin. Br J Oral
Maxillofac Surg 2007;45:463–466.
Schimmer BP, Parker KL. Adrenocorticotropic hormone; Adrenocortical steroids and their synthetic analogs;
Inhibitors of the synthesis and actions of adrenocortical hormones. In: Brunton LL, Lazo JS, Parker KL
(eds). Goodman and Gilman’s The Pharmacologic Basis of Therapeutics, ed 11. New York: McGraw-
Hill, 2005:1587–1612.
Seymour RA, Thomason JM, Ellis JS. The pathogenesis of drug-induced gingival overgrowth. J Clin
Periodontol 1996;23:165–175.
Sonis ST, Fazio RC, Fang LS. Oral Medicine Secrets. Philadelphia: Hanley & Belfus, 2003.
Streiff MB. Recommended Therapeutic Range and Duration of Warfarin Therapy. http://silkview.com/
refs/Warfarin%20INR%20Range%20and%20duration-5-31-05%20card%20version.pdf. Accessed 19
April 2016.

254
T-cell Modulation Group. http://www.tcells.org/scientific/tcell_activation. Accessed 19 April 2016.
Weber C. Guide to High Blood Pressure Stages and Classes. http://highbloodpressure.about.com/od/
newlydiagnosed/p/stage_pro.htm. Accessed 21 April 2016.
Wilson WR, Gewitz M, Lockhart PB, et al. Prevention of viridans group streptococcal infective endocarditis:
A scientific statement from the American Heart Association. Circulation 2021;143:e963–e978.
Wilson W, Taubert KA, Gewitz M, et al. Prevention of infective endocarditis: Guidelines from the American
Heart Association: A guideline from the American Heart Association Rheumatic Fever, Endocarditis,
and Kawasaki Disease Committee, Council on Cardiovascular Disease in the Young, and the Council
on Clinical Cardiology, Council on Cardiovascular Surgery and Anesthesia, and the Quality of Care and
Outcomes Research Interdisciplinary Working Group. Circulation 2007;116:1736–1754.
Wood NK, Goaz PW. Differential Diagnosis of Oral and Maxillofacial Lesions, ed 5. St Louis: Mosby, 1997.
Yagiela JA, Dowd FJ, Johnson B, Mariotti A, Neidle EA. Pharmacology and Therapeutics for Dentistry.
St Louis: Mosby, 2011:623.

Chapter 2
Kourkouta S, Dedi KD, Paquette DW, Mol A. Interproximal tissue dimensions in relation to adjacent
implants in the anterior maxilla: Clinical observations and patient aesthetic evaluation. Clin Oral Implants
Res 2009;20:1375–1385.
Tarnow DP, Cho SC, Wallace SS. The effect of inter-implant distance on the height of inter-implant bone
crest. J Periodontol 2000;71:546–549.
Teughels W, Merheb J, Quirynen M. Critical horizontal dimensions of interproximal and buccal bone
around implants for optimal aesthetic outcomes: A systematic review. Clin Oral Implants Res 2009;
20(suppl 4):134–145.

Chapter 3
Al-Faraje L. Surgical and Radiologic Anatomy for Oral Implantology. Chicago: Quintessence, 2013.
Avery JK. Oral Development and Histology. Baltimore: Williams & Wilkins, 1987.
Cho SC, Wallace SS, Froum SJ, Tarnow DP. Influence of anatomy on Schneiderian membrane perforations
during sinus elevation surgery: Three-dimensional analysis. Pract Proced Aesthet Dent 2001;13:160–163.
Craigmyle MBL. A Colour Atlas of Histology, ed 2. Chicago: Year Book Medical, 1995.
Hall-Craggs ECB. Anatomy as a Basis for Clinical Medicine. Munich: Urban & Schwarzenberg, 1985.
Kalpidis CD, Setayesh RM. Hemorrhaging associated with endosseous implant placement in the anterior
mandible: A review of the literature. J Periodontol 2004;75:631–645.
Logan BM, Reynolds PA. McMinn’s Color Atlas of Head & Neck Anatomy, ed 4. St Louis: Mosby, 2010.
McMinn RMH, Hutchings RT. A Colour Atlas of Human Anatomy [in Arabic]. St Louis: Mosby, 1980.
Miles DA. Color Atlas of Cone Beam Volumetric Imaging for Dental Applications. Chicago: Quintessence, 2008.
Norton NS. Netter’s Head and Neck Anatomy for Dentistry. Philadelphia: Saunders, 2006.
Piper SN, Maleck WH, Kumle B, Deschner E, Boldt J. Massive postoperative swelling of the tongue:
Manual decompression and tactile intubation as a life-saving measure. Resuscitation 2000;43:217–220.
Shah JT. Color Atlas of Head and Neck Surgery: Mouth, Pharynx, Larynx, Thyroid, Parotid, Soft Tissues &
Reconstructive Surgery. London: Wolfe Medical, 1990.
Ten Cate AR. Oral Histology: Development, Structure, and Function, ed 5. St Louis: Mosby, 1998.
Timmenga NM, Raghoebar GM, Liems RS, van Weissenbruch R, Manson WL, Vissink A. Effects of maxillary
sinus floor elevation on maxillary sinus physiology. Eur J Oral Sci 2003;111:189–197.

Chapter 4
Agliardi EL, Tetè S, Romeo D, Malchiodi L, Gherlone E. Immediate function of partial fixed rehabilitation
with axial and tilted implants having intrasinus insertion. J Craniofac Surg 2014;25:851–855.
Al-Faraje F. Surgical Complications in Oral Implantology: Etiology, Prevention, and Management. Chicago:
Quintessence, 2011.

255
BIBLIOGRAPHY

Anavi Y, Allon DM, Avishai G, Calderon S. Complications of maxillary sinus augmentations in a selective
series of patients. Oral Surg Oral Med Oral Pathol Oral Radiol Endod 2008;106:34–38.
Boyne PJ. Osseous Reconstruction of the Maxilla and the Mandible. Chicago: Quintessence, 1997.
Briggs RD, Wright ST, Cordes S, Calhoun KH. Smoking in chronic rhinosinusitis: A predictor of poor
longterm outcome after endoscopic sinus surgery. Laryngoscope 2004;114:126–128.
Buser D, Dahlin C, Schenk RK. Guided Bone Regeneration in Implant Dentistry. Chicago: Quintessence, 1994.
Cranin AN, Klein M, Simons A. Atlas of Oral Implantology, ed 2. St Louis: Mosby, 1999.
Das S, Becker AM, Perakis H, Prosser JD, Kountakis SE. The effects of smoking on short-term quality of
life outcomes in sinus surgery. Laryngoscope 2007;117:2229–2232.
de Papp A, Bone HG, Caulfield MP, et al. A cross-sectional study of bone turnover markers in healthy
premenopausal women. Bone 2007;40:1222–1230.
Devani P, Lavery KM, Howell CJ. Dental extractions in patients on warfarin: Is alteration of anticoagulant
regime necessary? Br J Oral Maxillofac Surg 1998;36:107–111.
Edwards BJ, Hellstein JW, Jacobsen PL. Updated recommendations for managing the care of patients
receiving oral bisphosphonate therapy: An advisory statement from the American Dental Association
Council on Scientific Affairs. J Am Dent Assoc 2008;139:1674–1677 [erratum 2009;140:522].
Francischone CE, Vasconcelos LW, Brånemark PI. Osseointegration and Esthetics in Single Tooth Reha-
bilitation. Chicago: Quintessence, 2000.
Granström G, Tjellström A, Albrektsson T. Postimplantation irradiation for head and neck cancer treatment.
Int J Oral Maxillofac Implants 1993;8:495–501.
Guler N, Delilbasi C. Ectopic dental implants in the maxillary sinus. Quintessence Int 2007;38:e238–e239.
Ihde S, Kopp S, Gundlach K, Konstantinovic VS. Effects of radiation therapy on craniofacial and dental
implants: A review of the literature. Oral Surg Oral Med Oral Pathol Oral Radiol Endod 2009;107:56–65.
Jensen OT. The Sinus Bone Graft, ed 2. Chicago: Quintessence, 2006.
Jensen OT, Shulman LB, Block MS, Iacono VJ. Report of the sinus consensus conference of 1996. Int J
Oral Maxillofac Implants 1998;13(suppl):11–45.
Jiménez-López V. Oral Rehabilitation with Implant-Supported Prostheses. Chicago: Quintessence, 1999.
Johnsson K, Hansson A, Granström G, Jacobsson M, Turesson I. The effects of hyperbaric oxygenation on
bone-titanium implant interface strength with and without preceding irradiation. Int J Oral Maxillofac
Implants 1993;8:415–419.
Kim YK, Hwang JY, Yun PY. Relationship between prognosis of dental implants and maxillary sinusitis
associated with the sinus elevation procedure. Int J Oral Maxillofac Implants 2013;28:178–183.
Lynch SE, Genco RJ, Marx RE. Tissue Engineering: Applications in Maxillofacial Surgery and Periodontics.
Chicago: Quintessence, 1999.
Nilsson P, Albrektsson T, Granström G, Röckert HO. The effect of hyperbaric oxygen treatment on bone
regeneration. An experimental study using the bone harvest chamber in the rabbit. Int J Oral Maxillofac
Implants 1988;3:43–48.
Nolan PJ, Freeman K, Kraut RA. Correlation between Schneiderian membrane perforation and sinus lift
graft outcome: A retrospective evaluation of 359 augmented sites. J Oral Maxillofac Surg 2014;72:47–52.
Oh E, Kraut RA. Effect of sinus membrane perforation on dental implant integration: A retrospective study
on 128 patients. Implant Dent 2011;20:13–19.
Pal US, Sharma NK, Singh RK, et al. Direct vs. indirect sinus lift procedure: A comparison. Natl J Maxillofac
Surg 2012;3:31–37.
Palacci P. Esthetic Implant Dentistry: Soft and Hard Tissue Management. Chicago: Quintessence, 2001.
Renouard F, Rangert B. Risk Factors in Implant Dentistry: Simplified Clinical Analysis for Predictable Treat-
ment, ed 2. Chicago: Quintessence, 2008.
Ruggiero S, Gralow J, Marx RE, et al. Practical guidelines for the prevention, diagnosis, and treatment of
osteonecrosis of the jaw in patients with cancer. J Oncol Pract 2006;2:7–14.
Si MS, Zhuang LF, Gu YX, Mo JJ, Qiao SC, Lai HC. Osteotome sinus floor elevation with or without grafting:
A 3-year randomized controlled clinical trial. J Clin Periodontol 2013;40:396–403.

256
Siervo S. Suturing Techniques in Oral Surgery. Chicago: Quintessence, 2008:211.
Valerin MA, Brennan MT, Noll JL, et al. Relationship between aspirin use and postoperative bleeding
from dental extractions in a healthy population. Oral Surg Oral Med Oral Pathol Oral Radiol Endod
2006;102:326.
Wallace SS, Mazor Z, Froum SJ, Cho SC, Tarnow DP. Schneiderian membrane perforation rate during sinus
elevation using piezosurgery: Clinical results of 100 consecutive cases. Int J Periodontics Restorative
Dent 2007;27:413–419.
Weinberg LA. Atlas of Tooth- and Implant-Supported Prosthodontics. Chicago: Quintessence, 2003.
Weiss CM, Weiss A. Principles and Practice of Implant Dentistry. St Louis: Mosby, 2001.
White LC, Kazi AA, Jang DW, Gurrola J, Kountakis SE. The effect of smoking on quality of life following
sinus surgery: 10-year follow-up. ORL J Otorhinolaryngol Relat Spec 2015;77:39–43.
Zarb GA, Bolender CL. Prosthodontic Treatment for Edentulous Patients: Complete Dentures and
Implant-Supported Prostheses, ed 12. St Louis: Mosby, 2004.

Chapter 5
Babbini M, Thomas ML. Pharmacology Test Prep: 1500 USMLE-Style Questions & Answers. New York:
Thieme Medical, 2015.
Bell WE. Orofacial Pains: Classification, Diagnosis, Management, ed 4. Chicago: Year Book Medical, 1989.
Costantinides F, Rizzo R, Pascazio L, Maglione M. Managing patients taking novel oral anticoagulants
(NOAs) in dentistry: A discussion paper on clinical implications. BMC Oral Health 2016;16:5.
Cunha BA, Ortega AM. Antibiotic failure. Med Clin North Am 1995;79:633–672.
Davis VL, Abukabda AB, Radio NM, et al. Platelet-rich preparations to improve healing. Part II: Platelet acti-
vation and enrichment, leukocyte inclusion, and other selection criteria. J Oral Implantol 2014;40:511–521.
deShazo RD, Nelson HS. An approach to the patient with a history of local anesthetic hypersensitivity:
Experience with 90 patients. J Allergy Clin Immunol 1979;63:387–394.
Dionne RA, Phero JC, Becker DE. Management of Pain & Anxiety in the Dental Office. Philadelphia:
Saunders, 2002.
Fonseca RJ, Marciani R, Turvey T. Oral and Maxillofacial Surgery, ed 2. Philadelphia: Saunders, 2008.
Gandy W. Severe epinephrine-propranolol interaction. Ann Emerg Med 1989;18:98–99.
Gómez-Moreno G, Guardia J, Cutando A. Interaction of paracetamol in chronic alcoholic patients. Impor-
tance for odontologists. Med Oral Patol Oral Cir Bucal 2008;13:E235–E238.
Goodman SB, Jiranek, W, Petrow E, Yasko AW. The effects of medications on bone. J Am Acad Orthop
Surg 2007;15:450–460.
Jeffcoat MK, Reddy MS, Wang IC, Meuninghoff LA, Farmer JB, Koth DL. The effect of systemic flurbiprofen
on bone supporting dental implants. J Am Dent Assoc 1995;126:305–311.
Kalafutova S, Juraskova B, Vlcek J. The impact of combinations of non-steroidal anti-inflammatory drugs
and anti-hypertensive agents on blood pressure. Adv Clin Exp Med 2014;23:993–1000.
Kozody R, Ready LB, Barsa JE, Murphy TM. Dose requirement of local anesthetic to produce grand mal
seizure during stellate ganglion block. Can Anaesth Soc J 1982;29:489–491.
Lang NP, Wilson TG, Corbet EF. Biological complications with dental implants: Their prevention, diagnosis
and treatment. Clin Oral Implants Res 2000;11(suppl 1):146–155.
Lee Y, Le CH, Oh U. Painful channels in sensory neurons. Mol Cells 2005;20:315–324.
Li XJ, Jee WS, Li YL. Flurbiprofen enhances growth and cancellous and cortical bone accumulation in
rapidly growing long bone. Bone 1989;10:35–44.
Lorenz K, Bruhn G, Heumann C, Netuschill, Brecx M, Hofmann T. Effect of two new chlorhexidine mouth-
rinses on the development of dental plaque, gingivitis, and discoloration: A randomized, investiga-
tor-blind, placebocontrolled, 3-week experimental gingivitis study. J Clin Peridontol 2006;33:561–567.
Marx RE. Oral & Intravenous Biphosphonate-Induced Osteonecrosis of the Jaws: History, Etiology, Preven-
tion, and Treatment. Chicago: Quintessence, 2007.

257
BIBLIOGRAPHY

Moeintaghavi A, Talebi-ardakani MR, Haerian-ardakani A, et al. Adjunctive effects of systemic amoxicillin


and metronidazole with scaling and root planing: A randomized, placebo controlled clinical trial. J
Contemp Dent Pract 2007;8(5):51–59.
Mombelli A. Microbiology and antimicrobial therapy of peri-implantitis. Periodontol 2000 2002;28:177–189.
Mombelli A, Lang NP. The diagnosis and treatment of peri-implantitis. Periodontol 2000 1998;17:63–76.
Munson ES, Wagman IH. Diazepam treatment of local anesthetic-induced seizures. Anesthesiology
1972;37:523–528.
Newman MG, van Winkelhoff AJ (eds). Antibiotic and Antimicrobial Use in Dental Practice, ed 2. Chicago:
Quintessence, 2001.
Nishizawa Y, Nakamura T, Ohta H, et al. Guidelines for the use of biochemical markers of bone turnover
in osteoporosis (2004). J Bone Miner Metab 2005;23:97–104.
Ogle OE, Dym H, Weinstock RJ (eds). Medical Emergencies in Dental Practice. Chicago: Quintessence, 2016.
Pallasch TJ. Clostridium difficile-associated diarrhea and colitis. J Calif Dent Assoc 1999;27:405–413.
Pallasch TJ. Global antibiotic resistance and its impact on the dental community. J Calif Dent Assoc
2000;28:215–233.
Parsons SE. Pharmaceutical Calculations. Pittsburgh: Parsons, 2012.
Schneider JP, Miller A. Oxycodone to oxymorphone metabolism. Practical Pain Management. http://www.
practicalpainmanagement.com/treatments/pharmacological/opioids/oxycodone-oxymorphone-metab-
olism. Accessed 21 April 2016.
Simon AM, O’Connor JP. Dose and time-dependent effects of cyclooxygenase-2 inhibition on fracture-heal-
ing. J Bone Joint Surg Am 2007;89:500–511.
Simon LS. Role and regulation of cyclooxygenase-2 during inflammation. Am J Med 1999;106(5B
suppl):37S–42S.
Sorsa T, Tjäderhane L, Konttinen YT, et al. Matrix metalloproteinases: Contribution to pathogenesis,
diagnosis and treatment of periodontal inflammation. Ann Med 2006;38:306–321.
Trevor AJ, Katzung BG, Masters SB. Katzung & Trevor’s Pharmacology Examination and Board Review,
ed 7. New York: McGraw-Hill, 2003.
Wang GK. Binding affinity and steroselectivity of local anesthetics in single batrachotoxin-activated Na+
channels. J Gen Physiol 1990;96:1105–1127.
Weinstein RD. True strength. J Bone Miner Res 2000;15:621–625.
Wu X, Al-Abedalla K, Rastikerdar E, et al. Selective serotonin reuptake inhibitors and the risk of osseointe-
grated implant failure: A cohort study. J Dent Res 2014;93:1054–1061.
Yagiela JA, Dowd FJ, Johnson BS, Mariotti AJ, Neidle EA. Pharmacology and Therapeutics for Dentistry,
ed 6. St Louis: Mosby, 2011.

Chapter 6
Borie E, Orsi IA, de Araujo CP. The influence of the connection, length and diameter of an implant on
bone biomechanics. Acta Odontol Scand 2015;73:321–329.
Goiato MC, Shibayama R, Gennari Filho H, et al. Stress distribution in implant-supported prostheses
using different connection systems and cantilever lengths: Digital photoelasticity. J Med Eng Technol
2016;40(2):35–42.
Romeed SA, Malik R, Dunne SM. Marginal bone loss influence on the biomechanics of single implant
crowns. J Craniofac Surg 2013;24:1459–1465.
Siadat H, Hashemzadeh S, Geramy A, Bassir SH, Alikhasi M. Effect of offset implant placement on the
stress distribution around a dental implant: A three-dimensional finite element analysis. J Oral Implantol
2015;41:646–651.

258
Chapter 7
Aquilino SA, Shugars DA, Bader JD, White BA. Ten-year survival rates of teeth adjacent to treated and
untreated posterior bounded edentulous spaces. J Prosthet Dent 2001;85:455–460.
Baldissara P, Llukacej A, Ciocca L, Valandro FL, Scotti R. Translucency of zirconia copings made with
different CAD/CAM systems. J Prosthet Dent 2010;104:6–12.
Beumer J III, Faulkner RF, Shah KC, Moy PK. Fundamentals of Implant Dentistry, Volume 1: Prosthodontic
Principles. Chicago: Quintessence, 2015.
Brånemark PI (ed). The Brånemark Novum Protocol for Same-Day Teeth: A Global Perspective. Chicago:
Quintessence, 2001.
Burns DR, Unger JW, Coffey JP, Waldrop TC, Elswick RK Jr. Randomized, prospective, clinical evalu-
ation of prosthodontic modalities for mandibular implant overdenture treatment. J Prosthet Dent
2011;106:12–22.
Cantor R, Curtis TA, Shipp T, Beumer J 3rd, Vogel BS. Maxillary speech prostheses for mandibular surgical
defects. J Prosthet Dent 1969;22:253–260.
das Neves FD, Fones D, Bernardes SR, do Prado CJ, Neto AJF. Short implants: An analysis of longitudinal
studies. Int J Oral Maxillofac Implants 2006;21:86–93.
Davis DM, Packer ME, Watson RM. Maintenance requirements of implant-supported fixed prostheses
opposed by implant-supported fixed prostheses, natural teeth, or complete dentures: A 5-year retro-
spective study. Int J Prosthodont 2003;16:521–523.
Duyck J, Van Oosterwyck H, Vander Sloten J, De Cooman M, Puers R, Naert I. Magnitude and distribution
of occlusal forces on oral implants supporting fixed prostheses: An in vivo study. Clin Oral Implants
Res 2000;11:465–475.
Garber DA, Salama MA. The aesthetic smile: Diagnosis and treatment. Periodontol 2000 1996;11:18–28.
Heitz-Mayfield LJ, Huynh-Ba G. History of treated periodontitis and smoking as risks for implant therapy.
Int J Oral Maxillofac Implants 2009;24(suppl):39–68.
Jiménez-López V. Oral Rehabilitation with Implant-Supported Prostheses. Chicago: Quintessence, 1999.
Johnsson AA, Sawaii T, Jacobsson M, Granström G, Turesson I. A histomorphometric study of bone reac-
tions to titanium implants in irradiated bone and the effect of hyperbaric oxygen treatment. Int J Oral
Maxillofac Implants 1999;14:699–706.
Kim JH, Kim KR, Kim S. Critical appraisal of implant impression accuracies: A systematic review. J Prosthet
Dent 2015;114:185.e1–192.e1.
Kim JS, Raigrodski AJ, Flinn BD, Rubenstein JE, Chung KH, Mancl LA. In vitro assessment of three types
of zirconia implant abutments under static load. J Prosthet Dent 2013;109:255–263.
Misch CE, Judy KW. Classification of partially edentulous arches for implant dentistry. Int J Oral Implantol
1987;4:7–13.
Murray C, Herson J, Daly T, Zimmerman S. Radiation necrosis of the mandible: A 10-year study. Int J
Radiat Oncol Biol Phys 1980;6:549–553.
Ogiso M, Tabata T, Kuo PT, Borgese D. A histologic comparison of the functional loading capacity of an
occluded dense apatite implant and the natural dentition. J Prosthet Dent 1994;71:581–588.
Parel SM, Sullivan DY. Esthetics and Osseointegration. Dallas: Osseointegration Seminars Inc, 1989.
Phoenix RD, Cagna DR, DeFreest CF. Stewart’s Clinical Removable Partial Prosthodontics, ed 3. Chicago:
Quintessence, 2003.
Preiskel HW. Overdentures Made Easy: A Guide to Implant and Root Supported Prostheses. Chicago:
Quintessence, 1996.
Renvert S, Quirynen M. Risk indicators for peri-implantitis. A narrative review. Clin Oral Implants Res
2015;26(suppl 11):15–44.
Salama M, Ishikawa T, Salama H, Funato A, Garber D. Advantages of the root submergence technique for
pontic site development in esthetic implant therapy. Int J Periodontics Restorative Dent 2007;27:521–527.
Shugaa-Addin B, Al-Shamiri HM, Al-Maweri S, Tarakji B. The effect of radiotherapy on survival of dental
implants in head and neck cancer patients. J Clin Exp Dent 2016;8:e194–e200.

259
BIBLIOGRAPHY

Stevens PJ, Frederickson EJ, Gress ML. Implant Prosthodontics: Clinical and Laboratory Procedures,
ed 2. St Louis: Mosby, 2000.
Tarnow D, Elian N, Fletcher P, et al. Vertical distance from the crest of bone to the height of the interprox-
imal papilla between adjacent implants. J Periodontol 2003;74:1785–1788.
Tarnow DP, Magner AW, Fletcher P. The effect of the distance from the contact point to the crest of bone
on the presence or absence of the interproximal dental papilla. J Periodontol 1992;63:995–996.
ten Bruggenkate CM, Asikainen P, Foitzik C, Krekeler G, Sutter F. Short (6-mm) nonsubmerged dental implants:
Results of a multicenter clinical trial of 1 to 7 years. Int J Oral Maxillofac Implants 1998;13:791–798.
Tsagkalidis G, Tortopidis D, Mpikos P, Kaisarlis G, Koidis P. Accuracy of 3 different impression techniques
for internal connection angulated implants. J Prosthet Dent 2015;114:517–523.
Venezia P, Torsello F, Cavalcanti R, D’Amato S. Retrospective analysis of 26 complete-arch implant-sup-
ported monolithic zirconia prostheses with feldspathic porcelain veneering limited to the facial surface.
J Prosthet Dent 2015;114:506–512.
Williams BH, Ochiai KT, Hojo S, Nishimura R, Caputo AA. Retention of maxillary implant overdenture bars
of different designs. J Prosthet Dent 2001;86:603–607.
Williamson EH, Lundquist D. Anterior guidance: Its effect on electromyographic activity of the temporal
and masseter muscles. J Prosthet Dent 1983;49:816–823.
Zarb GA, Bolender CL. Prosthodontic Treatment for Edentulous Patients: Complete Dentures and
Implant-Supported Prostheses, ed 12. St Louis: Mosby, 2004.

Chapter 8
Cha HS, Kim JW, Hwang JH, Ahn KM. Frequency of bone graft in implant surgery. Maxillofac Plast Reconstr
Surg 2016;38:19.
Deeb GR, Deeb JG. Soft tissue grafting around teeth and implants. Oral Maxillofac Surg Clin North Am
2015;27:425–448.
Dense-PTFE Membrane vs. Expanded-PTFE Membrane? https://www.osseonews.com/dense-ptfe-
membrane-vs-expanded-ptfe-membrane/Accessed 10 November 2022.
Louise F, Dragan O. Essentials of Maxillary Sinus Augmentation. London: Quintessence, 2018:17.
Stimmelmayr M, Allen EP, Reichert TE, Iglhaut G. Use of a combination epithelized-subepithelial connective
tissue graft for closure and soft tissue augmentation of an extraction site following ridge preservation or
implant placement: Description of a technique. Int J Periodontics Restorative Dent 2010;30:375–381.
Third-Generation PTFE Barriers Break the Stigma in Bone Regeneration Cases. https://decisionsindentistry.
com/article/unicare-biomedical/Accessed 10 November 2022.
Tolstunov L. Vertical Alveolar Ridge Augmentation in Implant Dentistry: A Surgical Manual. Hoboken, NJ:
Wiley-Blackwell, 2016.
Zhao R, Yang R, Cooper PR, Khurshid Z, Shavandi A, Ratnayake J. Bone grafts and substitutes in dentistry:
A review of current trends and developments. Molecules 2021;26:3007.

Chapter 10
Alberti A, Francetti L, Taschieri S, Corbella S. The applications of enamel matrix derivative in implant
dentistry: A narrative review. Materials (Basel) 2021;14:3045.
Ankrum JA, Ong JF, Karp JM. Mesenchymal stem cells: Immune evasive, not immune privileged. Nat
Biotechnol 2014;32:252–260.
Cervera-Maillo JM, Morales-Schwarz D, Morales-Melendez H, Mahesh L, Calvo-Guirado JL. Autologous
tooth dentin graft: A retrospective study in humans. Medicina (Kaunas) 2021;58:56.
Difference Between Amnion and Chorion. http://www.differencebetween.net/science/health/differ-
ence-between-amnion-and-chorion/Accessed 10 November 2022.
Dwek JR. The periosteum: What is it, where is it, and what mimics it in its absence? Skeletal Radiol
2010;39:319–323.

260
Freitas RM, Spin-Neto R, Marcantonio E Jr, Pereira LAVD, Wikesjö UME, Susin C. Alveolar ridge and maxil-
lary sinus augmentation using rhBMP-2: A systematic review. Clin Implant Dent Relat Res 2015;17(suppl
1):e192–e201.
Gupta A, Kedige SD, Jain K. Amnion and chorion membranes: Potential stem cell reservoir with wide
applications in periodontics. Int J Biomater 2015;2015:274082.
Harrison S, Vavken P, Kevy S, Jacobson M, Zurakowski D, Murray MM. Platelet activation by collagen
provides sustained release of anabolic cytokines. Am J Sports Med 2011;39:729–734.
Hassan M, Prakasam S, Bain C, Ghoneima A, Liu SSY. A randomized split-mouth clinical trial on effective-
ness of amnion-chorion membranes in alveolar ridge preservation: A clinical, radiologic, and morpho-
metric study. Int J Oral Maxillofac Implants 2017;32:1389–1398.
He L, Lin Y, Hu X, Zhang Y, Wu H. A comparative study of platelet-rich fibrin (PRF) and platelet-rich plasma
(PRP) on the effect of proliferation and differentiation of rat osteoblasts in vitro. Oral Surg Oral Med
Oral Pathol Oral Radiol Endod 2009;108:707–713.
Herford AS, Stoffella E, Stanford CM. Bone grafts and bone substitute materials. In: Torabinejad M,
Sabeti MA, Goodacre CJ (eds). Principles and Practice of Single Implant and Restoration. Philadelphia:
Saunders, 2014:75–86.
Ikawa T, Akizuki T, Shujaa Addin A, Fukuba S, Stavropoulos A, Izumi Y. Enamel matrix derivative in liquid
form as adjunct to natural bovine bone grafting at buccal bone dehiscence defects at implant sites: An
experimental study in beagle dogs. Clin Oral Implants Res 2019;30:989–996.
James AW, LaChaud G, Shen J, et al. A review of the clinical side effects of bone morphogenetic protein-2.
Tissue Eng Part B Rev 2016;22:284–297.
Kobayashi E, Flückiger L, Fujioka-Kobayashi M, et al. Comparative release of growth factors from PRP,
PRF, and advanced-PRF. Clin Oral Investig 2016;20:2353–2360.
Lynch Biologics website. https://www.lynchbiologics.com/gem-21s/Accessed 10 Nobember 2022.
Matichescu A, Ardelean LC, Rusu LC, et al. Advanced biomaterials and techniques for oral tissue engi-
neering and regeneration­—A review. Materials (Basel) 2020;13:5303.
Pallesen L, Schou S, Aaboe M, Hjørting-Hansen E, Nattestad A, Melsen F. Influence of particle size of
autogenous bone grafts on the early stages of bone regeneration: A histologic and stereologic study
in rabbit calvarium. Int J Oral Maxillofac Implants 2002;17:498–506.
Park SY, Kim KH, Kim S, Lee YM, Seol YJ. BMP-2 gene delivery-based bone regeneration in dentistry.
Pharmaceutics 2019;11:393.
Paz AG, Maghaireh H, Mangano FG. Stem Cells in Dentistry: Types of intra- and extraoral tissue-derived
stem cells and clinical applications. Stem Cells Int 2018; 2018:4313610.
Rutkowski JL, Thomas JM, Bering CL, et al. Analysis of a rapid, simple, and inexpensive technique used
to obtain platelet-rich plasma for use in clinical practice. J Oral Implantol 2008;34:25–33.
Singh P, Suresh DK. Clinical evaluation of GEM 21S(®) and a collagen membrane with a coronally advanced
flap as a root coverage procedure in the treatment of gingival recession defects: A comparative study.
J Indian Soc Periodontol 2012;16:577–583.
Soudi A, Yazdanian M, Ranjbar R, et al. Role and application of stem cells in dental regeneration:
A comprehensive overview. EXCLI J 2021;20:454–489.
Wakefield LM, Winokur TS, Hollands RS, Christopherson K, Levinson AD, Sporn MB. Recombinant latent
transforming growth factor beta 1 has a longer plasma half-life in rats than active transforming growth
factor beta 1, and a different tissue distribution. J Clin Invest 1990;86:1976–1984.
Woo EJ. Adverse events reported after the use of recombinant human bone morphogenetic protein 2.
J Oral Maxillofac Surg 2012;70:765–767.
Zhang W, Wang N, Yang M, et al. Periosteum and development of the tissue-engineered periosteum for
guided bone regeneration. J Orthop Translat 2022;33:41–54.

261
ALFARAJE
CONTENTS

ORAL IMPLANTOLOGY REVIEW, SECOND EDITION


1 Medical Evaluation of the Implant Patient

2 Oral Evaluation and Treatment Planning

3 Head and Neck Anatomy


LOUIE ALFARAJE, DDS
4 Surgical Procedures and Complications

5 Pharmacology

6 Biomechanics

7
8
Implant Prosthodontics
ORAL
IMPLANTOLOGY
Bone Grafting

9 Zygomatic and Pterygoid Implants

10

REVIEW
Blood Concentrates and Growth Factors

SECOND EDITION
978-1-64724-156-8

You might also like